You are on page 1of 673

PREPARED BY:-

Dr. Dler R. Abdulkareem


1
‫ميحرلا نمحرلا هللا مسب‬
‫و‌ميهرهبان‬
‫‌‬ ‫خشنده‌‬
‫‌‬ ‫‌‬
‫ناوى‌خوای‌به‬ ‫به‌‬
‫‌‬

‫‪In the name of ALLAH, most gracious,‬‬


‫‪most merciful‬‬

‫‪2‬‬
3
Educational Testing
and Assessment
Systems
Derm In-Review | Board Review Study Questions
www.dermatologyinreview.com

Prepared by:-
Dr. Dler R. Abdulkareem

Written and Developed by

4
Board Prep: Custom Quizzes & Timed
Practice Exams

Chapter No. of Page


CHAPTERS
No. Questions No.

All Questions 3451 672

1 Basic Science and Structure of the Skin 203 7

2 Immunodermatology 161 82

3 Genodermatoses 386 138

4 General Dermatology 423 285

5 Dermatopathology 353 445

6 Benign and Malignant Neoplasms 164 568

7 Medical Mycology 118 628

5
6
Chapter -1-
Basic Science and Structure of the Skin
1
A specific marker of Merkel cells is:
A. Cytokeratin 10
B. Cytokeratin 15
C. Cytokeratin 20
D. Loricrin
E. Envoplakin
►C

Cytokeratin 20 is a specific marker for the Merkel cell. Merkel cells are mechanoreceptors
located at body sites requiring high tactile sensitivity. Keratinocyte deformation results in a
secretion of chemokines by Merkel cells, which make synaptic connection with neurons.

2
Darier's sign is described as:
A. When an intact epidermis shears away from the underlying dermis, leaving a
moist surface
B. Spreading bulla phenomenon with pressure on an intact bulla
C. Swollen, itchy and or red after stroking the skin
D. Central depression within a lesion when squeezed along its margins
E. Disappearance of color when the lesion is pressed
►C

Swollen, itchy and or red after stroking the skin is referred to Darier's sign and can be seen in
systemic mastocytosis or urticaria pigmentosa. Nikolsky sign can be seen when an intact
epidermis shears away from the underlying dermis, leaving a moist surface (seen in pemphigus
vulgaris, staphylococcus scalded skin syndrome (SSSS), and toxic epidermal necrosis).
Spreading bulla phenomenon with pressure on an intact bulla is referred to as Asboe-Hansen
sign, commonly seen with pemphigus vulgaris. Central depression within a lesion when
squeezed is referred to as the dimpling sign and is seen in dermatofibromas. Disappearance of
color or blanching when the lesion is pressed is commonly found on vascular lesions

3
A salt split skin DIF is performed on a biopsy taken adjacent to the skin lesions shown. Where
would you expect staining to be seen?
A. Epidermal side

7
B. Dermal side
C. Epidermal and Dermal sides equally
D. In the lamina densa
E. In the anchoring plaques
►A

The image shown is bullous pemphigoid. On salt split skin DIF exams, deposits are seen on the
epidermal side of the split. If dermal deposits are seen, epidermolysis bullosa acquisita or anti-
epiligrin pemphigoid are potential diagnoses.

4
What does a western blot identify?
A. Protein
B. RNA
C. DNA
D. Genes
E. Cell membranes
►A

A northern blot identifies RNA and a southern blot identifies DNA.

5
Which one of the following is responsible for maintaining a barrier to water loss in the stratum
corneum?
A. Involucrin
B. Filaggrin
C. Loricrin
D. Transglutaminase
E. Odland bodies
►E

Odland bodies, also known as lamellar granules, keratinosomes, and membrane-coating granules,
are small organelles that are discharged from granular cells into the intracellular space of the
granular layer of the epidermis. These bodies have two known functions: they mediate stratum
corneum cell cohesion and they form a barrier to water loss. Odland bodies are round to oval,
measure approximately 300 to 500 nm in diameter, and possess a trilaminar membrane and a
laminated interior. They contain neutral sugars linked to lipids and/or proteins, hydrolytic
enzymes, and free sterols. Filaggrin is a breakdown product of filaggrin precursor, a component
of keratohyaline granules, which aggregates with keratin filaments and acts as a "glue" for
keratin filaments. Involucrin is a cysteine-rich protein synthesized in the cytoplasm of spinous

8
cells. The enzyme, transglutaminase, cross-links involucrin in the granular layer forming an
insoluble cell boundary that is resistant to denaturing and reducing chemicals. Loricrin, is a
highly insoluble sulfur- and glycine/serine-rich protein, which is the major protein comprising
the cornified cell envelope.

6
Dimple sign refers to:
A. When an intact epidermis shears away from the underlying dermis, leaving a
moist surface
B. Spreading bulla phenomenon with pressure on an intact bulla
C. Swollen, itchy and or red after stroking the skin
D. Central depression within a lesion when squeezed along its margins
E. Disappearance of color when the lesion is pressed
►D

Central depression within a lesion when squeezed is referred to as the dimpling sign and is seen
in dermatofibromas. Nikolsky sign can be seen when an intact epidermis shears away from the
underlying dermis, leaving a moist surface (seen in pemphigus vulgaris, staphylococcus scalded
skin syndrome (SSSS), and toxic epidermal necrosis). Spreading bulla phenomenon with
pressure on an intact bulla is referred to as Asboe-Hansen sign, commonly seen with pemphigus
vulgaris. Swollen, itchy and or red after stroking the skin is referred to Darier′s sign and can be
seen in systemic mastocytosis or urticaria pigmentosa. Disappearance of color or blanching when
the lesion is pressed is commonly found on vascular lesions

7
At any one time, the approximate proportion of hair follicles in anagen is:
A. 40%
B. 60%
C. 85%
D. 95%
E. 15%
►C

Most hair follicles are in anagen, and thus most hair follicles involve growing hair. The longer a
hair follicle is anagen, the longer the hair can grow in length. Hairs of the scalp grow
approximately 0.4 mm per day, and thus the date of your next hair cut can be accurately
calculated.

8
All mononuclear phagocytic cells in the dermis express:

9
A. CD3
B. CD6
C. CD34
D. CD68
E. CD20
►B

CD6 and CD11c are expressed on all mononuclear phagocytic cells in the dermis. CD3 is a T-
cell marker and CD20 is a B-cell marker. CD34 is expressed on mast cells and CD68 on
macrophages.

9
Homocystinuria is an autosomal recessive condition with findings including a marfanoid habitus,
downward dislocation of the lens, cardiovascular disease and mental retardation. It is caused by a
mutation in cystathionine beta-synthetase. What does this mutation in cystathionine beta-
synthetase cause other than an accumulation of homocystine?
A. Abnormal crosslinking of collagen
B. Abnormal development of elastin fibers
C. Melanocyte death
D. Pigmentation of cartilage
E. Black urine
►A

Excess homocystine leads to abnormal crosslinking of collagen, with only ~1/3 of crosslinking
activity compared to normal controls (OMIM 236200). The other listed findings are not features
of homocystinuria, though hypopigmentation can be a feature.

10
Which sebaceous gland is located on the areola?
A. Montgomery's tubercles
B. Tyson′s glands
C. Zeis glands
D. Meibomian glands
E. Fordyce's spots
►A

Montgomery's tubercles are sebaceous glands not associated with hair follicles present on the
areola. Tyson's glands on the labia minora and glans, meibomina and Zeis glands are present on
the eyelids , and Fordyce's spots are located on the buccal mucosa.

10
11
Which of the following statements about the direct immunofluorescence pattern in lichen planus
is correct?
A. The DIF is negative in the vast majority of cases
B. Deposition of IgG is within cytoid bodies in the superficial dermis
C. The DEJ deposition is granular
D. Deposition of fibrinogen is within cytoid bodies in the deep dermis
E. There is prominent deposition of IgM within the spinous layer of the epidermis.
►C

The DEJ deposition is granular. The DIF is positive in the vast majority of cases. Deposition of
IgM and fibrinogen is within cytoid bodies in the superficial dermis. There is no deposition of
IgM within the spinous layer of the epidermis.

12
Which of the following enzymes does not require copper for functioning?
A. Lysyl oxidase
B. ATP7a
C. Tyrosinase
D. Cystathione beta-synthase
E. Ferrochelatase
►E

All of the listed enzymes are copper containing or dependent except ferrochelatase. Lysyl
oxidase facilitates crosslinking of fibrillin in elastic fibers. ATP7a is deficient in Menkes Kinky
Hair Syndrome. Cystathione beta-synthase is defective in homocystinuria. Tyrosinase catalyzes
the first 2 steps, and at least 1 subsequent step, in the conversion of tyrosine to melanin.
Ferrochelatase mutation leads to excess protoporphyrin production and photosensitivity.

13
When evaluating a foreign body, which substance would be PAS negative and have no
bifringence on polarizing microscopy?
A. Silica
B. Talc
C. Zinc
D. Aluminium
E. Wood splinters
►D

11
Starch, cactus spines, and wood splinters are PAS positive. Silica, talc, zinc, and wood splinters
are positive for bifringence on microscopy.

14
The antibody target in ocular cicatricial pemphigoid is also mutated in:
A. Junctional epidermolysis bullosa, Herlitz type
B. Recessive dystrophic epidermolysis bullosa
C. Junctional epidermolysis bullosa with myotonic dystrophy
D. Dominant dystrophic epidermolysis bullosa
E. Junctional epidermolysis bullosa with pyloric atresia
►E

JEB with pyloric atresia and ocular cicatricial pemphigoid both have mutations in Beta4integrin.
Herlitz type JEB is due to laminin 5. Recessive and dominant dystrophic EB is due to COL17A1
(type VII collagen). JEB with myotonic dystrophy is due to plectin.

15
Which of the following is true regarding BPAg1?
A. It is pathogenic in cicatricial pemphigoid
B. It is a member of the plakin family
C. It is pathogenic in pemphigoid gestationis
D. It is not pathogenic in paraneoplastic pemphigus
E. It coprecipitates with plakoglobin
►B

BPAg1 is a member of the plakin family, which includes envoplakin, periplakin, desmoplakin,
plectin, and BPAg1. BPAg2 is pathogenic in cicatricial pemphigoid and pemphigoid gestationis.
BPAg1 is pathogenic in paraneoplastic pemphigus. Desmoglein 3 is pathogenic in pemphigus
vulgaris and coprecipitates with plakoglobin.

16
The finding on DIF that reflects binding of the Ro and La antigens in subacute cutaneous lupus
erythematosus is:
A. Granular fluorescence throughout the cytoplasm and nucleus of basal
keratinocytes
B. Cytoid bodies
C. Immune deposits along the DE junction
D. Granular deposits along the basement membrane
E. A "chicken-wire" pattern within the epidermis
►A

12
Granular fluorescence throughout the cytoplasm and nucleus of basal keratinocytes reflect the
binding of Ro and La antigens and is unique to SCLE. Cytoid bodies and Immune deposits along
dermal-epidermal junction are seen in both DLE and SCLE. Granular deposits along the
basement membrane are seen in dermatitis herpetiformis and a chicken-wire pattern of staining is
seen in pemphigus vulgaris.

17
Which of the following cells are required for wound healing?
A. Neutrophil
B. Macrophage
C. Eosinophil
D. Langerhans cell
E. Lymphocyte
►B

The macrophage is required for wound healing. The macrophages debride tissue, secrete
collagenase and stimulate expression of FGF, IL-1, TGF-beta, PDGF and TGF-alpha thus
facilitating transition from inflammation to repair.

18
Regarding the stratum germinativum (basale):
A. Intermediate filaments in basal cells insert into only hemidesmosomes
B. Keratins 1 and 10 are expressed
C. Not all basal cells have the potential to divide
D. Microfilaments assist in downward movement of cells
E. Plectins regulate adhesion and initiation of differentiation.
►C

Not all basal cells have the potential to divide. Stem cells give rise to transient amplifying cells
which give rise to the epidermal keratinocytes. Intermediate filaments in basal cells insert into
desmosomes and hemidesmosomes. Keratins 5/14 are predominantly expressed. Microfilaments
assist in upward movement of cells. Integrins regulate adhesion and initiation of differentiation.

19
Meibomian glands are:
A. Eccrine glands localized to the vermillion border of the lips
B. Sebaceous glands found on the areola of the breast
C. Sebaceous glands found on the eyelids
D. Apocrine glands found in the anogenital regions

13
E. Apocrine glands found on the eylelids
►C

Sebaceous glands enlarge at puberty in response to increased levels of androgens. They are
holocrine glands. Meibomian glands are modified sebaceous glands found in the eyelids. Free
sebaceous glands not associated with hairs are found in the nipple and areola and are called
Montgomery′s tubercles. Fordyce′s condition involves free sebaceous glands on the vermillion
border of the lips and on the buccal mucosa. Sebaceous glands are found everywhere on the skin
except the palms and soles.

20
Epidermolysis bullosa simplex (EBS), Weber Cockayne type, is caused by what defect?
A. Collagen VII
B. Alpha-6-beta-4 integrin
C. Keratins 1 & 10
D. Keratins 5
E. Plectin
►D

All subtypes of EBS are caused by a defect in keratins 5 & 14 except EBS with muscular
dystrophy which is caused by a defect in plectin. An alpha-6-beta-4 integrin defect is seen in
junctional epidermolysis bullosa (JEB) with pyloric atresia. Collagen VII defect is seen in EB
aquisita (EBA), Bart's syndrome and dominant EB (DEB). Keratin 1 & 10 defects are not seen in
any of the EB subtypes.

21
Mutations in which of the following genes will produce red hair?
A. MC1-R
B. Agouti
C. Hairless
D. c-kit
E. Tyrosinase
►A

MC1-R is the receptor that binds alpha-MSH, and ACTH>alpha-MSH. Defects in this receptor
will produce a phenotype including red hair. Agouti is not involved in this process. Agouti
describes the banding of hairs seen in some mammals such as dogs, foxes, and mice, which is
due to alternating production of eumelanin and pheomelanin occurs. This has not been described
in humans (Bolognia, p942). The hairless gene is associated with alopecia totalis. C-kit

14
mutations are associated with piebaldism and urticaria pigmentosa. Tyrosinase defects are
associated with Type I Oculocutaneous albinism.

22
Which of the following is true in or associated with Apert Syndrome?
A. Secondary to a mutation in FGFR3
B. AR
C. Acne localized to buttocks and thighs
D. Cutaneous/ocular depigmentation
E. Synostoses
►E

Apert Syndrome is an AD syndrome secondary to a mutation in FGFR2. Also known as


acrocephlosyndactyly, this condition includes synostoses of the hands, feet, back and skull as
well as generalized acne. Mosaicism of this gene causes nevus comedonicus. There is associated
cutaneous and ocular hypopigmentation.

23
Which of the following markers are specific and reliable for Merkel cells?
A. CD20
B. HMB-45
C. CD34
D. CD3
E. CK20
►E

Cytokeratin (CK) 20 is a reliable marker for Merkel cells. CD20 is a marker for B-cells and CD3
is a T-cell marker. HMB-45 is used in staining for immature melanosomes and is reactive in
melanoma. CD34 is used to stain dermatofibrosarcoma protuberans.

24
Dermal dendrocytes:
A. Are responsible for immediate-type hypersensitivity reactions.
B. Actively synthesize and release IgG.
C. Present antigen to naïve T cells in the lymph node.
D. Are the primary cell found in a glomangioma.
E. Are the primary cell found in an angiosarcoma.
►C

15
A dermal dendrocyte is a mononuclear phagocytic cell that is a type of antigen presenting cell. It
is derived from the bone marrow and found both in the papillary and upper reticular dermis. This
cell is highly phagocytic and synonymous with the melanophage that has ingested pigment.
Dermal dendrocytes are likely very important to the afferent limb of the immune response.

25
LEMD3 is mutated in which of the following syndromes?
A. Buschke Ollendorf
B. Albright Hereditary Osteodystrophy
C. Goltz
D. McCune-Albright
E. Menkes
►A

LEMD3 or MAN1 is mutated in Buschke-Ollendorf syndrome. Common skin findings include


dermatofibrofibrosis lenticularis disseminata. The radiographic finding commonly seen is
osteopoikilosis. The gene defect was recently described as MAN1/LEMD3, a gene encoding for
a nuclear membrane protein. Albright Hereditary Osteodystrophy is caused by inactivation of
GNAS1, Goltz is an x-linked dominant condition with unknown pathogenesis, McCune-Albright
syndrome is caused by an activation of GNAS1. Menke's Syndrome is caused by a defect in
ATP7A/MKN on Xq13.

26
Granulation tissue primarily contains:
A. Collagen I
B. Collagen III
C. Collagen IV
D. Fibrin
E. Collagen VII
►B

Granulation tissue begins to form four days after injury and is composed of new capillaries,
macrophages, fibroblasts, blood vessels and primarily collagen type III. Formulation of
granulation tissue is dependent on the presence of fibronectin and progresses through an orderly
sequence of matrix deposition: fibronectin to collagen III then finally collagen I.

27
Sneddon Wilkinson disease is caused by a defect in what antigen?
A. Desmocollin 1
B. Desmoglein 1

16
C. Desmogelin 3
D. BPAg1
E. BPAg2
►A

Sneddon- Wilkinson disease, also known as sub-corneal pustular dermatosis is due to a defect in
desmocollin 1. Desmoglein 1 defect is seen in pemphigus foliaceous and staph scalded skin
syndrome. Desmoglein 3 defect is seen in pemphigus vulgaris and intraepidermal neutrophilic
IgA dermatosis. BPAg1 is seen in paraneoplastic pemphigus and bullous pemphigoid. BPAg2 is
seen in bullous pemphigoid, cicatricial mucous membrane pemphigoid and linear IgA disease.

28
Which epidermal layer do pilar cysts generally not have a:
A. Stratum corneum
B. Stratum granulosum
C. Stratum spinosum
D. Stratum basale
E. Stratum lucidum
►B

Pilar cysts do not have a granular layer, they do have all the other layers.

29
Type VII collagen in found in anchoring fibrils and also in:
A. Fetal skin
B. Bone
C. Amnion
D. Aorta
E. Blood vessels
►C

Type VII collagen is present in anchoring fibrils and amnion. Fetal skin and blood vessels
contain type III collagen. The aorta contains type VI collagen. Bone contains type I collagen.

30
What is the major component of the anchoring fibril?
A. Type I Collagen
B. Type III Collagen
C. Type IV Collagen
D. Type VII Collagen

17
E. Laminin 5
►D

Anchoring fibrils are made of Type VII collagen. Anchoring plaques are made of Type IV
collagen, and interact with a network of Type I And Type III collagen fibers in the dermis.

31
1 melanocyte has contact with:
A. 6 keratinocytes
B. 12 keratinocytes
C. 18 keratinocytes
D. 30 keratinocytes
E. 36 keratinocytes
►E

One melanocyte and 36 keratinocytes make up one epidermal melanin unit. This melanocytes
transfers pigment to these keratinocytes.

32
Which hormone is homologous to alpha-MSH (melanocyte stimulating hormone)?
A. Insulin
B. Human growth factor
C. Prolactin
D. Thyroid stimulating hormone
E. Adrenocorticotropic hormone
►E

Both alpha-MSH and ACTH are cleavage products of proopiomelanocortin (POMC).

33
Sebaceous carcinoma stains positive for:
A. EMA
B. CEA
C. S100
D. cKIT
E. CD1a
►A

Sebaceous carcinomas stain positive for EMA. The other answer choices are incorrect as they do
not stain positive in sebaceous carcinoma.

18
34
What is the major function of urocanic acid?
A. Bacteriocidal acid produced by stratum corneum
B. Primarily a UVB filter
C. Primarily a UVA filter
D. Helps degrade free fatty acids
E. Aids in protecting the skin from dermatophytes
►C

Urocanic acid a byproduct of filaggrin degradation and has a peak absorbtion of 345 nm. It
serves as a major UVA filter.

35
Once a keratinocyte leaves the basal cell layer, the normal transit time to stratum corneum is at
least:
A. 7 days
B. 14 days
C. 21 days
D. 28 days
E. 35 days
►B

Once a basal cell leaves the basal layer in humans, normal transit time to stratum corneum is at
least 14 days. Transit time through the stratum corneum to desquamation requires 14 more days.

36
Which keratins are expressed in the stratum germinativum and are present but not made de novo
in the stratum spinosum?
A. K4, 13
B. K1,10
C. K2e, 10
D. K3,12
E. K5,14
►E

K5 and K14 are expressed in the stratum germinativum (basal layer) and are defective in
epidermolysis bullosa simplex. They are still present in the stratum spinosum but are not made
de novo in this layer. K4 and 13 are found in mucosal epithelium. K1 and 10 are expressed in

19
suprabasal keratinocytes, K2e and 10 in the upper spinous and granular layers, and K3 and K12
in the cornea.

37
The triple helix of the collagen molecule is largely maintained due to its amino acid composition.
The polypeptide chains of collagen are repeating triplets of Glycine-X-Y. The X and Y positions
can be occupied by multiple amino acids, but are most often:
A. Alanine and asparagine
B. Tyrosine and threonine
C. Histidine and ornithine
D. Leucine and isoleucine
E. Proline and hydroxyproline
►E

Collagen fibers provide tensile strength to the skin and allow it to serve as a protective organ
against the external environment. Collagen makes up 80% of the dry weight of the dermis. The
structure of collagen is comprised of three alpha chains arranged into a triple helix. The repeating
amino acid triplets of the triple helix of collagen are most often Glycine-Proline-Hydroxyproline.

38
Which eponym describes vestigial lines of pigmentary demarcation?
A. Futcher lines
B. Wallace's lines
C. Langer's lines
D. Lines of Blaschko
E. Dermatome
►A

Futcher lines are vestigial lines in which the dorsal surface has more melanocytes than ventral
surface. Wallace's lines are the well-demarcated lines around the margin of the foot and hand.

39
The main collagen component of the basement membrane is:
A. Collagen IV
B. Collagen III
C. Collagen I
D. Tenascin-X
E. Collagen VII
►A

20
Collagen IV is the main collagen component of basement membranes. Collagen I is the main
collagen of mature dermis, bone and tendon. Collagen III is found in fetal skin, blood vessels and
intestines. Tenascin-X is mutated in some forms of Ehlers-Danlos syndrome and is not
associated with the basement membrane. Collagen VII makes up anchoring fibrils and amnion.

40
Eccrine glands are found in all the following areas of the body except:
A. Axillae
B. Palms
C. Labia minora
D. Scalp
E. Cutaneous lip
►C

Eccrine glands are sweat glands enervated by cholinergic sympathetic nerves mediated by
acetylcholine. They absent on modified skin which lacks appendages like the vermillion border,
nail beds, glans penis, inner aspect of the prepuce and the labia minora.

41
Anchoring filaments originate at the hemidesmosomes and insert into the:
A. Desmosome
B. Sub basal dense plate
C. Lamina lucida
D. BPAG 180
E. Lamina densa
►E

Anchoring filaments (smaller than anchoring fibrils) stretch from the plasma membrane through
the subbasal dense plaque and the lamina lucida to the lamina densa.

42
Asboe-Hansen Sign refers to:
A. When an intact epidermis shears away from the underlying dermis, leaving a
moist surface
B. Spreading bulla phenomenon with pressure on an intact bulla
C. Swollen, itchy and or red after stroking the skin
D. Central depression within a lesion when squeezed along its margins
E. Disappearance of color when the lesion is pressed
►B

21
Spreading bulla phenomenon with pressure on an intact bulla is referred to as Asboe-Hansen
sign, commonly seen with pemphigus vulgaris. Nikolsky sign can be seen when an intact
epidermis shears away from the underlying dermis, leaving a moist surface (seen in pemphigus
vulgaris, staphylococcus scalded skin syndrome (SSSS), and toxic epidermal necrosis). Swollen,
itchy and or red after stroking the skin is referred to Darier′s sign and can be seen in systemic
mastocytosis or urticaria pigmentosa. Central depression within a lesion when squeezed is
referred to as the dimpling sign and is seen in dermatofibromas. Disappearance of color or
blanching when the lesion is pressed is commonly found on vascular lesions

43
The hair follicle is the site of production or conversion of which of the following hormones?
A. All the options are correct
B. dihydrotestosterone (DHT)
C. prolactin
D. adrenocorticotropin hormone (ACTH)
E. alpha-melanocyte stimulating hormone (MSH)
►A

The hair follicle converts gonadal or aderenal testosterone to dihydrotestosterone locally via 5-
alpha reductase. It also cleaves proopiomelanocortin to ACTH, alpha-MSH, and B-endorphin via
prohormone convertase. It also secretes corticotropin releasing hormone (CRH) as well as
prolactin.

44
Telogen effluvium:
A. Involves a diffuse alopecia affecting more than 50% of the scalp
B. Often results from antimetabolites used during cancer chemotherapy
C. Is an inflammatory alopecia devoid of scarring
D. Is a patchy alopecia affecting less than 50% of the scalp
E. Results from sudden illness or surgery
►E

Telogen effluvium is an excessive loss of club hairs from the normal resting follicles of the
scalp. The follicle is pushed from anagen to catagen to telogen. There is no inflammatory process
involved. Causes of telogen effluvium are illness, surgery, parturition, fever, drugs, traction,
starvation, and hypervitaminosis A. Usually, the hair loss only involves less than 50% of the
scalp. There is no specific therapy, and most cases are self-resolving within months.

45
Fragmentation and/or loss of elastic fibers in not seen in:

22
A. Cutis laxa
B. Marfan's syndrome
C. Anetoderma
D. Psuedoxanthoma elasticum
E. Buschke-Ollendorf syndrome
►E

Cutis laxa results from decreased desmosine and lysyl oxidase and demonstrates fragmentation
and loss of elastic fibers. Marfan's sydnrome results from decreased fibrillin I and demonstrates
fragmentation of elastic fibers. Pseudoxanthoma elasticum demonstrates increased
glycosaminoglycans on elastic fibers and the accumulation of fragmented and calcified elastic
fibers. Anetoderma has decreased desmosine and demonstrates loss and fragmentation of elastic
fibers. Buschke-Ollendorf syndrome exhibits increased desmosine and an increased amount of
thickened elastic fibers.

46
What is the most important cell for wound healing?
A. Fibroblasts
B. Neutrophils
C. Macrophages
D. Lymphocytes
E. Mast cells
►C

Macrophages are the most important cell for wound healing. They secrete TGFs, cytokines, IL-1,
TNF, and PDGF. Neutropenic or lymphopenic patients do not have impaired wound healing,
whereas macrophage-deficient (quantity or function) patients heal poorly. Neutrophils however
are the first cell type to flood the wound during phase I of inflammation (within first 6-8 hours);
this movement is facilitated by TGF-beta. Fibroblasts migrate into the wound by stage II
(granulation) where they produced glycosaminoglycans and fibronectin.

47
Which of the following polypeptides is found in the lamina lucida?
A. plakoglobin
B. desmoplakin
C. keratocalmin
D. demoyokin
E. laminin 5
►E

23
Laminin-5 is a basement membrane extracellular matrix protein that mediates attachment
substrate for both adhesion and migration in a wide variety of cell types, including epithelial
cells, fibroblasts, neurons and leukocytes and is a preferred adhesion substrate for epithelial cells
(Koshikawa et al., 2001). The remaining listed items are part of the desmosomal plaque in the
epidermis.

48
Which of the following abnormalities would be present in a patient with Harlequin ichthyosis?
A. lamellar granules are uniformly absent
B. keratohyaline granules are normal in all types of harlequin fetus
C. demoyokin mutation
D. band 6 protein is absent
E. plakoglobin is abnormal
►A

In Harlequin ichthyosis, the lamellar granules are uniformly abnormal or absent. On electron
microscopy, there is no evidence of the lipid lamellae which form due to lamellar granule
discharge into the region between the granular and cornified cells (Fitz v6, p493). This is
suggestive of a primary defect in lipid synthesis and protein dephosphorylation resulting in faulty
lamellar body formation and secretion (Bolognia, ch. 57). Harlequin fetus/ichthyosis can be
divided into three types based on the microscopic appearance of the keratohyaline granules. Type
I - normal, Type II - too small to be seen by light microscopy, Type III - absent keratohyaline
granules. Demoyokin, band-6 protein (a plaque component) and plakoglobin are desmosomal
proteins and not involved in Harlequin ichthyosis. Recently, the gene defect was localized a
defect in the ABCA12 protein.

49
Sebaceous glands secrete sebum via:
A. Holocrine mechanism
B. Exocrine mechanism
C. Endocrine mechanism
D. Exostosis
E. Mecrocrine mechanism
►A

The sebaceous lobules have basal germinative cells and central sebocytes, which gradually
become more distended with lipid vacuoles until they are shed into the lumen (holocrine
secretion).

50

24
Which signaling molecule mediates the transition of hair cycling from telogen to anagen phase?
A. Foxn1
B. Fgf5
C. Sonic hedgehog
D. Dihydotestosterone
E. 5a-reductase
►C

Sonic hedgehog (Shh), a signaling molecule secreted by ectodermal cells of the developing hair
follicle, appears to be critical in mediating the transition from telogen to anagen during postnatal
hair cycling

51
Which keratins are upregulated in hyperproliferative disease such as psoriasis?
A. Keratins 1 and 10
B. Keratins 2e and 10
C. Keratins 5 and 14
D. Keratins 6 and 16
E. Keratins 8 and 18
►D

Keratins 6 and 16 are upregulated in hyperproliferative keratinocytes, outer root sheath, and oral
epithelium. A defect in these keratins may result in pachyonychia congenita.

52
Which of the following cytokines are secreted from keratinocytes?
A. IL-2
B. IL-3
C. IL-17
D. IL-22
E. IL-23
►E

IL-2 is a cytokine that primarily produced by T lymphocytes and acts on other T lymphocytes as
a growth factor for both survival and differentiation. IL-3 is also produced by T lymphocytes and
acts as a growth factor for cells of the myeloid lineage. IL-17 is an effector cytokine that is
implicated in the pathogenesis of psoriasis and produced by Th17 cells. IL-22 is also an effector
cytokine produced by T lymphocytes that contributes to epidermal acanthosis in psoriasis. All of
these cytokines are not secreted from epidermal keratinocytes. IL-23, which promotes Th17

25
effector function, is secreted by epidermal keratinocytes and upregulated in psoriasis. The p40
subunit of IL-23 is the target of drug ustekinimab used to treat psoriasis.

53
Which of the following keratins would most likely be expressed in the nail bed?
A. K6a/16
B. K6b/17
C. K1/9
D. K2e/10
E. K4/13
►B

Keratins 6b & 17 are expressed in the nail bed. K6a/16 is expressed in the outer root sheath and
in hyperproliferative keratinocytes, 1/9 in palmoplantar suprabasalar keratinocytes, 2e/10 in the
upper spinous and granular cell layers, and 4/13 are expressed in mucosal epithelium.

54
Fibrofolliculomas are associated with:
A. Birt-Hogg-Dube Syndrome
B. Cowdens
C. Cutis Laxa
D. Marfans
E. Brook Spiegler
►A

Fibrofolliculomas are associated with Birt-Hogg-Dube Syndrome and not in the other answer
choices.

55
The major protein component of the cornified envelope is:
A. Envoplakin
B. Desmoplakin
C. Plectin
D. Loricrin
E. Transglutaminase
►D

Loricrin is the major component of the cornified envelope (CE). The proteins of the CE are
synthesized in the spinous and granular layers. The CE is primarily a protein/lipid polymer
formed within the differentiating layer of keratinocytes. The CE eventually exists outside of the

26
cornified cells after the granular cell undergoes a programmed destruction (apoptosis). Self-
destructing granular cells are called transition cells.

56
Sebaceous glands are located in each of the following locations except:
A. Nipple
B. Labia minora
C. Palms
D. Eyelids
E. Buccal mucosa
►C

Sebaceous glands secrete their contents in a holocrine fashion and are primarily under the
influence of androgens. They secrete triglycerides, phospholipids, esterified cholesterol, waxes
but not free cholesterol. They may be found in association with hair follicles or in some areas of
modified skin such as the nipple/areola, labia minora, prepuce, vermilion border, and eyelids.

57
Which part of the nose is not innervated by V2 branch of Cranial nerve V?
A. nasal columella
B. nasal ala
C. nasal tip
D. nasal dorsum
E. all sensory of the nose is innervated by V2
►C

This is innervated by the anterior ethmoidal branch of V1. The infraorbital nerve of V2
innervates the nasal ala. The nasopalantine branch of V2 innervates the columella.

58
Which of the following statements is true about eccrine glands?
A. Postganglionic sympathetic fibers with acetylcholine as the principal
neurotransmitter
B. Postganglionic sympathetic fibers with norepinephrine as the principal
neurotransmitter
C. Postganglionic parasympathetic fibers with acetylcholine as the principal
neurotransmitter
D. Postganglionic parasympathetic fibers with norepinephrine as the principal
neurotransmitter

27
E. Postganglionic sympathetic fibers with both norepinephrine and acetylcholine as
the principal neurotransmitters
►A

Eccrine glands are innervated by postganglionic sympathetic fibers with acetylcholine as the
principal neurotransmitter. This explains why medications associated with anticholinergic side
effects may be associated with hypohidrosis.

59
Itch is most commonly transmitted by:
A. C-polymodal nociceptor class nerves
B. A-delta class nerves
C. A-beta class nerves
D. Parasympathetic postganglionic fibers
E. A-beta and A-delta fibers
►A

Itch is transmitted primarily by C-polymodal nociceptor class nerves. These are small diameter
unmyelinated nerves that carry pain, thermal, mechanical and pruritic stimuli. A-delta fibers
carry pain, thermal, mechanical and in some cases pruritic stimuli. A-beta fibers carry light touch
and motion stimuli. Parasympathetic post-ganglionic fibers do not contribute to cutaneous
pruritus

60
Hair follicle development in the human embryo begins during:
A. 1st trimester
B. 2nd trimester
C. 3rd trimester
D. At the blastocoele stage
E. Within 2 weeks of fertiization
►A

Hair follicles begin in the third month of fetal life as a downgrowth of cells from the epidermis
(3rd month=12th week=1st trimester).

61
The mechanism of action of ipilimumab can be described as
A. Inhibitor of CTLA-4
B. Antibody to CD27
C. Fusion protein binding B7

28
D. Antibody to CD 8 T cell
E. Inhibitor of CD27
►A

Ipilimumab is an antibody to CTLA-4, this interaction causes inhibition of CTLA-4 which


normally inhibits T cell co-activation by binding to B7 (which normally binds to CD28).

62
Which of the following groups of adhesion proteins are found in both the hemidesomsome-
anchoring filament complexes and lamina densa?
A. Plectin
B. Heparin sulfate proteoglycan
C. Laminin 5
D. Nidogen
E. Type VII collagen
►C

Laminin 5 is found both in the hemidesomsome-anchoring filament complexes and lamina


densa. Plectin is found in the hemidesomsome-anchoring filament complexes. Heparin sulfate
proteoglycan is found only in the lamina densa, collagen VII in the sublamina densa, and
nidogen in the lamina densa.

63
What is the location of the unbound corticosteroid receptor?
A. Cytoplasm
B. Nucleus
C. Mitochondria
D. Plasma membrane
E. golgi apparatus
►A

Both androgen and corticosteroid receptors localize to the cytoplasm. Estrogen receptors are
found in the nucleus. Progesterone receptors are distributed in both the nucleus and the
cytoplasm.

64
Which of the following options is characteristic of integrins?
A. Defects in the alpha-6 domain of integrin result in epidermolysis bullosa simplex
with muscular dystrophy
B. Alpha-6-Beta-4 integrin is found at sites where desmogleins attach

29
C. Expression is seen in all layers of the epidermis
D. The extracellular alpha6 domain binds collagen 7
E. These proteins coordinate linkage between intermediate filaments and
extracellular matrix of the basement membrane
►E

Defects in the BETA-4 (not alpha-6) domain of integrin result in junctional epidermolysis
bullosa with pyloric atresia. Its expression is seen in the basal cell layer and binds to laminins.

65
What is the most abundant collagen found on fetal skin?
A. Type I Collagen
B. Type II Collagen
C. Type III Collagen
D. Type IV Collagen
E. Type VII Collagen
►C

Type III collagen is found in the fetal skin. It is also present in the gastrointestinal tract, blood
vessels, and the basement membrane. A defect in this collagen results in the Ehlers-Danlos,
vascular type.

66
Mutations in which of the following proteins results in epidermolysis bullosa simplex associated
with muscular dystrophy:
A. Uncein
B. Plectin
C. a6b4 integrin
D. Laminin 5
E. Collagen type IV
►B

EBS with muscular dystrophy is due to a plectin mutation. There are many types of EBS.
Anchoring filaments exist within the lamina lucida. They are primarily comprised of laminin 5
and BP180. Laminin 5 is a cross-shaped assembly of 3 classes of polypeptides, a, b, g. The
anchoring filaments function as a structural network to which other proteins attach, and they
function as signaling molecules that transmit morphogenetic information to transmembrane
proteins of the basal cell layer (such as the integrins). Laminin 5 is also called epiligrin and binds
to the a6b4 integrin at the hemidesmosome.

30
67
Acid keratins are expressed on which of the following chromosomes?
A. 12
B. 17
C. 9
D. 16
E. 3
►B

Acidic keratins (K9-20) are expressed from chromosome 17. Basic keratins (K1-8) are on
chromosome 12. The other listed chromosomes are not involved in keratins.

68
The most common type of pityriasis rubra pilaris in childhood is type:
A. I
B. II
C. III
D. IV
E. V
►D

Type IV, or circumscribed juvenile, accounts for 25% of total PRP cases. The most common
type is Type I, classical adult, which accounts for 55% of cases. Types II, III, and V account for
less than 10% each.

69
Which of the following is true regarding the development of hair follicles?
A. The first primordial hair follicles form at 15 weeks gestation
B. The first hair follicles form on the scalp and eyelashes
C. Follicles develop in a cephalad to caudal direction
D. New follicles develop during the first 3 months postpartum
E. The eyebrows develop late in gestation
►C

The first primordial hair follicles form at 9 weeks gestation on the eyebrows, upper lip, and chin.
The remaining follicles develop at 4-5 months in a cephalad to caudal direction. New follicles
cannot develop in adult skin.

70
All of the following are true regarding the formation of hair except:

31
A. The inner root sheath keratinizes by means of trichohyalin granules
B. Henle‘s layer is outside of Huxley‘s layer
C. The outer root sheath is a downward extension of the epidermis
D. Huxley‘s layer contains melanin
E. The hair matrix becomes the hair and the inner root sheath
►D

The inner root sheath is composed of three layers, which are the inner root sheath cuticle, Huxley
layer, Henle layer. None of these layers contain melanin. All three layer keratinize by
trichohyaline granules and disintegrate when they reach the isthmus of the hair follicle.

71
As you move upward through the epidermis toward the stratum corneum, which of the following
is true?
A. calcium increases, phospholipids decrease, sphingolipids decrease
B. calcium increases, phospholipids increase sphingolipids increase
C. calcium increases, phospholipids decrease, sphingolipids increase
D. calcium decreases, phospholipids decrease, sphingolipids decrease
E. calcium decreases, phospholipids increase sphingolipids decrease
►C

Calcium increases, phospholipids decrease, sphingolipids increase. Calcium is needed for


desmosome formation and enzyme activation. Phospholipid content of epidermis decreases with
differentiation but neutral lipids and sphingolipids (ceramide) increase with differentiation.

72
The first cell type to migrate into a new wound in great numbers is the:
A. Neutrophil
B. Monocyte
C. Macrophage
D. Lymphocyte
E. Mast cell
►A

Neutrophils migrate with monocytes concurrently, but arrive first in great numbers because of
their abundance in circulation. Chemoattractants for the PMNs are fibrinogen, fibrin split
products, C5a and leukotrienes. If wound contamination is controlled, PMN migration ceases
within a few days and they become entrapped within the wound clot, undergo apoptosis or are
phagocytosed by macrophages.

32
73
Which of the following diseases has decreased or absent lamellar granules?
A. Flegel's
B. Psoriasis
C. Lamellar ichthyosis
D. Epidermolytic hyperkeratosis
E. Pemphigus vulgaris
►A

Flegel's disease has decreased or absent lamellar granules. hyperkeratosis lenticularis perstans,
this is a rare, possibly AD disorder with multiple disc-like keratotic papules predominenty on the
distal extremities and feet of older individuals (Bolognia, p 1714).

74
Choose the correct answer regarding melanin and skin color:
A. In black and brown skin the melanosomes are smaller in diameter and length
B. Facultative skin color is the amount of cutaneous melanin pigment generated
according to cellular genetics
C. In white skin the melanosomes form groups within the secondary lysosomes
D. Eumelanin produces a yellow chromophore
E. The number of melanocytes increases with one exposure to UVA/visible light
►C

Melanocytes of dark skin synthesize melanosomes larger than those produced in light skin. The
number of melanocytes in the epidermis is the same, regardless of the person's race or color: it is
the number and size of the melanosomes or pigment granules, continuously synthesized by these
melanocytes, that determine differences in skin color. The size of th melanosome is the principle
factor in determining how the melanosomes will be distributed within the keratinocytes. The
larger the melanosomes of dark skin are individually dispersed within the cytoplasm of
keratinocytes: smaller melanosomes of light skin are packaged in membrane-bound complexes
within keratinocytes. Eumelanin is in dark oval melanosomes found in black hair

75
Epidermolysis bullosa simplex with muscular dystrophy is due to which defective molecule
A. Plectin
B. Keratin 5 and 14
C. Laminin 5
D. Collagen VII
E. Integrin alpha 6 or beta 4
►A

33
EBS with muscular dystrophy is due to a mutation in plectin, plectin is also found in skeletal
muscle. Keratin 5 and 14 are defective in EBS, EBS herpetiformis, EBS weber cockayne, EBS
koebner. Laminin 5 is defective in Herlitz Junctional EB. Collagen VII is mutated in dominant
dystrophic EB, Barts syndrome and EB acquisita. Integrin alpha 6/beta 4 is mutated in junctional
epidermolysis bullosa with pyloric atresia.

76
Which hair condition is associated with abnormal regulation of the peroxisome proliferator-
activated receptor-Ɣ (PPARƔ):
A. Alopecia areata
B. lichen planopilaris
C. telogen effluvium
D. seborrheic dermatitis
E. androgenic alopecia
►B

Lichen planopilaris is a chronic, scarring condition that results in permanent loss of hair and
destruction of the hair follicles. Defect in lipid metabolism and peroxisome biogenesis may
contribute to LPP pathogenesis. Medications that act as PPAR agonists, such as pioglitazone,
may have a role in controlling the progression of this disease. (Karnik P, et al. (2009) Hair
follicle stem cell-specific PPAR deletion causes scarring alopecia. J Invest Dermatol
129(5):1243-57)

77
Which of the following statements about Laminins is correct?
A. Laminins span from the plasma membrane of basal keratinocytes to the lamina
lucida
B. Laminins provide little structural support in the basement membrane
C. Laminins provide signaling molecules that interact with other proteins to
transmit morphogenetic information to the cell's interior
D. Laminin 5 is also called plectin
E. Laminin 5 is the only laminin found in the basement membrane
►C

Laminins do transmit signals to the cell's interior. They span from the plasma membrane of basal
keratinocytes to the lamina DENSA, not lucida. Laminin 5 (also known as epiligrin) is part of a
structural network in the basement membrane that also includes laminin 6.

78

34
Anchoring fibrils are primarily composed of:
A. Type I collagen
B. Type III collagen
C. Type IV collagen
D. Type VII collagen
E. Type II collagen
►D

Anchoring fibrils are found in the sublamina densa and are made up of collagen type VII. This
collagen type is mutated in dystrophic epidermolysis bullosa, and targeted in epidermolysis
bullosa acquisita and bullous lupus erythematosus.

79
All of the following are consider intermediate filament except:
A. Microtubules
B. Keratins
C. Vimentin
D. Desmin
E. Peripherin
►A

Intermediate filaments are composed of keratins, vimentin, desmin, peripherin, neurofilaments,


nuclear laminins, and nestin. These are part of cytoskeletal elements.

80
Hair grows at:
A. 0.004 mm/day
B. 0.04 mm/day
C. 0.4 mm/day
D. 4 mm/day
E. 10 mm/day
►C

Hair grows at an average of 0.4 mm/day. The other options are incorrect.

81
Regarding dermal-epidermal junction, which of the following statements is true:
A. There are no anchoring filaments in lamina lucida
B. Lamina fibroreticularis lies above lamina densa

35
C. Lamina fibroreticularis comprises of anchoring fibrils and the elastic
microfibrils
D. Blood vessels cross the dermal-epidermal junction to reach the epidermis
E. Lamina lucida is an electron-dense layer
►C

Lamina fibroreticularis comprises of anchoring fibrils and the elastic microfibrils. Lamina
fibroreticularis lies below lamina densa. Blood vessels don't cross the dermal-epidermal junction
to reach the epidermis. Basement membrane contain both: 1)Lamina basale which include
a.lamina lucida and b.lamina densa( which contain collagene IV & other glycoproteins like
laminins,fibronectin etc.) 2)Lamina fibroreticularis contain collagen type III and anchoring
fibrils (collagen type VII)

82
Which of the following is not a specialized type of sebaceous gland?
A. Moll's gland
B. Meibomian gland
C. Gland of Zeis
D. Montgomery's tubercle
E. Fordyce spot
►A

There are several types of specialized sebaceous glands that are not associated with a hair
follicle. They include Montgomery's areolar tubercle, Fordyce spots of the lip, Glands of Zeis of
the cutaneous eyelid, and Meibomian glands of the eyelid. Moll's gland of the eyelid are a
modified apocrine gland.

83
Desmosine and isodesmosine are typical amino acids found in:
A. Collagen fibers
B. Anchoring fibril
C. Elastic fibers
D. Heparan sulfate
E. Anchoring plaques
►C

Desmosine and isodesmosine are typical amino acids found in elastic fibers. They crosslink
fibrillin. Anchoring fibrils are composed of collagen VII and collagen fibers and have the most
typical amino acids of proline and hydroxyproline. Heparan sulfate do not typically contain these
amino acids.

36
84
When do melanocytes begin to synthesize melanin?
A. 2nd month of gestation
B. 3rd month of gestation
C. 4th month of gestation
D. 5th month of gestation
E. 6th month of gestation
►B

Melanocytes begin to synthesize melanin in the 3rd month of gestation.

85
Elastic fibers contain the specific amino acids:
A. Lysine and proline
B. Leucine and isoleucine
C. Alanine and phenylalanine
D. Desmosine and isodesmosine
E. Glycine and proline
►D

Desmosine and isodesmosine are the typical amino acids of elastic fibers. Elastic fibers are
comprised of elastin that is wrapped by fibrillin microfibrils. Elastic fibers form a complex
meshwork extending from the lamina densa of the dermo-epidermo junction through the dermis.
Elastic fibers return the skin to a normal shape after being stretched.

86
Acid keratins (K10-20) are expressed on which of the following chromosomes?
A. 17
B. 18
C. 19
D. 10
E. 12
►A

Acid keratins are expressed on chromosome 17 and basic keratins on chromosome 12. The other
listed options do not have active keratin functions.

87

37
Which of the following statements about darkly pigmented races versus lighter pigmented races
is correct?
A. The number of melanosomes in melanocytes are the same
B. The individual melanosomes have the same degree of melanization
C. The melanosomes are equal in size melanosomes
D. There are equal numbers of melanocytes
E. There is a faster rate of melanosome degradation
►D

Individuals with darker pigmentation have an equal ratio of melanoyctes to keratinocytes. Darker
pigmentation is related to an increased number of melanosomes, increased melanization, greater
size of melanosomes, and slower degradation.

88
The first primordial hair follicles form on the eyebrows, upper lip and chin at which gestational
age?
A. 7 weeks
B. 9 weeks
C. 12 weeks
D. 16 weeks
E. 20 weeks
►B

The first primordial hair follicles form at 9 weeks gestation on the eyebrows, upper lip and chin.
The remaining follicles develop at 4-5 months in a cephalad to caudal direction. New follicles
cannot develop in adult skin.

89
A patient presents with painless firm nodules. Pathology reveals no epidermal involvement. The
patient is ultimately diagnosed with sarcoid. What is this type of involvement called?
A. Darier-Roussy disease
B. Lofgren′s syndrome
C. Heerfordt′s syndrome
D. Lupus Pernio
E. Kveim-Siltzbach disease
►A

Darier Roussy disease refers to subcutaneous sarcoid. Lofgren′s syndrome is sarcoid presenting
as erythema nodosum, hilar lymphadenopathy, fever, migrating polyarthritis, and acute iritis.

38
Heerfordt′s is parotid gland enlargement, uveitis, fever and cranial nerve palsies. Kveim-
Siltzbach antigen is used rarely in the US but can aid in the diagnosis of sarcoid.

90
Keratin filaments in basal cells insert into:
A. Desmosomes
B. Adherens junctions
C. Connexins
D. Lamellar granules
E. Odland bodies
►A

Keratin filaments insert into desmosomes and hemidesmosomes in the basal cell layer and into
desmosomes in the layers above. Connexins do not bind keratin filaments. Odland bodies and
lamellar granules are synonyms. These are a "membrane bounded organelle, specialized for the
storage and secretion various substances (surfactant phospholipids, glycoproteins and acid
phosphates) which are arranged in the form of tightly packed, concentric, membrane sheets or
lamellae. Has some similar properties to, but is distinct from, a lysosome."

91
The embryonic periderm becomes part of the
A. vernix caseosa
B. stratum corneum
C. stratum basale
D. dermis
E. hair follicle
►A

During the second trimester of fetal development, the periderm is sloughed from most of the skin
surface revealing the underlaying epidermis. The periderm becomes part of the protective
coating, vernix caseosa, together with the shed lanugo, sebum and other amniotic fluid materials.
Bolognia p.38

92
At what estimated gestational age are all layers of the keratinized epidermis identifiable?
A. 8 weeks
B. 12 weeks
C. 16 weeks
D. 20 weeks
E. 24 weeks

39
►E

At 24 weeks, all the layers of the mature epidermis can be identified, and the epidermis is
keratinized.

93
Regarding sebaceous glands:
A. These glands are present at birth at their adult size
B. Size of the gland is proportional to the size of the associated hair follicle
C. Are always associated with a hair follicle
D. Are found everywhere on the skin except palms and soles
E. Are unilobular glands
►D

Sebaceous glands are found everywhere on the skin except on the palms and soles. They are
multilobular, emptying into the sebaceous duct. Most sebaceous glands are associated with hair
follicles, but free glands exist, especially on the lip (Fordyce spots), on the nipple/areola
(Montgomery′s tubercles), Meibomian glands of the eyelids and Tyson′s glands on the genitalia.
The size of the sebaceous glands are not related to the size of the associated follicle. The
sebaceous gland enlarges at puberty in response to increased androgens.

94
Surgery, Parturition, Fever, Kwashiokor and Hypervitaminosis A are all causes of:
A. Telogen effluvium
B. Anagen effluvium
C. Alopecia areata
D. Androgenetic alopecia
E. Both telogen and anagen effluvium
►A

All of the listed stressors can induce telogen effluvium. Other causes include traction and some
drug exposures. Usually will only involve up to 50% of scalp hairs and will resolve within 2-3
months.

95
Glomus cells are primarily found:
A. on hands/feet
B. on the trunk
C. on the lateral thighs
D. on the genital skin

40
E. on the face
►A

Glomus cells are derived from Susquet-Hoyer canals which function to shunt blood from the
arterioles to venules and are primarily found on hands and feet. There are two types of glomus
tumors, solitary and multiple. The solitary types tend to have paroxysmal pain which can be
extreme. Multiple glomus tumors can be a autosomally dominant trait with incomplete
penetrance and are less likely to be painful. Both have a predominance on the distal extremities,
especially sub-ungual. Two tests are helpful in diagnosing glomus tumors: Hildreth sign -
disappearance of pain following application of a tourniquet proximally. Love test - eliciting pain
by applying pressure to a precise area with the tip of a pencil.

96
Which of the following amino acids are typically found in elastic fibers?
A. Desmosine
B. Isoleucine
C. Proline
D. Hydroxyproline
E. Glycine
►A

Desmosine and isodesmosine are typical amino acids found in elastic fibers. Glycine, proline and
hydroxyproline are found as components of collagen. Isoleucine is not specific for elastin fibers.

97
Melanocytes are derived from:
A. Bone marrow
B. Neural crest
C. Mesodermal precursors
D. Endodermal precursors
E. Yolk sac derived
►B

Melanocytes are derived from neural crest precursors and migrate to the epidermis, hair matrix,
retinal pigment epithelium, ear (stria vascularis), leptomeninges, and mucous membranes. The
other options listed are incorrect and are not involved with melanocytes.

98
Keratinocytes in the basal layer of the epidermis attach to the basement membrane zone at
hemidesmosomes via what intermediate filament molecules?

41
A. Keratins 1 and 10
B. Keratins 5 and 14
C. Desmogleins 1 and 3
D. Desmoplakin and Desmoglobin
E. BPAG1 and BPAG2
►B

Keratins 5 and 14 are intermediate filaments found in the basal layer of the epidermis that play a
role in attaching basal layer keratinocytes to the basement membrane zone. Keratins 1 and 10 are
found in more superficial layers of the epidermis. Desmogleins, desmoplakin, and desmoglobin
are all components of desmosomes which attach keratinocytes to other surrounding
keratinocytes. BPAG1 and BPAG2 are basement membrane proteins.

99
The proteins that make up the cornified cell envelope are synthesized in the:
A. Stratum corneum
B. Stratum basale
C. Melanocytes
D. Stratum granulosum
E. Langerhans cells
►D

The proteins that make up the cornified cell envelope are synthesized in the spinous and granular
cell layer. The stratum basale is not involved in production of these proteins and there is no
protein synthesis in the stratum corneum. Melanocytes and Langerhans cells are components of
the epidermis but are not involved in this process.

100
Which of the following make up the major protein of the cornified cell envelope?
A. Loricrin
B. Involucrin
C. Envoplakin
D. Filaggrin
E. Laminin V
►A

Loricrin is the major protein component of the cornified cell envelope (CE). Involucrin is cross-
linked by transglutaminase in the granular layer to form an insoluble cell boundary. Envoplakin
may link the CE to desmosomes and to keratin filaments. Filaggrin is thought to promote
aggregation and disulfide bonding of keratin filaments in CE. It is degraded into urocanic acid

42
and pyrrolidone carboxylic acid. Both of which hydrate the stratum corneum and block UV
radiation. Laminin V is found in the basement membrane and is not involved in the formation of
the cornified cell envelope.

101
Direct immunofluorescence is of no value in the diagnosis of:
A. Neonatal LE
B. Lichen planus
C. Mixed connective tissue disease
D. SLE
E. Erythema multiforme
►A

DIF is of no value in the diagnosis of scleroderma, morphea, and neonatal LE. DIF of lichen
planus is positive in the vast majority, with granular DEJ deposition and IgM and fibrinogen
staining within cytoid bodies in the superficial dermis. In MCTD DIF shows IgG deposits within
epidermal cell nuclei, and rarely along the DEJ. In SLE, while serology is more reliable, DIF can
show DEJ deposition in the lupus band test. DIF of erythema multiforme shows immunoglobulin
within superficial vessel walls, DEJ, and cytoid bodies.

102
Elastic fibers are present in the dermis and are responsible for providing tissue resiliency. They
are comprised of elastin as well as microfibrillar proteins including:
A. Fibrillins and fibulins
B. Hyaluronic acid
C. Collagen II
D. Laminin 5 and BPAG2
E. Desmoplakin and plakoglobin
►A

Elastic fibers are comprised of elastin as well as microfibrillar proteins including fibrillins and
fibulins. Hyaluronic acid is a glycosaminoglycan found in the extracellular matrix of the dermis.
Collagen II is the predominant collagen found in cartilage. Laminin 5 and BPAG2 are important
anchoring filaments in the basement membrane zone. Desmoplakin and plakoglobulin are
proteins found in the anchoring plaques of desmosomes in the epidermis.

103
During a salt split skin test, if the location of the deposition if found on the roof (lamina lucida),
which of the following could be a diagnosis?
A. Bullous pemphigoid

43
B. Anti-epiligrin cicatricial pemphigoid
C. Epidermolysis bullosa acquisita
D. Bullous eruption of lupus erythematosus
E. Bullous eruption of tinea
►A

Bullous pemphigoid is one of the many bullous diseases that may be found on the ′roof′ of the
blister, the others include pemphigoid gestationis, linear IgA bullous dermatosis, cicatricial
pemphigoid. The other answer (except for bullous tinea, in which a salt split would not be
performed) are those found on the ′′floor′ of the blister (lamina densa).

104
Which cytokeratin would you expect to be preferentially expressed in the keratinocytes of a
psoriatic plaque:
A. K1
B. K5
C. K12
D. K15
E. K16
►E

K16 and K6 act as markers for hyperproliferative keratinocytes. They are found in skin disease
such as psoriasis, warts, actinic keratoses, and SCC's.

105
Glomus cells are composed of what?
A. Modified fibroblasts
B. Modified skeletal muscle cells
C. Vascular smooth muscle cells
D. Monocytes
E. Neuronal cells
►C

Glomus cells are derived from Suquet-Hoyer canals. They are vascular smooth muscle cells that
allow the rapid shunting of blood from the arterioral to venular system, bypassing the capillaries.
This process occurs primarily on the palms and soles.

106
Which of the following statements about plectin is correct?
A. Links fillagrin to the plasma membrane

44
B. Crosslinks proteins in adherens junction
C. Mutations result in junctional epidermolysis bullosa with pyloric atresia
D. Plectin is a member of the plakin family
E. Plectin is a constituent protein of the desmosomal plaque
►D

Plectin is a hemidesmosomal protein and is not present in the desmosome. It links intermediate
filaments to the plasma membrane, crosslinks hemidesmosomal proteins, mutations result in
epidermolysis bullosa simplex with muscular dystrophy and is a member of the plakin family.

107
The target for GABEB and bullous pemphigoid is:
A. Type XVII collagen
B. Type VII collagen
C. Laminin 5
D. Integrin subunit β4
E. Type XII collagen
►A

The target for GABEB and bullous pemphigoid is type XVII collagen or BPAG2. It is also the
target protein in pemphigoid gestationis, CP and linear IgA.

108
The formation of granulation tissue depends on the presence of:
A. Neutrophils
B. Fibronectin
C. Collagen type I
D. Platelets
E. Collagen type IV
►B

Granulation tissue forms approximately four days after injury. It is composed of new capillaries,
macrophages, fibroblasts, and blood vessels. The formation is dependent on the presence of
fibronectin. There tends to be an ordered sequence of matrix deposition. Fibronectin is deposited
first followed by collagen 3 and then collagen 1. Granulation tissue primarily contains type 3
collagen.

109
The cutaneous immunofluorescence pattern in patients with Senear-Usher syndrome is:
A. Intercellular IgG and C3

45
B. Linear IgG and C3 along the basement membrane zone
C. Intercellular IgG on guinea pig esophagus
D. Intercellular IgG and C3 and linear IgG and C3 along the basement membrane
zone
E. Linear IgG and C3 along the basement membrane zone and intercellular IgG on
guinea pig esophagus
►D

Senear-Usher syndrome, or pemphigus erythematosus, is a variant of pemphigus foliaceus


characterized by crusted papules and plaques with a seborrheic distribution (malar region, scalp,
upper chest and back). They demonstrate in-vivo IgG and C3 deposition on keratinocyte cell
membranes and the basement membrane zone, in addition to circulating anti-nuclear antibodies.

110
Which element is necessary for function of matrix metalloproteinases?
A. Iron
B. Nitrogen
C. Manganese
D. Magnesium
E. Zinc
►E

Matrix metalloproteinases are required for normal tissue architecture and normal turnover of the
extracellular matrix. All of them have zinc at the active site and require octahedral binding of
calcium ions to maintain structural integrity.

111
Eulanin fibers:
A. Run parallel in bands within the superficial papillary dermis
B. Run perpendicular from the dermo-epidermal junction within the superficial
papillary dermis
C. Run parallel in bands within the reticular dermis
D. Run perpendicular in bands within the deep dermis
E. Run perpendicular in bands within the reticular dermis
►C

Eulanin fibers are elastic fibers that have less elastin and more fibrillin and run parallel in thin
bands within the reticular dermis. Oxytalin fibers contain no elastin and run perpendicular from
the dermo-epidermal junction within the superficial papillary dermis. Elastic fibers turn over
slowly in the skin, and are damaged by ultraviolet radiation.

46
112
Anagen effluvium is
A. Rarely seen following administration of cancer chemotherapeutic agents
B. Stimulus induces the abrupt cessation of mitotic activity in the rapidly dividing
hair matrix cells
C. Occurs within 24 to 48 hours of exposure
D. Entirely irreversible
E. Patients being treated with nitrosurea agents are usually spared
►B

It is frequently seen following administration of cancer chemotherapeutic agents. Stimulus


induces the abrupt cessation of mitotic activity in the rapidly dividing hair matrix cells. Occurs
within days to weeks following exposure to the inciting agent and is entirely reversible. Frequent
causes include antimetabolites, alkylating agents, miotic inhibitors, thallium, boron. Frequent
causes include doxorubicin, nitrosureas and cyclophosphamide.

113
Cutaneous warts in renal transplant recipients show increased expression of which keratin?
A. K2
B. K5
C. K9
D. K13
E. K16
►D

Keratins are divided into acidic (type I) and basic (type II) subtypes. Keratin 13 is a type I
keratin which is expressed suprabasally in adult epithelia and is associated with terminal
differentiation. Renal transplant recipients have an increased risk of developing skin cancers and
multiple warts. Factors contributing to this increased risk include human papillomavirus
infection, immunosuppressive therapy, and exposure to ultraviolet radiation. Increased
expression of K13 has been demonstrated in high-risk papillomas in this patient population.

114
The main permeability barrier in the lamina densa is:
A. heparan sulfate proteoglycan
B. collagen IV
C. laminin 5
D. nidogen
E. alpha-6-beta-4 integrin

47
►A

All of the listed proteins are present in the lamina densa except alpha-6-beta-4 integrin, which
connects the hemidesmosome to laminin 5 in the lamina lucida. The heparin sulfate
proteoglycans (perlecan) are negatively charged, thus serve as a permeability barrier. The other
listed proteins do not serve this function in the lamina densa. Collagen IV is the main basic
basement membrane scaffold. Defects in Collagen IV have been linked to Alport′s and
Goodpasture′s syndrome. Nidogen has a ′dumbbell′ shape and binds both laminins and collagen
IV in the lamina densa.

115
How soon does epithelialization begin after a skin wound occurs?
A. Minutes
B. Hours
C. 2 days
D. 4 days
E. 6 days
►B

Re-epithelialization begins hours after an injury occurs. Keratinocytes from residual epithelial
structures leapfrog each other. One to two days after injury, cells at the wound margin proliferate
and begin to migrate into the wound.

116
The following is a target in both junctional EB with pyloric atresia and ocular CP:
A. Integrin subunit β4
B. Laminin 5
C. Type VII Collagen
D. Type XVII collagen
E. BPAG2
►A

Integrin subunit β4 is the target for both junctional EB with pyloric atresia and ocular CP.

117
Which of the following cytokines is secreted by adipocytes?
A. IL-2
B. Il-4
C. IL-6
D. IL-10

48
E. IL-12
►C

Adipocytes secrete tumor necrosis factor (TNF)ɑ ±, interleukin-6 (IL-6), leptin, adiponectin,
angiotensinogen, and resistin. Specifically it has been shown that IL-6 mRNA levels in human
subjects are significantly greater in adipose tissue than in other tissues known to express IL-6. It
is thought that circulating IL-6 levels may be higher in obese patients, contributing to the
development and progression of vascular disease in this patient population.

118
Embryologically, epidermal stratification occurs at approximately what estimated gestational
age?
A. 4 weeks
B. 8 weeks
C. 12 weeks
D. 16 weeks
E. 20 weeks
►B

Epidermal stratification begins at about 8 weeks estimated gestational age and is completed by
the 2nd trimester. Epideral stratification occurs when an 'intermediate layer' develops between
the epidermal basal cells and the overlying layer of periderm cells. The periderm is an embryonic
structure that covers the primitive epidermis until keratinization begins, at which point the
periderm sloughs off and contributes to the protective covering of the newborn, the vernix
caseosa. This intermediate layer is highly proliferative, such that by 24-25 weeks estimated
gestaional age, the epidermis consists of 4 or 5 layers, in addition to the degenerating periderm.

119
Type 1 collagen is the most prevalent collagen in skin, accounting for 80% or more of the total
collagen in the adult dermis. The next most predominant collagen in adult human dermis is:
A. Type II collagen
B. Type III collagen
C. Type IV collagen
D. Type VII collagen
E. Type XVII collagen
►B

Type 1 collagen is the most prevalent collagen in skin, accounting for 80% or more of the total
collagen in the adult dermis. Type III collagen is the next most predominant collagen in human
dermis, accounting for approximately 10%. Type IV collagen is found in basement membranes.

49
Type VII collagen is found in human dermis as anchoring fibrils. Type XVII collagen, also
known as BPAG2, is an important transmembrane protein in the basement membrane zone.

120
Which of the following domains is targeted by the autoantibodies in bullous pemphigoid?
A. NC16A of BP180
B. Laminin 5
C. Plectin
D. alpha-6 integrin
E. NC16A of BP230
►A

The NC16A domain of BP180 is the target of autoantibodies in bullous pemphigoid. It interacts
with alpha-6 integrin extracellularly.

121
What component is the major barrier in the stratum corneum?
A. Sebum
B. Squalene
C. Collagen
D. Ceramide
E. Triglycerides
►D

The major component of lamellar granules of the keratinocytes is ceramide. These play a major
role in the barrier function of the skin.

122
Which of the following skin conditions involves elastin?
A. Rothmund-Thompson
B. Epidermolytic Hyperkeratosis
C. Pseudoxanthoma elasticum
D. Progeria
E. Classic type Ehlers-Danlos syndrome
►C

There are many genodermatoses that involve elastin. Pseudoxanthoma elasticum shows
increased glycosaminoglycans on elastic fibers, calcium deposition and accumulation of
fragmented and calcified elastic fibers. Cutis laxa shows decreased desmosine and lysyl oxidase.
In Marfan sydrome there is decreased fibrillin I and fragmentation of elastic fibers. Buschke-

50
Ollendorf syndroms shows increased desmosine and increased amount of thickend elastic fibers.
Other elastin disease are congenital contractural arachnodactyly from a mutation in fibrillin II
and anetoderma which shows decreased desmosine and loss/fragmentation of elastic fibers.
Classic type Ehlers-Danlos syndrome is caused by a defect in collagen V and not in elastin.
Progeria is caused by a defect in lamin A. EHK is caused by a defect in keratin 1/10. Rothmund
Thompson is caused by a defect in RecQL4, a helicase gene.

123
Keratohyalin granules contain:
A. Desmoplakin
B. Envoplakin and Keratin 6
C. Profilaggrin and loricrin
D. Numerous Golgi apparati
E. Involucrin
►C

Keratohyalin granules are found in the stratum granulosum (the granular layer), and contain the
proteins profilaggrin and loricrin. Profilaggrin is converted to filaggrin during the transformation
of the granular layer to the cornified layer. This is a calcium-dependent process. Loricrin
comprises 75% of the cornified envelope′s mass.

124
Nikolsky sign can be seen when:
A. When an intact epidermis shears away from the underlying dermis, leaving a
moist surface
B. Spreading bulla phenomenon with pressure on an intact bulla
C. Swollen, itchy and or red after stroking the skin
D. Central depression within a lesion when squeezed along its margins
E. Disappearance of color when the lesion is pressed
►A

Nikolsky sign can be seen when an intact epidermis shears away from the underlying dermis,
leaving a moist surface (seen in pemphigus vulgaris, staphylococcus scalded skin syndrome
(SSSS), and toxic epidermal necrosis). Spreading bulla phenomenon with pressure on an intact
bulla is referred to as Asboe-Hansen sign, commonly seen with pemphigus vulgaris. Swollen,
itchy and or red after stroking the skin is referred to Darier′s sign and can be seen in systemic
mastocytosis or urticaria pigmentosa. Central depression within a lesion when squeezed is
referred to as the dimpling sign and is seen in dermatofibromas. Disappearance of color or
blanching when the lesion is pressed is commonly found on vascular lesions

51
125
Odland bodies:
A. Contain keratins
B. Are found intercellularly in the basal cell layer
C. Are exclusively intracellular
D. Crosslink with keratins 5 and 14
E. Are secretory granules with features of lysosomes
►E

Odland bodies (lamellar granules) are lamellated bodies containing ceramide, are found
intercellularly in upper level keratinocytes; they discharge contents into the extracellular space at
the junction of the granular and horny layers, establish a barrier to water loss, and with filaggrin
mediate stratum corneum adhesion. Recent studies suggest that they are a type of secretory
granule with features of lysosomes. (They contain lamellar-body-derived enzymes that are
important in desquamation.)

126
The strength of a scar:
A. Is 5% at 1 week
B. Is 20% at 3 weeks
C. Is 70% at 1 year
D. All of these options are correct
E. None of these options are correct
►D

A scar has 5% strength at 1 week, 20% at 3 weeks and 70% at 1 year. It will never recover
strength to the level of pre-injury.

127
A 78 year-old man is diagnosed with cicatricial pemphigoid. Which of the following antigens
being implicated in his disease should trigger a work-up for malignancy?
A. BPAg2
B. Laminin 5
C. Laminin 6
D. Beta-4 integrin
E. Type VII collagen
►B

Involvement of Laminin 5 would raise the suspicion for malignancy when found. ~30-40% of
cases may have associated malignancy. Patients with antiepiligrin (laminin 5) CP indicated an

52
increased risk of malignancy that approximates that for adults with dermatomyositis. The risk is
particularly high in the first year of disease. Reports of both lung and gastric carcinomas are seen
in the literature with some feeling that gastric malignancy is most common.

128
During hair follicle development, the WNT signaling pathway is one of the earliest molecular
pathways involved in hair follicle initiation. What is the downstream mediator of WNT
signaling?
A. Smoothened
B. Beta-catenin
C. Keratin 16
D. p53
E. HLA-B27
►B

Beta-catenin is the downstream mediator of WNT signaling. Through a series of signals, WNT
proteins inhibit the degradation of beta-catenin in the cytoplasm. After being translocated to the
nucleus, beta-catenin then forms a complex with LEF/TCF transcription factors, which
ultimately results in expression of downstream genes. Activation of this pathway is necessary for
epithelium to have the potential to develop a hair follicle.

129
In the epidermis, the cell most responsible for antigen detection and processing is the:
A. Keratinocytes
B. Merkel Cell
C. Melanocyte
D. Langerhans cell
E. CD4+ T cell
►D

The Langerhans cell is a bone narrow-derived, antigen-presenting cell found in all layers of the
epidermis, oral mucosa, esophagus, and vagina. Langerhans cells ingest and process antigens,
mature, migrate to a local lymph node, and then present the antigen to a na′ve (or resting) T cell,
activating that T cell. The Langerhans cell is central to the pathogenesis of atopic dermatitis,
psoriasis, allergic contact dermatitis, and certain infections, such as Leishmaniasis.

130
Krause end bulbs are:
A. Adapting mechanoreceptors found on weight-bearing sites that respond to
vibrational stimul

53
B. Found on the vermillion border of the lips
C. Located in the dermal papillae of digital skin
D. Located in the deep dermis and within the subcutis in weight-bearing sites of the
body
E. Found at the orifice of the hair follicle and particularly sensitive to cold
►B

Krause end bulbs are mucocutaneous receptors found on the glans penis, clitoris, labia minora,
perianal area, and vermillion border of the lips. Papillary nerve endings are free nerve endings
found at the orifice of the hair follicle and particularly sensitive to cold. Meissner's corpuscles
are ovoid, elongated mechanoreceptors located in the dermal papillae of digital skin that detect
touch and light pressure. Pacinian (Vater-Pacini) corpuscles exist in the deep dermis and within
the subcutis in weight-bearing sites and function as adapting mechanoreceptors that respond to
vibrational stimuli.

131
Which cell type is required for wound healing?
A. Macrophage
B. Lymphocyte
C. Mast cell
D. Dermal dendrocyte
E. Neutrophil
►A

The macrophage is required for wound healing. Without macrophages, there is no healing.
Macrophages debride tissue through phagocytosis and digestion of organisms, tissue debris and
effete PMN's. They secrete collagenase and secrete growth factors that facilitate transition from
inflammation to repair.

132
Which component of hair is positive for citrulline?
A. Outer root sheath
B. Inner root sheath
C. Cortex
D. Glassy vitreous layer
E. Medulla
►B

54
The inner root sheath stains red because it contains citrulline. The cortex and medulla along with
the cuticle make up the hair shaft. The outer root sheath is the most peripheral cellular structure.
The glassy vitreous layer is the basement zone equivalent of hair and is the outermost layer.

133
What percentage of the dry weight of skin in elastin?
A. 2
B. 4
C. 6
D. 8
E. 10
►B

Elastin fibers make up 4% of dry weight of the skin, forming a complex meshwork extending
from lamina densa of the dermoepidermal junction through the dermis and into the hypodermis.
Elastins help return the skin to the normal configuration after being stretched. Elastic fibers are
90% elastin wrapped by fibrillin microfibrils. Fibrillin is mutated in Marfan syndrome. The
typical amino acids found in elastic fibers are desmosine and isodesmosine. Elastic fibers turn
over slowly in the skin and are damaged by ultraviolet radiation.

134
On electron microscopy, which cell demonstrates cytoplasmic projections and secretory
granules?
A. Langerhans cell
B. Keratinocyte
C. Mast cell
D. Melanocyte
E. Macrophage
►C

Electron microscopy of mast cells demonstrates large long villi at the periphery and round or
oval secretory granules. Langerhans cells show a folded nucleus and rarely phagocytized
melanosomes on electron microscopy. Their characteristic feature is the presence of Birbeck
granules whose disk shape with one or two vesicles at either end represents a tennis raquet.
Examination of melanocytes shows an absence of tonofilaments or desmosomes with
characteristic melanosomes in various stages of formation. Macrophages often contain
phagocytized material within phagosomes.

135

55
Merkel cells are mechanoreceptors found in areas of high-tactile sensitivity. This
immunohistochemical marker is restricted to Merkel cells in the skin and is thus a reliable
marker for these cells:
A. Keratin 7
B. Keratin 20
C. S-100
D. Factor XIIIa
E. LYVE-1
►B

Keratin 20 is reliable immunohistochemical markers for Merkel cells as it is restricted to these


cells in the skin. Keratin 7 can be used as a marker for Paget's Disease. S-100 is frequently used
to stain neural cells and melanocytes. Factor XIIIa can be used to differentiate a dermatofibroma
from dermatofibroma sarcoma pertuberans (positive in DF; negative in DFSP). LYVE-1 is a
marker for lymphatics.

136
In epidermolysis bullosa simplex, where on the blister does the signal localize on a salt split skin
test?
A. Roof
B. Floor
C. Middle
D. Diffuse
E. No localization
►A

In dystrophic EB the signal localizes to the roof of the blister. In junctional EB, the roof has
BPAG2 while the floor has type IV collagen.

137
Which type of collagen in mutated in osteogenesis imperfecta?
A. Collagen I
B. Collagen II
C. Collagen III
D. Collagen IV
E. Collagen VII
►A

Collagen type I is mutated in osteogenesis imperfecta. The COL1A2 gene is located on 7q22 and
the COL1A1 is on 17q22. The other listed collagens are not involved in osteogenesis imperfecta.

56
Collagen II is in cartilage and vitreous, Collagen III is in fetal skin, blood vessels and intestines,
Collagen IV is in basement membranes and Collagen VII in anchoring fibrils and amnion.

138
A 25-year-old man presents with a new diagnosis of plaque-type psoriasis. keratotic follicular
papules involving the bilateral dorsal arms, thighs, and cheeks. A biopsy of an unaffected region
of skin would likely exhibit which of the following findings:
A. Increased filaggrin
B. Increased loricrin
C. Increased involucrin
D. Diminished lamellar bodies
E. Decreased Transglutaminase I activity
►C

In psoriasis, involucrin levels are increased. In contrast, filaggrin and loricrin levels are
diminished. Decreased lamellar bodies are implicated in Flegel‘s disease and Harlequin
icthyosis. Tissue transglutaminase I is faulty in lamellar icthyosis and non-bullous congenital
icthyosiform erythroderma.

139
Which of the following dermal cells always express CD11c and CD6?
A. Mononuclear phagocytic cells
B. Fibroblasts
C. Mast cells
D. Erythrocytes
E. Glomus cells
►A

The mononuclear phagocytic cell includes monocytes, macrophages and dermal dendrocytes. All
phagocytic skin macrophages express CD11c and CD6. Fibroblasts do not produce a CD marker.
Mast cells are derived from bone marrow-residing CD34+ stem cells. They do not produce
CD11c or CD6.

140
Which of the following statements about elastic fibers is true?
A. Elastic fibers form 35% of the dry weight of the skin
B. Elastic fibers are 90% elastin wrapped in fibrillin
C. Collagen 1 is mutated in Marfan syndrome
D. Oxytalan fibers run parallel within the superficial papillary dermis
E. Elaunin fibers run perpendicular in thin bands within the reticular dermis

57
►B

Elastic fibers are responsible for much of the elasticity of the dermis. They are essentially 90%
elastin wrapped in fibrillin. They form 4% of the dry weight of the skin. Fibrillin 1 is mutated in
Marfan syndrome. Oxytalan fibers run PERPENDICULAR from the DEJ within the superficial
papillary dermis. Elaunin fibers run parallell in thin bands within the reticular dermis.

141
What is the average duration of the telogen cycle in terminal scalp hair?
A. 2-6 years
B. 2-3 weeks
C. 3 months
D. 6 months
E. 9 months
►C

The average duration of the telogen phase of the hair cycle is 3 months; this feature explains why
telogen effluvium is typically observed 3 months following a traumatic event or serious illness.
The average duration of the anagen phase of the hair cycle is 2-6 years, whereas that of the
catagen cycle is 2-3 weeks.

142
Upon presentation of an antigen in the skin surface, a hapten forms. The first cell to take up the
hapten is:
A. B cells
B. Langerhans cells
C. keratinocytes
D. T cells
E. Mast cell
►B

Most of the contact allergens are low-molecular weight chemicals, which after penetrating into
the skin, have to couple with host proteins to be able to act as full antigens. These are called
haptens. Upon epicutaneous application to a naive host, Langerhans cells take up the hapten,
process it and migrate towards the regional lymph nodes, where the anitgen is presented to the
naive T cells.

143
Apocrine chromhidrosis results from which of the following contents of apocrine sweat?
A. Lipofuschin

58
B. Squalene
C. Cholesterol
D. Fatty acids
E. Ammonia
►A

Chromhidrosis refers to the secretion of pigmented sweat, most commonly yellow, green or
black. It reflects the rich lipofuschin content of apocrine sweat. Extrinsic apocrine chromhidrosis
results from staining of sweat and garments by chromogenic bacteria, such as Corynebacterium
spp.

144
Which of the following protein plays a major role in wound healing?
A. Uncein
B. Fibronectin
C. Nidogen
D. Entactin
E. Band-6 protein
►B

Fibronectin is a key player in wound healing, initially secreted by myofibroblasts. The


bed/matrix of fibronectin provides an adherent base for migration into the wound, provides
scaffolding for collagen fibrils and mediates wound contraction. Band-6 protein is a constituent
of the desmosomes intracellular plaque. Entactin and nidogen are synonyms and are found in the
dermal-epidermal junction, binding to laminin 1s alpha chain. Uncein is associated with
anchoring filaments.

145
Moving from internally to externally choose the correct description of the hair follicle:
A. Inner root sheath cuticle - Huxley's layer - Henle's layer - medulla - cortex - hair
shaft cuticle
B. Hair shaft cuticle - cortex - medulla - Henle's layer - Huxley's layer _ inner root
sheath cuticle
C. Henle's layer - Huxley's layer - inner root sheath cuticle - hair shaft cuticle -
cortex - medulla
D. Medulla _ cortex _ hair shaft cuticle _ inner root sheath cuticle _ Huxley's layer
_ Henle's layer
E. Inner root sheath cuticle _ outer root sheath cuticle _ hair shaft cuticle _ cortex _
Huxley's layer _ Henle's layer
►D

59
146
Which of the following statements about glomus cells is correct?
A. Tumors of glomus cells are most commonly found on the tongue
B. Are of neural origin
C. Allow rapid shunting of blood from the arterioles to venules, bypassing
capillaries
D. Tumors composed of glomus cells are asymptomatic
E. Tumors composed of glomus cells are often malignant
►C

Tumors composed of glomus cells are usually PAINFUL, not asymptomatic. The are derived
from Suquet-Hoyer canals and allow rapid shunting of blood from the arterioles to venules.
Glomus cells are part of glomus tumors which are most often solitary, purple dermal nodules on
the extremities. Most often, they are seen on the fingers and toes. They are usually painful and
rarely malignant. As opposed to a glomus tumor, glomangiomas are usually painless. They also
most often occur on extremities, but can also occur on the trunk.

147
Defects in what kind of structural protein lead to pyloric atresia associated with junctional
epidermolysis bullosa:
A. Collagen
B. Elastin
C. Loricrin
D. Integrin
E. Plectin
►D

Junctional epidermolysis bullosa with pyloric atresia involves a defect in the β4 subunit of the
a6b4 integrin. The expression of this protein is limited to the basal layer of the epidermis. This
integrin is a transmembrane protein that coordinates a link between the intermediate filaments
(keratins) and the extracellular matrix of the basement membrane. The β4 domain mediates an
interaction with both plectin and BP180; its absence prevents hemidesmosomal assembly.

148
Platelets release which of the following factors to promote new tissue growth?
A. Neutrophil chemotactic factor
B. IL-1
C. ADP

60
D. TGF-alpha
E. FGF
►D

Platelets release PDGF, TGF-alpha and TGF-beta which promote new tissue growth. FGF and
IL-1 are released from monocytes as growth factors (monocytes also release PDGF, TGF-alpha
and TGF-beta). ADP is released from platelets, but is not functioning as a growth factor.
Neutrophil chemotactic factor is released from mast cells and is an inflammatory mediator.

149
Which of the following is the most common genetic alteration seen in mucosal melanomas?
A. GNAQ
B. Cyclin Dependant Kinase 4/6
C. BRAF
D. KIT
E. MDM2
►D

Mucosal melanomas tend to have an activating mutation in KIT. GNAQ mutations are seen in
both uveal melanomas and blue nevi. Cyclin dependant kinase 4/6 binds cyclin D and together
phosphorylate Retinoblastoma. MDM2 targets p53 for ubiquitination.

150
What structure delineates the anatomic region between the nail bed and the distal groove, where
the nail plate detaches of the distal portion of the digit?
A. Nail matrix
B. Proximal nail fold
C. Lunula
D. Eponychium
E. Hyponychium
►E

The hyponychium is the structure that delineates the anatomic region between the nail bed and
the distal groove, where the nail plate detaches of the distal portion of the digit. The corneal layer
of the hyponychium accumulates in part under the free margin of the nail plate. In most cases,
the hyponychium is covered by the distal nail plate, however, it may become visible in the case
of nail biters.

151
The greatest density of mast cells is found in the:

61
A. Stratum spinosum
B. Stratum basale
C. Papillary dermis
D. Reticular dermis
E. Subcutaneous fat
►C

Mast cells are found in the greatest density in the papillary dermis, in sheaths of the appendages,
and around blood vessels and nerves of the subpapillary plexus. These cells are derived from
CD34+ stem cells residing in the bone marrow. They produce and store in secretory granules
many inflammatory mediators such as histamine, heparin, tryptase, chymase, carboxypeptidase,
neutrophil chemotactic factor and eosinophilic chemotactic factor of anaphylaxis. They also
release, without storing, growth factors, cytokines and leukotrienes. Giemsa or Leder stains are
used to identify mast cells in biopsy specimens. Giemsa stains mast cell granules purple. Leder
stains granules red.

152
BP230 is a member of of which of the following families?
A. Plakin
B. Integrin
C. Laminin
D. Collagen
E. Elastin
►A

BP230 is a member of the plakin family and is homologous to desmoplakin and is intracellular.
It attaches intermediate filaments to hemidesmosomal plaque. The remaining options are not part
of the plakin family.

153
Retinoids upregulate transcription of which types of collagen?
A. 1 and 3
B. 1 and 4
C. 1 and 7
D. 3 and 7
E. 4 and 7
►C

Retinoids upregulate the transcription of collagens one and seven thereby strengthening the
dermis.

62
154
Sebaceous glands:
A. Respond to chemical stimuli such as hormones
B. Respond to cholinergic neural activity, exclusively
C. Respond to adrenergic neural activity, exclusively
D. Respond to both adrenergic and cholinergic stimuli
E. Respond to the local release of cytokines from inflammatory cells
►A

Sebaceous glands are androgen-responsive holocrine glands that enlarge at puberty. Meibomian
glands of the eyelids are modified sebaceous glands. Sebaceous glands are found everywhere on
the skin except the palms and soles. Fordyce′s condition involves free sebaceous glands on the
vermillion border of the lips and on the buccal mucosa. Eccrine glands are thermoregulatory
structures that respond to cholinergic stimulation.

155
Sebaceous glands secrete sebum through which of the following secretory mechanisms?
A. Holocrine
B. Merocrine
C. Apocrine
D. Holocrine and Merocrine
E. Holocrine and Apocrine
►A

Sebaceous glands exhibit holocrine secretion, whereby the sebocytes disintegrate in transit to the
gland center, releasing their sebum contents. Merocrine secretion refers to the formation of
intracellular secretory vesicles that translocate to the apical cell surface for secretion. Apocrine
secretion refers to the process whereby secretory contents are packaged using the apical cell
membrane, and 'pinched off' to achieve secretion.

156
Mast cells are derived from bone marrow ____+ cells?
A. CD3
B. CD6
C. CD20
D. CD34
E. CD68
►D

63
CD34+ cells in the bone marrow are the precursors of mast cells. CD3 and CD20 are T and B
cell markers respectively. CD6 is found on mononuclear phagocytic cells in the dermis, CD68 is
a macrophage marker.

157
Homocystinuria has abnormal crosslinking of collagen because of a mutation in:
A. Cystathione synthase
B. Type I collagen N-peptidase gene
C. Lysyl hydroxylase
D. Tenascin X
E. Lysyl oxidase
►A

Homocystinuria is caused by a mutation in cystathione synthase. The main skin findings are a
malar flush, livedo reticularis and leg ulcerations. A characteristic eye finding is the downward
displacement of the lens. The other options are involved in abnormalities associated with Ehlers-
Danlos syndrome. Lysyl hydroxylase is deficient in Kyphoscoliosis type of EDS. Tenascin X is
involves in ~3% of Classical type EDS cases. Dermatosparaxis type EDS has recessive
mutations in the type I collagen N-peptidase gene.

158
Direct immunofluorescence staining of intercellular spaces and the basement membrane zone, in
combination, is seen in:
A. Paraneoplastic pemphigus
B. Anti-epiligrin pemphigoid
C. Pemphigus vegetans
D. Pemphigus foliaceous
E. IgA pemphigus
►A

Paraneoplastic pemphigus and drug-induced pemphigus demonstrate direct immunofluorescence


staining of the intercellular space and the BMZ, in combination. Anti-epiligrin pemphigoid
demonstrates BMZ staining (dermal staining on salt-split skin). Pemphigus vegetans, pemphigus
foliaceous, and IgA pemphigus all show intercellular space deposition without staining of the
BMZ.

159
Tissue contraction begins:
A. At 3rd day of wound healing
B. During the 2nd week of wound healing

64
C. After the first month of wound healing
D. After the 3rd month of wound healing
E. After the 9th month of wound healing
►B

Wound healing tends to be a predictable process that begins initially with the inflammatory
stage. During this stage, clot formation occurs. This is the initial step in wound healing. Platelets,
neutrophils and macrophages all migrate to the wound and secrete many mediators of wound
healing. Epithelialization then begins hours after injury. This is followed by granulation tissue
formation (four days after injury) and angiogenesis (first week of repair). Wound contraction
ensues during the second week of healing.

160
The epidermis is comprised of what type of cells?
A. Keratinocytes, Melanocytes, Merkel cells, Langerhan cells
B. Keratinocytes, Endothelial cells, Merkel cells, Langerhan cells
C. Keratinocytes, Melanocytes, Neutrophils, Langerhan cells
D. Keratinocytes, Melanocytes, Merkel cells, Goblet cells
E. Keratinocytes, Endothelial cells, Merkel cells, Goblet cells
►A

The adult epidermis is composed of three basic cell types: Keratinocytes, melanocytes, and
Langerhans cells. An additional cell, the Merkel cell, can be found in the basal layer of the palms
and soles, the oral and genital mucosa, the nail bed, and the follicular infundibula.

161
A child presents with a 1 cm yellow-red nodule on the face. Pathology shows Touton giant cells.
What is the most frequent site of extracutaneous involvement in this disease?
A. Eye
B. Lung
C. Bone
D. CNS
E. Visceral
►A

The eye is the most frequent site of extracutaneous juvenile xamthogranuloma. The second most
common site of extracutaneous disease is the lungs. Ocular involvement is typically unilateral.

162
During embryogenesis, periderm cells of the fetus contain which of the following substances?

65
A. Ceramide
B. Glycogen
C. Free fatty acids
D. Porphyrins
E. Sebum
►B

In week 7 of embryogenesis the surface ectoderm produces two layers. The external layer is the
periderm which contains glycogen and gives rise to the stratum corneum by week 21. The other
layer is the stratum germinativum.

163
Desmoglein 1 is the antigen in which of the following autoimmune diseases of the skin:
A. Pemphigus foliaceus
B. Bullous impetigo
C. Dermatitis herpetiformis
D. Bullous pemphigoid
E. Pemphigoid gestationis
►A

The antigen implicated in pemphigus foliaceus is Desmoglein 1. Desmoglein 1 is targeted in


bullous impetigo, but this is an infectious condition, not an autoimmune disease. The antigen in
dermatitis herpetiformis is transglutaminase 3. The antigens for both bullous pemphigoid and
pemphigoid gestationis are BPAG1 and BPAG2.

164
Which of the following is true about melanosomes?
A. Spheroid melanosomes have concentric lamellae
B. Spheroid melanosomes synthesize brown-black eumelanin
C. Elliptical melanosomes have microvesicular structure
D. Elliptical melanosomes synthesize yellow or red pheomelanin.
E. The melanosomes are positioned after the Golgi apparatus in the secretory
pathway.
►E

Melanosomes are organelles related to lysosomes and are positioned after the Golgi apparatus in
the secretory pathway. Elliptical melanosomes have concentric lamellae and synthesize brown-
black eumelanin. Spheroid melanosomes have microvesicular structure and synthesize yellow or
red pheomelanin.

66
165
The microflora of pilosebaceous unit consist of which of the following:
A. Pityrosporum ovale
B. Staphylococcus aureus
C. Escherichia coli
D. Pseudomonas aeruginosa
E. Corynebacterium diphtheriae
►A

All the above bacteria and fungi are found within sebaceous glands; the Malasssezia spp. and P.
ovale are found within the acroinfundibulum, S. epidermidis is found within the
midinfundibulum, and Propionibacterium spp. deep within the follicle.

166
Each of the following is true about melanosomes except:
A. Most characteristic organelle of the melanocyte
B. Tyrosinase activity decreases as melanosomoes mature
C. Are transferred to keratinocytes via phagocytosis
D. Are singly dispersed in the basal layer in white skin
E. Are larger in size in black skin compared to white skin
►D

Several differences exist that may explain the heterogeneity of skin color. The number of
melanocytes are the same; however, there are several differences in the melanosomes.
Differences in skin color can be attributed to five factors. In racially heavily pigmented skin, (1)
there is greater production of melanosomes in melanocytes, (2) individual melanosomes show a
higher degree of melanization, (3) melanosomes are larger, (4) the melanosomes are dispersed to
a greater degree in the keratinocytes, and (5) there is a slower rate of degradation.

167
Which cadherin is responsible for adhesion of Langerhan cells to the epidermis?
A. E-cadherin
B. P-cadherin
C. N-cadherin
D. Desmoglein
E. Desmocollin
►A

E-cadherin is responsible for the adhesion of Langerhan cells to the epidermis. There are two
major subclasses of cadherins which mediate cell adhesion and play a fundamental role in

67
normal development, classic (E-,P-,N-cadherin) and desmosomal (desmoglein and desmocollin).
They depend on calcium for their function.

168
Which of the following is a member of the armadillo family of linking proteins?
A. E-cadherin
B. Periplakin
C. Envoplakin
D. Desmocollin
E. Plakoglobin
►E

Plakoglobin is an example of an armadillo protein, which links the cytoskeleton associated


linking proteins (such as plakins in the case of intermediate filaments and alpha-catenin in the
case of actin) to the transmembrane adhesion molecules, termed cadherins. E-cadherin and
desmocollin are examples of cadherins. Periplakin and envoplakin are examples of plakin linking
proteins.

169
Each of the following is true about the basement membrane zone except:
A. Anchoring filaments attach the basal cell membrane to the lamin lucida
B. Can be visualized on light microscopy with PAS staining
C. Lamina densa is composed of type IV collagen
D. Contains laminin 1 and laminin 5
E. Anchoring fibrils are composed of type VII collagen
►A

The basement membrane zone is seen on staining with PAS stain. It appears as a homogenous
band approximately 1 micron thick at the dermo-epidermal junction. The hemidesmosomal
complex and basement membrane zone play an integral role in maintaining cellular adhesion.
Anchoring filaments (primarily composed of laminin 5 and BPAG2) attach the basal cell
membrane to the lamina densa NOT lamina lucida.

170
Human sebum is distinguished from lipids of internal organs by the presence of:
A. Cholestrol
B. Cholestrol esters
C. Squalene
D. Wax esters
E. Glycerides

68
►D

As human sebum exits the sebaceous gland, its major constituents are squalene, cholesterol,
cholesterol esters, triglycerides, and wax esters. With passage through the hair follicle,
triyglycerides in the sebum become hydrolyzed by bacterial enzymes, so that by the time the
sebum reaches the skin surface, it contains free fatty acids, mono- and diglycerides in addition to
the original components. Human sebum is distinguished by the presence of wax esters and
squalene. The lipids of human internal organs contain no wax esters and little squalene. The
squalene that is produced in internal organs is quickly converted to lanosterol and then to
cholesterol, so it does not remain in its original form. Human sebaceous glands do not convert
squalene to sterols.

171
Anagen effluvium is best described as:
A. An abrupt transition from anagen to catagen in rapidly dividing hair matrix cells
B. A cessation of mitotic activity in rapidly dividing hair matrix cells
C. An abrupt transition of telogen to anagen in resting hair matrix cells
D. A cessation of mitotic activity in resting hair matrix cells
E. A scarring alopecia affecting only anagen stage follicles
►B

Anagen effluvium results from an outside stimulus – most often an antimetabolite,


chemotherapeutic drug – inducing an abrupt cessation of hair matrix cell mitotic activity. This
process occurs within days to weeks of the stimulus, and is reversible with cessation of the drug
therapy.

172
Red or blonde hair pigmentation primarily results from:
A. The presence of eumelanin
B. The absence of melanin
C. The presence of pheomelanin
D. The reduced activity of tyrosinase
E. The reduced activity of DOPA dehydroxylase
►C

Hair color is determined by melanocytes. The melanocytic activity of follicular melanocytes is


coupled to anagen ′ hair is only pigmented when it is growing. Pigment is produced in the matrix
area of follicle, above the follicular papilla. Eumelanin is the pigment of brown/black hairs, and
pheomelanin is the pigment of red/blonde hairs. Intensity of color is proportional to the amount

69
of pigment. The absence of pigment produces white hair, and markedly reduced pigment
produces gray hair.

173
Which of the following elements is necessary for melanin production?
A. Copper
B. Selenium
C. Iron
D. Zinc
E. Calcium
►A

Melanin is synthesized from tyrosine vis the action of the enzyme tyrosinase (tyrosine to DOPA
to DOPAquinone to melanin). Tyrosinase is a copper containing enzyme. The other elements are
not specifically required for melanin synthesis.

174
Acral melanomas are particularly concerning because they:
A. Are diagnosed at a later stage
B. Invade perineurally
C. Cannot be resected
D. Metastasize frequently even at shallow Breslow depth
E. Do not respond to ipilimumab
►A

Acral melanoma is problematic because it is diagnosed at a later stage. It accounts for 5-10% of
all melanomas.

175
Which mechanoreceptor found in hair bearing areas sense deep touch and vibration?
A. Merkel cell
B. Meissner corpuscle
C. Vater-Pacini corpuscle
D. Krase end-bulb
E. Free nerve ending
►C

Vater-Pacini corpuscles sense deep touch and vibration. Merkel cells are slow adapting type I
mechanoreceptors found among basal keratinocytes. Meissner corpuscles are found in the dermal

70
papilla, especially in the palms and soles. Krase end-bulbs are mucocutaneous end-organs found
on the glans penis, prepuce, clitoris, labia minora, and vermillion border of the lip.

176
Which sebaceous gland is located on the eyelids in association with eyelashes?
A. Montgomery's tubercles
B. Tyson's glands
C. Zeis glands
D. Meibomian glands
E. Fordyce's spots
►C

Zeis glands are sebaceous glands associated with eyelashes. Montgomery's tubercles are present
on the areola, Tyson's glands on the labia minora and glans, and Fordyce's spots are located on
the buccal mucosa.

177
The following protein is the target for both dytrophic EB and Bullous SLE:
A. Type VII Collagen
B. Laminin 5
C. Integrin subunit β4
D. Type XVII collagen
E. BPAG2
►A

Type VII collagen is the target for both dystrophic EB and bullous SLE. It is also the target for
EB acquisita.

178
Which of the following stains would you expect to be positive in a normal eccrine unit?
A. S-100
B. Prussian blue
C. Giemsa
D. Verhoeff von Gieson
E. Steiner
►A

Eccrine glands stain S-100 and CEA positive. The remaining stains would not be expected to
stain normal eccrine sweat glands. Prussian blue (Perl‘s) is an iron stain which stains iron or
hemosiderin bright blue. Giemsa stains mast cell granules purple (heparin in the granules) and

71
can also be useful in staining in Leishmaniasis. Verhoeff von Gieson is a stain for elastic tissue
that stains blue-black. Steiner stain is a silver stin for spirochetes similar to a Warthin Starry or
Dieterle stain.

179
All of the following pertain to Odland bodies EXCEPT:
A. Contain squalene
B. Are found intracellularly in upper level keratinocytes
C. Discharge their contents into the extracellular space at the junction of the
granular and cornified layers
D. Establish a barrier to water loss
E. Mediate stratum corneum adhesion in conjunction with filaggrin.
►A

Odland bodies mediate stratum corneum adhesion in conjunction with FILAGGRIN. They
discharge their contents into the extracellular space at the junction of the granular and horny
layers establish a barrier to water loss. They are first found intracellularly in upper level
keratinocytes and contain ceramides not squalene.

180
The function of glycosaminoglycans/proteoglycans in the dermis is:
A. Regulate water-binding capacity
B. Interact with dermal dendrocytes
C. Facilitate COLVII binding to the anchoring plaques
D. Facilitate mast cell degranulation
E. Regulate lymphocyte trafficking
►A

The function of glycosaminoglycans/proteoglycans in the dermis is to regulate water-binding


capacity. The other listed options are not functions of glycosaminoglycans / proteoglycans.

181
What is the significance of the critical line of Auber?
A. It is the location of the insertion of the erector pili muscle
B. the bulk of the mitotic activity in the hair occurs above this line
C. the inner root sheath is formed above this line
D. It is the widest diameter of the hair bulb
E. It is where keratinization first occurs in the hair
►D

72
The critical line of Auber is at the widest diameter fo the hair bulb. Below this line, the bulk of
mitotic activity that gives rise to the hair and the inner root sheath occurs. The erector pili muscle
inserts in the isthmus region of the follicle. Keratinization first occurs above this line.

182
Which of the following glands is not under neural control?
A. Sebaceous glands
B. Apocrine glands
C. Eccrine glands
D. Salivary glands
E. Molls glands
►A

Sebum is secreted continuously on the skin under hormonal glands. It contains squalene,
cholesterol, cholesterol esters, wax, and triglycerides.

183
Meibomian glands are:
A. Modified sebaceous glands
B. Found everywhere except on the palms and soles
C. Sebaceous lobules that feed into a lactiferous duct
D. Modified ceruminous glands
E. Modified apocrine glands
►A

Meibomian glands of the eyelids are modified sebaceous glands. Sebaceous glands are found
everywhere on the skin except the palms and soles. Montgomery's areolar tubercles consist of
several sebaceous lobules feeding into a lactiferous duct. Ceruminous glands are apocrine glands
of the external ear canal. Apocrine glands in the eyelids are Moll's glands.

184
Which protein is the largest component of the cornified cell envelope?
A. Keratin
B. Involucrin
C. Profilaggrin
D. Loricrin
E. Ceramide
►D

73
The cornified cell envelope is a durable, protein-lipid polymer that eventually acts as a
mechanical and chemical barrier on the exterior of cornified cells. In the upper spinous layer,
keratohyaline granules release profilagrin and loricrin. Profilaggrin is cleaved to filaggrin and
subsequently aggregates keratin filaments. Loricrin is the major protein component of the
cornified cell envelope and is bound to the cell membrane by transglutaminases (in addition to
other structural proteins like involucrin, keratins, elafin, cystatin A and desmosomal peptides).
This forms the highly insoluble proteinaceous component of the cornified cell envelope.

185
The classical types of Ehlers-Danlos Syndromes (Type 1 and 2) lead to varying degrees of
hyperextensible skin, easy bruising, wide, atrophic scars, and hypermobile joints. The underlying
defect in this disorder is a mutation in:
A. Elastin
B. Fibrillin 1
C. ABCC6
D. Collagen V
E. Cystathionine beta-synthase
►D

Classical types of Ehrlers-Danlos are manifested by varying degrees of hyperextensible skin,


easy bruising, wide, atrophic scars, and hypermobile joints. The genetic defect is in Collagen V.
Elastin is mutated in Cutis Laxa. ABCC6 is defective in Pseudoxanthoma elasticum.
Cystathionine beta-synthase is mutated in homocystinuria.

186
People with darker skin show:
A. Smaller, more concentrated melanosomes
B. A more rapid degradation of melanosomes
C. A lessened production of melanosomes within melanocytes
D. A higher degree of dispersion of melanosomes in keratinocytes
E. A grouping of melanosomes with a low degree of melanization
►D

More darkly pigmented races show a greater production of melanosomes in the melanocyte,
melanosomes with a higher degree of melanization, larger melanosomes, a higher degree of
dispersion of melanosomes in the keratinocytes, and a slower rate of melanosome degradation.

187
The desmosomal connections of the epidermis are dependent on which of the following ions?
A. Iron

74
B. Zinc
C. Selenium
D. Calcium
E. Sodium
►D

The desmosomal connections in the epidermis are calcium dependent. The other options are not
required for these connections.

188
Apocrine glands are found in all of the following areas of the body except:
A. Axillae
B. Breasts
C. Eyelid
D. Palms
E. Perineum
►D

Apocrine glands operate by decapitation secretion and are activated by epinephrine and
norepinephrine. They are located in a few distinct areas of the body, which include axillae,
anogenital region, Moll‘s glands of the eyelids, mammary glands of the breast and the
ceruminous glands of the external auditory canal.

189
Pick the correctly paired keratin with its structure:
A. K1/K10 - basal cells
B. K3/K12 - esophagus
C. K4/K13 - cornea
D. K5/K14 - suprabasal cells
E. K16/K6 - palms and soles
►E

Keratin 1 and 10 are found in the stratum spinosum. K5 and K14 are found in the basal layer. K3
and K12 are found in the suprabasilar cells of the cornea. K4 and K13 are found in the non-
cornifying cells of stratified mucosa. K16 and K6 are found in the palms and soles.

190
Dystrophic epidermolysis bullosa results from mutations in:
A. Collagen type I
B. Collagen type II

75
C. Collagen type III
D. Collagen type V
E. Collagen type VII
►E

Dystrophic epidermolysis bullosa results from mutations in Type 7 collagen. Type I collagen
mutations are associated with osteogenesis imperfecta. Antibodies to Type II collagen are
associated with relapsing polychondritis. Mutations in type V collagen result in the classic form
of Ehlers Danlos while mutations in type III collagen result in the vascular form of Ehlers
Danlos.

191
Where are glomus cells derived from?
A. Mesenchyme
B. Bone marrow
C. Susquet-Hoyer canal
D. Skeletal muscle
E. Dermis
►C

Glomus cells are vascular smooth muscle cells derived from the arterial portion of the glomus
body, or the Sucquet-Hoyer canal, which is an arteriovenous shunt in the dermis that contributes
to temperature regulation. Glomus are found primarily on the palms and soles and function to
allow the rapid shunting of blood from the arteioles to venules. Disease processes involving
glomus cells include glomus tumor and glomangioma.

192
For the treatment of drug induced linear IgA bullous dermatosis, after stopping offensive
medication, what other treatments may be used?
A. Interferon alpha
B. Interferon gamma
C. Psoralen plus UVA
D. Interleukin 2
E. Dapsone
►E

Dapsone may be used to treat patients with drug induced linear IgA bullous dermatosis which
persist after the offending medication has been discontinued. The other medications listed are
rare causes of drug induced linear IgA bullous dermatosis and so should not be used for
treatment.

76
193
Keratinocytes are derived from which of the following:
A. Endoderm
B. Mesoderm
C. Ectoderm
D. Neural Crest
E. Bone marrow precursors
►C

As implied by the root 'ecto', a prefix meaning "outer", the keratinocytes of the epidermis are
derived from the ectoderm. The other layers do contribute cell populations that are present in the
skin.

194
Apocrine glands:
A. Are coiled glands
B. Have a two segment ducts that empties onto the skin
C. Are present everywhere on the skin except on the palms and soles
D. Function from birth
E. Secretions are initially odorless
►E

Apocrine glands are tubular glands that demonstrate decapitation secretion. Like eccrine glands,
the ducts are composed of three segments: intraepidermal duct, intradermal duct and secrectory
portion. The duct usually leads to a pilosebaceous follicle above the entrance of the sebaceous
duct. They are found in the axillae, anogenital region, external ear canal (ceruminous glands), in
the eyelids (Moll‖s glands) and in the breast (mammary glands). These glands are functional
starting at puberty. The initial secretions are odorless with the odor being derived from C6 &
C11 acids. The most abundant being 3-methyl-2-hexenoic acid.

195
S100 staining would be negative in?
A. langerhans cells
B. eccrine cells
C. schwann cells
D. adipocytes
E. keratinocytes
►E

77
S100 is sensitive for melanocytes and melanoma, but not specific as it also stains langerhans
cells, eccrine cells, schwann cell, adipocytes, and chondrocytes. It is negative in keratinocytes
and can help discriminate melanoma from pagetoid bowens or spindle cell squamous cell
carcinoma

196
Keratinocytes have been shown to secrete all of the following cytokines except:
A. IL-1
B. IL-6
C. IL-8
D. TNF-alpha
E. IL-2
►E

Keratinocytes have been shown to secrete all of the above cytokines, except IL-2, IL-4, and IFN-
gamma.

197
Numerous neuromediators are involved in cutaneous neurobiology and many play a role in the
development of inflammation in the skin. One such mediator can be induced by application of
capsaicin to the skin. Which of the following is the correct neuromediator?
A. Noradernaline
B. Substance P
C. Neurokinin A
D. Acetylcholine
E. Pro-opiomelanocortin
►B

Substance P is a neuromediator that binds the tachykinin receptor. It is released upon stimulation
of sensory nerve fibers. It has numerous cutaneous functions, including development of skin
edema, erythema, and pruritus, upregulation of adhesion molecule expression, release of
proinflammatory mediators, etc. Capsaicin causes release of substance P. This has been utilized
pharmacologically in conditions such as zoster, in which consistent application of capsaicin leads
to depletion of substance P, which in turn can help to reduce/eliminate post-herpetic neuralgia.
The other answer choices are also neuromediators that are involved in cutaneous inflammation,
but their release is not induced by capsaicin.

198
Which sebaceous gland is located on the buccal mucosa and vermilion border of the lips?
A. Montgomery's tubercles

78
B. Tyson's glands
C. Zeis glands
D. Meibomian glands
E. Fordyce's spots
►E

Fordyce's spots are located on the buccal mucosa and vermilion border of the lips. Montgomery's
tubercles are present on the areola, Tyson's glands on the labia minora and glans, meibomian and
Zeis glands are present on the eyelids.

199
Regarding the stratum spinosum, which of the following is correct?
A. No keratin 1/10 is present
B. New synthesis of K5/14 occurs in this layer
C. The "spines" seen on pathology are due to desmosomal connections between
keratinocytes
D. This layer contains melanocytes
E. This layer contains the cornified cell membrane
►C

The "spines" seen on pathology are due to desmosomal connections between keratinocytes, NOT
hemidesmosomes. The hemidesmosomes are present in the cells at the base of the basal layer and
are part of the connection between the epidermis and basement membrane. Keratin expression
continues in the spinous layer. Keratins 1/10 are synthesized, not keratins 5/14. Keratin 5/14,
however, is still present. Keratohyaline granules are typically seen in the granular layer.

200
Which of the following medications is concentrated in the eccrine glands?
A. Cyclophosphamide
B. Cytarabine
C. Ciprofloxacin
D. Cephalexin
E. All of the answers are correct
►E

The above listed drugs, as well as beta-lactam antibiotics, antifungals such as ketoconazole and
griseofulvin, are known to be secreted into eccrine sweat ducts. This feature may explain the
development of neutrophilic eccrine hidradenitis and eccrine squamous syringometaplasia in the
context of chemotherapy.

79
201
In which of the following locations would you be least likely to identify melanocytes?
A. Stria vascularis of the ear
B. Iris
C. Leptomeninges
D. Retina
E. Pericardium
►E

Melanocytes are found in the stria vascularis of the ear, iris, leptomeninges and retina. There are
no normal populations of melanocytes found in the pericardium.

202
Apocrine glands:
A. Demonstrate holocrine secretion
B. Demonstrate decapitation secretion
C. Are fully functional at birth
D. Are diffusely distributed on the body
E. Are thermoregulatory
►B

Apocrine glands show decapitation secretion. Like eccrine glands, apocrine glands are composed
of three segments, the intraepidermal duct, the intradermal duct, and the secretory portion. The
duct of the apocrine gland usually leads to a pilosebaceous follicle above the entrance of the
sebaceous duct. Apocrine glands are found in the axillae, anogenital region, external ear canal
(ceruminous glands), in the eyelids (Moll‘s glands), and in the breast (mammary glands).
Apocrine glans are functional only at puberty. Their initial secretion is odorless.

203
Tyrosinase is the enzyme that catalyzes the conversion of tyrosine to DOPA and DOPA to
DOPAquinone. The enzyme contains which of the following ions?
A. Zinc
B. Copper
C. Selenium
D. Iron
E. Magnesium
►B

Tyrosinase is a copper containing enzyme that is responsible for the conversion fo tyrosine to
DOPA and DOPA to DOPAquinone.

80
81
Chapter -2-
Immunodermatology
1
The immunoglobulin most commonly found in mucous secretions is:
A. IgA
B. IgD
C. IgE
D. IgG
E. IgM
►A

IgA is found in mucous membrane secretions and is able to agglutinate antigens and activate the
alternate but not the classic complement pathway. IgG is the antibody that can cross the placenta
and the most common antibody found in circulation. IgD is not found in circulation other than in
hyper-IgD syndrome, an autosomal recessive disorder caused by mutations in the mevalonate
kinase gene. A significant elevation of serum IgD is seen in 95% of these patients. IgE is an
anaphylactic antibody that is involved in nearly all immediate allergic and anaphylactic type
reactions. IgM is the antibody produced in the early stages of antibody responses. It is a
pentamer which can agglutinate antigen and active the classic complement pathway.

2
A 24 year old female patient is referred for management of chronic idiopathic urticaria. Many
cases of this disease are associated with autoantibodies against what?
A. IgE receptor
B. Tryptase
C. Histamine receptor
D. TNF-alpha receptor
E. IL-6 receptor
►A

Many cases of idiopathic urticaria have circulating autoantibodies directed against the chain of
high affinity IgE receptor on the mast cell surface.

3
The pharmacologic activity of tacrolimus includes:
A. Phosphorylation of NFAT (nuclear factor of transcription)
B. Binding and inhibition of NF kappa B
C. Inhibition of interleukin-1 gene transcription

82
D. Activation of calcineurin
E. Inhibition of interleukin-2 gene transcription
►E

Tacrolimus is a macrolide similar to cyclosporine and pimecrolimus (Elidel). Calcineurin is a


phosphatase that dephosphorylates subunit on NFAT (nuclear factor of activated T cells).
Dephosphorylated NFAT is active and causes the increase in IL2. Tacrolimus binds
macrophillin1 which blocks calcineurin so NFAT is not dephosphorylated ie it remains
phosphorylated and is inactive therefore preventing transcription of IL 2.

4
This patient says the rash is spreading and not controlled with topical therapy. You give him a
course of oral treatment that lasts:
A. 1 week
B. 2 weeks
C. 3 weeks
D. 4 weeks
E. 5 weeks
►C

Generally, for poison ivy dermatitis, if patients are given a course of oral steroids, the course
should be at least 3 weeks long, as if the duration is shorter, patients may develop a rapid
rebound.

5
Anti Jo-1 antibodies are directed against which of the following?
A. Topoisomerase
B. Lysyl oxidase
C. Gyrase
D. Histidyl transfer RNA synthetase
E. Telomerase
►D

Anti Jo-1 antibody is typical of autoimmune diseases that involve muscle, including
dermatomyositis.

6
Anti-epiligrin (laminin 5) antibodies may be seen in:
A. Pemphigoid gestationis
B. Pemphigus vegetans

83
C. Fogo selvagem
D. Cicatricial pemphigoid
E. Paraneoplastic pemphigus
►D

Patients with cicatricial pemphigoid have been reported to have anti-epiligrin antibodies.

7
Subacute cutaneous lupus erythematosus has been associated with the ingestion of which of the
following drugs?
A. Phenytoin
B. Allopurinol
C. Terbinafine
D. Trimethoprim/sulfamethoxazole
E. Auranofin
►C

SCLE has been reported to be associated with terbinafine. This condition is often associated with
anti-Ro (SS-A) and anti-La (SS-B) antibodies.

8
This patient has a lichenified plaque in the lower mid abdomen as well as these two exczematous
plaques. This patient needs:
A. Patch testing
B. A steroid
C. An antiviral
D. A KOH scraping
E. An antifungal
►A

This patient is likely allergic to the nickel in her belt buckle and possibly to metals that she
places in her pocket. She needs patch testing. While a topical steroid might help resolve pruritus,
the patient ultimately needs to be diagnosed with her allergy and avoid nickel.
Dimethylglyoxime can be used as an indicator to detect nickel in metals.

9
A previously healthy child presents with palpable purpura, arthritis, and vomiting. You suspect a
hypersensitivity vasculitis characterized by:
A. Perivascular IgA
B. P-ANCA autoantibodies

84
C. Granulomas and eosinophilia
D. Infiltration of destruction of vessels by atypical lymphocytoid and plasmacytoid
cells
E. Nectrotizing granulomatous vasculitis
►A

The child has findings of Henoch-Schonlein purpura, a hypersensitivity vasculitits that presents
clinically with a triad of purpura, joint pain, and GI complaints. Histologically it is characterized
by perivascular deposition of IgA.

10
Which immunoglobulin is most efficient at fixing complement?
A. IgM
B. IgE
C. IgD
D. IgA
E. IgG
►A

IgM is the first antibody produced by B cells. It is secreted by plasma cells as a pentamer. Due to
its size, IgM doesn't enter tissue well. It is the most efficient immunoglobin at fixing
complement.

11
Gene rearrangement analysis is useful for determining:
A. Lymphocyte clonality in mycosis fungoides
B. Lymphocyte activity
C. Gene Function
D. Gene Mutations
E. T cell receptor status
►A

Gene rearrangement studies are useful to detect clonality in antigen specific cell types (B cells, T
cells).

12
Major histocompatibility complex class I molecules bind to:
A. Peptides derived from proteins synthesized and degraded in the cytosol
B. Peptides derived from proteins degraded in endocytic vesicles
C. Peptides external to the cell membrane

85
D. Immunoglobulin E
E. None of these answers are correct
►A

Major histocompatibility complex (MHC) class I molecules bind to peptides derived from
proteins synthesized and degraded in the cytosol. They present these processed peptides to CD8+
T-cells. MHC class II molecules bind stably to peptides derived from proteins degraded in
endocytic vesicles. CD4+ T-cells recognize the MHC class II molecules. Immune activation
against the foreign antigens or pathogens taken up by the cell is the result of these interactions.
Peptides external to the cell are not recognized by MHC molecules. IgE does have a receptor on
the cell surface, especially basophils and mast cells, but is not recognized by the MHC complex.

13
The most definitive HLA association with psoriasis is:
A. HLA-Cw6
B. HLA-B27
C. HLA-B13
D. HLA-B17
E. HLA-B37
►A

HLA-Cw6 is associated with a 9-15x greater risk for developing psoriasis. All of the other HLA
antigens listed are associated with various types of psoriasis, but at with lesser strength of
association.

14
Immunocytomas are:
A. Low grade B-cell lymphomas
B. Aggressive B-cell lymphomas
C. Low grade T-cell lymphomas
D. Aggressive T-cell lymphomas
E. NK cell lymphomas
►A

These indolent tumors present as solitary or multiple nodules usually on the extremities. The
cells have been reported to have CD-20 (B cell marker) and have been reported to be bcl-2
positive.

15
Anti-Ro (SS-A) antibodies are most commonly found in:

86
A. Mixed connective tissue disease
B. Eosinophilic fasciitis
C. Drug-induced systemic lupus erythematosus
D. Homozygous C2 deficiency
E. Neonatal lupus erythematosus
►E

Ro 60 kDa autoantigen is a major target for patients with SLE and Sjogren‖s syndrome. Neonatal
lupus occurs in newborns of mothers with anti-Ro antibodies and leads to children with
photosensitive skin lesions and a cardiac conduction defect, a third degree heart block.

16
Which of the following cytokines shifts the immune response towards TH2?
A. IL-4
B. IL-5
C. IL-10
D. TNF
E. IFN-gamma
►A

IL-4 is the cytokine that polarizes the immune response towards Th2. IL-5 is an eosinophil
growth factor. IL-10 is a general down-regulator of immunity. TNF and IFN-gamma are TH1
cytokines, not TH2 cytokines. IFN-gamma is secreted by TH1 cells, and is the main
macrophage-activating cytokine.

17
Which of the follwing is a chemotactic factor for eosinophils?
A. TNF
B. IL2
C. C5a
D. Plasminogen activator
E. IL8
►C

Eosinophil chemotactic factors include all of the following: Histamine, soluble immune
complexes, C5a, and HETE

18
Herpes simplex virus-related erythema multiforme has been associated with an increased
frequency of:

87
A. HLA-B7
B. HLA-B8
C. HLA-B13
D. HLA-B15
E. HLA-B27
►D

Erythema multiforme associated with herpes simplex has been reported to have an increase of
HLA-B15.

19
A homeless patient presents with a scaling, pustular periorifical eruption around the mouth and
genitalia. What lab abnormality is associated with this condition?
A. Increased zinc level
B. Increased copper level
C. Decreased copper level
D. Decrease alkaline phosphatase level
E. Increased alkaline phosphatase level
►D

In a homeless patient with poor nutrition, acrodermatitis enteropathica due to zinc deficiency can
presents with a scaling eruption of the periorificial regions. In addition to a low zinc level, levels
of alkaline phosphatase, a zinc dependent enzyme, are decreased.

20
Which of the following immunoglobulins cannot activate the complement pathway?
A. IgM
B. IgG1
C. IgG2
D. IgG3
E. IgG4
►E

Immunoglobulins (Ig) differ in their ability to activate complement. IgM is the largest Ig, is the
major Ig in the primary immune response, and consists of a pentamer that activates the classic
complement pathway. IgG is the most abundant Ig and the major Ig in the secondary immune
response. Four subclasses of IgG exist based on the amino acid residue sequences of their
constant region, IgG1 through IgG4. IgG1 and IgG3 are potent activators of the classic
complement pathway, IgG2 is less effective and IgG4 is unable to do so.

88
21
Which of the following is an example of a delayed hypersensitivity reaction?
A. Allergic contact dermatitis
B. Anaphylaxis
C. Latex allergy
D. Transfusion reaction
E. Serum sickness
►A

There are 4 types of Hypersensitivities: ("ACID") Type I: Anaphylactic and Atopic: Examples -
Urticaria, Asthma, and Allergic Rhinitis Type II: cytotoxic: examples - Transfusion reactions,
ABO incompatibility, Rh disease (erythroblastosis fetalis), Autoimmune reactions, Hemolytic
disease of newborn, Goodpasture's syndrome Type III: Immune complex, Serum sickness, and
arthrus reactions: examples -PAN, glomerulonephritis, SLE, Rheumatoid arthritis, and serum
sickness Type IV: Delayed (cell-mediated)typesL examples -TB skin test, transplant rejection,
contact dermatitis, interactions and skin repsonsiveness to bacteria, fungi, viruses, and protozoa,
photo-allergies, insect bites, etc

22
This woman should have a workup for:
A. Lymphoma
B. Nephrolithiasis
C. Pancreatic cancer
D. Hemochromatosis
E. Thalassemia
►D

Porphyria cutanea tarda has been shown to be associated with hemochromatosis. Patients with
hemochromatosis have mutations in the HFE gene, and early detection of mutations can improve
life expectancy for these patients.

23
Langerhans cell adhesion to keratinocytes is mediated by what adhesion molecule?
A. E-cadherin
B. N-cadherin
C. Desmoglein 3
D. P-cadherin
E. B7
►A

89
Langerhans cells are found in the epidermis and on mucosal surfaces. After antigen exposure and
activation, they migrate to regional lymph nodes where they mature into antigen presenting cells.
Adhesion to keratinocytes in the epidermis is mediated by e-cadherin.

24
When attempting to identify Langerhans cells in a specimen, which marker is most helpful?
A. CD1
B. CD4
C. CD7
D. CD8
E. CD20
►A

CD1 is a surface antigen specific for epidermal Langerhans cells. It is not expressed in other
epidermal structures. The other characteristic ultrastructural feature of Langerhans cells is the
Birbeck granule. CD4 is found on T-helper cells and occasionally on Langerhans cells. CD7 is a
T-cell marker that is often lost in cutaneous T-cell lymphoma. CD8 is found on cytotoxic T-cells.
CD20 is a B-cell marker.

25
Elaboration of which of the following cytokines is characteristic of TH2 response?
A. Interferon-8 (gamma)
B. Interleukin-1
C. Interleukin-2
D. Interleukin-4
E. Interleukin-12
►D

Interleukin-4 (IL-4) is a cytokine involved in B-cell proliferation. Along with IL-5 and IL-13, IL-
4 is classified as a TH2 cytokine.

26
Which cytokine is most important in recruiting neutrophils?
A. Interleukin-1
B. Interleukin-2
C. Interleukin-4
D. Interleukin-8
E. Interleukin-10
►D

90
Chemotaxis is the process of cells moving through a gradient of towards increasingly higher
concentrations. IL-8 has activating and chemoattractant properties on neutrophils.

27
Which monoclonal gammopathy is most commonly associated with erythema elevatum
diutinum?
A. IgA
B. IgD
C. IgE
D. IgG
E. IgM
►A

A report of 13 patients indicated IgA is most commonly associated with EED.

28
Which cytokine is not upregulated in atopic dermatitis patients?
A. IL-13
B. IL-4
C. IL-5
D. IL-10
E. IFN-gamma
►E

Interferon gamma is Th1 cytokine which downregulates Th2 responses. The remaining are Th2
cytokines active in atopic dermatitis. IL-4 is a B-cell growth factor and active in signaling
isotope switching from IgM to IgE. IL-5 is an eosinophil growth factor. IL-10 downregulates
Th1 immunity and IL-13 signals isotope switching along with IL-4.

29
Dermatitis Herpetiformis is most commonly associated with which HLA?
A. HLA-DR3
B. HLA-B27
C. HLA-B8
D. HLA-Bw35
E. HLA-DQ(A1*0501, B1*02)
►E

91
HLA-DQ (A1*0501, B1*02) genes are present on 90% of all patients with Dermatitis
Herpetiformis. The remainder of patients have the HLA-DQ8 gene. Other associations include:
HLA-B8, HLA-DR3 and DR5/DR7.

30
The most common autoimmune disorder in patients with chronic hepatitis C infection is:
A. Autoimmune thyroiditis
B. Idiopathic thrombocytopenic purpura
C. Rheumatoid arthritis
D. Sjӧgren‖s syndrome
E. Systemic lupus erythematosus
►A

Autoimmune thrombocytopenia has been reported with hepatitis C infection but not as
commonly as autoimmune thyroiditis.

31
A 27 year old gentleman with a known history of chronic plaque psoriasis complains of pain and
stiffness of his joints, particularly in his hands. Which HLA subtype is associated with psoriatic
arthritis?
A. B27
B. Cw6
C. B17
D. B51
E. B8
►A

Psoriatic arthritis, as well as generalized psoriasis,is associated with HLA-B27. HLA-Cw6 is


associated with psoriasis, Bw35 with cutaneous lichen planus, B8 with oral lichen planus, and
B51 with Behcet's disease.

32
Topical tacrolimus and pimecrolimus are used to treat atopic dermatitis and other inflammatory
skin conditions. On which of the following ions is the inflammatory pathway blocked by these
medications dependent?
A. Sodium
B. Potassium
C. Calcium
D. Selenium
E. Zinc

92
►C

Calcium. Both pimecrolimus and tacrolimus penetrate cutaneous T-cells, forming a complex
with calcineurin, which blocks the activation of NF-AT, thus blocking the transcription of a
variety of genes with a resultant decrease in T-cell mediated inflammation. This pathway is
calcium dependent. The other ions are not involved in this process.

33
Natural killer (NK) cells eliminate infected cells in all of the following ways except:
A. NK cells adhere to and kill target cells coated with IgG
B. NK cells secrete perforins
C. NK cells secrete granzyme
D. NK cells secrete myeloperoxidase
E. NK cells do not target cells expressing major histocompatibility (MHC) class I
molecules
►D

NK cells focus on the destruction of infected or malignant cells. They achieve this via
recognition of IgG on target cells, the so-called 'antibody-dependent cellular toxicity.' NK cells
also eliminate cells by secreting perforin, which makes holes in the cell membrane, through
which granzyme is injected. Granzyme induces apoptosis by activating the caspase cascade. In
addition, NK cells do not target cells expressing MHC class I molecules on their surface; some
virus downregulate MHC class I molecules to evade recognition by cytotoxic T cells, which may
make them susceptible to NK cell attack. Neutrophils kill ingested organims using
myeloperoxidase.

34
An elderly gentleman with metastatic colon cancer is prescribed erlotinib, an epidermal growth
factor receptor monoclonal antibody. What is the most common cutaneous finding seen as a
result of this class of medication?
A. Acneiform eruption
B. Acral erythema
C. Morbilliform eruption
D. Hypotrichosis
E. Urticaria
►A

Erlotinib, along with cetuximab and gefitinib, are epidermal growth factor receptor inhibitors
that are used for the treatment of multiple visceral malignancies, including metastatic colon
cancer. They have all been associated with an acneiform eruption.

93
35
Tacrolimus is a non-steroidal anti-inflammatory medication that works by inhibiting calcineurin
activity through complexing with what binding protein?
A. FK506
B. TGF-beta
C. NF-kappa-B
D. SRE
E. IL-23
►A

Tacrolimus and pimecrolimus are non-steroidal calcineurin inhibitors that act as anti-
inflammatory medications. In dermatology, they are most commonly used in topical
preparations. These medications form complexes with FK506 binding protein, which inhibits
calcineurin activity. A key regulatory step in the activation of T cells is the activation of
calcineurin via calmodulin.

36
Keratinocytes express what class of major histocompatibility complex under normal conditions?
A. MHC Class I
B. MHC Class II
C. MHC Class III
D. MHC Class IV
E. MHC Class V
►A

Keratinocytes express MHC Class I molecules and therefore can be attacked by CD8+ Tc cells,
in particular after viral infection. Although keratinocytes do not express MHC Class II molecules
under normal conditions, they can be induced to do so in the setting of inflammatory conditions.

37
Which component of the classical pathway of complement acts as an opsonin?
A. C3b
B. C3a
C. C5a
D. C5b
E. C1 INH
►A

94
The classical pathway of complement is activated by antigen-antibody complexes. C3a is a
neutrophil chemoattractant. C3b is an opsonin. C5a is an anaphylatoxin. C5b forms a part of the
membrane attack complex.

38
Which of the following immune-mediated events has been demonstrated in psoriasis vulgaris?
A. Clonal expansion of CD8+ T cells
B. Decrease dermal Langerhans cells
C. Downregulation of keratin 16
D. Increase Th2 CD4+ T cells
E. Decreased production of interferon-gamma
►A

The involvement of T cells in the pathophysiology of psoriasis vulgaris is well-recognized.


Availability of monoclonal antibodies has allowed for extensive characterization of T cell
subsets and other mediators increased in psoriasis lesions. CD8+ T cells are highly concentrated
in psoriatic epidermis and studies have demonstrated increased IL-2R and HLA-DR surface
molecules indicative of persistent activation. Clonal expansion of CD8+ T cells has been
observed suggesting that this subset is the major antigen-reactive population.

39
Which cytokine is responsible for fever in patients with sunburn?
A. IL-1
B. IL-5
C. IL-10
D. IL-11
E. TNF-beta
►A

IL-1 is a pyrogenic cytokine responsible for the fever in sunburn. It also is causes B cell
maturation and proliferation and NK cell activation.

40
Which of the following substances is located in the core of an eosinophil?
A. Eosinophilic cationic protein
B. Eosinophil-derived neurotoxin
C. Eosinophil peroxidase
D. Major basic protein
E. Chymase
►D

95
Major basic protein is the only protein located in the core of an eosinophil. Eosinophil cationic
protein, eosinophil-derived neurotoxin, and eosinophil peroxidase are all located in the matrix.
Chymase is a mediator stored in granules of a mast cell.

41
Which paraprotein is found most commonly in patients with pyoderma gangrenosum?
A. IgG
B. IgA
C. IgM
D. IgE
E. IgD
►B

IgA paraproteinemia has been reported in over 10-18% of pyoderma gangrenosum.

42
Which of the following TH2 cytokines is a B cell growth factor?
A. IL-4
B. IL-5
C. IL-10
D. IL-13
E. IFN-gamma
►A

IL-4 is a B cell growth factor. IL-5 is an eosinophil growth factor. IL-10 is a general down-
regulator of TH1 immunity. IL-13 (along with IL-4) promotes an isotype switch from IgM to
IgE. IFN-gamma is not a TH2 cytokine. It is secreted by TH1 cells, and is the main macrophage-
activating cytokine.

43
Which is not a feature of mast cells?
A. Expresses c-kit
B. Expresses CD-3
C. Produces IL-8
D. Produces prostaglandin D2
E. Stains with napththol chloro-acetate esterase
►B

Mast cells are an integral portion of immediate type hypersensitivity. CD3 is a T cell marker.

96
44
Which of the following is associated with hepatitis C infection?
A. Essential mixed cryoglobulinemia
B. Rheumatoid arthritis
C. Relapsing polychondritis
D. Wegener‘s granulomatosis
E. Dermatomyositis
►A

Hepatitis C infections can present as urticaria or papable purpura and cryoglobulinemia.

45
All of the following stains can be reactive in this condition except
A. CDXIIIa
B. CD31
C. CD34
D. Ulex europaeus
E. Factor VIII-related antigen
►A

The picture shows Kaposi's scarcoma(KS). It is controversial whether KS represents neoplasia or


hyperplasia; all clinical variants are viewed as a virally induced disease - human herpesvirus 8
(HHV-8) is the suspected agent. Cutaneous lesions present as variably distributed pink patches,
blue-violet to black nodules or plaques, and polyps, depending on clinical variant and stage.
Variable staining can occur with CD31, CD34, Ulex europaeus and factor VIII-related antigen.
CD XIIIa is positive in dermatofibroma.

46
Angiocentric NK/T-cell lymphoma in children may present as:
A. Papular acrodermatitis of childhood
B. Acropustulosis of infancy
C. Childhood dermatomyositis
D. Hydroa vacciniforme
E. En coup de sabre
►D

Hydroa vacciniforme a photodermatitis that typically occurs with sun exposure in the spring has
been reported with NK/T cell lymphomas in childhood.

97
47
Histamine is a biologic amine produced by which of the following cells?
A. Monocytes
B. Eosinophils
C. Basophils
D. Platelets
E. Basophils and Platelets
►E

In the skin, histamine is mainly contained within the granules of dermal mast cells. Histamine is
present in mast cells, basophils, and platelets.

48
The elicitation of nickel contact dermatitis requires signaling by which of the following?
A. TLR2
B. TLR4
C. LFA-1
D. TNF-alpha
E. IL-4
►B

Allergies to nickel (Ni(2+)) are the most frequent cause of contact hypersensitivity (CHS) in
industrialized countries. The efficient development of CHS requires both a T lymphocyte-
specific signal and a proinflammatory signal. Ni(2+) triggers an inflammatory response by
directly activating human Toll-like receptor 4 (TLR4). Studies with mutant TLR4 proteins
revealed that the non-conserved histidines 456 and 458 of human TLR4 are required for
activation by Ni(2+) but not by the natural ligand lipopolysaccharide.

49
Which of the following is the target antigen in pemphigus vulgaris?
A. Desmoglein 3
B. Desmoplakin
C. loricrin
D. Type XVII collagen
E. Desmoglein 1
►A

Desmoglein 3 is the dominant target auto-antigen in pemphigus vulgaris and is in the cadherin
family.

98
50
Which cytokine is primarily responsible for stimulation of neutrophils?
A. IL-1
B. IL-4
C. IL-5
D. IL-6
E. IL-8
►E

IL-8 is primarily responsible for the stimulation of neutrophils. IL-5 stimulates eosinophils. IL-4
stimulates mast cells and IgE isotype switching.

51
Which virus is most closely associated with Kaposi‖s sarcoma in HIV-infected patients?
A. Human herpes virus 2
B. Cytomegalovirus
C. Human herpes virus 6
D. Human herpes virus 8
E. Epstein-Barr virus
►D

HHV-8 has been repeatedly associated with all forms of Kaposi‖s sarcoma.

52
Which component of the T cell receptor is associated with superantigen recognition?
A. D-beta
B. J-alpha
C. J-beta
D. V-alpha
E. V-beta
►E

Superantigens are able to bypass many elements of the normal immune response. They are not
processed by antigen presenting cells. Instead, they bind directly to the MHCII complex and
interact with T cells in a relatively non-specific fashion. Whereas conventional antigens require
recognition in all 5 elements of the T-cell receptor (V-alpha, J-alpha, V-beta, D-beta, J-beta),
superantigens are recognized by V-beta alone.

53
Which of the following cytokines is primarily involved in activating eosinophils?

99
A. IL-5
B. TNF-alpha
C. IL-10
D. IL-2
E. Interferon-gamma
►A

Eosinophils provide many functions of the immune system, including protection against
helminths. They are activated by IL-5.

54
Relapsing polychondritis is an autoimmune disease associated with immunity to which type of
collagen?
A. I
B. II
C. III
D. IV
E. VII
►B

Relapsing polychondritis has been reported to have autoantibodies to type II collage.

55
A deficiency of this complement component may result clinically in susceptibility to pyogenic
infections, glomerulonephritis, and partial lipodystrophy:
A. C1 Esterase Inhibitor
B. C3
C. C4
D. C50
E. Properidin
►B

C3 is the central component of the complement cascade. It plays a key role in the opsonization of
bacteria. An autosomal recessive deficiency of C3 may result in susceptibility to pyogenic
infections, glomerulonephritis, and partial lipodystrophy.

56
Which of the following cytokines, together with IL-4, promotes isotype switching from IgM to
IgE?
A. IL-5

100
B. IL-10
C. IL-13
D. TNF
E. IFN-gamma
►C

IL-5 is an eosinophil growth factor. IL-10 is a general down-regulator of TH1 immunity. IL-13
(along with IL-4) promotes an isotype switch from IgM to IgE. TNF and IFN-gamma are TH1
cytokines. IFN-gamma is secreted by TH1 cells, and is the main macrophage-activating
cytokine.

57
A 26 year-old man presents with a history of recurrent episodes of targetoid, erythematous,
edematous macules, patches, and plaques on the arms, legs, palms, and soles. The most likely
etiologic agent is:
A. Parvovirus B19
B. Herpes simplex virus
C. Coxsackievirus
D. Cytomegalovirus
E. Human immunodeficiency virus
►B

This description of the skin disorder is compatible with either erythema multiforme or erythema
elevatum diutinum, both of which have been reported in association with HSV.

58
Efalizumab is an antibody which is directed against LFA1 on the T-cell, blocking this molecule's
interaction with:
A. CD40
B. B7
C. LFA3
D. ICAM-1
E. P-selectin
►D

ICAM-1 on the antigen presenting cell interacts with LFA1 on the T-cell. By interrupting this
interaction, T-cell activation is blocked by preventing the pairing of LFA1 with ICAM-1. This
prevents costimulatory signals from being given to the T-cell. T-cell ability to traffic into the
skin is also inhibited in the arrest stage of trafficking.

101
59
This skin disease has been shown to be associated with reduced Beta 2 defensin. The diagnosis
is:
A. Atopic dermatitis
B. Psoriasis
C. Lepromatous leprosy
D. Tuberculoid leprosy
E. Subacute cutaneous lupus erythematosus
►A

The answer is atopic dermatitis. Cathelicidin 37 (LL-37) and Human Beta 2 Defensins (HBD-2)
have been reported reduced in atopic dermatitis and normal or elevated in psoriasis. This could
be one factor that predisposes atopics, but not psoriatics to bacterial and viral infections.

60
Which common contact allergen is detected via the dimethylglyoxime test?
A. Benzocaine
B. Chromates
C. Formaldehyde
D. Nickel
E. Rhus
►D

Nickel is found in virtually all metals of common use. It is the most common sensitizer in
women, and is detected by the dimethylglyoxime test.

61
Which component of the classical pathway of complement acts as a neutrophil chemoattractant?
A. C3a
B. C3b
C. C5a
D. C5b
E. C1 INH
►A

The classical pathway of complement is activated by antigen-antibody complexes. C3a is a


neutrophil chemoattractant. C3b is an opsonin. C5a is an anaphylatoxin. C5b forms a part of the
membrane attack complex.

62

102
Which of the following features of IgG is true?
A. IgG is not an opsonizing antibody
B. IgG is the only class of immunoglobulin that can pass through the placenta
C. IgG cannot activate the complement cascade
D. IgG represents 15% of the total protein in serum
E. IgG is the second immunoglobulin synthesized by the fetus
►B

IgG opsonizes bacteria, fixes complement, neutralizes bacterial toxins and viruses, crosses the
placenta. It has the highest serum concentration of all immunoglobulins.

63
A 5 year old patient is seen at a tertiary referral center for evaluation of a complex syndrome.
Ultimately, he is diagnosed with IPEX syndrome (immune dysregulation, polyendocrinopathy,
enteropathy, X-linked syndrome.) What gene is mutated in IPEX syndrome?
A. FOXP3
B. LFA-3
C. TGF-beta
D. B7
E. Mac-1
►A

IPEX syndrome is a rare x-linked disorder. Mutation of FOXP3 results in absence of regulatory
T cells and is the cause of this syndrome, which includes immune dysregulation,
polyendocrinopathy, and enteropathy.

64
Which cytokine is the main macrophage-activating cytokine?
A. TNF
B. IFN-gamma
C. IL-4
D. IL-10
E. lymphotoxin
►B

IFN-gamma is the main macrophage-activating cytokine, and is secreted by TH1 cells.

65
Which of the following is not true about the effects of ultraviolet radiation on the immune
system?

103
A. UV radiation causes an increase in number of Langerhans cells in the epidermis
B. UV radiation causes nuclear DNA damage
C. Effects can be demonstrated by the example of reactivation of latent herpes
simplex infection after sun exposure
D. UV radiation acts to suppress the immune system both locally and
systematically
E. Effects can be demonstrated by the ability of an antigen to induce an allergic
hypersensitivity reaction when applied to skin which has been exposed to low
doses of UV radiation
►A

Effects of UV radiation on the immune system: UV radiation inhibits function of Langerhans


cells (A-False). It causes DNA damage, can reactivate HSV, suppresses immune system, and can
induce allergic hypersensitivity reaction

66
What is the mechanism of action of Ipilimumab?
A. BRAF inhibitor
B. anti-CTLA 4 receptor antibody
C. blocks TNF alpha
D. inhibition of hedgehog signaling pathway
E. inhibits IL2
►B

Ipilimumab is an anti-CTLA 4 receptor antibody. Side effects include rash and coitis.
Vemurafenib is a BRAF inhibitor specifically for patients with V600E mutation. Vismodegib
inhibits the hedgehog signaling pathway.

67
Which of the following dietary supplements may inhibit platelet function?
A. Vitamin A
B. Vitamin C
C. Vitamin D
D. Vitamin E
E. Vitamin K
►D

Supplemental vitamin E can inhibit platelet function and predispose to hemorrhagic stroke. It can
be particularly hazardous in patients with beta-thalassaemia mutations.

104
68
Serum IgA antibodies to tissue transglutaminase occur in:
A. Bullous pemphigoid
B. Linear IgA disease
C. Pemphigus foliaceus
D. Bullous lupus erythematosus
E. Dermatitis herpetiformis
►E

Autoantibodies to tissue transglutaminase are an area of active investigation in both celiac


disease and dermatitis herpetiformis.

69
The endothelial ligand for cutaneous lymphocyte antigen (CLA) is:
A. Intercellular adhesion molecule 1 (ICAM-1)
B. L-selectin
C. E-selectin
D. Vascular cell adhesion molecule (VCAM-1)
E. Leukocyte functional antigen (LFA 3)
►C

Cutaneous lymphocyte antigen (CLA) allow memory T cells to home to the skin, where it binds
to its ligant E-selectin on cutaneous microvessels. Transmigration of memory T cells into the
dermis, however, further requires interaction between leukocyte functional antigen 1 (LFA-1)
and ICAM-1, and B-integrin very late antigen 4 (VLA-4) and VCAM-1. L-selectin is expressed
on post-capillary venules in the lymph nodes and serves as the attachment points for naive T
cells.

70
Which of the following causes a photoallergic contact dermatitis that is exacerbated by UVA
radiation?
A. Ascorbic acid
B. Titanium dioxide
C. Oxybenzone
D. Zinc oxide
E. Dihydroxyacetone
►C

Oxybenzone is the most common sunscreen agent causing photoallergic contact dermatitis.
Patients sensitive should be instructed to avoid sunscreens containing oxybenzone.

105
71
For this patient, you request that the lab perform indirect immunofluorescence using what
substrate?
A. Monkey esophagus
B. Rat bladder
C. Guinea pig esophagus
D. Mouse epithelium
E. Hep-2 cells
►C

This patient has pemphigus folicaceus, and indirect immunofluorescence works best on guinea
pig esophagus.

72
Rituximab works by targeting the CD20 antigen which is predominantly expressed on which of
the following cells?
A. Plasma cells
B. B cells
C. B cells and plasma cells
D. T cells
E. Dendritic cells
►B

Rituximab targets the CD20 antigen which is predominantly expressed on mature B cells but not
plasma cells, T cells, or dendritic cells. Therefore, even when used in antibody-mediated diseases
(such as pemphigus vulgaris), the response to therapy requires a long duration of time as the drug
targets B cells which must then differentiate into plasma cells that ultimately produce the
pathogenic antibodies in their secreted form.

73
Which antibody can bind the FcER1 portion of mast cells, basophils, Langerhans cells, dermal
dendritic cells?
A. IgA
B. IgD
C. IgE
D. IgG
E. IgM
►C

106
IgE is an anaphylactic antibody that is involved in nearly all immediate allergic and anaphylactic
type reactions and commonly seen in elevated levels in patients with atopic dermatitis. Mast
cells, basophils, Langerhans cells, dermal dendritic cells as well as monocytes from atopic
individuals all express high-affinity FcERI receptor which can bind IgE. More recently, it
became clear that can bind monomeric IgE via the high-affinity FcRI IgG is the antibody that can
cross the placenta and the most common antibody found in circulation. IgA is found in mucous
membrane secretions and is able to agglutinate antigens and activate the alternate but not the
classic complement pathway. IgD is not found in circulation other than in hyper-IgD syndrome,
an autosomal recessive disorder caused by mutations in the mevalonate kinase gene. A
significant elevation of serum IgD is seen in 95% of these patients. IgM is the antibody produced
in the early stages of antibody responses. It is a pentamer which can agglutinate antigen and
active the classic complement pathway.

74
What is the best screening test for hereditary angioedema?
A. C1 esterase
B. C4
C. CH50
D. C2
E. C3
►B

C4 is the best screening test for hereditary angioedema (Quinke's edema). There are two types of
hereditary angioedema. In type I, there are low antigenic and functional levels of a NORMAL
C1 esterase inhibitor protein. In type II, there is a normal or elevated level of a
DYSFUNCTIONAL C1 esterase inhibitor. The low C4 level is a result of continuous activation
and consumption of complement components.

75
With which HLA type is psoriasis most definitively linked?
A. HLA-B51
B. HLA-B8
C. HLA-DR4
D. HLA-DR3
E. HLA-Cw6
►E

Psoriasis is linked with HLA-Cw6. Patients with this HLA type have a relative risk of having
psoriasis that is 9-15 times normal.

107
76
The major histocompatibility complex (MHC)consists of a linked set of genes encoding for
MHC Class I, Class II, Class III, and Class IB. Which of the following is/are true:
A. Class II molecules are present on all nucleated cells
B. Class I molecules are present on erythrocytes
C. Class I molecules are expressed on B8 cells, monocytes and dendritic cells
D. The level of Class I and II expression can be modulated by cytokines
E. The MHC region is located on chromosome 17 in humans
►D

The major hisotcompatibility complex (MHC) consists of a linked set of genes encoding for
MHC Class I, Class II, Class III, and Class IB. The level of Class I and II expression can be
modulated by cytokines.

77
All of the following statements regarding major histocompatibility complex molecules are true
EXCEPT:
A. MHC class II molecules bind stably to peptides derived from proteins sythesized
and degraded in the cytosol
B. MHC class I molecules bearing viral peptides are recognized by cytotoxic T-
cells that subsequently kill the infected cell
C. Class I molecules such as HLA-A, B, and C, are present on all nucleated cells
D. The MHC is located on chromosome 6 in humans
E. MHC Class II molecules bearing peptides are recognized by TH1 or TH2 cells.
►A

MHC class I molecules bind stably to peptides derived from proteins sythesized and degraded in
the cytosol, while MHC class II molecules bind stably to peptides derived from proteins
degraded in endocytic vesicles. MHC class I molecules bearing viral peptides are recognized by
cytotoxic T-cells that subsequently kill the infected cell. MHC Class II molecules bearing
peptides trigger helper T-cells and are recognized by TH1 or TH2 cells. Class I molecules such
as HLA-A, B, and C, are present on all nucleated cells, whereas Class II molecules are on B
cells, monocytes, dendritic cells, and are inducible on keratinocytes and endothelial cells. The
MHC is located on chromosome 6 in humans, its polygenic and polymorphic nature contributes
to the ability of the immune system to respond to a multitude of different and rapidly evolving
pathogens.

78
Eight complement receptors have been described. CR1 is the main receptor for which component
of complement?

108
A. C3b
B. Factor B
C. Properdin
D. C1
E. C5b
►A

CR1 (also known as CD35) is the main receptor for C3b. It plays an important role in mediating
clearance of immune complexes, phagocytosis, and immune adherence of antibody-coated
bacteria to erythrocytes.

79
The most likely target for exfoliative toxin A in bullous impetigo is:
A. Desmoglein 3
B. Laminin 5
C. Desmoglein 1
D. Collagen VII
E. Cesmocollin
►C

Desmoglein 1, the dominant target antigen of the autoantibody involved in pemphigus foliaceus,
is also the target of the exfoliative toxin of group II staphylococcus aureus that is usually
responsible for bullous impetigo.

80
Linear IgA disease is most closely associated with which of the following medications?
A. Erythromycin
B. Vancomycin
C. Streptomycin
D. Azithromycin
E. Clarithromycin
►B

Vancomycin is an antibiotic frequently used in subjects allergic to penicillin. It has been reported
to produce subepidermal blistering disease with linear IgA deposits.

81
Which of the following is a criterion for the diagnosis of Behcet'sdisease?
A. Inflammatory bowel disease
B. Uveitis

109
C. Conjunctivitis
D. Nasal septal perforation
E. Lobular panniculitis
►B

Behcet's is a triad that includes oral ulcerations, genital ulcerations and uveitis. Behcet's also
displays arthritis and gastrointestinal disease. Blindness is the most-feared outcome, and relates
to the uveitis.

82
A patient with Stage 3 mycosis fungoides is referred to a tertiary referral center for further
management. Denileukin diftitox is offered as a potential therapy. This medication binds to what
receptor on the T cell?
A. IL-2
B. MHC II
C. CD20
D. TNF-alpha
E. CD19
►A

Denileukin diftitox is a fusion of a fragment of diphtheria toxin and IL-2. It binds to high affinity
IL-2 (CD25) receptors on T cells. As the medication is internalized, the toxin leads to cell death.
This medication has been approved for cutaneous T cell lymphoma.

83
The antibody produced in the early stages of antibody responses is:
A. IgA
B. IgD
C. IgE
D. IgG
E. IgM
►E

IgM is the antibody produced in the early stages of antibody responses. It is a pentamer which
can agglutinate antigen and active the classic complement pathway. IgG is the antibody that can
cross the placenta and the most common antibody found in circulation. IgA is found in mucous
membrane secretions and is able to agglutinate antigens and activate the alternate but not the
classic complement pathway. IgD is not found in circulation other than in hyper-IgD syndrome,
an autosomal recessive disorder caused by mutations in the mevalonate kinase gene. A

110
significant elevation of serum IgD is seen in 95% of these patients. IgE is an anaphylactic
antibody that is involved in nearly all immediate allergic and anaphylactic type reactions.

84
Eosinophils are typically seen in the cutaneous infiltrate of:
A. Krabbe‖s disease
B. Kaposiform hemangioendothelioma
C. Kawasaki‖s disease
D. Kimura‖s disease
E. Ki-1 lymphoma
►D

Eotaxin, which attracts eosinophils, is produced by T cells in Kimura‖s disease.

85
Allergic contact dermatitis is caused by T-cell response to topical exposure to compounds that
form complexes with host proteins (haptens). Which cell is responsible for initial sensitization of
the T-cells?
A. Melanocyte
B. Langerhans Cell
C. B-cell
D. Mast Cell
E. Macrophage
►B

The Langerhans cell phagocytizes the haptens in the epidermis or dermis, begins the process of
maturation and migrates to the regional lymph node, where na‖ve T-cells are educated to expand
to a clone of CLA positive memory T-cells. These cells then can expand and extravasate with
subsequent exposure to the same allergen. Melanocytes, B-cells, Mast cells and macrophages are
not involved in the sensitization process of allergic contact dermatitis.

86
Which of the following diseases does NOT respond with a Th1-type responses?
A. Leishmaniasis which self-resolves
B. Lepromatous Leprosy
C. Tuberculoid Leprosy
D. Allergic contact dermatitis
E. Psoriasis
►B

111
Lepromatous leprosy is Th2 biased. The remaining conditions are Th1 predominant responses.
Leishmaniasis which show strong cell-mediated immunity to the parasite and with self-resolving
lesions are Th1 responses. In indolent/progressive leishmaniasis, a Th2 pattern is predominant.

87
A very common pentadecacatechol sensitizer is found in all of the following plants/plant
components except:
A. Gingko fruit pulp
B. Poison sumac
C. Mango fruit pulp
D. Cashew oil
E. Indian marking nut oleoresin
►C

The sensitizer in rhus dermatitis is a pentadecacatechol found in the oleoresin of anacardaciae


plants, including poison ivy/oak/sumac, mango rinds (but not pulp), cashew nutshells and oil,
Indian marking nut oleoresin, gingko fruit pulp, and Japanese lacquer tree oleoresin.

88
A male infant presents with thrombocytopenia, eczema, and recurrent infections. You suspect
which of the following immunodeficiency disorders?
A. Ataxia telangiectasia
B. Di-George anomaly
C. Hyper-IgM syndrome
D. Leiner‖s disease
E. Wiskott-Aldrich syndrome
►E

Wiskott-Aldrich syndrome is an X-linked recessive disorder that presents with


thrombocytopenia, small defective platelets, eczema, autoimmune disease, infections, and
lymphoreticular malignancy.

89
Which of the following diseases occur with an increased frequency in persons deficient in C2?
A. Psoriasis
B. Dermatitis Herpetiformis
C. Androgenetic Alopecia
D. DLE
E. Leiner's disease
►D

112
Among the complement deficiencies, C2 deficiency is most frequently seen. Most of these
patients are healthy. Diseases that occur with increased frequency in patients with C2 deficiency
are SLE, SLE-like syndrome, frequent infections, anaphylactoid purpura, lethal dermatomyositis,
vasculitis, disseminated cutaneous lupus erythematosus, and cold urticaria.

90
Herpes gestationis is most commonly associated with which HLA?
A. HLA-DR3
B. HLA-B27
C. HLA-B51
D. HLA-DR9
E. HLA-DQ8
►A

HLA-DR3 is the most commonly found HLA association in herpes gestationis. HLA-DR4 is also
found in addition to HLA-DR3 in about 50% of patients. There is nearly 100% incidence of anti-
HLA antibodies patients affected by herpes gestationis.

91
All of the following statements regarding Toll receptors are true EXCEPT:
A. Toll receptors are present on macrophages and dendritic cells
B. Toll 2 receptors are typically activated by lipopolysaccharide
C. Toll 4 receptors are typically activated by gram negative bacteria
D. Nuclear factor kappa B (NFKB) is the final common pathway of toll receptors
E. None of the above (all are true)
►B

Bacteria can induce inflammation through activation of Toll receptors, which are present on a
variety of cutaneous cells including macrophages, dendritic cells, keratinocytes, and mast cells.
Typically, Toll 2 receptors are activated by peptidoglycans and lipoproteins on the surface of
gram positive bacteria and yeast. Toll 4 receptors are activated by lipopolysaccharides on the
surface of gram negative bacteria. Nuclear factor kappa B is a final common pathway of toll
receptors and other immune receptors involved in initiating a variety of proinflammatory
cytokines.

92
Which of the following cytokines has been shown to be critical for epidermal acanthosis in
psoriasis?
A. TNF-alpha

113
B. TGF-beta
C. IL-2
D. FGF
E. IL-22
►E

IL-22 has recently been shown to be critical in causing epidermal acanthosis. Interleukin (IL)-23,
a cytokine involved in the development of IL-17-producing T helper cells (Th17 cells), was
found to have a potential function in the pathogenesis of psoriasis. IL-22 is preferentially
produced by Th17 cells and mediates the acanthosis induced by IL-23. IL-23 can directly induce
the production of IL-22 in human naive T cells. Furthermore, IL-22 mediates IL-23-induced
acanthosis and dermal inflammation through the activation of Stat3.

93
Imiquimod induces which of the following cytokines?
A. Interleukin-10
B. Interferon-alpha
C. Interleukin-2
D. Interleukin-4
E. Interleukin-5
►B

Imiquimod is a topical immunomodulator that increases a variety of cytokines including


interferon-alpha.

94
Which systemic anti-inflammatory agent targets CD2+ activated T cells for apoptosis?
A. Etanercept
B. Infliximab
C. Efalizumab
D. Alefacept
E. None of the above
►D

Alefacept is a soluble form of LFA-3 that blocks the immunologic synapse between CD2 on the
T cell and LFA-3 on the antigen presenting cell. Furthermore, alefacept targets CD2+ activated T
cells for apoptosis.

95
The target antigen of chronic bullous disease of childhood is:

114
A. BPAG 1 9230 kd BPAG)
B. 97 kd LAD-1 (a component of BPAG2)
C. Collagen type VII
D. Alpha 6 beta 4 integrin
E. Plectin
►B

This rare chronic bullous disease of childhood is a subepidermal blistering disease with a
homogeneous IgA deposits at the epidermal basement membrane. This occurs in children usually
less than 5 years of age.

96
Psoriatic arthritis is most commonly associated with which HLA?
A. HLA-B27
B. HLA-Cw6
C. HLA-Aw19
D. HLA-Bw35
E. None of these options are correct
►A

HLA-B27 is associated with an increase in psoriatic arthritis as well as pustular psoriasis and
acrodermatitis continua of Hallopeau.

97
Dermatitis herpetiformis is associated with which HLA type(s)?
A. HLA-B8
B. HLA-DR3
C. HLA-DQw2
D. All of the above
E. None of the above
►D

Dermatitis herpetiformis is associated with HLA-B8, -DR3, and -DQw2.

98
This patient developed an acute vesicular rash after eating a mango. She has returned for a
routine follow-up. She needs to be careful of exposure to:
A. Ginkgo fruit
B. Croton
C. Ragweed

115
D. Tea tree oil
E. All of these answers are correct
►A

Patients allergic to the peel of a mango can also be allergic to other plants/products of the
Anacardiaceae family. Cross-reactions can occur with exposure to any plants of the genus
Toxicodendron, to the oil from the cashew nut shell, to the Brazilian pepper tree, to lacquer from
the Japanese lacquer tree, to ink from the Indian marking nut, and to the fruit pulp of the ginkgo
tree, and others.

99
Psoriasis affects over 2% of the world‖s population and has a strong association with which
HLA class I haplotype?
A. HLA-DR4
B. HLA-DR1
C. HLA-CW6
D. HLA-B27
E. HLA-DQ6
►C

HLA-CW6 has been seen in up to 90% of patients with early onset psoriasis, 50% with late onset
psoriasis and only 7.4% of the general population. HLA-DR1 and DR4 are both related to
Rheumatoid Arthritis and have a 7x increased relative risk (RR) for developing disease. HLA-
B27 is linked to ankylosing spondylitis, postinfection arthridites and Reiter‖s disease, with
increase relative risks of 100x, 10-20x, and 35x respectively.

100
Which cytokine is up-regulated in this geometric, eczematous dermatitis?
A. IL-2
B. IL-4
C. IL-1
D. TNF-Alpha
E. IFN-Gamma
►B

The geometric pattern of erythema suggests "outside job". Allergic contact dermatitis is a
delayed type hypersensitivity reaction mediated by Th2. The cytokines up-regulated in this
process are IL-4, 5, 10.

101

116
Major histocompatibility complex (MHC) Class I molecules:
A. Are inducible on keratinocytes
B. Complexed with antigen trigger cytotoxic T cells
C. Are recognized by receptors on CD4+ T cells
D. Bear peptides derived from pathogens taken up into vesicles
E. All of the above
►B

MHC Class I molecules are present on all nucleated cells. They are recognized by receptors on
surfaces of CD8+ T cells, and, when complexed with antigen, trigger cytotoxic T cells. The other
statements apply to MHC Class II molecules.

102
MHC Class II molecules are present on which of the following cell types:
A. B cells
B. T cells
C. NK cells
D. Mast cells
E. All of the above
►A

MHC Class II molecules are on B cells, monocytes, dendritic cells, and are inducible on
keratinocytes and endothelial cells.

103
The gene for NEMO (NF-kappa bets essential modulator) is mutated in:
A. Papillon-Lefevre syndrome
B. Pachyonychia congenital type II
C. Dyskeratosis congenital
D. Noonan's syndrome
E. Incontinentia pigmenti
►E

NEMO/IKK gamma is an essential component of the nuclear factor kappa B pathway, which is a
common signaling pathway for many cytokines. Mutation in this pathway have been found to
cause incontinentia pigmenti.

104
All of the following statements regarding Natural Killer (NK) cells are true EXCEPT:
A. NK cells have properties of innate and acquired immunity

117
B. NK cells express CD 3 molecules
C. NK cells are large granular lymphocytes
D. NK cells mediate tumor lysis
E. NK cells mediate lysis of viral-infected cells
►B

NK cells do not express CD 3 molecules. The other statements are true. Nk cells express CD2.

105
Which cytokine is upregulated in lesions of tuberculoid leprosy?
A. IL 2
B. IL 4
C. IL 5
D. IL 10
E. None of the answers are correct
►A

Tuberculoid leprosy is characterized by a type I immune related response. Tuberculoid leprosy


has increased amounts of TH1 cytokines including IL-2, interferon gamma, and IL-12. These
lead to a t-cell and macrophage mediated cytotoxic response. IL-4, IL-5, and Il-10 are typically
downregulated in tuberculoid leprosy.

106
Which antibody is most commonly found in circulation of patients with atopic dermatitis?
A. IgA
B. IgD
C. IgE
D. IgG
E. IgM
►C

IgE is an anaphylactic antibody that is involved in nearly all immediate allergic and anaphylactic
type reactions and commonly seen in elevated levels in patients with atopic dermatitis. Mast
cells, basophils, langerhans cells, dermal dendritic cells as well as monocytes from atopic
individuals all express high-affinity FcRI receptor which can bind IgE. More recently, it became
clear that can bind monomeric IgE via the high-affinity FcRI IgG is the antibody that can cross
the placenta and the most common antibody found in circulation. IgA is found in mucous
membrane secretions and is able to agglutinate antigens and activate the alternate but not the
classic complement pathway. IgD is not found in circulation other than in hyper-IgD syndrome,
an autosomal recessive disorder caused by mutations in the mevalonate kinase gene. A

118
significant elevation of serum IgD is seen in 95% of these patients. IgM is the antibody produced
in the early stages of antibody responses. It is a pentamer which can agglutinate antigen and
active the classic complement pathway.

107
All of the following statements regarding mast cells are true EXCEPT:
A. Mast cells reside near small blood vessels
B. Mast cells protect mucosal surfaces against pathogens
C. Mast cells release substances that affect vascular permeability
D. Mast cells have receptors for certain fragments of complement on their surface
E. None of the above (all are true)
►E

Mast cells are the central cell in immediate-type hypersensitivity, and are very important in
cutaneous inflammation. They reside near small blood vessels, and when activated release
substances that affect vascular permeability. They orchestrate allergic responses and protect
mucosal surfaces against pathogens. They have receptors for IgE and certain fragments of
complement on their cell surface.

108
In adult patients with Henoch-Schönlein purpura with IgA vasculitis, which of the following
complications is most likely to occur?
A. Pulmonary hemorrhage
B. Hemorrhagic cystitis
C. Peripheral neuropathy
D. Mesangial nephropathy
E. Facial edema
►D

A urinalysis should be ordered in patients with palpable purpura. Red blood cells and proteinuria
are often seen in Henoch-Schönlein syndrome.

109
Langerhans cells are characterized by the racket-shaped Birbeck granules seen on electron
microscopy. What role do Birbeck granules perform in cells?
A. Endosomal recycling
B. Peptide packaging
C. Cellular trafficking
D. Initiation of apoptosis
E. Scaffolding for RNA synthesis

119
►A

Langerhans cells are important dendritic antigen-presenting cells in the epidermis and mucosa.
The characteristic Birbeck granules are part of the endosomal recycling compartment.

110
The most useful pair of immunohistochemical stains to distinguish between Dermatofibroma
protuberans (DFSP) and a fibrous histiocytoma would be:
A. Synaptophysin, chromogranin
B. CD34, factor XIIIa
C. CD34, CD31
D. CD31, CD3
E. Vimentin, synaptophysin
►B

Dermatofibroma, the more common benign histiocytoma, is usually CD34 negative and factor
XIIIa positive. DFSP is usually CD34 positive and factor XIIIa negative.

111
A patient with chronic atopic dermatitis is found to have elevated IgE levels. Which cytokine
promotes isotype switching to IgE?
A. IL-4
B. IL-6
C. IL-5
D. Interferon-gamma
E. IL-23
►A

In a typical humoral response, isotype switching occurs subsequent to exposure to antigen.


Switching is regulated by T cell derived cytokines. IgG is promoted by IL-4, IL-6, IL-2 and IFN-
gamma. IgA is promoted by IL-5 and TGF-beta. IgE is promoted by IL-4.

112
Which of the following best describes the mechanism of action for infliximab?
A. Inhibition of calcineurin-mediated dephosphorylation of transcription factors
B. Inhibition of retinoic acid 4-hydroxylase activity
C. Inhibition of tumor necrosis factor alpha activity
D. Selective elimination of activated T cells via binding to high affinity IL-2
receptor
E. Selective T-cell up-regulation of IL-4 and IL-5 production

120
►C

Infliximab is a monoclonal antibody given intravenously that binds to membrane-bound and


soluble TNF-alpha.

113
The best screen for classical pathway complement deficiency or dysfunction is:
A. CD4/CD8 ratio
B. CH50
C. C2 esterase levels
D. Total C4 levels
E. Serum electophoresis
►B

The appropriate screening test for a complement deficiency is the CH50. The CH50 will detect
deficiencies in the classical pathway and membrane attack pathways.

114
Which of the following immunoglobulin crosses the placenta?
A. IgA
B. IgD
C. IgE
D. IgG
E. IgM
►D

IgG is the antibody that can cross the placenta and the most common antibody found in
circulation. IgA is found in mucous membrane secretions and is able to agglutinate antigens and
activate the alternate but not the classic complement pathway. IgD is not found in circulation
other than in hyper-IgD syndrome, an autosomal recessive disorder caused by mutations in the
mevalonate kinase gene. A significant elevation of serum IgD is seen in 95% of these patients.
IgE is an anaphylactic antibody that is involved in nearly all immediate allergic and anaphylactic
type reactions. IgM is the antibody produced in the early stages of antibody responses. It is a
pentamer which can agglutinate antigen and active the classic complement pathway.

115
Imiquimod has been used for many dermatologic conditions. It works by binding to toll-like
receptor 7. What is the native ligand of TLR-7?
A. Single-stranded RNA
B. TNF-alpha

121
C. IL-10
D. IL-23
E. Interferon-alph
►A

Toll-like receptors are pattern recognition receptors involved in innate immunity. They are most
highly expressed on monocytes, dendritic cells, and B cells. Imiquimod works through binding to
toll-like receptor 7. The native ligand for TLR7 is single-stranded RNA.

116
Which of the following HLA alleles is most strongly associated with Behcet‘s disease?
A. HLA-B27
B. HLA-B51
C. HLA-DQw2
D. HLA-DR1
E. HLA-DR4
►B

HLA-B51 has been reported with increased relative risks in European, Asian and Middle Eastern
populations.

117
All of the following statements regarding neutrophils are true EXCEPT:
A. Neutrophils' major function is antigen presentation
B. Neutrophils have receptors for IgG and complement
C. Neutrophils are granulocytes
D. Neutrophils are the most abundant leukocytes
E. Neutrophils play an important role in wound healing
►A

Neutrophils' major function is phagocytosis. The other statements are correct.

118
Of the following complement components, the most powerful neutrophil chemoattractant is:
A. C3
B. C5a
C. C3a
D. C5b, C6, C7, C8, C9
E. C4a
►B

122
The complement system plays an important role in innate immunity. Of this group, C5a is the
most powerful neutrophil chemoattractant. C3 represents the endpoint for the classic, alternative
and lectin pathways, and results in the generation of immunologically active substances. C3a and
C4a cause mast cell degranulation. Assembly of C5b, C6, C7, C8, C9 components results in the
membrane attack complex (MAC), which perforates cell membranes causing death by osmotic
lysis.

119
Patients with a type I reaction to latex may have cross-reactions with which of the following
foods?
A. Avocado
B. Horseradish
C. Cashews
D. Parsnips
E. Garlic
►A

Latex allergy is a TH2 form of contact sensitivity that is IGE mediated. Using the old Gel and
Coombs classification, it was classified as type I hypersensitivity.

120
All of the following statements are true regarding cells of the innate immune system EXCEPT:
A. Neutrophils have receptors for IgG
B. Basophils are a type of granulocyte, as are neutrophils
C. IL-5 downregulates the functions of eosinophils
D. Langerhans cells are poorly phagocytic
E. Langerhans cells express CD1 on their surface
►C

Neutrophils, eosinophils, and basophils are collectively known as granulocytes. Neutrophils have
receptors for IgG and complement. IL-5 enhances all functions of eosinophils. Langerhans cells
are dendritic cells found in high concentration in epithelial surfaces and some areas of lymph
nodes and spleen. They have a high density of class II MHC molecules and express CD1 on their
surface. They are poorly phagocytic.

121
This lesion is hypoesthetic and is associated with which of the following:
A. Gamma interferon
B. Interleukin 2

123
C. Interleukin 12
D. All of these answers are correct
E. None of these answers are correct
►D

The lesion is high immune or tuberculoid leprosy, which is associated with TH1 cytokines
including IFN-gamma, IL2, IL12, IL15, IL18 and IL23.

122
A patient with severe chronic psoriasis has failed multiple systemic therapies. Alefacept is
initiated for a three month trial. This medication induces apoptosis of T cells by binding to what
T cell receptor molecule?
A. CD2
B. CD45RO
C. LFA-3
D. CTLA-4
E. p40
►A

Alefacept is a human fusion protein of LFA-3 with Fc protion of IgG1. It binds CD2 on
CD45RO memory effector T cells, thus blocking the 2nd signal of T cell activation and leading
to apoptosis. A provider must monitor the CD4 T cell count while a patient is on this medication.

123
Purpuric contact dermatitis is most likely to be associated with:
A. Nickel
B. Formaldehyde
C. P-phenylenediamine
D. Propylene glycol
E. Sorbic acid
►B

Textile dermatitis has been reported to be associated with purpuric contact dermatitis.

124
Which of the following is true of Langerhan cells?
A. They are the primary antigen presenting cell in the epidermis
B. They are primarily involved in the innate immune response
C. They do not express the CD1a marker
D. They contain intranuclear birbeck granules

124
E. They are increased on the palms, soles, genitalia, and buccal mucosa
►A

Langerhans' cell function primarily in the afferent limb of the immune response by providing for
the recognition, uptake, processing, and presentation of antigens to sensitized T lymphocytes.
They are normally found scattered among keratinocytes of the stratum spinosum, or prickle cell
layer. of the epidermis. They can be stained with CD1a or S-100. They originate in the bone
marrow. They contain intracytoplasmic Birbeck granules.

125
Which cytokine is chemotactic for neutrophils?
A. IL-2
B. IL-3
C. IL-5
D. IL-6
E. IL-8
►E

IL-8 is chemotactic for neutrophils. The other cytokines elicit other types of immune cells, IL-2
(T-ells), IL-3 (mast cells), IL-5 (eosinophils), IL-6 (plasma cells).

126
This patient had antibodies to 180 kd antigen. The likely diagnosis is:
A. Bullous pemphigoid
B. Pemphigus vulgaris
C. Pemphigus foliaceous
D. Erythema multiforme
E. Bullous lichen planus
►A

The answer is bullous pemphigoid, which shows a characteristic antibasement membrane


antibody to Bp180.

127
The main cytokine secreted by Th1 CD8+ effector T-cells is:
A. IL-1
B. IL-2
C. IFN-gamma
D. IL-4
E. IL-5

125
►C

Interferon-gamma is the main cytokine secreted by Th1 CD8+ effector T-cells and is active in
blocking viral replication. IL-2 is also released in Th1 reactions and causes T cell proliferation
and differentiation, macrophage, NK, & Tc cell activation. It also acts as a Th1 autocrine growth
factor. IL-1 is a pro-inflammatory cytokine and a pyrogen. IL-4 is a B-cell growth factor and
active in signaling isotope switching from IgM to IgE. IL-5 is an eosinophil growth factor. Both
IL-4 and -5 are secreted by Th2 T-cells.

128
A patient had antibodies to desmoglein 3, but no antibodies to desmoglein 1 or desmplakin. The
likely diagnosis is:
A. Pemphigus vulgaris
B. Pemphigus foliaceous
C. Bullous pemphigoid
D. Dermatitis herpetiformis
E. Erythema multiforme
►A

The answer is pemphigus vulgaris, which typically has autoantibodies to desmoglein 3. While
mixed forms of pemphigus can occur, the histology in this case confirmed pemphigus vulgaris.

129
Which systemic anti-inflammatory agent specifically blocks the ability of T cells to leave the
vasculature and enter the skin?
A. Etanercept
B. Infliximab
C. Efalizumab
D. Alefacept
E. None of the above
►C

Efalizumab is a humanized monoclonal antibody to CD11a that blocks the immunologic synapse
between LFA-1 on the T-cell and ICAM-1 on the antigen presenting cell. By targeting LFA-1
efalizumab specifically blocks the ability of T cells to leave the vasculature and enter the skin.

130
All of the following statements regarding Langerhans cells are true EXCEPT:
A. They are highly phagocytic
B. They express CD1 on their surface

126
C. They are found in some areas of lymph nodes and spleen
D. They have a high density of Class II molecules on their surface
E. None of these answers are correct
►A

Langerhans cells are dendritic cells found in high concentrations in epithelial surfaces and some
areas of lymph nodes and spleen. They express CD1 on their surface and have a high density of
Class II MHC molecules. They are poorly phagocytic.

131
An 8 month-old baby with diffuse purpura is admitted to the hospital for her third episode of
bacterial meningitis. Which component of her immune system is impaired?
A. CD4 + T cells
B. Natural killer cell activation
C. CD8 + T cells
D. Complement activation
E. Antibody production
►D

Predisposition to sporadic and occasionally recurrent meningococcal disease occurs in patients


with congenital or acquired complement deficiencies, particularly late acting components C5-9

132
Which of the following is the most abundant leukocyte?
A. Neutrophil
B. Eosinophil
C. Mast cell
D. Lymphocyte
E. Monocyte
►A

Neutrophils are the most abundant leukocyte. The major function of neutrophils is phagocytosis.

133
This disease is most associated with:
A. Gamma interferon
B. IL-4
C. IL-5
D. IL-13
E. None of these answers are correct

127
►A

The answer is gamma interferon, which is the TH1 cytokine associated with a hyperproliferative
epidermis and with micaceous scale as seen in psoriasis.

134
Which of the following suture materials induces the least inflammation?
A. Surgical gut
B. Polyglycolic acid (Dexon)
C. Polyglycan 910 (Vicryl)
D. Polypropylene (Prolene)
E. Silk
►D

Prolene is a nonabsorbable suture material that evokes only minimal inflammation.

135
Anti-centromeric antibodies are associated with which rheumatologic disease?
A. CREST
B. Mixed connective tissue disease
C. SLE
D. Dermatomyositis/polymyositis
E. Progressive systemic sclerosis
►A

Anti-centromeric antibodies are associated with CREST syndrome. Anti-RNP antibodies are
associated with MCTD, anti-dsDNA, ssDNA, and Sm associated with SLE, anti-Jo-1 associated
with polymositis, and anti-Scl 70 associated with PSS.

136
Which cytokine is predominantly responsible for the Jarish-Herxheimer reaction?
A. TNF-alpha
B. IL-2
C. IL-5
D. IL-6
E. IL-12
►A

The Jarish-Herxheimer reaction may occur after the initiation of treatment of many systemic
infections including syphillis. It is an indirect drug induced effect caused by bacterial endotoxins

128
or microbial antigens liberated by the destruction of micro-organisms. Clinical manifestations
may include fever, lymphadenopathy, arthralgias, and exacerbation of pre-existing cutaneous
lesions. Tumor necrosis factor-alpha is the primary cytokine that mediates this reaction.

137
IL-23 plays a critical role in the pathogenesis of psoriasis. Which of the following cytokines is
critical for IL-23-mediated epidermal hyperplasia in psoriasis?
A. IL-2
B. IL-6
C. IL-12
D. TNF-alpha
E. IFN-gamma
►B

IL-23 facilitates the differentiation and induces complete maturation of Th17 cells. Lesional
psoriatic skin has increased levels of IL-23. IL-6 is essential for development of the IL-23-
elicited responses and is required for the development of epidermlal hyerplasia.

138
Calcipotriene-induced improvement in psoriasis is associated with increased lesional levels of
which cytokine?
A. Interleukin-2
B. Interleukin-8
C. Tumor necrosis factor
D. Interleukin-10
E. Interluekin-12
►D

Psoriasis is generally described as a TH1 autoimmune disease where IL-12/IFN-gamma pathway


is dominant. IL-10 is the prototype of TH2 and calcipotriene application results in increased
levels of IL-10, thus decreasing TH1 disease.

139
Which T-cell subset is commonly found in Sezary syndrome?
A. CD4+/CD7+
B. CD8+/CD7-
C. CD4-/CD7+
D. CD4+/CD7-
E. CD8+/CD7+
►D

129
Sezary syndrome is the leukemia phase of mycosis fungoides (cutaneous T-cell lymphoma) and
is usually a TH2-biased CD4+ leukemia.

140
T-cell anergy occurs if:
A. Stimulation by a MHC Class III molecule is involved
B. MHC/TCR engagement occurs without costimulatory molecules
C. FasL is bound on the T-cell
D. MHC Class I or II is bound in the presence of IL-2
E. A HLA-DM facilitator is not involved with the binding
►B

MHC/TCR engagement occurs without costimulatory molecules. MHC Class III molecules are
not involved in this process. The MHC III region encodes for soluble proteins of the complement
cascade and the tumor necrosis family. Fas-FasL interactions promote apoptosis on the target
cell, not stimulation. IL-2 is a stimulatory molecule, produced by Th1 T-cells. It does not
produce anergy. HLA-DM facilitates loading of proteins processed by antigen presenting cells
into the MHC class II molecule before being brought to the cell surface.

141
Which of the following B cell receptors is involved in immunoglobulin isotype switching?
A. CD40
B. CD19
C. CD20
D. CD154
E. CD22
►A

CD40 activation on B cells by CD40-ligand (CD154) on T cells induces isotype switching from
an IgM to IgG response. Defects in the expression of CD40-ligand result in an
immunodeficiency state (hyper-IgM syndrome) characterized by low levels of IgG, IgA and IgE,
but elevated IgM. CD19, CD20, CD22 are pan-B cell markers.

142
Toll-like receptors (TLRs) have been found to play an important role in innate immunity. This
has been utilized in the development of medications frequently used in dermatology. The
mechanism of what medication involves activation of TLR7.
A. Clobetasol
B. Tacrolimus

130
C. Cyclosporine
D. 5-Fluorouracil (5-FU)
E. Imiquimod
►E

TLRs recognize pathogen-associated molecular patterns (PAMPs) present in a variety of


pathogens and activate signaling pathways involved in innate immunity, as well as augmenting
adaptive immunity. The mechanism of action of imiquimod involves activation of these
pathways via TLR7. None of the other medications listed above involve TLRs as their main
mechanism of action.

143
Chronic idiopathic urticaria is associated with which HLA type(s)?
A. HLA-DR4
B. HLA-DRB4
C. HLA-DQ8
D. all of the above
E. none of the above
►D

Chronic idiopathic urticaria is associated with HLA -DR4, -DRB4, and -DQ8.

144
This patient recently developed this rash. You decide to patch test her, but in the meanwhile you
tell her to avoid:
A. Chamomile
B. Primin
C. Abietic acid
D. Benzocaine
E. Cinnamon
►E

This patient likely has a fragrance allergy. Patients allergic to fragrances also need to avoid
certain spices like cinnamon.

145
IL-23 plays a critical role in the pathogenesis of psoriasis. Which of the following cytokines is
critical for IL-23-mediated epidermal hyperplasia in psoriasis?
A. IL-2
B. IL-12

131
C. IL-17A
D. TNF-alpha
E. IFN-gamma
►C

IL-23 and Th17 cells producing IL-17A and IL-22 are found in excess in skin affected by
psoriasis. IL-6, IL-22, and IL-17A have all been shown to be critical in mediating epidermal
hyperplasia in psoriasis in response to IL-23.

146
Sensation is intact in this lesion, but a Fite stain is positive. This lesions is associated with which
of the following:
A. IL-4
B. IL-5
C. IL-10
D. All of these answers are correct
E. None of these answers are correct
►D

The lesion is low immune or lepromatous leprosy, which is associated with TH2 cytokines
including IL-4, IL-5, IL-10, and IL-13.

147
The putative mechanism of action of topical macrolide immunomodulators is inhibition of:
A. Lymphokine production
B. Prostaglandin secretion
C. Antigen presentation
D. Neutrophil migration
E. Lymphocyte migration
►A

Tacrolimus and pimecrolimus are topical macrolide immunomodulators that inhibit lymphokine
or cytokine production via binding to macrophilin. This complex inhibits calcineurin, a
phosphatase involved in the activation of NF-AT. This suppresses the production of IL-2 and
IFN-gamma (TH1 cytokines) as well as IL-4, 5 and 13 (TH2 cytokines). In addition, they
decrease the expression of IgE receptors on Langerhans cells and reduce mast cell degranulation.

148
Which Th2 cytokine is involved in the proliferation and differentiation of B cells?
A. IL-4

132
B. IL-5
C. IL-6
D. IL-10
E. IL-13
►C

The Th2 response is important in promoting humoral immunity. IL-4 promoted Th2
differentiation, isotype switching and inhibition of macrophages. IL-5 activates eosinophils. IL-6
assists in the proliferation and differentiation of B cells. IL-10 decreases expression of MHC II.
IL-13 is related to IL-4, and is implicated in allergic inflammation.

149
The Epstein-Barr virus utilizes which cell surface receptor to gain access to cells?
A. CR2
B. B7
C. T-cell receptor
D. TLR7
E. MHC II
►A

EBV utilizes the CR2 receptor for cell entry and infection. the CR2 receptor (also known as
CD21) presents antigen to B cells and is a co-receptor for B cell signalling.

150
Which cell surface component of T cells is part of the \"signal 2\" which in addition to T cell
receptor binding leads to T cell activation?
A. CD28
B. B7
C. CD80
D. TNF-alpha receptor
E. CD1a
►A

Activation of T cells requires two signals. The first is T cell receptor binding to MHC molecule
on the antigen presenting cell. THe second signal involves CD28 on T cell interacting with B7
molecules on the antigen presenting cells. CD80 is expressed on Langerhans cells after
activation. CD1a is a Langerhans cell surface marker.

151
What cytokine is most critical for the development and maturation of eosinophils.

133
A. Interleukin-2
B. Interleukin-4
C. Interleukin-5
D. Interferon-alpha
E. Interferon-gamma
►C

IL-5 is critical in the development and maturation of eosinophils. IL-2 is an autocrine factor for
activated T cells. IL-4 is involved in B-cell class switching and Th2 differentiation. IFN-alpha is
a major part of the antiviral response. IFN-gamma is involved in macrophage activation and
specific isotype switching.

152
Mononuclear phagocytes residing in tissues:
A. Are called macrophages
B. Phagocytose foreign antigens and degrade them into peptides
C. Present antigen to T-cells
D. Produce cytokines, which recruit other inflammatory cells
E. All of the above
►E

Mononuclear phagocytes are components of the innate immune system. Once in tissues they are
called macrophages. All of the above are correct.

153
The classical complement pathway:
A. Can be activated in the absence of antibody
B. Can be activated by IgG4
C. Can be activated by IgM
D. Includes C3 and factor B
E. Does not cause membrane damage
►C

The classical complement pathway is activated by antigen-antibody complexes, while the


alternate complement pathway can be activated in the absence of antibody. IgM and IgG are
capable of activating the classical pathway, but IgG4 does not activate complement. The proteins
of the classical pathway are C1, C2, C3, and C4. Factor B, Factor D, properdin, and C3 are
proteins in the alternate pathway. Biologic activities of complement include opsonization,
chemotaxis, anaphylaxis, immune complex solubilization, membrage damage, and B cell
activation.

134
154
Which cytokine is responsible for activating natural killer cells?
A. Interleukin 4
B. Interleukin 2
C. Interleukin 5
D. Interferon-gamma
E. Tumor necrosis factor-alpha
►B

Activation of natural killer cells occurs via interleukin 2 (IL-2). IL-2 is a key component of the
Th1 (cell-mediated) immunity. It acts to promote growth, proliferation and activation of T cells,
B cells and natural killer cells.

155
Proteins in the alternate complement pathway include:
A. Factor B
B. properdin
C. C3
D. Factor B and properdin
E. Factor B, properdin and C3
►E

Proteins in the alternate complement pathway are Factor B, Factor D, properdin, and C3. The
alternate complement pathway is active against pathogenic microorganisms, virus-infected cells,
and neoplastic cells. The proteins of the classical pathway are C1, C2, C3, and C4.

156
All the following conditions exhibit a T-helper cell 1 (Th1) cytokine secretion profile except:
A. Psoriasis
B. Systemic lupus erythematosus
C. Granulomatous leprosy
D. Rheumatoid arthritis
E. Multiple sclerosis
►B

Systemic lupus erythematous is associated with a Th2 cytokine profile, whereas the other
conditions are associated with a Th1 cytokine profile. Th2 cells are typified by secretion of IL-4,
IL-5, IL-6, IL-9, IL-10 and IL-13, whereas Th1 cells are typified by secretion of IFN-gamma,
TNF-beta, and IL-2.

135
157
A 44 year old female complains of intra-oral burning lesions. Examination reveals erythematous
plaques with white striae on the buccal mucosa consistent with oral lichen planus. No cutaneous
lesions are seen. Which of the following HLA types is associated with oral lichen planus?
A. B8
B. Cw6
C. Bw35
D. B27
E. B51
►A

Oral lichen planus is a relatively common condition that can cause significant morbidity. Oral
lichen planus is associated with HLA-B8. HLA-Cw6 is associated with psoriasis, Bw35 with
cutaneous lichen planus, B27 with psoriatic arthritis, and B51 with Behcet\'s disease.

158
Naive T cells express which of the following surface molecules:
A. CD19
B. CD20
C. CD79
D. CD45RO
E. CD45RA
►E

Naive T cells express CD45RA whereas memory T cells express CD45RO. CD19, 20, and 79 are
B cell surface markers.

159
The T cell surface receptor CD2 binds to what partner on endothelial cells to assist with
adhesion?
A. LFA-3
B. B7
C. E-cadherin
D. CD40
E. ICAM-1
►A

CD2 molecules are T cell surface receptors that bind to LFA-3 on endothelial cells and antigen
presenting cells to participate in adhesion and activation.

136
160
Pemphigus is associated with which HLA type(s)?
A. HLA-DRw6
B. HLA-B8
C. HLA-B51
D. All of these answers are correct
E. None of these answers are correct
►A

Pemphigus is associated with HLA-DR4 or DRw6.

161
The human major histocompatibility complex (MHC) is located on chromosome:
A. 2
B. 6
C. 9
D. 11
E. 17
►B

The MHC is highly polygenic and polymorphic, and this contributes to the ability of of the
immune system to respond to a multitude of different and rapidly evolving pathogens. The MHC
is located on chromosome 6 in humans.

137
Chapter -3-
Genodermatoses
1
Patients with Darier‖s disease are at increased risk for:
A. Kaposi‖s varicelliform eruption
B. Melanoma
C. Decreased life span
D. Basal cell carcinoma
E. Lipid abnormalities
►A

Kaposi‖s varicelliform eruption is the condition in which viral infection occurs in a patient with
a pre-existing chronic dermatitis. Darier‖s disease is an autosomal dominant genodermatosis
caused by a mutation in ATP2A2 which encodes SERCA2. Cutaneous manifestations of warty,
hyperkeratotic papules in a seborrheic dermatitis, which may be infected with HSV or bacteria.

2
A patient with coarse facies, broad nasal bridge, and extensive eczema might be expected to have
which abnormal laboratory value?
A. Hypertriglyceridemia
B. High Copper levels
C. Low Zinc levels
D. High IgE levels
E. Anemia
►D

Patients‖ with Hyper IgE Syndrome (Job syndrome) have impaired regulation of the IgE
function and increased susceptibility to infections. In addition to recurrent cutaneous infections
including cold abscesses, patients have widespread eczematous dermatitis, recurrent
sinopulmonary infections and typically have coarse facies with broad nasal bridge and a
prominent nose.

3
Refsum syndrome is due to a deficiency in phytanyl coenzyme A hydroxylase. Treatment for this
condition is:
A. Diet low in green vegetables, dairy and ruminant fats
B. Diet high in green vegetables, dairy and ruminant fats
C. Enzyme replacement
D. No treatment is available at this time

138
E. Avoid phenylalanine
►A

Treatment is with a diet low in green vegetables, dairy and ruminant fats is the treatment of
choice for Refsum syndrome. Avoidance of specific amino acids is not helpful.

4
A 16 year-old girl presents with a family history of Gardner syndrome. Her mother is very
concerned that her daughter may have the syndrome as it runs in her family and she has many
skin complaints. Gardner syndrome has been linked to defects in beta-catenin mediated
transcription. Which of the following ocular finding is diagnostic for Gardner syndrome?
A. Lisch nodules
B. Lester iris
C. Congenital Hypertrophy of the Retinal Pigment Epithelium
D. Angioid streaks
E. Retinal detachment
►C

CHRPE (Congenital Hypertrophy of the Retinal Pigment Epithelium) is the characteristic eye
finding for Gardner syndrome. Lisch nodules are seen in Neurofibromatosis I, Lester iris in Nail-
Patella syndrome and angioid streaks are present in Pseudoxanthoma elasticum. Retinal
detachment is not part of Gardner syndrome.

5
Which of the following eye findings is caused by the rupture of Bruch's membrane?
A. Angioid streaks
B. Blue sclerae
C. Retinal detachment
D. Ruptured globe
E. Keratoconus
►A

The rupture of Bruch's membrane causes angioid streaks in pseudoxanthoma elasticum. Bruch's
membrane is the innermost layer of choroid with a central layer of elastic fibers. The other
findings are found in Ehlers-Danlos syndrome and are not related to Bruch's membrane.

6
Which genetic defect could explain cutaneous findings in addition to abnormal immunoglobulin
levels, recurrent respiratory infections, hypogonadism, and an increased risk of leukemia and
lymphoma?

139
A. RecQL3
B. ERCC6
C. WAS gene
D. NADPH oxidase
E. Adenosine deaminase
►A

Bloom's syndrome is an autosomal recessive disorder caused by mutations in the RecQL3 gene
encoding a DNA helicase. Clinically, individuals with Bloom's syndrome have a
photodistributed erythema with telangectasia on the malar eminences. The may also have
decreased IgM and IgA levels, hypogonadism, and an increased risk for leukemia and
lymphoma.

7
A patient with Carney complex has distal arthrogryposis and is autosomal dominant. The heart
findings are:
A. Atrial myxomas
B. Ventricular hypertrophy
C. Dilated blood vessels
D. Third degree block
E. Second degree block
►A

A patient with Carney complex has atrial myxomas. Early diagnosis can save lives. It is a
mutation in the PRKAR1A gene.

8
Which of the following diseases is seen only in females and mosaic males?
A. Griscelli syndrome
B. Piebaldism
C. Hypomelanosis of Ito
D. Incontinentia pigmenti
E. Carney complex
►D

Incontinentia pigmenti (Bloch-Sulzberger syndrome) is an X-linked dominant disease that is


lethal in males unless they are a mosaic. There are four stages: vesicular, verrucous,
hyperpigmented and hypopigmented. Also seen are peg and conical teeth, eye abnormalities,
CNS defects, and alopecia. This condition is caused by mutations in the NEMO gene.

140
9
What is the most likely gene mutation in this individual who has migratory patches and fixed
plaques as depicted in this picture?
A. Connexin 26
B. Connexin 30.3 and 31
C. Calcium ATPase 2C1
D. Loricrin
E. Keratins 1 and 10
►B

Erythrokeratodermis variabilis is an autosomal dominant disease characterized by transient


patches of geographic erythema and fixed hyperkeratotic plaques. The disease is due to gene
defects in connexin 30.3 and 31.

10
A 27 year old presents with some mildly itchy spots on his posterior neck. Biopsy reveals
elastosis perforans serpiginosa. Other notable exam findings include tall stature, joint laxity,
pectus excavatum, and a high-arched palate. Which gene is most likely mutated in this patient?
A. Fibrillin 1
B. Fibronectin
C. Collagen 5
D. ABCC6
E. Fibulin 4
►A

Marfan syndrome is an autosomal dominant disorder characterized by tall stature,


arachnodactyly, pectus excavatum, high-arched palate, joint laxity, ectopia lentis, aortic
dilatation, striae, and elastosis perforans serpiginosa. The syndrome is caused by a defect in
fibrillin 1 or 2. Fibronectin is defected in Type 10 Ehlers-Danlos syndrome. Collagen 5 defects
can cause Ehlers-Danlos types 1 or 2. ABCC6 is mutated in pseudoxanthoma elasticum. Cutis
laxa is caused by defects in fibulin 4.

11
A patient has lusterless hair, diffuse hypopigmentation, doughy skin with severe psychomotor
retardation has a defect in the:
A. ATP7A gene
B. GJB2 gene
C. Connexin 26
D. c-kit gene
E. PAX1

141
►A

This patient has Menkes Disease that is an X-linked recessive disorder caused by a mutation of
the ATP7A gene. This gene encodes copper-transporting ATPase. The disease is lethal in males.
Girls develop variably depigmented lusterless hair. Polaroscopy reveals monilethrix, pili torti,
trichorrhexis nodosa. In addition, there is diffuse cutaneous hypopigmentation, doughy skin and
an exaggerated cupid bow.

12
Findings of eyelid papules (string of pearls) and a hoarse cry in infants is characteristic of which
of the following syndromes?
A. Lipoid proteinosis
B. Amyloidosis
C. Pseudoxanthoma elasticum
D. Disseminated xanthomas
E. None of these answers are correct
►A

Findings of the eyelid string of pearls and a hoarse cry during the first years of life (due to vocal
cord infiltration) is characteristic of Lipoid Proteinosis (Urbach-Wiethe disease or Hyalinosis
cutis et mucosae). It is an autosomal recessive condition with mutations in the extracellular
matrix protein 1 gene. Other findings include calcifications of the temporal lobe and
hippocampus, hairloss, atrophic scars and waxy papules on the face, verrucous nodules and a
thick tongue. The other conditions could be considered on the differential for Lipoid Proteinosis,
but do not have the findings described above.

13
What phenotype results from a low activity of double stranded RNA adenosine deaminase?
A. Waardenberg's syndrome type 2
B. Piebaldism
C. Tietz syndrome
D. dyschromatosis symmetrica hereditaria
E. oculocutaneous albinism type 4
►D

Dyschromatosis symmetrica hereditaria (or acropigmentation symmetrica of Dohi) is an


autosomal dominant disease with hypo and hyper pigmented macules and patches on the dorsal
hands and feet associated with a low activity of double stranded RNA adenosine deaminase.

14

142
What is the mode of transmission for lamellar ichthyosis?
A. Autosomal dominant
B. Autosomal recessive
C. X-linked dominant
D. X-linked recessive
E. Sporadic
►B

Lamellar ichthyosis which is characterized by collodian membrane in newborns and platelike


scale in children and adults is an autosomal recessive syndrome. The gene defect is
transglutaminase 1 (TGM1).

15
A triangular-shaped lunula is a characteristic finding in which disease?
A. Darier's disease
B. Dyskeratosis congenita
C. Epidermal nevus syndrome
D. Incontinentia pigmenti
E. Nail-patella syndrome
►E

Nail-patella syndrome, also called hereditary osteo-onychodysplasia is a rare autosomal


dominant condition caused by a defect in the LMX1B gene. It is characterized by triangular
lunulae, palmoplantar hyperhidrosis, renal dysplasia, glomerulonephritis, and hyperpigmentation
of the papillary margin of the iris, an ophthalmologic finding also known as Lester iris. Other
nail findings include micronychia with hemionychia, anonychia, and longitudinal fissures. Bony
findings include absent or hypoplastic patella, posterior iliac horns, radial head subluxation,
thickened scapulae, and scoliosis. Nail findings in Darier's disease include red and white
longitudinal bands, subungual hyperkeratosis and V-shaped nicking of the distal nail plate.
Dystrophic nails with longitudinal ridges, pterygium, and atrophic or absent nails can be found in
dyskeratosis congenita. Dystrophic changes of the nails can be seen in approximately 5-10% of
patients with incontinentia pigmenti.

16
Adenosine deaminase deficiency is associated with which of the following disorders?
A. Wiskott Aldrich syndrome
B. Gout
C. Job syndrome
D. Xeroderma pigmentosum
E. Severe combined immunodeficiency

143
►E

Adenosine deaminase deficiency is associated with severe combined immunodeficiency. The


most common inheritance is x-linked recessive. It is a mixed group of disorders all sharing
defects in cell-mediated and humoral immunity. Skin findings include: candidal infections,
mucocutaneous, bacterial pyodermas, seborrheic-like dermatitis/lichen planus-like
sclerodermatous changes, aplastic thymus and pneumonias. The other listed syndromes are not
associated with adenosine deaminase deficiency.

17
A teenage female presents with the complaint of "nail fungus". On exam, she has triangular
lunulae, palmoplantar hyperhidrosis, micronychia and an absent patella. Which of the following
gene defects is most likely in this patient?
A. LMX1B
B. COL5A1
C. EEC1
D. LKB1
E. NEMO
►A

This patient has nail-patella syndrome. The gene defect is an autosomally transmitted defect in
LMX1B. This gene is involved in dorsal/ventral limb patterning and is in close proximit to the
COL5A1 gene. EEC1 is defective in ectrodactyly-ectodermal dysplasia-cleft lip/palate syndrome
and LKB1 in Peutz Jeghers syndrome.

18
Pseudoxanthoma elasticum is caused by mutations in which of the following genes?
A. Fibrillin 1
B. Elastin gene
C. Lysyl oxidase
D. ABCC6 gene
E. Collagen 5
►D

Pseudoxanthoma elasticum is caused by mutations in the ABCC6 gene, which is an ATP-using


cell transporter. Elastin and lysyl oxidase mutations cause cutis laxa, fibrillin 1 mutations cause
Marfan syndrome, and collagen 5 mutations cause Ehlers-Danlos syndrome.

19
Which of the following is NOT a characteristic skin finding in patients with Down Syndrome?

144
A. Syringomas
B. Elastosis perforans serpiginosa
C. Single palmar crease
D. Flat nipples
E. Small tongue
►E

Down syndrome is caused by nondisjunction and results in trisomy 21. Clinical features include
single palmar crease, flat nipples, increased nuchal skin folds, syringomas, elastosis perforans
serpiginosa, xerosis, epicanthic folds of eyes, protruding scrotal tongue and fissured thickened
lips.

20
Which eye findings would be expected in an individual with this disorder associated with
atherosclerosis?
A. Dendritic corneal ulcerations
B. Salt and pepper retinitis pigmentosa
C. Ectopia lentis
D. Angioid streaks
E. Keratoconus
►D

Pseudoxanthoma elasticum is an autosomal rescessive or autosomal domminant disease caused


by a mutation in ABCC6 (adenosine triphosphate-binding cassette subfamily C member 6).
Associated findings include gastric artery hemorrhage, angiod streaks, retinal hemorrhage,
atherosclerotic disease, and a possible increased risk of first trimester miscarriage.

21
Defects in Fibrillin 2 are linked with:
A. Congenital contractural arachnodactyly
B. Cutis Laxa
C. Arthrochalasis multiplex congenita
D. Occipital horn syndrome
E. Lipoid proteinosis
►A

Fibrillin 2 defects arelinked primarily with congenital contractural arachnodactyly. This


syndrome is associated with long limbs, arachnodactyly, scoliosis and crumpled ears.
Occasionally, fibrillin 2 can be associated with Marfan syndrome also. The other conditions are
not linked to fibrillin mutations.

145
22
A 32 year-old woman is 5 weeks pregnant and is diagnosed with hyperthyroidism. Her doctor
gives her a prescription for on methimazole 10 mg PO tid. Which of the following fetal
abnormalities could be caused by this exposure?
A. Aplasia cutis congenita
B. Meningocele
C. Encephalocele
D. Spina bifida
E. Dermoid cyst
►A

In-utero methimazole exposures has been linked to aplasia cutis congenita and should not be
used in pregnant women. The FDA pregnancy class is D. The other listed options are not linked
with maternal methimazole usage.

23
Spontaneous mutations causing tuberous sclerosis are more likely to occur in which gene?
A. TSC2
B. TSC1
C. TSC 3
D. TSC 4
E. TSC 5
►A

Spontaneous mutations are four times more likely to occur in TSC2. In familial TS half
mutations are in TSC1 and half in TSC2.

24
This syndrome is characterized by lentigines, ECG conduction defects, ocular hypertelorism,
pulmonic stenosis, abnormal genitalia, retardation of growth and deafness:
A. Moynahan syndrome
B. Nail patella syndrome
C. Bloch-sulzberger syndrome
D. Incontinentia pigmenti achromians
E. Gardner syndrome
►A

146
Moyahan syndrome is also known as Leopard syndrome and can have all the findings of
lentigines, ECG conduction defects, ocular hypertelorism and pulmonic stenosis. They also get
abnormal genitalia, retardation of growth and deafness.

25
Patients that are diagnosed with Peutz-Jeghers syndrome must have a colonoscopy for
hamartomatous polyps transforming to carcinoma every:
A. 10 years
B. 8 years
C. 5 years
D. 2 years
E. 1 year
►D

Patients that are diagnosed with Peutz-Jeghers syndrome need to have a colonscopy every 2
years to assure that the hamartomatous polyps do not progress to carcinomas. It is an autosomal
dominant disorder with hyperpigmented macules in the oral mucosa.

26
A patient has Conradi-Hunermann-Happle syndrome with congenital ichthyosiform
erythroderma, ventricular septal defect, and asymmetric limb shortening. The bone finding for
the disease is:
A. Chondrodysplasia punctata
B. Bowing of the knees
C. Invisible bone syndrome
D. Thickened epiphyses
E. Distal calcification
►A

Patients with Conradi-Hunermann-Happle syndrome has chondrodysplasia punctata that is X-


linked dominant. They have ventricular septal defects and asymmetric limb shortening.

27
Premature aging seen in Cockayne‖s syndrome is caused by a mutation in which gene?
A. Transglutaminase
B. ATM gene
C. Patched gene
D. NEMO gene
E. DNA helicase gene
►E

147
Cockayne‖s syndrome is caused by a mutation in a DNA helicase gene, CSA or ERCC8. The
condition is defined by growth deficiency, premature aging, and pigmentary retinal degeneration.
75% of patient have photosensitive eruptions and severe cataracts before the age of 3 are
associated with poor prognosis.

28
Keratins 1 and 10 are important intermediate filaments in suprabasal keratinocytes. Genetic
mutation of these keratins can lead to which of the following disorders?
A. Netherton syndrome
B. Sjogren-Larsson syndrome
C. Pachyonychia congenita type 1
D. Epidermolytic hyperkeratosis / Bullous ichthyosiform erythroderma
E. White sponge nevus
►D

Epidermolytic hyperkeratosis is caused by a genetic mutation in keratins 1 and 10. White sponge
nevus is caused by a genetic mutation in keratins 4 and 13. Pachyonychia congenita is caused by
a genetic mutation in keratins 6a and 16. Netherton syndrome and Sjogren-Larsson syndrome are
not disorders of keratins.

29
What is the inheritance pattern of a dermatosis with acantholytic dyskeratosis?
A. Autosomal dominant
B. Autosomal recessive
C. X-linked dominant
D. X-linked recessive
E. Sporadic
►A

Hailey-Hailey, or Familial Benign Pemphigus, is an autosomal dominant genodermatosis which


is caused by a mutation in ATP2C1. Vesicles and erythematous plaques develop in the skin folds
such as axillae and groin area. Darier's disease is an autosomal dominant genodermatosis due to
a mutation in ATP2A2 and is often in a seborrheic distribution.

30
Which PPK is associated with a right-sided cardiomyopathy?
A. Haim-Munk
B. Papillon-Lefevre
C. Naxos

148
D. Olmstead
E. Huriez
►C

Naxos is associated with right sided cardiomyopathy. It is an AR, non epidermolytic PPK in
which patients have congenital wooly hair and fragile desmosomes. These patients are at risk for
heart failure and death.

31
Yellow-brown depositions in Descemet's membrane of the corneas is diagnostic for:
A. Hemochromatosis
B. Neurofibromatosis
C. Hepatolenticular degeneration
D. Pseudoxanthoma elasticum
E. Diabetes mellitus
►C

Hepatolenticular degeneration or Wilson Disease is caused by a defect in biliary excretion of


copper leading to accumulation in the liver, brain, cornea, pretibial hyperpigmentation,
hepatomegaly and cirrhosis. The Kayser-Fleischer ring is the yellow-brown copper deposition in
Descemet's membrane of the cornea. Other findings include ataxia, dysarthria and dementia.
Hemochromatosis does not have an eye finding. Neurofibromatosis has Lisch nodules and
Pseudoxanthoma elasticum has angioid streaks of the retina.

32
What cutaneous manifestation is associated with familial cerebral cavernomas?
A. Verrucous hemangioms
B. Glomeruloid hemangiomas
C. hyperkeratotic cutaneous capillary-venous malformations(HCCVM)
D. segmental facial hemangiomas
E. Tufted angiomas
►C

Familial cerebral cavernomas are due to a defect in the CCM gene which encodes the KRIT-1
protein. These patients often times have hyperkeratotic cutaneous capillary-venous
malformations.

33
Anodontia is a bone finding seen in which of the following conditions:
A. Hypomelanosis of Ito

149
B. Letterer-Siwe disease
C. Tuberous sclerosis
D. Jackson Sertoli syndrome
E. Hyper-IgE syndrome
►A

Hypomelanosis of Ito, or Incontinentia pigmenti achromians is a condition characterized by


marble-cake hypopigmentation, epilepsy, alopecia, scoliosis and mental/motor retardation. The
characteristic dental abnormality is anodontia. The remaining syndromes are not associated with
anodontia.

34
A 4-year old boy presents with generalized white scale. The mother reports that her son was born
with a tight membrane enveloping his body. Peripheral blood smear is within normal limits.
What is the most likely diagnosis?
A. Congenital ichthyosiform erythroderma
B. Neutral lipid storage disease
C. Lamellar ichthyosis
D. Netherton syndrome
E. Ichthyosis vulgaris
►A

The most likely diagnosis is Congenital ichthyosiform erythroderma. In neutral lipid storage
disease, the peripheral blood smear would demonstrate lipid vacuoles in leukocytes and
monocytes. Lamellar ichthyosis is characterized by plate-like scale in children/adults. Netherton
syndrome is characterized by ichthyosis linearis circumflexa. Ichthyosis vulgaris does not
typically present with collodian baby.

35
Cutaneous osteomas are seen in which syndrome?
A. Waardenburg syndrome
B. LEOPARD syndrome
C. Carney complex
D. Albright hereditary osteodystrophy
E. Gaucher‖s syndrome
►D

Albright hereditary osteodystrophy is caused by mutations in the Gs subunit of adenylate


cyclase. There is calcification and ossification due to pseudohypoparathyroidism, absent 4th
knuckle, and hypogonadism.

150
36
A patient is diagnosed with Leopard syndrome with cafe au lait macules, melanoma, local
hypopigmentation, triangular face, frontal bossing and low ears. The cardiac findings are:
A. AV block
B. Mitral valve prolapse
C. Tricuspid valve prolapse
D. Atrial fibrillation
E. Ventricular fibrillation
►A

Patients that have Leopard syndrome have AV blocks, arrhythmias, bundle branch block,
ventricular hypertrophy. The patient has ocular hypertelorism, pulmonic stenosis, abnormal
genitals, and deafness.

37
A patient has cutaneous amyloidosis, pancreatic tumors, parathyroid tumors, pheochromocytoma
and medullary carcinoma. The patient also had notalgia paresthetica as a child. The child most
likely has:
A. MEN Type IIA
B. MEN Type I
C. MEN Type IIB
D. MEN Type IIC
E. MEN Type III
►A

This patient has MEN type IIA also known as "Sipple's Syndrome". It is an autosomal dominant
disorder caused by the RET gene. Work up should include calcitonin, calcium, PTH and urine
catecholamines.

38
A patient with renal cell carcinoma caused by mutations in fumarate hydratase deficiency likely
suffers which of the following conditions?
A. Von-Hippel-Lindau syndrome
B. Cowden syndrome
C. Birt-Hogg-Dube syndrome
D. Familial multiple cutaneous leiomyomatosis
E. Multiple endocrine neoplasia
►D

151
Familial multiple cutaneous leiomyomatosis is an autosomal dominant condition caused by
mutations in the fumarate hydratase gene. Clinically, there are multiple cutaneous leiomyomas,
uterine leiomyomas and leiomyosarcomas, as well as renal cell carcinomas.

39
A 45 year old woman with history of uterine fibroids presents with painful firm papules of the
left upper arm. Biopsy of one of these lesions reveals smooth muscle fascicles. She notes a
history of renal cell cancer in her family. The gene implicated in this syndrome has which of the
following functions?
A. Telomere maintenance
B. Involved in the Kreb's cycle
C. Inhibits mTOR signalling
D. Impairs folliculin function
E. Encodes a deubiquitinating enzyme
►B

The patient scenario describes Reed's syndrome (multiple cutaneous and uterine leiomyomatosis)
characterized by uterine and cutaneous leiomyomas, in addition to papillary renal cell cancer.
This autosomal dominant disorder is due to mutations within the fumarate hydratase gene, which
catalyses the conversion of fumarate to malate in the Kreb\'s cycle. It is also thought to act as a
tumor suppressor gene. With regards to the other answer choices, telomere maintenance is
disrupted in dyskeratosis congenita. The tuberous sclerosis genes hamartin (TSC1) and tuberin
(TSC2) inhibit mTOR signalling. Mutations in the FLCN gene, with encodes folliculin (thought
to act as a tumor suppressor), cause Birt Hogg Dube syndrome. The CYLD gene, a
deubiquitinating enzyme, has been implicated in Brooke-Spiegler syndrome.

40
Epidermolysis bullosa with muscular dystrophy is caused by mutations in which of the
following?
A. Keratins 5 and 14
B. Plectin
C. Loricrin
D. Collagen 7
E. Collagen 17
►B

Epidermolysis bullosa with muscular dystrophy is caused by mutations in plectin.

41
Retinal hemangioblastomas are found in which syndrome:

152
A. Osler-Weber-Rendu disease
B. Von-Hippel Lindau disease
C. Kasabach-Merritt syndrome
D. Klippel-Trenaunay Weber syndrome
E. Sturge-Weber syndrome
►B

Von Hippel-Lindau syndrome is an autosomal dominant condition caused by a defect in the VHL
tumor suppressor gene. This disease is characterized by retinal hemangioblastomas, often
resulting in visual impairment and blindness if left untreated. In addition, many tumors are seen
including pheochromocytoma, renal cell carcinoma, and hemangioblastomas of the cerebellum,
medulla, and spinal cord. Pancreatic and renal cysts are also a feature of this condition. Finally,
polycythemia can occur as a result of erythropoietin production by renal cell carcinoma. Von
Hippel-Lindau syndrome is a progressive, universally fatal condition which presents most often
in the fourth decade of life.

42
Patients with hemochromatosis are at increased risk for which of the following?
A. Vibrio vulnificus infections
B. Yersenia infections
C. Polyarthritis
D. Generalized metallic-grey hyperpigmentation
E. All of these options are correct
►E

Patients with hemochromatosis have increased intestinal iron absorption leading to systemic iron
overload. Signs include a generalized metallic-grey hyperpigmentation, koilonychia, alopecia
(especially pubic/axillary hair) cardiac failure/arrhythmias/heart block, hepatomegaly with
cirrhosis, diabetes (bronze diabetes), polyarthritis with chondrocalcinosis and are susceptible to
Vibrio vulnificus and Yersinia infections.

43
What is the gene mutation that most commonly causes herlitz junctional epidermolysis bullosa?
A. LAMB3
B. COL7A1
C. NEMO
D. ATP2A2
E. ATP 2C1
►A

153
LAMB3 is the most common mutation, it is usually a nonsense mutation causing a premature
stop codon resulting in the absence of laminin 332. Non herlitz type is due to a missense or
splice site mutation resulting in rudimentary/decreased laminin 332. COL7A1 is mutated in
dystrophic epidermolysis bullosa. NEMO gene is mutated in incontinnentia pigmenti. ATP2A2
gene is mutated in Dariers. ATP2C1 is mutated in Hailey Hailey.

44
Which of the following syndromes demonstrate atrophoderma vermiculatum?
A. Tuzun Syndrome
B. ROMBO Syndrome
C. Nicolau-Balus Syndrome
D. Braun-Falco-Marghescu Syndrome
E. All of these answers are correct
►E

All of the listed syndromes include atrophoderma vermiculatum as part of their constellation of
symptoms. Atrophoderma vermiculatum is characterized by honeycomb pattern of atrophic scars
on the face. Tuzun Syndrome also has scrotal tongue. ROMBO has BCCs, milia, peripheral
vasodilation, trichoepitheliomas. Nicolau-Balus has eruptive syringomas and milia. Braun-Falco-
Marghescu has keratosis pilaris and palmoplantar hyperkeratosis.

45
A patient presents with focal symmetric palmoplantar keratoderma, thickened, hyperkeratotic
fingernails and toenails with a "pincer" appearance and frequent staph and candida paronychial
infections, follicular hyperkeratosis of the knees and elbows and oral leukokeratosis. The patients
mother and grandfather both have similar skin findings. What is the defect in PC type I?
A. Keratin 6a/16
B. Keratin 6b/17
C. Keratin 1/10
D. Keratin 2e/10
E. None of these options are correct
►A

Keratin 6a/16 are defective in PC type I. K6b17 is defective in PC type II, K1/10 in Unna-Thost
PPK and epidermolytic hyperkeratosis, K2e/10 in Ichthyosis bullosa of Siemens.

46
A 2 year old female patient has linear atrophy in a Blasckoid distribution, alopecia, nail
dystrophy, abnormal teeth, and colobomas. What is the inheritance pattern of this condition?
A. X-linked dominant

154
B. X-linked recessive
C. Autosomal dominant
D. Autosomal recessive
E. Mitochondrial
►A

Focal dermal hypoplasia, or Goltz syndrome, is an x-linked dominant condition with a defect in
the PORCN gene. It is characterized by linear atrophy in a Blasckoid distribution, alopecia, nail
dystrophy, abnormal teeth, and colobomas.

47
Mutations in calcium transporters cause which pair of diseases?
A. Erythrokeratodermia variabilis and progressive symmetric erythrokeratodermia
B. Lamellar ichthyosis and nonbullous congenital ichthyosiform erythroderma
C. Refsum syndrome and Sjogren-Larsson syndrome
D. Chondrodysplasia punctata and CHILD syndrome
E. Darier‖s disease and Hailey-Hailey disease
►E

Darier‖s disease and Hailey-Hailey disease are caused by mutations in the SERCA calcium
ATPase. The former is characterized by hyperkeratotic papules in seborrheic areas, palmar
keratoses and pits, red-white longitudinal nail bands, v-shaped distal nail nicks, and
cobblestoning of oral and rectal mucosae. The latter is characterized by acantholytic erosions in
skin folds.

48
Cutaneous meningiomas have been associated with what syndrome?
A. Neurofibromatisis
B. Cowden
C. Neurocutaneous melanosis
D. Gorlin
E. Glomangiomatosis
►A

Cutaneous meningiomas are sperated into 3 types. Type 1 are thought to be misplaced meningeal
cells, or rudimentary mengioceles. Type II develop along the course of cranial nerves. Type III
lesions represent cutaneous metastasis or an underlying primary meningioma. Cutaneous
meningiomas have been associated with cranial developmental anomalies and in
neurofibromatosis.

155
49
Which of the following is caused by a defect in a gap junction protein?
A. Epidermolysis bullosa simplex
B. Hailey-hailey
C. Erythrokeratoderma variabilis
D. Dyskeratosis congenita
E. Bullous ichthyosis of siemens
►C

Erythrokeratoderma variabilis is also known as Mendes da Costa Syndrome. It is caused by a


defect in connexin 31, a gap junction protein. EB simplex is caused by a mutation in keratins 5 &
14, Hailey-Hailey is caused by a mutation in calcium transporters, dyskeratosis congenita is
caused by a defect in rRNA synthesis, and bullous icthyosis of Siemens is caused by mutations
in keratin 2e.

50
A patient has a bird head facies, "Mickey Mouse" ears, photosentivity, deafness and basal
ganglia calcifications has:
A. Cockayne syndrome
B. Xeroderma pigmentosum
C. Tay's syndrome
D. Louis Bar syndrome
E. Bloch-sulzberger disease
►A

The patient has Cockayne syndrome. It is autosomal recessive associated with a defect in
nucleoside excision repair with a mutation in the XPG gene. It is similar to XP with
photosensitivity, neurologic degeneration, deafness, retinal atrophy, basal ganglia calcifications
and peripheral neuropathy.

51
A 16 month-old girl presents with patchy alopecia, whorled erythematous scaly eruption, and
asymmetric limb shortening. What laboratory or radiologic test may aid in diagnosis?
A. Brain MRI
B. Alkaline phosphatase
C. Chest radiograph
D. Bone films
E. Complete blood count
►D

156
The patient has Conradi-Hunermann Syndrome. This is a X-linked dominant disorder
characterized by ichthyosiform erythroderma in Blaschko's lines in infancy which resolves with
follicular atrophoderma, patchy alopecia, short stature, cataracts, scoliosis, assymetric limb
shortening. Bone films will demonstrate stippled epiphyses. Ichthyosis and stippled epiphyses
resolve after infancy.

52
Which of the following is correct about nevoid basal cell carcinoma syndrome?
A. known as Gardner syndrome
B. autosomal recessive
C. multiple keratoacanthomas
D. calicification of the falx
E. odontogenic steatomas
►D

Nevoid basal cell carcinoma syndrome, also known as Gorlin syndrome, is an autosomal
dominant disease, with abnormalities in the PTCH gene. It presents with multiple basal cell
cancers early in life, frontal bossing, hypertelorism, odontogenic keratocysts, and bifid ribs. The
presence of intracranial calcification of the falx cerebri is characteristic.

53
A 7 year old boy presents to your office with short and sparse hair. He is also short for his age. A
hair mount reveals alternating light and dark bands under polarized light. Which of the following
statements is correct regarding this condition?
A. Eyebrows and eyelashes are not affected
B. There is no increased risk of skin cancer
C. Amino acid analysis of hair reveals high sulfur content
D. There is no increased hair fragility.
E. Dark bands represent air
►B

The patient described has trichothiodystrophy, with the hair finding of trichoschisis. These
patients have photosensitivity but no increased risk of skin cancer. They also have intellectual
impairment, decreased fertility, short stature, and progeria-like facies. Low sulfur content of the
hair is found. Eyebrows and eyelashes are also affected. Dark bands in pili annulati, not
trichoschisis, are air cavities.

54
Which of the following metals is deficient in the serum of patients with Menkes kinky hair
syndrome?

157
A. Copper
B. Iron
C. Selenium
D. Zinc
E. Biotin
►A

Menkes kinky hair syndrome is transmitted in an X-linked recessive manner and is caused by a
mutation in ATP7A, an ATP-dependent copper tranporter. This defect results in low serum
levels of copper. These individuals will have hair abnormalities such as sparse, hypopigmented
brittle hair, eyelashes and eyebrows, lax skin, a "cupid's bow" upper lip, CNS progressive
deterioration, seizures, skeletal abnormalities and tortuous arteries. The other listed items are not
associated with Menkes syndrome.

55
A child has ichthyosis and is found to have leukocytes with vacuoles filled with lipids on
peripheral smear. She most likely has:
A. Chanarin-Dorfman syndrome
B. Ichthyosis hystrix
C. Naxos disease
D. Ichthyosis bullosa of Siemens
E. Ichthyosis vulgaris
►A

This child has Chanarin-Dorfman syndrome, also called neutral lipid storage disease. This is an
autosomal recessive disorder characterized by accumulation of triglycerides in the cytoplasm of
leukocytes, muscle, liver, fibroblasts, and other tissues. Patients generally have normal blood
lipid levels and a finely scaling ichthyosis. They can also have extracutaneous involvement such
as cataracts, decreased hearing, myopathy, and neurologic abnormalities.

56
Which type of porphyria is associated with hyponatremia?
A. Acute intermittent porphyria
B. Porphyria cutanea tarda
C. Variegate porphyria
D. Hereditary coproporphyria
E. Erythropoietic protoporphyria
►A

158
Acute intermittent porphyria can cause hyponatremia due to the syndrome of inappropriate
antidiuretic hormone secretion.

57
A patient has macular telangiectasias on the oral mucosa, face and acral surfaces. These patients
also have epistaxis, melena related to angiomas in the GI tract and AV malformation in other
organs. The gene mutation is:
A. ALK1, ACVRL1
B. PTEN
C. TCS1, TCS2
D. Endokinase
E. Antigen kinase-1
►A

Patients with hereditary hemorrhagic telangiectasia or Osler-Weber Rendu have a mutation in


Endoglin or Activin receptor like kinase -1 (ALK1, ACVRL1) gene that result in HHT1 and
HHT2. Both genes play a role in the vascular system.

58
What medication may exacerbate this autosomally dominant, acnatholytic disorder?
A. Phenytoin
B. Lithium
C. Oral contraceptives
D. Anti-malarials
E. Corticosteroids
►B

Darier's disease is autosomal dominant condition characterized by hyperkeratotic papules


coalescing into warty plaques and cobblestoned papules on mucosal surfaces. The cutaneous
manifestations may be exacerbated by lithium.

59
In alkaptonuria there is a blue-black discoloration in the sclera near the insertion of the rectus
muscles. This sign is called the:
A. Osler sign
B. Hutchinson sign
C. Blue sign
D. Amyloid sign
E. Ochronosis
►A

159
Alkaptonuria is am autosomal recessive disease with a deficiency in homogentistic acid oxidase.
The osler sign is a blue black discoloration in the sclera near the insertion of the rectus muscles,
oil droplet opacities in the cornea, pigmented pingucela, granules in episclera.

60
Ivory-colored papules between the angles of the scapulae are characteristic of which syndrome:
A. Hurler
B. Scheie
C. Morquio
D. Hunter
E. Sanfilippo
►D

These syndromes are all mucopolysaccharidoses. These papules are characteristic of Hunter
syndrome which is caused by a deficiency in iduronate sulfatase.

61
A child presents with macroglossia, exopthalmos and gigantism. He has a history of omphalocele
repair and has circular depressions on the rim of the posterior helices. Although this syndrome is
most often transmitted in a sporadic manner, 15% of cases have defects in which gene?
A. KIP2
B. ATM
C. VHL
D. FLT4
E. FOXC2
►A

KIP2 can be mutated in 15% of cases of Beckwith-Wiedemann Syndrome. KIP2 is a cyclin-


dependent kinase inhibitor gene which acts as a negative regulator of cell proliferation. These
children have an increase risk of Wilms' tumor and organomegaly. In addition to the circular ear
depression, they can also have a linear earlobe crease. The remaining genes are mutated in other
syndromes with vascular disorders as a component: ATM in ataxia telangectasia, VHL in Von
Hippel-Lindau, FLT4 in Hereditary lymphedema and FOXC2 in Lymphedema-distichiasis
syndrome.

62
Hyper IgE syndromes are associated with genetic deficiency of which of the following?
A. WASp
B. IL-17 receptor

160
C. AIRE
D. FOXP3
E. Dock8
►E

To date, three genetic etiologies of hyper IgE syndromes have been identified: STAT3, DOCK8,
and Tyk2. All of these hyper IgE syndromes are characterized by eczema, sinopulmonary
infections, and greatly elevated serum IgE. However, each has distinct clinical manifestations.
Mutations in STAT3 cause autosomal dominant HIES (Job\'s syndrome), which is unique in its
diversity of connective tissue, skeletal, and vascular abnormalities. DOCK8 deficiency is
characterized by severe cutaneous viral infections such as warts, and a predisposition to
malignancies at a young age. Hyper IgE syndrome associated with Tyk2 deficiency is
characterized by nontuberculous mycobacterial infections.

63
In one variant of epidermolysis bullosa simplex, those affected have muscular dystrophy in
addition to the skin findings. Which protein mutation has been linked to this finding?
A. Plakoglobin
B. Plakophilin
C. Plectin
D. Desmoglein
E. Desmocollin
►C

A plectin mutation is linked with this subtype of epidermolysis bullosa simplex.

64
A patient has premature graying and a mutation in laminin A. This patient also has a bird like
face, early wrinkling, dyspigmentation, atrophy of skin, decreased subcutaneous fat and
atherosclerosis. This patient has:
A. Progeria
B. Hypotrichosis simplex
C. Costello syndrome
D. Marie-Unna
E. Cartilage hair hypoplasia
►A

Patients with premature aging with thin hair, bird like face, early wrinkling, and atherosclerosis
has progeria. THey also have dyspigmentation, atophy of the skin and decreased subcutaneous
fat.

161
65
You receive a hospital consult from the gastroenterology service for a 42-year old woman with
esophageal cancer. They would like your opinion on the yellow, thickened areas on her palms
and soles in areas of pressure. When you speak with her, she says that her father had similar
problems and it runs in her family. Which of the following is defective?
A. TOC gene
B. Desmoplakin
C. Plakoglobin
D. Connexin 30.3
E. Connexin 31
►A

This case describes Howell-Evans syndrome. This AD syndrome characteristically has a PPK in
areas of pressure, oral leukoplakia and esophageal carcinomas. Desmoplakin is defective in
Carvajal syndrome and plakoglobin in Naxos syndrome. Connexin 31 and 30.3 are linked to
erythrokeratoderma variabilis, which includes a PPK, but not esophageal carcinoma.

66
A 6-year-old boy presents with brachyonychia and three firm subcutaneous nodules with a bluish
hue about the trunk. Biopsy of a representative lesion is consistent with a pilomatricoma. What is
the most likely associated syndrome?
A. Noonan syndrome
B. Carney Complex
C. Nail-Patella syndrome
D. Werner syndrome
E. Rubinstein-Taybi syndrome
►E

Rubenstein-Taybe syndrome is caused by a amutation in CREB-Binding protein and presents


with brachyonychia, eruptive keloids, and multiple pilomatricomas. Nail-Patella syndrome is
associated with anonychia, along with hypoplastic patellae, triangular lunulae, radial subluxation,
Lester iris, iliac horns, and glomerulonephritis. The other disorders are not associated with
brachyonychia.

67
What is the underlying gene defect for this transgrediens form of palmoplanter keratoderma
A. SLURP-1
B. TOC gene
C. Plakoglobin

162
D. Keratin type 1
E. Keratin type 9
►A

Attached picture is Mal de Meleda (keratosis palmoplantaris transgrediens) which is an


autosomal recessive form of diffuse PPK, associated with keratotic plaques that extend to the
dorsal aspects of the hands and feet ("transgrediens") and may overlie joints . Hyperhidrosis,
superinfection, and occasionally perioral erythema, brachydactyly, and nail abnormalities are
associated. Mal de Meleda is due to mutations in ARSB, which encodes SLURP-1. The other
choices represent gene defects for "non-transgrediens" forms of PPK (Plakoglobin in Naxos
syndrome, TOC gene in Howel-Evans syndrome, K1 in non-epidermolytic PPK "Unna-Thost",
and K9 in epidermolytic PPK "Vorner"

68
Patients with x-linked icthyosis are more prone to getting which two malignancies?
A. Pancreatic and acute lymphocytic leukemia (ALL)
B. Pancreatic and acute myelogenous leukemia (AML)
C. Testicular and AML
D. Testicular and ALL
E. Renal cell and ALL
►D

Patients with x-linked icthyosis have a 20% chance of having cryptorchidism and are more prone
to both testicular cancer and acute lymphocytic leukemia.

69
Dyshcromatosis symmetrica hereditaria (DSH) is a disorder characterized by asymptomatic
hypo- and hyperpigmented macules on the face and dorsal extremities. Mutations in which of the
following genes is associated with this condition?
A. DKC
B. PTEN
C. ADAR1
D. ERCC4
E. STK11
►C

Dyshcromatosis symmetrica hereditaria (DSH) is a rare autosomal dominant pigmentary disease


characterized by asymptomatic hypopigmented and hyperpigmented macules distributed on the
face and dorsal extremities. It is most common in patients of Asian descent but has also been
reported in patients of European and South American descent. The condition is typically limited

163
to the skin but has been associated in rare instances with neurological deterioration and brain
calcificiations. The lesions tend to appear in infancy and childhood and stabilize during
adolescence. Mutations in the double-stranded RNA-specific adenosine deaminase gene
(ADAR1 or DSRAD) underlie DSH. The ADAR1 protein catalyzes the deamination of
adenosine to inosine in double-stranded RNA which is important for alternative splicing or
alternations of codons for protein translation. However, the precise mechanism by which these
mutations lead to actual disease remain unknown. There are two diseases that are
phenoptypically similar to DSH and previously thought to be related. The first is dyschromatosis
universalis hereditaria (DUH) which is characterized by a mixture of hyperpigmented and
hypopigmented macules that are widespread. DSH can be distinguished from DUH by its
localization to the face and distal acral sites. The second disease is acropigmentatio retiticularis
(AR, also known as acropigmentation of Kitamura). AR is also characterized by
hyperpigmentated macules on the dorsal hands and feet as well as palmar pits. However, in
contrast to DSH, it is notably devoid of hypopigmented macules. Suzuki et al. sequenced 20
cases of patients presenting with DSH, DUH, and AR. All patients with DSH demonstrated
mutations in ADAR1, however, none of the patients with DUH or AR demonstrated mutations.
Based on these studies, they proposed that these diseases are in fact distinct and separate from
DSH. There are no effective therapies for DSH. However, it is important to distinguish DSH
from other more serious disorders that can present with pigment alterations such as dyskeratosis
congenita and xeroderma pigmentosum. Both of these conditions present with photosensitivity
and squamous cell carcinomas which are not present in DSH.

70
Which of the following is caused by a defect in keratins 4 & 13?
A. White sponge nevus
B. Epidermolysis bullosa simplex
C. Epidermolysis bullosa simplex with myotonic dystrophy
D. Junctional EB with pyloric atresia
E. Clouston‖s syndrome
►A

White sponge nevus is caused by a defect in keratins 4 & 13. The remaining entities have the
corresponding defects:
EB simplex→►►►keratins 5 & 14
EB simplex with myotonic dystrophy→►►►plectin
Junctional EB with pyloric atresia→►►►Integrin ɑ-6, β4
Clouston‖s syndrome (hidrotic ectodermal dysplasia)→►►►connexin 30

71
A patient with multiple lentigines and blue nevi may also have:

164
A. Deafness
B. Pulmonary valve stenosis
C. Atrial myxoma
D. Mental retardation
E. GI malignancy
►C

This patient may have a constellation of features associated with NAME syndrome, otherwise
known as Carney complex or LAMB syndrome. This condition is inherited in an autosomal
dominant pattern and is due to a defect in the PRKAR1A gene. This condition is characterized
by the following features: blue Nevi, Atrial myxomas, cutaneous Myxomas, and Ephelides. In
addition, testicular tumors are seen as well as sexual precocity. Finally, patients may have
endocrine abnormalities including pigmented nodular adrenocorticoal disease and Cushing
syndrome as well as pituitary adenomas. Deafness, pulmonary stenosis, GI malignancies, and
mental retardation are not features of this condition.

72
AKT1 mutation is seen in which of the following condition?
A. Gardner syndrome
B. Proteus syndrome
C. Incontinentia pigmenti
D. Noonan syndrome
E. Beckwith-Wiederman syndrome
►B

Proteus syndrome is a sporadic condition due to mutation in AKT1. Clinical features include
subcutaneous lymphovenous malformations, capillary malformations, lipomas, connective tissue
nevi of palms/soles, hemihypertrophy, frontal bossing, hyperostoses of epiphyses & skull
(especially external auditory canal), scoliosis, bilateral ovarian cystadenomas, and parotid
monomorphic adenomas.

73
Which of the following is a potentially serious complication of the blue rubber bleb nevus
syndrome?
A. Development of chondrosarcomas
B. Development of angiosarcomas
C. Gastrointestinal hemorrhage
D. Development of lymphedema
E. Development of fluid retention
►C

165
Blue rubber bleb nevus syndrome is characterized by multiple tender venous malformations of
skin and gastrointestinal tract, which can lead to gastrointestinal bleeding.

74
What finding is seen on brain imaging of patients with Papillon-Lefevre Syndrome?
A. Tram track calcifications
B. Calcification of the falx cerebri
C. Calcification of the hippocampus
D. Calcification of the dura
E. Agenesis of the corpus callosum
►D

Pappilon Lefevre Syndrome is an autosomal recessive syndrome characterized by transgredient


PPK and periodontitis. There is a defect in cathepsin C. One sees dural calcification at the
tentorium and choroid plexus. Tram track calcifications are seen in STurge-WEber.
CAlcification of the falx cerebri and agenesis of the corpus callosum is seen in basal cell nevus
syndrome. Hippocampal calcification is seen in lipoid proteinosis.

75
A patient is diagnosed with tuberous sclerosis with seizures, mental retardation and
hypopigmented macules has a mutation in the
A. TSC1 and TSC2 gene
B. PTEN gene
C. p53 gene
D. WEC gene
E. KI gene
►A

Patients with tuberous sclerosis have a mutation in the TSC1 and TSC2 gene. The encode for
tumor suppressor proteins hamartin (TSC1) and tuberin (TSC2). Other skin findings include
hypopigmented macules, facial angiofibromas, collagenomas, shagreen patch, periungual
fibromas, Koenen's tumors and forehead plaques.

76
Angioid streaks on retinal exam are characteristic of which of the following syndromes?
A. Pseudoxanthoma elasticum
B. Choroid malformations
C. Eyelid papillomas
D. Lester iris

166
E. Salt & pepper retinitis pigmentosa
►A

Angioid streaks are characteristic of pseudoxanthoma elasticum. They are caused by rupture of
Bruch's membrane of the choroid. Choroid malformations are found in Sturge-Weber syndrome,
eyelid papillomas in xeroderma pigmentosum, Lester iris in Nail-patella syndrome and salt &
pepper retinitis pigmentosa in Refsum syndrome.

77
Which of the following is NOT a complication of Kasabach-Merritt Syndrome:
A. Thrombocytopenia
B. CHF
C. Ataxia
D. Disseminated intravascular coagulation
E. Gastrointestinal bleeding
►C

Kasabach-Merritt Syndrome results from platelet trapping. It occurs with tufted angiomas and
kaposiform hemangioendotheliomas. Hematologic complications include thrombocytopenia,
microangiopathic hemolytic anemia, DIC, and acute hemorrhage. The presence of large
angiomas can lead to high output failure (CHF) and they can also compress on surrounding
structures.

78
A patient with this autosomal recessive disorder caused by a defect in helicase is an increased
risk for which malignancy?
A. Acute leukemia
B. Renal cell carcinoma
C. Medullary thyroid carcinoma
D. Squamous cell carcinoma of the lung
E. Prostate carcinoma
►A

Bloom's syndrome is an autosomal recessive disorder caused by a mutation in DNA helicase. It


is characterized by photodistributed erythema in a butterfly distribution, malar hypoplasia with a
prominent nose, high pitched voice, and an increased risk for malignancy (acute leukemia,
lymphoma, and GI adenocarcinoma.)

79
Papillon-Lefevre and Haim-Munk syndromes have which of the following symptoms?

167
A. Right-ventricular cardiomyopathy
B. Pseudoainhum
C. Esophageal cancer
D. Eccrine syringofibradenoma
E. Periodontitis with tooth loss
►E

Right-ventricular cardiomyopathy is associated with Naxos syndrome, pseudoainhum is


associated with Vohwinkel syndrome, esophageal cancer is associated with Howel-Evans
syndrome, and eccrine syringofibradenomas are associated with Schopf-Schulz-Passarge
syndrome. Periodontitis with tooth loss is associated with Papillon-Lefevre and Haim-Munk
syndromes, which are caused by mutations in Cathepsin C.

80
Most common malignancy to develop in a patient with tricholemmomas, acral verrucous papules
and cobble-stoning of buccal and gingival mucosa?
A. Thyroid cancer
B. Breast cancer
C. Colon cancer
D. Lymphoma
E. Melanoma
►B

Cowden's syndrome is an autosomally dominant inherited defect of PTEN. Patient may present
with multiple trichilemmoma, hamartomatous tumors of the breast, thyroid and endometrium,
acral keratoses and papillomatous papules. Breast cancer is the most common cancer to arise
while thyroid carcinoma is the second most common.

81
A patient has multiple basal cell carcinomas and palmoplantar pits with bifid ribs. This patient
also has jaw cysts in the first decade of life. The gene mutation in this patient is:
A. PATCH
B. PTEN
C. MCS1, MCS2
D. p53
E. UBO
►A

168
This patient has basal cell nevus syndrome or Gorlin Syndrome. The mutation is in the PATCH
gene that encodes PTC protein involved in the sonic hedge hog pathway. It leads to multiple
basal cell carcinomas in the adulthood and also palmoplantar pits in nearly all patients.

82
A seven month old infant diagnosed with eczema on her face returns for a diaper-rash follow-up.
A one-month trial of topical antifungals has failed to improve the infant's systems. The part of
the physical exam that might prove most useful include:
A. Stool samples
B. Fontanelle examination
C. Examination of palms and soles
D. Cardiac ausculatation
E. Hearing test
►C

Acrodermatitis enteropathica presents itself in infancy once breastfeeding has stopped. It can
clinically mimic atopic dermatitis, seborrheic dermatitis and candidiasis. Clinical features
include scaly red rash around mouth, eyes, and palms, diarrhea, stomatitis, glossitis, alopecia,
and failure to thrive.

83
Giant lysosomal granules are seen in which disease?
A. Chediak-Higashi syndrome
B. Griscelli syndrome
C. Piebaldism
D. Incontinentia pigmenti
E. Carney complex
►A

Chediak-Higashi syndrome is caused by an autosomal recessive mutation in a lysosomal


transport gene (LYST, CHS1). This disorder is characterized by oculocutaneous albinism, ataxia,
muscle weakness, and giant lysosomal granules. There is an accelerated phase characterized by
lymphohistiocytic infiltration of reticuloendothelial system, pancytopenia and death.

84
What is the inheritance pattern of chronic granulomatous disease?
A. Autosomal recessive
B. Autosomal dominant
C. X-linked recessive
D. X-linked dominant

169
E. Autosomal recessive and X linked recessive
►E

Chronic granulomatous disease is inherited in an autosomal recessive and x-linked recessive


manner. There are mutations 5 mutations total. If the mutation is present in CYBA (a
cytochrome subunit), NCF1 & 2 (neutrophil cytosol factors 1 & 2 & 4) it is inherited in an
autosomal recessive manner. The mutation in CYBB is X linked recessive.

85
The presence of natal teeth and pincer nails suggests which disease entity?
A. Congenital syphillis
B. Thalidomide exposure in utero
C. Incontinentia pigmenti
D. Pachyonychia congenita
E. Anhidrotic ectodermal dysplasia
►D

Pachyonychia congentia is an autosomal dominant condition characterized by a constellation of


findings affecting ectodermal structures. These include the presence of natal teeth, steatocystoma
multiplex, follicular hyperkeratosis of the knees, elbows and extensor extremities, eruptive vellus
hair cysts, and oral leukokeratosis which is not pre-malignant. In addition, nail findings include
twenty-nail dystrophy, subungual hyperkeratosis with increase transverse curvature ("pincer
nails") and candidal paronychia. There are two forms of pachyonychia congenital: Type 1
(Jadassohn-Lewandowsky syndrome) caused by defects in keratin 6a and 16, and Type 2
(Jackson-Lawler type) caused by defects in keratins 6b and 17. Anhidrotic ectodermal dysplasia
is associated with peg-shaped teeth, hypoanodontia, and a non-specific nail dystrophy. Likewise,
incontientia pigmenti also is characterized by anodontia and peg-shaped teeth and dystrophic
changes of the nail. Finally congenital syphilis is a well-recognized cause of pegged teeth. Limb
deformities are the most serious sequelae of thalidomide exposure in utero.

86
The combination of gastrointestinal polyposis, nail atrophy, alopecia, generalized pigmentation
of skin, and melanotic macules of the fingers is characteristic of which of the following
syndromes?
A. Nicolau-Balus syndrome
B. Peutz-Jeghers syndrome
C. Cronkhite-Canada syndrome
D. Cowden syndrome
E. Bannayan-Riley-Ruvalcaba syndrome
►C

170
Cronkhite-Canada syndrome is a sporadic gastrointestinal polyposis syndrome associated with
nail atrophy, alopecia, generalized pigmentation of the skin, and melanotic macules on the
fingers.

87
Non-bullous icthyosiform erythroderma is caused by which of the following mutations:
A. Transglutaminase-1 gene (TGM1)
B. 12R-lipoxygenase gene (ALOX12B)
C. Lipoxygenase-3 gene (ALOXE3)
D. Both 12R-lipoxygenase gene (ALOX12B) and lipoxygenase-3 gene (ALOXE3)
are correct
E. All of these answers are correct
►E

Non-bullous congenital erythroderma (NCIE)is an autosomal recessive disorder characterized by


a collodion baby presentation at birth, and generalized erythroderma with fine white scale,
palmoplantar keratoderma, and heat intolerance. NCIE may be caused by mutations in
transglutaminase-1 gene (TGM1), the 12R-lipoxygenase gene (ALOX12B), and the
lipoxygenase-3 gene (ALOXE3). Mutations in the keratinocyte TGM1 gene interferes with
normal cross-linking of structural proteins and the lipid envelope, leading to defective
cornification and desquamation. ALOXE3 functions as an epoxy alcohol synthase using the
product of ALOX12B as the preferred substrate; either gene can be the site of mutations causing
NCIE.

88
A 2 year old girl presents with sunken eyes, large ears, microcephaly and a photodistributed
eruption on her face. Eye exam reveals ―salt and pepper‖ retina. The gene responsible for this
syndrome codes for a:
A. Transcription factor
B. Surface glycoprotein
C. Lysosomal protease
D. DNA helicase
E. Mismatch repair gene
►D

The patient described has Cockayne syndrome, an autosomal recessive disorder believed to be
due to a mutation in either DNA helicase ERCC6 or defective ERCC8 which does nucleotide
excision repair. UV irradiated cells have decreased DNA and RNA synthesis and increased

171
chromosomal breaks. Clinical features include cachectic dwarfism with microcephaly, thin nose,
large ears, photosensitive eruption, cataracts, salt & pepper retina, and diffuse demyelination.

89
In biopsies from blisters in patients with junctional epidermolysis bullosa, the split is found in
the:
A. Basal cell layer of the epidermis
B. Lamina lucida
C. Lamina densa
D. Squamous cell layer of the epidermis
E. None of the answers are correct
►B

The split seen in junctional epidermolysis bullosa is in the lamina lucida. The other locations can
be involved in blistering disease, but not junctional epidermolysis bullosa.

90
The following enzyme defect is most commonly seen in CHILD Syndrome.
A. 3-beta-hydroxysteroid dehydrogenase
B. 3-beta-hydroxysteroid isomerase
C. Aryl sulfatase E
D. NAD oxido reductase
E. DNA helicase
►A

CHILD Syndrome is a X-linked dominant disorder characterized by unilateral ichthyosiform


erythroderma, ipsilateral limb deformity, and ipsilateral organ hypoplasia. The most commom
gene defect is NSDHL which encodes 3-beta hydroxysteroid dehydrogenase. EBP gene defects
which encode 3-beta-hydroxysteroid isomerase have been described, however this is the usual
defect in Conradi-Hunermann Syndrome. Aryl sulfatase E is mutated in X-linked recessive
chondrodysplasia punctata.

91
Medulloblastoma is seen in which syndrome?
A. Gardner‖s syndrome
B. Multiple endocrine neoplasia 2b
C. Muir-Torre syndrome
D. Basal cell nevus syndrome
E. Neurofibromatosis Type 1
►D

172
Basal cell nevus syndrome is an autosomal dominant syndrome caused by a mutation in PTC
gene, which acts in the Sonic hedgehog pathway. Cutaneous manifestations of this
genodermatosis include basal cell carcinomas, palmoplantar pits, epidermoids cysts. Other
findings include odotogenic cysts, frontal bossing, bifid ribs, calcification of the falx cerebri and
medulloblastomas.

92
Birt-Hogg-Dube syndrome is most strongly associated with which of the following
malignancies?
A. Basal cell carcinoma
B. Medulloblastoma
C. Renal cell carcinoma
D. Trichoepithelial carcinoma
E. Eccrine syringofibroadenoma
►C

Birt-Hogg-Dube syndrome is characterized by multiple fibrofolliculomas, trichodiscomas, acro-


collagenomas, lipomas, and oral fibromas. Patients develop renal cell carcinoma, colon cancer,
and medullary thyroid carcinoma.

93
Familial macular and lichen amyloidosis is a feature of which of the following conditions?
A. Sipple syndrome
B. Peutz-Jeghers syndrome
C. Marfan syndrome
D. Dyskeratosis congenita
E. Birt-Hogg-Dube syndrome
►A

Sipple syndrome (MEN 2a) is caused by autosomal dominant mutations in the ret
protooncogene. Patients develop parathyroid cancers, pheochromocytomas, and medullary
cancer of the thyroid gland. Familial macular and lichen amyloidosis is also a feature of this
syndrome.

94
A 17 y/o man presents with facial acne that he would like treated. You notice that he has fine
brown scale on his neck and do a complete skin exam. This scale is present on the remainder of
his body, sparing his palms, soles and flexural areas. He informs you that his uncles on his
mother‘s side have similar skin findings. He is not concerned about the skin and would like to

173
proceed with acne treatment only. What other clinical exam should you perform to screen for a
potential malignancy to which this patient is at higher risk for acquiring?
A. Testicular
B. Abdominal
C. Lymph node
D. Lung
E. Rectal
►A

Men with x-linked ichthyosis are at increased risk of testicular cancer and cryptorchidism. A
testicular exam is simple to perform and a good screening exam for detecting testicular
abnormalities. The remaining exams are not useful as these patients are not at higher risk for
other types of cancer.

95
Nail patella syndrome is inherited in an autosomal dominant fashion. The eye finding for this
disease with hyperpigmentation of the pupillary margin of the iris seen in 45% of patients is also
called:
A. Lester iris
B. Heterochromia of the iris
C. Microcornea
D. Glaucoma
E. Cataracts
►A

Nail patella syndrome is also known as hereditary osteonychodysplasia (HOOD) and is inherited
in an autosomal dominant fashion. The defect is in the gene LMX1B. The ocular abnormality is
Lester iris seen in 45% of patients.

96
Ichthyosis hystrix is characterized by the following gene defects?
A. Keratins 1 and 9
B. Keratins 1 and 10
C. Keratins 5 and 14
D. Keratins 6 and 16
E. None of these answers are correct
►B

174
Ichthyosis hystrix or extensive epidermal nevi occurs secondary to a somatic mosaicism for
keratins 1 and 10. If the mosaicism occurs on gonadal cells, offspring may have full blown
epidermolytic hyperkeratosis (EHK).

97
A double row of eyelashes is associated with:
A. Lymphedema-distichiasis syndrome
B. Cornelia de Lange syndrome
C. Rubinstein-Taybi syndrome
D. Russell-Silver syndrome
E. Hunters syndrome
►A

A double row of eyelashes is defined as distichiasis and is associated with the Lymphedema-
distichiasis syndrome. This syndrome is transmitted in an autosomal dominant fashion and is
related to a mutation in FOXC2. Findings include late onset lymphedema, distichiasis, corneal
irritation, ectropion, webbed neck and congenital heart defects. The remaining syndromes do not
include distichiasis as a feature.

98
An infant presents with poikiloderma on his face, buttocks, arms and legs. He is also noted to
have a hypoplastic thumb and no radius. Yearly ophthalmologic examination is indicated
because of the infant is at risk for developing:
A. Glaucoma
B. Cataracts
C. Subcapsular lens displacement
D. Copper deposition
E. Macular degeneration
►B

The patient described has Rothmund-Thomson syndrome (or poikiloderma congentiale), an


autosomal recessive disease localized to chromosome 8 and believed to be due to a DNA
helicase mutation (RECQL4). 40-50% of patients will develop juvenile cataracts before puberty.
Other clinical features include alopecia, dystrophic nails, short stature, hypogonadism and dental
dysplasia.

99
Which of the following is NOT part of the Carney complex?
A. Peg or conical teeth
B. Cardiac, cutaneous or mammary myxomas

175
C. Pigmented skin lesions
D. Endocrine abnormalities
E. Primary pigmented nodular adrenocortical disease
►A

Peg/conical teeth are not part of the Carney complex. This is found in incontinentia pigmenti and
anhidrotic ectodermal dysplasia. The remaining skin findings are part of this complex sometimes
known as NAME syndrome. It consists of multiple, diffuse mucocutaneous lentigines, cardiac
and subcutaneous myxomas and endocrine abnormalities may be present. Other findings include:
testicular tumors, thyroid disease, primary pigmented nodular adrenocortical disease,
psammomatous melanotic schwannomas and hormone-secreting pituitary adenomas.

100
A patient has soft compressible blue tumors that are seen in the trunk and arms. There is also
nocturnal pain. This condition is also caused by a mutation in the VMCM1 gene:
A. Blue Rubber Bleb Nevus Syndrome
B. Kaposi's Sarcoma
C. Henoch-Schonlein Purpura
D. Hereditary Hemorrhagic Telangiectasia
E. Olser-Weber Rendu
►A

This patient has all the clinical signs of blue rubber bleb nevus syndrome. These patient can also
have melena that occurs with gastrointestinal hemangioma rupture. Other involvement are in the
lung, eye and CNS.

101
The arylsulfatase E gene is mutated in which disease?
A. X-linked ichthyosis
B. Refsum syndrome
C. Haim-Munk syndrome
D. Naxos syndrome
E. Griscelli syndrome
►A

Arylsulfatase E is also known as steroid sulfatase and is mutated in X-linked ichthyosis. This
condition is inherited in a X-linked recessive pattern. Clinical findings include: brown scale
sparing palms, soles and flexures, comma-shaped corneal opacities, failure of labor progression
and cryptorchidism. It is also mutated in X-linked recessive type chondrodysplasia punctata.

176
102
The syndrome characterized by generalized mild hyperkeratosis, erythematous keratotic plaques,
palmoplantar keratoderma, non-progressive sensorineural deafness, progressive bilateral keratitis
with secondary blindness is:
A. KID syndrome
B. Vohwinkel syndrome
C. Erythrokeratoderma variabilis
D. CHILD syndrome
E. Refsum syndrome
►A

KID syndrome is described above. It is an autosomal dominant mutation in connexin 26.


Vohwinkel syndrome is also a connexin 26 mutation, but is characterized by diffuse
honeycombed palmoplantar keratoderma, pseudoainhum, starfish-shaped keratotic plaques over
joints and deafness. Erythrokeratoderma variabilis is an autosomal dominant mutation in
connexin 31 and 30.3 characterized by erythematous migratory patches, fixed hyperkeratotic
plaques and a palmoplantar keratoderma. CHILD syndrome is an X-linked dominant mutation
condition due to a mutation in NAD(P)H Steroid dehydrogenas-like protein, lethal in males.
Unilateral ichthyosiform erythroderma, limb/visceral hypoplasias are characteristic. Refsum
syndrome is an autosomal recessive condition with a mutation in phytanoyl coenzyme A
hydroxylase characterized by mild ichthyosis, cerebellar ataxia, peripheral neuropathy, retinitis
pigmentosa (salt & pepper) and deafness.

103
The most common cardiovascular defect in patients with Noonan syndrome is:
A. Atrial septal defect
B. Ventricular septal defect
C. Enlarged aorta
D. Pulmonic valve stenosis
E. Aortic stenosis
►D

Noonans syndrome is similar to cardiofaciocutaneous syndrome as they both have mutations in


PTPN11. Noonan patients have short stature, ptosis, hypertelorism, low-set ears, thick lips and
curly hair. Pulmonic valve stenosis is the most common cardiovascular defect, with atrial septal
defects also seen.

104
This form of EB simplex has a defect in actin assembly, not keratin formation.
A. Dowling Meara

177
B. Weber-Cockayne
C. Koebner
D. Kindler
E. Ogna
►D

Kindler syndrome has been reclassified as a subtype of EB simplex due to congenital blistering
being the first symptom. These patients do not demonstrate defects in keratin proteins. Instead,
the KIND1 gene defect leads to disruption of actin assembly.

105
Which of the following syndromes is associated with hematologic abnormalities?
A. Sturge Weber disease
B. Klippel-Trenaunay-Parks-Weber
C. Kasabach-Merritt syndrome
D. Blue rubber bleb nevus syndrome
E. Zollinger-Ellison syndrome
►C

Kasabach-Merritt syndrome is associated with hematologic abnormalities, such as


thrombocytopenia, microangiopathic hemolytic anemia, disseminated intravascular coagulation.
The condition develops from platelet-trapping within a large hemangioma, most commonly a
kaposiform hemangioendothelioma in the retroperitoneal location.

106
Patients that have been diagnosed with hypohidrotic ectodermal dysplasia have the hair findings
of:
A. Longitudinal groove on electron microscopy
B. Trichorrhexis nodosum
C. Monilethrix
D. Beaded hair
E. Pili trianguli et canaliculi
►A

Patients with hypohidrotic ectodermal dysplasia have longitudinal groove on electonr


microscopy. They also have peg teeth with problems with sweating.

107
The most common ocular association with cutis marmorata telangiectatica congenital is:
A. Cataracts

178
B. Glaucoma
C. Retinoblastoma
D. Corneal opacity
E. Angioid streaks
►B

Glaucoma is the most common associated eye finding in CMTC patients. Glaucoma is also seen
in patients with neurofibromatosis type 1 and Sturge Weber patients.

108
A child presents with the hair finding seen in the image in addition to brittle nails, keratosis
pilaris, abnormal teeth and cataracts. Which of the following abnormalities is the most likely
mutated?
A. Keratin 1/10
B. Keratin hHb1/hHb6
C. Keratin 6/16
D. Keratin 6/17
E. Keratin 2e
►B

Keratin hHb1/hHb6 is defective in monilethrix, which is described above. Keratin 1/10 defects
are found in epidermolytic hyperkeratosis, K6/16 in inflamed skin and pachyonychia congenita
type I, K6/17 in pachyonychia congenita type II and K2e in Ichythosis bullosa of Siemens.

109
A patient with Bloom Syndrome is most likely to have which laboratory abnormalities:
A. Decreased immunoglobulins
B. Macrocytic anemia
C. Elevated IgE
D. Thrombocytopenia
E. Positive ANA
►A

Bloom syndrome is an autosomal recessive disorder due to a mutation in the BLM gene which
codes for a DNA helicase. Patients have impaired DNA repair after UV exposure and increased
photosensitivity. Clinical features include photodistributed erythema, cheilitis, high-pitched
voice, hypogonadism, and increased risk for leukemia, lymphoma and GI adenocarcinoma.
Laboratory evaluation reveals decreased IgA, IgM and IgG leading to increased risk of
respiratory infections

179
110
Which of the following laboratory test might prove useful in the diagnosis of Fabry disease:
A. Complete blood count with differential
B. Fasting lipids
C. Urinary sediment exam with polarizing light microscopy
D. Stool guaiac
E. Bleeding time
►C

Patients with Fabry disease have a defect in the alpha-galactosidase A enzyme, leading to an
accumulation of glycosphingolipids in all tissues. Although patients are at increased risk for
myocardial infrctions and strokes, the serum lipid levels are normal. Ischemic events occur as a
result of glycosphingolipid accumulation in endothelial cells leading to swelling. In the brain,
strokes occur from direct vessel occlusion or stretching and distention of branches of
dolichoectatic parent vessels. Deposits in the kidneys leads to progressive renal failure with urine
exam exhibiting proteinuria and birefringent lipid globules (―maltese crosses‖) seen with
polarizing light microscopy.

111
Osteopathia striata is seen in which of the following disorders?
A. Gaucher‘s disease
B. Albright‘s syndrome
C. Aplasia cutis congenita
D. Focal dermal hypoplasia
E. Scleroderma
►D

Focal dermal hypoplasia (Goltz syndrome) is an X-linked dominant disorder that is lethal in
males. There is linear atrophy following Blaschko‘s lines with areas of fat herniation with
underlying osteopathia striata, which is radiologically characterized by linear bony hyperdensity.
Other features include mucocutaneous papillomas and pits, alopecia, nail dystrophy, tooth
abnormalities, and colobomas.

112
Christ-Siemens-Touraine Syndrome is most commonly linked with defects in which of the
following genes?
A. NEMO
B. Ectodysplasin (EDA)
C. ERCC2
D. ATP7A

180
E. None of these options are correct
►B

Ectodysplasin (EDA) on Xq12-q13 is transmitted in an X-linked recessive fashion and is the


most common cause of anhidrotic ectodermal dysplasia (Christ-Siemens-Touraine syndrome or
hypohidrotic ectodermal dysplasia). NEMO can be linked to this syndrome and is associated
with immunodeficiency, but this is a rare association. ERCC2 is associated with
trichothiodystrophy and ATP7A with Menkes kinky hair syndrome. They are not associated with
anhidrotic ectodermal dysplasia.

113
A patient with a port wine stain covering one enlarged leg likely has which of the following
associated symptoms?
A. Lymphatic and deep venous insufficiency
B. Visceromegaly with omphalocele
C. Bilateral retinal hemangioblastomas
D. Enchondromas
E. Distichiasis
►A

Klippel-Trenaunay-Weber syndrome is a sporadic condition characterized by port-wine stains


typically covering one lower extremity that is enlarged with underlying lymphatic and deep
venous insufficiency.

114
The most common neoplasm seen in Maffucci Syndrome is:
A. Enchondromas
B. Angiosarcomas
C. Osteosarcomas
D. Lymphangiosarcomas
E. chondrosarcoma
►A

Maffucci syndrome comprises of superficial and deep venous malformations, enchondromas, and
short stature. Enchondromas are the most common neoplasm, while chondrosarcomas are the
most common malignancies.

115
Which ocular finding may be seen in a patient with this skin condition?
A. Comma-shaped corneal opacities

181
B. Retinitis pigmentosa
C. Congenital hypertrophy of the retinal pigmented epithelium
D. Angioid streaks
E. Pingueculae
►D

Pseudoxanthoma elasticum is caused by a defect in connective tissue. Angioid streaks develop


when a rupture occurs in Bruch's membrane.

116
The gene defect in LEOPARD syndrome is:
A. PTPN11
B. PRKAR1A
C. KIP2
D. ATM
E. Neurofibromin
►A

PTPN11 gene is mutated in LEOPARD syndrome. This syndrome consists of the complex of
Lentigines, EKG abnormalities, Ocular hypertelorism, Pulmonary stenosis, Abnormal genitalia,
Retardation of growth and Deafness. PRKAR1A is associated with the Carney complex of
diseases. KIP2 is found in 15% of cases of Beckwith-Wiedermann syndrome. ATM is mutated in
ataxia telangectasia syndrome and Neurofibromin in Neurofibromatosis type I.

117
Ectopia lentis (downward displacement of the lens) is characteristic of:
A. Marfan syndrome
B. Homocystinuria
C. Phenylketonuria
D. Multiple Carboxylase deficiency
E. Cutis laxa
►B

Ectopia lentis (downward displacement) is seen in homocystinuria. Upward displacement is seen


in Marfan syndrome. There are no changes in the lens in phenylketonuria or multiple
carboxylase deficiency.

118
Which of the following is defective in Ehlers-Danlos syndrome (EDS) with congenital adrenal
hyperplasia?

182
A. Tenascin-X
B. Lysyl oxidase
C. Lysyl hydroxylase
D. None of these answers are correct
E. All of these answers are correct
►A

Tenascin-X defects are associated with EDS and with congenital adrenal hyperplasia. The
phenotype is that of typical EDS with hyperextensible skin, hypermobile joints, and tissue
fragility. Lysyl oxidase is defective in X-linked EDS (type V) and Occipital horn syndrome (type
IX). Lysyl hydroxylase is defective in ocular-scoliotic (type VI) EDS.

119
A child with phenylketonuria likely presents with which cutaneous problems?
A. Blue-gray generalized hyperpigmentation
B. Alopecia universalis
C. Generalized hypopigmentation
D. Generalized hyperpigmentation
E. Leg ulcers
►C

Phenylketonuria is an autsomal recessive disorder caused by a mutation on the long arm of


chromosome 12. A deficiency of phenylalanine hydroxylase or its cofactor tetrahydrobiopterin
leads to accumulation of phenylalanine. Clinical features include generalized hypopigmentation,
eczematous dermatitis, sclerodermoid changes, seizures, psychomotor delay, urine with ―mousy‖
odor, mental retardation.

120
What is the classic radiologic findings associated with this disorder?
A. Dural calcifications
B. Calcifications of the falx-cerebri
C. Tram-track calcifications of the temporal and occipital cortex
D. Osteopatha striata
E. Osteopoikilosis
►C

Sturge-Weber syndrome is a sporadic disroder characterized by a facial capillary malformation


in a trigeminal nerve distribution. Patients with Sturge-Weber may have cerebral atrophy,
ipsilateral vascular malformations of the leptomeninges, seizures, and glaucoma. The classic
radiologic finding is tram-track calcifications of the temporal and occipital cortex.

183
121
A 7 year old girl with abnormally short hair has progressive seizures, lethargy, ataxia, and mental
retardation. A blood test reveals elevated levels of ammonia. What is the affected gene?
A. Argininosuccinase
B. ATP7A
C. Ectodysplasin A
D. Connexin 30
E. p63
►A

Argininosuccinic aciduria is an autosomal recessive disorder caused by mutations in


argininosuccinase. It is characterized by trichorrhexis nodosa, hyperammonemia, hepatomegaly
and vomiting, seizures, lethargy, coma, ataxia, and mental retardation.

122
A 20-year-old woman presents with hypodontia, sparse hair, palmoplantar hyperkeratosis, and
nail dystrophy. Examination of her eyelids reveal multiple, translucent-appearing papules. The
most likely gene defect is:
A. WNT10A
B. PTCH
C. CYLD
D. BRAF
E. PTEN
►A

Schopf-Schulz-Passarge syndrome is characterized by multiple apocrine hydrocystomas (most


commonly appearing on the eyelids) and syringofibroadenomas, in addition to hypodontia,
hypotrichosis, onychodystrophy, and palmoplantar keratoderma. It is inherited as an autosomal
recessive condition and is caused by mutations in WNT10A, which encodes a cutaneous
signaling molecule involved in ectodermal appendageal development.

123
A teenage female presents with the complaint of "nail fungus". On exam, she has triangular
lunulae, palmoplantar hyperhidrosis, micronychia and an absent patella. Which of the following
screening tests should you order first?
A. Urinalysis
B. CBC
C. Fasting lipids
D. Renal ultrasound

184
E. X-ray of the knees, elbows and pelvis
►A

Patients with nail-patella syndrome can have glomerulonephritis and renal dysplasia leading to
renal failure. Screening with a urinalysis is a reasonable first test. If this is abnormal, referral for
a renal ultrasound could be useful. Fasting lipids and a CBC are not indicated.

124
The porphyrias are a group of diseases related by abnormal heme synthesis. Which is the only
porphyria that is inherited in an autosomal recessive manner?
A. Congenital erythropoietic porphyria
B. Variegate porphyria
C. Porphyria cutanea tarda
D. Acute intermittent porphyria
E. Erythropoietic protoporphyria
►A

The porphyrias are generally inherited in an autosomal dominant manner. Congenital


erythropoietic porphyria is unique in being inherited in an autosomal recessive manner. Helpful
mnemonic: all porphyrias are autosomal dominant exCEPt one.

125
A 4 year old girl suffers multiple fractures. She also has thin skin, easy bruising, and blue sclera.
An echocardiogram reveals mitral valve prolapse. What type of osteogenesis imperfecta does she
most likely have?
A. Type I
B. Type II
C. Type III
D. Type IV
E. Type V
►A

Osteogenesis imperfect is a defect in collagen I which leads to thin skin, easy bruising, blue
sclera, and multiple fractures. Mitral valve prolapse is seen especially in Type I.

126
Which one of the following is the most common oncogenic virus in patients with
epidermodysplasia verruciformis?
A. HPV-5
B. HPV-8

185
C. HPV-13
D. HPV-16
E. HPV-33
►A

Epidermodysplasia verruciformis is a rare autosomal recessive disorder in which an impaired


cellular immunity allows widespread infection with certain subtypes of the human papilloma
virus (HPV). Some of these lesions have a tendency for malignant transformation, most
commonly those verruca caused by HPV type 5.

127
Collagen III is mutated in which type(s) of Ehlers-Danlos syndrome (EDS)?
A. All of the answers are correct
B. None of the answers are correct
C. Benign Hypermobile (type III)
D. Vascular (type IV)
E. Periodontitis (type VIII)
►A

Collagen III is mutated in all three types of EDS listed. Benign hypermobile type EDS is
associated with hypermobile joints and is autosomal dominant (AD) in transmission. Vascular
type EDS is associated with arterial and visceral rupture leading to early death, and visible
venous patterns. It is transmitted autosomal recessive (AR) or AD. Periodontitis type EDS is
associated with mild EDS symptoms and periodontitis.

128
Painful crises and 'whorled' corneal opacities are seen with which of the following enzyme
abnormalities?
A. Homogentisic acid oxidase
B. Alpha-galactosidase A
C. Glucocerebrosidase
D. Iduronate sulfatase
E. Glucoronidase
►B

Painful crises and whorled corneal opacities are found in Fabry disease which is caused by a
defect in alpha-galactosidase A. The remaining conditions do not have these findings.

129
Underlying defect for the disease shown in picture is

186
A. ATP2A2
B. ATP2C1
C. BPAG1
D. BPAG2
E. Collagen type 17
►B

The disease shown in image is Hailey-Hailey disease (Familial Benign Pemphigus) which is an
autosomal dominant genodermatosis, caused by mutation in ATP2C1, encoding a calcium pump
protein related to SERCA2. It is characterized by recurrent vesicles and erosions, which most
commonly appear on the sides and back of the neck, in the axillae, in the groin, and in the
perianal regions. The disorder is not seen before puberty and usually has its onset in the late
teens or early 20s. In the intertriginous area lesions tend to form erythematous plaques with dry
crusting and soft, flat, and moist granular vegetations. Burning or pruritus is common, and,
particularly in the intertriginous areas, lesions tend to become irritating, painful, and exceedingly
uncomfortable. ATP2A2 is underlying defect in Darier's disease, other choices are defects seen
in pemphigoid and epidermolysis bullosa.

130
Lamellar ichthyosis is caused by mutations in which of the following genes?
A. Steroid sulfatase
B. Fatty aldehyde oxidoreductase
C. Phytanoyl coenzyme A hydroxylase deficiency
D. Arylsulfatase E
E. Transglutaminase
►E

Nonbullous congenital ichthyosiform erythroderma and lamellar ichthyosis are caused by


mutations in the transglutaminase gene.

131
A 11 year-old female patient with hypoparathyroidism is referred to your clinic secondary to
chronic mucocutanous candidiasis which is refractory to standard treatments. The patient also
has malabsorption and severe chronic diarrhea. You determine that she has autoimmune
polyendocrinopathy-candiasis-ectodermal dystrophy syndrome(APECED). Since only two of
three major criteria are needed to make this diagnosis, this patient having demonstrated
hypoparathyroidism and chronic mucocutanous candidiasis is diagnosed with APECED. What is
the third major criteria that would qualify a patient to meet the diagnosis of APECED?
A. Ectodermal dysplasia
B. Insulin dependent diabetes mellitus

187
C. Chronic autoimmune hepatitis
D. Addison's disease
E. Hypothyroidism
►D

This patient has APECED, also know as autoimmune polyendocrinopathy syndrome type 1(APS
type 1). Two of three major criteria are needed to make this diagnosis - chronic mucocutaneous
candidiasis, hypoparathyroidism and Addison's disease. They usually present within the first 5
years of life with chronic mucocutaneous candidiasis, then before the age of ten with
hypoparathyroidism, and finally in early adulthood with Addison's disease. They often present
with chronic diarrhea and malabsorption. Ectodermal dysplasia usually does not present until the
fifth decade. Autoimmune skin disease, such as vitiligo and alopecia areata are not uncommon.

132
Acropigmentation of Dohi is characterized by:
A. Reticulated pigmentation of the axillae, neck, and groin
B. Linear palmar pits and pigmented macules on volar and dorsal hands and feet
C. Pigmented and depigmented macules on the distal dorsal extremities and face
D. Hyperpigmented macules on the lips and oral mucosa
E. Flaccid, superficial pustules that burst and leave pigmented macules
►C

Patients with Acropigmentation of Dohi (dyschromatosis symmetrica hereditaria) are usually


from Europe, India, or the Carribean. They develop pigmented and depigmented macules on
dorsal distal extremities and face. This disorder is due to mutation in DSRAD gene. Reticulated
pigmentation of the axillae, neck, and groin is seen in Dowling-Degos' disease. Linea palmar pits
and pigmented macules on volar and dorsal hands an feet is seen in Reticulate acropigmentation
of Kitamura. Hyperpigmented macules on oral mucosa and lips can be seen in Peutz-Jeghers,
Cronkite-Canada, and Laugier-Hunziker syndromes. Flaccid, superficial pustules that burst and
leave pigmented macules is seen in transient neonatal pustulosis.

133
Beckwith-Wiedemann syndrome is characterized by which of the following triads?
A. Hemangioblastomas, renal cysts and renal cell carcinoma
B. Epistaxis, telangictases, and gastrointestinal tract bleeding
C. Enlarged limb, port wine stain, and deep venous thrombosis
D. Omphalocele, venous malformations, and ataxia
E. Exomphalos, macroglossia, and gigantism
►E

188
Beckwith-Wiedemann syndrome is also known as EMG syndrome as it includes exomphalos,
macroglossia, and gigantism. It is usually a sporadic condition but is sometimes caused by
autosomal dominant mutations in p57. Clinical features include facial capillary malformations,
macroglossia, visceromegaly with omphalocele, and hemihypertrophy associated with tumors
(especially Wilm‘s tumors).

134
A patient with multiple sebaceous adenomas should be screened with which of the following
examinations?
A. Retinal examination
B. Laryngoscopy
C. Colonoscopy
D. MRI of the spine
E. Renal ultrasound
►C

Muir-Torre syndrome is an autosomal dominant disorder caused by the HMSH2 and MLH1
DNA mismatch repair genes. Clinically, there are numerous sebaceous adenomas, epitheliomas
and carcinomas and multiple keratoacanthomas associated with indolent colon and other visceral
adenocarcinomas. Patients and first-degree relatives should be screened by colonoscopy as
colonic adenocarcinomas may precede the development of cutaneous tumors.

135
Hereditary Hemorrhagic Telangiectasia syndrome is transmitted in an autosomal dominant
fashion and can have two variants. Type I is linked to defects in HHT1, the endoglin gene. Type
II is linked to defects in HHT2, the ALK1 gene. What feature that differentiates type I from type
II clinically?
A. Type I families have an increase incidence of pulmonary arteriovenous fistulas
B. Type II families have an increased incidence of pulmonary arteriovenous fistulas
C. Type I families have an increased incidence of hepatic arteriovenous
malformations
D. Type II families have a decreased incidence of hepatic arteriovenous
malformations
E. None of the answers are correct
►A

There is an increased incidence of pulmonary arteriovenous fistulas in HHT type I. Type II has
an increased incidence of hepatic arteriovenous malformations. Note ALK1 ACVRL1.

136

189
Which of the following elastic tissue diseases demonstrates calcified elastic fibers?
A. Cutis laxa
B. Marfan syndrome
C. Anetoderma
D. Pseudoxanthoma elasticum
E. Buschke-ollendorf syndrome
►D

Pseudoxanthoma elasticum is usually an autosomally recessive inherited condition due to a


defective transport protein, ABCC6. The clinical manifestations of the disease arise from
fragmented and calcified fibers of the skin, eyes and arteries. Patients may have yellow papules,
loose redundant skin, angioid streaks and hemorrhage. Histologically, the hallmark of
pseudoxanthoma elasticum is calcified elastic fibers.

137
A patient presents with starfish keratoses, pseudoainhum, honeycombed PPK, and generalized
ichthyosis. What is the most likely genetic defect?
A. Connexin 31
B. Connexin 26
C. Connexin 30
D. Loricrin
E. Connexin 33
►D

The patient has Vohwinkel syndrome. This is an autosomal dominant syndrome with 2 clinical
variants. The variant described above with generalized ichthyosis is due to a loricrin mutation. In
the classic form with nonprogressive hearing loss connexin 26 is mutated.

138
Patients with progeria typically die of which of the following conditions?
A. Infection
B. Metastatic carcinoma
C. Atherosclerotic heart disease
D. Nail atrophy
E. Progressive systemic sclerosis
►C

Progeria (Hutchinson-Gilford syndrome) is a sporadic condition characterized by lipoatrophy,


sclerodermoid skin, alopecia, nail atrophy, craniomegaly with small face, muscle/bone wasting,
and severe premature atherosclerosis resulting in early death.

190
139
Mutations affecting the VEGF receptor-3 cause which of the following disorders?
A. Hereditary lymphedema (Nonne-Milroy disease)
B. Lymphedema-distichiasis syndrome
C. Lymphedema and ptosis
D. Noonan syndrome
E. Hereditary hemorrhagic telangiectasias
►A

Hereditary lymphedema (Nonne-Milroy disease) is an autosomal dominant condition caused by


mutations in the FLT4 gene which encodes for VEGF receptor-3. There is congenital
lymphedema and chylous ascites, scrotal swelling, intestinal tract protein loss, persistent bilateral
pleural effusion, and hypoproteinemia.

140
Which of the following is a feature of Neurofibromatosis type II?
A. Congenital hypertrophy of the retinal pigment epithelium
B. Lisch nodules
C. Juvenile posterior subcapsular lenticular opacities
D. Lester iris
E. Optic gliomas
►C

Neurofibromatosis type II is an autosomal dominant disorder caused by mutations in


schwannomin/merlin. Clinical features include cutaneous schwannomas and neurofibromas,
bilateral vestibular schwannomas, and juvenile posterior subcapsular lenticular opacities. Optic
gliomas are usually seen in NF1.

141
Which syndrome is due to a defective secreted mammilian Ly6/uPAR-related protein-1?
A. Netherton's syndrome
B. Refsum's syndrome
C. Sjogren-Larsson syndrome
D. Mal de Meleda syndrome
E. Haim-Munk syndrome
►D

Mal de Meleda, also known as keratoderma palmoplantaris transgrediens, is due to a defect in


secreted mammilian Ly6/uPAR-related protein or SLURP-1.

191
142
You are examining a child with mild albinism, immunodeficiency and silver grey highlights in
his hair. You diagnose the child with Chediak-Higashi syndrome. Why are you confident that
this isn‘t Griscelli syndrome?
A. Giant lysosomal granules are present in neutrophils in the blood smear
B. Griscelli syndrome does not have albinism as a feature
C. Griscelli syndrome has no changes in hair color
D. All of these answers are correct
E. None of these answers are correct
►A

Chediak-Higashi syndrome and Griscelli syndrome have similar features including silver-grey
highlights of hair, immunodeficiency, mild albinism and an accelerated phase of disease.
Examining a peripheral blood smear is helpful in distinguishing between these two syndromes.
Patients with the LYST defect (a lysosomal storage transport gene) have Chediak-Higashi
syndrome and will have giant lysosomal granules visible in white blood cells on a blood smear.

143
Marfan's syndrome is characterized by striae distensae, abdominal wall hernia, elastosis
perforans serpiginosa, and tall stature. The genetic defect is:
A. Fibrillin 1
B. Intermediate fibers 1
C. Actin 5
D. Req12
E. PTPN11
►A

Marfan's syndrome is caused by a mutation in the fibrillin 1, it is a major component of


intermediate fibrils found in the skeletal, ocular and cardiovascular system. The cardiac
irregularities are aortic root aneurysms, rupture and dissection.

144
A 9 year old boy has sparse, short, brittle hair. A blood test reveals low serum copper. What is
the inheritance pattern of this disease?
A. X-linked recessive
B. X-linked dominant
C. Autosomal dominant
D. Autosomal recessive
E. Mitochondrial

192
►A

Menkes kinky hair syndrome is an x-linked recessive disorder caused by defects in ATP7A, an
ATP-dependent copper transporter. It is characterized by pili torti, trichorrhexis nodosa, short,
britle hair, lax skin, CNS deterioration, seizures, and tortuous arteries.

145
Primary pigmented nodular adrenocortical disease and psammomatous melanotic schwannomas
are characteristic of which of the following syndromes?
A. Hypomelanosis of Ito
B. Carney complex
C. McCune-Albright syndrome
D. Gaucher‘s syndrome
E. Tuberous sclerosis
►B

Carney complex is an autosomal dominant disorder caused by mutations in PRKAR1A (protein


kinase A regulatory subunit 1-alpha). Key features include cardiac, cutaneous and mammary
myxomas, pigmented skin lesions, endocrine abnormalities (pituitary, testicular, thyroid, etc),
primary pigmented nodular adrenocortical disease, and psammomatous melanotic schwannomas.

146
Which of the following medications is a teratogen associated with a aplasia cutis congenita?
A. Propranolol
B. Methimazole
C. Lithium
D. Alcohol
E. Warfarin
►B

Aplasia cutis congenita is characterized by well-demarcated erosions at birth that heal with
atrophic, alopecic scars. Some cases are caused by medications, with methimazole considered a
teratogen particularly associated with this condition.

147
Junctional epidermolysis bullosa with pyloric atresia is associated with mutations in:
A. The alpha-6 subunit of integrin
B. The beta-4 subunit of integrin
C. Both subunits of integrin can have mutations causing this type of junctional
epidermolysis bullosa

193
D. Plectin
E. Laminin 5
►C

Both subunits of integrin can have mutations causing this type of junctional epidermolysis
bullosa. Plectin is associated with epidermolysis bullosa simplex with muscular dystrophy.
Laminin 5 is mutated in Herlitz and non-Herlitz types of junctional epidermolysis bullosa.

148
What is the most common ocular findings seen in this condition?
A. Angioid streaks
B. Phakomas
C. Lisch nodules
D. Bitot‖s spots
E. Coloboma
►A

Pseudoxanthoma elasticum, PXE, is an inherited disorder that affects selected connective tissue
in some parts of the body. Elastic tissue in the body becomes mineralized, that is, calcium and
other minerals are deposited in the tissue. The most common ocular finding is angioid streaks.
The other ocular findings are seen in these syndromes: Phakomas is seen in tuberous sclerosis,
Lisch nodules in neurofibromatosis, Bitot‖s spots in Vitamin A deficiency and coloboma in focal
dermal hypoplasia.

149
A 2 year old patient is small for his age. He has thin, pale, translucent skin, with prominent
veins. His father died from aortic rupture when he was in his twenties. Vascular Ehlers-Danlos is
caused by mutations in what gene?
A. Type III Collagen
B. Tenascin-X
C. Type V Collagen
D. Lysyl hydroxylase
E. Type I Collagen
►A

Vascular Ehlers-Danlos (Type 4) is characterized by the skin findings discussed above, as well
as the onset of severe complications of fragile blood vessels and organs in early adulthood. The
disease is inherited in an autosomal dominant manner. Defects are seen in type III collagen.
Tenascin-X mutations are seen in hypermobile Ehlers-Danlos (Type 3), type V collagen in

194
classical Ehlers-Danlos (Type 1 and 2), lysyl hydroxylase in kyphoscoliosis Ehlers-Danlos (Type
6), and type I collagen in Ehlers-Danlos types 1, 2, and 7.

150
You are consulted on a patient with possible Nethertons Syndrome. Which location of the body
would most likely have hairs demonstrating trichorrhexis invaginata?
A. Scalp
B. Eyebrow
C. Eyelash
D. All of these answers are correct
E. None of these answers are correct
►B

Eyebrow hair is most common site with hairs demonstrating trichorrhexis invaginata.

151
A 3 year old boy has white hair of the central frontal scalp and depigmented symmetrical patches
on the knees since birth. No ocular abnormalities or deafness are noted. What is true of this
disorder?
A. It is an autosomal recessive disorder of melanocyte development
B. There is typically no progression of depigmented patches
C. Regression of the white forelock has been noted
D. One form of this condition is associated with iris pigmentary abnormalities
E. Results from a mutation in a tumor suppressor gene
►B

The patient in this scenario has piebaldism, an autosomal dominant disorder of pigmentation
caused most commonly by dominant negative missense mutations in the KIT proto-oncogene.
Piebaldism almost always has no progression of the depigmented patches, and is thought to be a
static disoder. One exception was noted in a family with a novel Val620Ala (1859T>C) mutation
in the KIT gene where development of new depigmented patches did occur. Regression of the
white forelock has been noted in select cases. Piebaldism is thought to be unresponsive to
systemic treatments or phototherapy. If the patient presented with either eye abnormalities or
deafness, Waardenburg syndrome would be considered in the diagnosis.

152
Mutation in lamin A (nuclear envelope protein) has been found in:
A. Peutz-Jeghers syndrome
B. Buschek-Ollendorf syndrome
C. Progeria (Hutchinson-Gilford)

195
D. Albright‖s syndrome
E. Marfan syndrome
►C

Progeria (Hutchinson-Gilford syndrome) is a sporadic condition characterized by lipoatrophy,


sclerodermoid skin, alopecia, nail atrophy, craniomegaly with small face, muscle/bone wasting,
and severe premature atherosclerosis resulting in early death. Recent studies have shown that
mutations in nuclear envelope protein lamin A is associated with progeria.

153
Which of the following immunoglobulins is commonly decreased in Wiskott-Aldrich
syndrome?
A. IgA
B. IgD
C. IgE
D. IgM
E. IgG
►D

IgM is decreased in WAS. IgA, IgD and IgE levels are all elevated. IgG is not abnormal in WAS.

154
A 16 year-old girl presents with a family history of Gardner syndrome. Her mother is very
concerned that her daughter may have the syndrome as it runs in her family and she has many
skin complaints. Gardner syndrome has been linked to defects in beta-catenin mediated
transcription. Which of the following genes dysfunction is responsible?
A. APC
B. STK11
C. CYLD
D. PTCH
E. ABCC6
►A

APC gene has been associated with defects in beta-catenin mediated transcription. The
remaining genes are not associated with Gardner syndrome.

155
Which of the following are cutaneous features of Marfan syndrome?
A. Loose skin and crumpled ears
B. Dermatofibrosis lenticularis and striae

196
C. Fat herniation and cutaneous atrophy
D. Sclerodermoid changes and dyspigmentation
E. Striae and elastosis perforans serpiginosa
►E

Marfan syndrome is an autosomal dominant disorder caused by mutations in fibrillin 1 and 2.


Patients have tall stature, arachnodactyly, pectus excavatum, high-arched palate, joint laxity,
ectopia lentis with upward dislocation, dilated aorta with rupture, mitral valve prolapse, striae,
and elastosis perforans serpiginosa.

156
A patient that has patchy leukoderma containing smaller spots of hyperpigmentation and a white
forelock with CNS abnormalities with impaired motor coordination has a defect in the:
A. c-KIT gene
B. PAX 3 gene
C. MITF gene
D. SOX10 gene
E. EDN3 gene
►A

This patient has piebaldism, which is autosomal dominant caused by a defect in the porto-
oncogene c-KIT. The gene encodes tyrosine kinase receptor on melanocytes. It is associated with
CNS abnormalities with motor coordination, cerebellar ataxia, mental retardation, and deafness.

157
A patient has multiple erythematous papules with central keratotic plug with many annular or
serpiginous patters on the neck. This condition is associated with all the following except for:
A. Marfan's syndrome
B. Down's syndrome
C. Osteogenesis imperfecta
D. Ehlers-Danlos syndrome
E. Pseudoxanthoma elasticum
►E

The patient has elastosis perforans serpiginosa and is associated with medications and certain
systemic conditions. It is associated with the medicine penicillamine, Marfan's syndrome,
Down's, osteogenesis imperfecta, and Ehler's Danlos syndrome. It is often self limiting.

158

197
A patient you notice has unruly hair that looks like spun-glass hair. She is found to have
uncombable hair syndrome, which is a condition with the associated hair findings of:
A. Pili trianguli et canaliculi
B. Beaded hair
C. Trichorrhexis nodosa
D. Longitudinal groove on electron microscopy
E. Monilethrix
►A

Uncombable hair syndrome has the characteristic findings of spun-glass hair or cheveux
incoiffables. It is associated with pili trianguli et canaliculi and can be inherited in an autosomal
dominant or autosomal recessive fashion.

159
All of the following disorders are exacerbated by UV radiation except:
A. Bloom syndrome
B. Hartnup‘s disease
C. Refsum syndrome
D. Cockayne syndrome
E. Rothmund-Thomopson syndrome
►C

Refsum‘s syndrome is an autosomal recessive disorder caused by mutations in phytanoyl-CoA


hydroxylase. Clinically, patients have mild icthyosis, cerebellar ataxia, polyneuropathy, salt and
pepper retinitis pigmentosa, sensorineural deafness, and arrhythmias with heart block. They are
not overly sensitive to UV radiation.

160
What protein is deficient in the condition shown?
A. Calcium ATP‘ase IIC1
B. Calcium ATP‘ase IIA2
C. PEX-7
D. SPINK5
E. Desmoglein 3
►A

The picture shown is Hailey-Hailey disease. This is an autosomally dominant condition with a
defect in Calcium ATP‘ase IIC1. On H&E stain, an acantholytic ―dilapidated brick wall‖
appearance is seen. Calcium ATP‘ase IIA2 is defective in Darier‘s Disease, PEX-7 in refsum and

198
rhizomelic chondrodysplasia punctata, SPINK5 in Netherton‘s disease and Desmoglein 3 in
pemphigus vulgaris.

161
Which gene is defective in Wiskott-Aldrich syndrome?
A. WAS
B. CYBA
C. CYBB
D. NCF1
E. NCF2
►A

The WAS gene is defective in Wiskott-Aldrich syndrome. WAS is an Arp2/3 complex


interacting protein. The remaining options are genes related to Chronic Granulomatous Disease
and are not active in the pathogenesis of Wiskott-Aldrich syndrome.

162
The hair abnormality shown in the image is characteristic of which of the following diseases?
A. Naxos disease
B. Trichothiodystrophy
C. Bjornstad syndrome
D. Monilethrix
E. All of the options are correct
►B

The hair abnormality shown is trichoschisis, clean breaks of the hair shaft which occurs in
trichothiodystrophy. Also commonly seen is the "tiger-tail" banding pattern of the hair when
placed under polarized light. The other options do have hair shaft abnormalities, but not
trichoschisis.

163
A patient with Klinefelter Syndrome may be expected to experience which of the following:
A. Recurrent pulmonary infections
B. Recurrent leg ulcers
C. Pulmonary valve stenosis
D. Gastroesophageal reflux
E. Scarring alopecia
►B

199
Klinefelter syndrome results from nondisjunction during meiosis, leading to the XXY genotype.
Patients are characteristically tall (long lower extremities) with scant body and pubic hair.
Klinefelter patients have numerous varicosities predisposing them to recurrent leg ulcers.

164
What is the gene defect in this condition, which is also called Mendes da Costa syndrome?
A. Keratin 1 and 10
B. SPINK5
C. SLURP-1
D. Connexin 26
E. Connexin 31
►E

Mendes da COsta syndrome is also called Erythrokeratoderma Variabilis. It is an autosomal


dominantly inherited due to a mutation in connexin 31 or connexin 30.3. It is characterized by
transient geographic patches of erythema and hyperkeratotic plaques.

165
Pruritus is Sjogren Larsson syndrome is attributed to accumulation of what molecule(s)?
A. Bile salts
B. Histamine
C. Leukotriene
D. All of these answers are correct
E. None of these answers are correct
►C

Accumulation of leukotriene B4 contributes to pruritus in Sjogren Larsson syndrome.


Leukotriene inhibitors may be helpful in controlling symptoms.

166
Which of the following is true regarding tuberous sclerosis?
A. Confetti-like macules are typically present at birth
B. Facial angiofibromas are the most common cutaneous manifestation
C. Hypomelanotic macules (ash leaf spots) have a decreased number of
melanocytes
D. Periungual fibromas are considered a major feature in the diagnosis of tuberous
sclerosis
E. 6 or more hypomelanotic macules (ash leaf spots) are considered a major feature
in the diagnosis of tuberous sclerosis
►D

200
The earliest and most common cutaneous manifestation of tuberous sclerosis are hypomelanotic
macules (ash leaf spots), typically presenting at birth or early infancy. 3 or more of these lesions
are considered a major criteria in diagnosis. Melanocyte numbers are normal. Confetti-like
macules, on the other hand, are usually not apparent until the second decade of life. Facial
angiofibromas occur in approximately 75% of patients, and tend to become more prominent with
age. Periungual fibromas are considered a major feature for diagnosis, usually presenting around
puberty to early adulthood.

167
Urticaria pigmentosa is linked to a defect in the c-kit protooncogene. What autosomal dominant
skin disease also has been linked to this defect?
A. Piebaldism
B. Hypomelanosis of Ito
C. Waardenburg syndrome
D. Hermansky-Pudlak syndrome
E. Incontinentia pigmenti
►A

Piebaldism is linked to a defect in the c-kit protooncogene. Hypomelanosis of Ito has whorled
hypopigmentation, occasional CNS defects, scoliosis and anodontia, a sporadic mutation.
Waardenburg syndrome has 4 types, 1-3 are AD, 4 is AR. Type 1 & 3 have defects in PAX-3, 2
in MITF, and 4 in Sox10, endothelin-3 ligand or receptor genes. Hermansky-Pudlak syndrome is
AR and most commonly linked to defects in HPS, a lysosomal transport protein and AP3B1, a
protein important in endocytic/exocytic sorting. Incontinentia pigmenti is an X-linked dominant
syndrome with a defect in the NEMO gene.

168
Exposure to what medicine in-utero has been most closely associated with aplasia cutis
congenita?
A. Methimazole
B. Levothyroxine
C. Magnesium
D. Isotretinoin
E. Trimethoprim/sulfamethoxazole
►A

Aplasia cutis congenita presents as well-demarcated erosions at birth with heal with atrophic,
alopecic scars. It can be inherited in autosomal dominant, autosomal recessive, or sporadic
forms. Aplasia cutis congenia can be caused by teratogens, particular methimazole.

201
169
Nevoid basal carcinoma syndrome (Gorlin syndrome) is autosomal dominant transmitted
mutation of the patched gene. Symptoms include innumberable basal cell carcinomas, painful
odontogenic jaw keratocysts, palmoplantar pits, frontal bossing, bifid ribs and what other bony
abnormality?
A. Polyostotic fibrous dysplasia
B. Stippled epiphyses
C. Calcification of falx cerebri
D. Osteopoikilosis
E. Sphenoid wing dysplasia
►C

Calcification of falx cerebri is seen in Gorlin's syndrome. CHILD syndrome and


chondrodysplasia punctata both can exhibit stippled epiphyses. Polyostotic fibrous dysplasia is
found in McCune-Albright syndrome, osteopoikilosis in seen in Buschke-Ollendorf syndrome.
Sphenoid wing dysplasia is seen in neurofibromatosis type I.

170
A neonate has a papulopustular rash that progresses into a impetiginized eczematous dermatitis
and abnormally high IgE level. Patients with this sydnrome can have an IgE that is as:
A. 10x the normal value
B. 5x the normal value
C. 2x the normal value
D. 3x the normal value
E. 15x the normal value
►A

This patient has Jobs syndrome. It is characterized with papulopustular rash that progresses to
chronic impetiginized eczematous dermatitis and IgE levels that are 10x the normal level. It is
also highly suggestive of autosomal dominant hyper IgE syndrome.

171
Retention of primary teeth a dental finding of which of the following conditions?
A. Hypomelanosis of Ito
B. Letterer-Siwe disease
C. Tuberous sclerosis
D. Jackson Sertoli syndrome
E. Hyper-IgE syndrome
►E

202
Hyper-Immunoglobulin E syndrome is an autosomal dominant condition with impaired
regulation of IgE function and deficient neutrophil chemotaxis. There is increased susceptibilty
to infections and increased IgE serum levels. Retained primary teeth and lack of development of
secondary teeth are characteristic findings. The remaining conditions do not have this as a
prominent finding.

172
Ichthyosis bullosa of Siemens is a condition characterized by fragile blisters at birth,
hyperkeratotic plaques on elbows/knees later in life, and a gene mutation in:
A. Keratin 2e
B. Keratin 5/14
C. Keratin 6a/16
D. Keratin 1/10
E. None of these options are correct
►A

Keratin 2e is mutated in ichythosis bullosa of Siemens. Keratin 5 and 14 are defective in


epidermolysis bullosa simplex, 6a/16 in Pachyonychia congenital type I (Jadassohn-
Lewandowsky), 1/10 in epidermolytic hyperkeratosis and Unna-Thost PPK.

173
A sporadic syndrome affecting transcriptional coactivator CREB-binding protein is:
A. Rubinstein-Taybi syndrome
B. Cornelia de Lange syndrome
C. Nonne-Milroy disease
D. Maffucci syndrome
E. Blue rubber bleb nevus syndrome
►A

Rubinstein-Taybi syndrome is caused by a sporadically transmitted defect in transcriptional


coactivator CREB-binding protein. This gene is responsible for encoding a nuclear protein which
acts as a co-activator of cAMP regulated gene expression. Findings of this syndrome include:
capillary malformation, short stature, broad thumbs, craniofacial abnormalities including beaked
nose, mental retardation, strabismus, congenital heart defects and cryptorchidism. The other
listed conditions are not related to this defect.

174
Which of the following is not a feature of Cockayne syndrome?
A. Dwarfism

203
B. Salt and pepper retina
C. Increased sister chromatid exchange
D. Increased risk of skin cancer
E. Poikiloderma
►D

Patients with Cockayne syndrome do not have an increased risk of skin cancer. All the other
findings are associated with Cockayne syndrome, as well as photosensitivity, deafness, cataracts,
and sunken facies. Increased chromatid exchange is also seen in Bloom syndrome,
dyskeratosis congenita, and Fanconi's anemia.

175
Xeroderma pigmentosum (XP) variant is different than classic XP in which of the following
way?
A. Defective DNA nucleotide excision repair of the global genome
B. Defective post-replication repair
C. Increased chromosomal breakage and sister chromatid exchanges
D. Defective DNA nucleotide excision repair of actively transcribing genes
E. Low IgM
►B

XP variant is DNA nucleotide excision repair proficient, but the defect is in post replication
repair of DNA. Increased chromosomal breakage and sister chromatid exchanges is found in
Bloom‘s syndrome, an autosomal recessive syndrome caused by a defect in BLM gene, whose
product functions as a helicase. Clinical findings include: Telangiectasias, short stature, malar
erythema, recurrent infection, increased frequency of leukemia and lymphoma, normal
intelligence. Defective DNA nucleotide excision repair of actively transcribing genes is a feature
of Cockayne‘s syndrome, an autosomal recessive syndrome with clinical findings including:
Cachexia, short stature, pigmentary retinal degeneration, progressive deafness and no increase in
neoplasms. Xeroderma pigmentosum has seven different complementation groups (A-G), each
associated with a different form of impairment of DNA nucleotide excision repair.

176
A 13 year old girl has woolly hair, keratoderma of the soles, edema of the lower extremities, and
swelling of the abdomen. A cardiac evaluation reveals right-sided cardiomyopathy. What is the
most likely defective protein?
A. Plakoglobin
B. Plectin
C. Desmocollin
D. Desmoglein

204
E. Desmoplakin
►A

Naxos disease is caused by a defect in the plakoglobin protein. It is characterized by woolly hair,
keratoderma, and right-sided cardiomyopathy.

177
The development of which malignancy is most commonly associated with lymphomatoid
papulosis?
A. Non-Hodgkin's lymphoma
B. Mycosis fungoides
C. Multiple myeloma
D. Immunoblastic lymphoma
E. Waldenstrom's macroglobulinemia
►B

Lymphomatoid papulosis is a recurrant eruption of unclear etiology characterized by the


appearance of red-brown papules and nodules which spontaneously disappear in 3 to 8 weeks. It
is notable for histologic features which suggest a CD30 positive malignant lymphoma. There is
controversy regarding whether lymphomatoid papulosis (LyP) is a malignant, premalignant or
benign condition. The disease may last from months to years and in up to 20% of patients, it may
be preceded by, followed by, or associated with another type of cutaneous malignancy.
Generally, this is mycosis fungoides, a CD30-positive large T-cell lymphoma or Hodgkin's
disease. Because of this potential risk, long-term follow-up of these patients is required.

178
Which of the following syndromes is characterized by follicular atrophoderma, hypohidrosis,
hypotrichosis and multiple basal cell carcinomas?
A. Bazex-Christol-Dupre syndrome
B. Rombo syndrome
C. Rasmusen syndrome
D. Gorlin syndrome
E. Incontinentia Pigmenti
►A

Bazex-Christol-Dupre syndrome has the findings of follicular atrophoderma, hypohidrosis,


hypotrichosis and multiple basal cell carcinomas (BCC). Rombo syndrome is associated with
BCC and hypotrichosis, but not the other listed findings. The atrophoderma in Rombo syndrome
is vermicular, not follicular. Rasmusen syndrome is not associated with BCC.

205
179
A patient has epidermal cysts, demoid tumor, fibromas, lipomas and dental cysts. The patient is
diagnosed with an autosomal dominant syndrome with the characteristic eye finding of
congenital hypertrophy of retinal epithelium (CHRPE). This person will develop adenomatous
polyposis of the colon that will transform to carcinoma by the age of 30:
A. 100% of the time
B. 90% of the time
C. 80% of the time
D. 70% of the time
E. 50% of the time
►A

This patient has Gardner's syndrome with a mutation in the tumor suppressor APC gene. These
patients with develop adenenomatous polyposis of the colon and rectum with 100%
transformation to carcinoma by the age of 30.

180
The x-linked recessive type of dyskeratosis congenita is:
A. Dyskerin
B. TERC
C. CDKN2A
D. PTEN
E. Menin
►A

The dyskerin gene, whose product is involved in ribosomal RNA synthesis, is mutated in X-
linked recessive dyskeratosis congenita. TERC is linked with autosomal domininant transmission
of the syndrome. CDKN2A is involved in familial dysplastic nevi/melanoma syndrome, PTEN in
Cowden syndrome and Menin in MEN type I.

181
The gene defect in Griscelli Syndrome is:
A. Myosin Va or Rab27a
B. LYST or CHS1
C. P gene
D. TRP1
E. None of these answers are correct
►A

206
Myosin Va or Rab27a are defective in Griscelli syndrome, an AR syndrome with mild albinism,
pancytopenia, immunodeficiency, neurologic symptoms and an accelerated phase similar to
Chediak-Higashi syndrome. LYST/CHS1 is defective in Chediak-Higashi syndrome. The P-gene
is mutated in oculocutaneous albinism type 2 and TRP in oculocutaneous albinism type 3.

182
Comma shaped corneal opacities are seen in what disease?
A. Refsum Syndrome
B. Sjogren-Larson Syndrome
C. Pseudoxanthoma elasticum
D. X-linked ichthyosis
E. Proteus syndrome
►D

X-linked ichthyosis is a X-linked recessive disorder secondary to steroid sulfatase deficiency


characterized by brown adherent scale. Additional findings include comma-shaped corneal
opacities, cryptorchisdism, and failure to progress during labor.

183
Pseudoxanthoma elasticum (PXE) can be transmitted in an autosomal dominant, recessive or
sporadic manner. Which of the following genes is mutated in PXE?
A. ABCC6
B. MAN1
C. Collagen III
D. Collagen V
E. Collagen I
►A

ABCC6 is mutated in PXE. Other findings include fragmented and calcified elastin in skin, eyes,
arteries. There is the appearance of plucked chicken skin on the flexures and yellow papules on
the mucous membranes. Angioid streaks are present in the eye. Other findings include gastric
hemorrhage and arterial disease. The others are not involved in PXE.

184
The NEMO gene is defective in Bloch-Sulzberger syndrome. What other syndrome has been
linked with defects in the NEMO gene?
A. Hypohidrotic ectodermal dysplasia with immune deficiency
B. Hypomelanosis of Ito
C. Tuberous sclerosis
D. Waardenburg syndrome

207
E. Piebaldism
►A

Hypohidrotic ectodermal dysplasia with immune deficiency, is caused by mutations in the


NEMO (IKK-gamma gene). As opposed to the X-linked dominant inheritance of Bloch-
Sulzberger syndrome (incontinentia pigmenti), this is a X-linked recessive disorder.
Hypomelanosis of Ito is sporadically inherited and is not linked with a gene defect. Tuberous
sclerosis is autosomal dominant and has been linked to defects in tuberin and hamartin tumor
suppressor genes. Waardenburg syndrome has four subtypes, linked with the PAX-3, MITF and
SOX10/endothelin-3 receptor genes. Piebaldism is linked to defects in the c-kit protooncogene.

185
Deficiency of filaggrin is the strongest known predisposing genetic factor for the development of
atopic dermatitis. What is the mode of inheritance of mutations in filaggrin?
A. Autosomal dominant
B. Autosomal semidominant
C. Autosomal recessive
D. X-linked dominant
E. X-linked recessive
►B

Autosomal dominant means that a mutation in a gene on one of the two chromosomes in any
autosomal chromosome pair results in disease. Autosomal recessive means that mutations in the
same gene on both of the chromosomes in the pair must occur for the disease to occur.
Autosomal semidominant means that a mutation in one gene of the autosomal chromosome pair
results in a mild version of the disease, while mutations in the gene on both chromosomes results
in the full-blown disease. Ichthyosis vulgaris is caused by mutations in filaggrin and inherited in
a autosomal semidominant manner. This is the genetic explanation for why the phenotype of
ichthyosis vulgaris can vary significantly across individuals.

186
Dystrophic epidermolysis bullosa is associated with mutations in collagen VII. Trauma or
friction induced blistering in these patients have a plane a splitting in the:
A. Sublamina densa
B. Stratum spinosum
C. Lamina lucida
D. Stratum basale
E. None of these answers are correct
►A

208
The split in dystrophic epidermolysis bullosa is found in the sublamina densa, where the collagen
VII anchors the epidermis to the anchoring plaques in the dermis. The remaining options are
incorrect.

187
Connexin 30 (GJB6 gene) is defective in which of the following syndromes?
A. KID syndrome
B. Vohwinkel syndrome
C. Vohwinkel syndrome variant
D. Clouston syndrome
E. Erythrokeratoderma variabilis
►D

Clouston syndrome is associated with a defect in Connexin 30 (GJB6 gene). Findings include
palmoplantar keratoderma with transgradiens, dystrophic nails, sparse hair with absent body,
eyelash, eyebrow hair after puberty. KID syndrome and Vohwinkel syndrome are associated
with a defect in Connexin 26 (GJB2). Vohwinkel syndrome variant is associated with a loricrin
defect. Erythrokeratoderma variabilis has mutations in Connexin 31 (GJB3) and 30.3(GJB4).

188
Adenosine deaminase deficiency is seen in which immunodeficient disease?
A. Wiskott-Aldrich syndrome
B. Chronic granulomatous disease
C. Job syndrome
D. Severe combined immunodeficiency syndrome
E. Leineri's disease
►D

Severe combined immunodeficiency is a heterogeneous group of disorders characterized by


decreased humoral and cell mediated immunity. Patients may have recurrent infections including
cutaneous ones, GVHD (due to in utero cmaternal lymphocytes), sepsis, oral candidiasis, and
diarrhea. Implicated genes include the IL-2 receptor (x-linked recessive form) and adenosine
deaminase deficiency (autosomal recessive form).

189
A patient has recurrent infections and is found to have a disorder of phagocytic cells from an
inability of phagocytes to undergo the respiratory burst needed to kill certain types of bacteria
and fungi. Carriers of this disease are also at risk for which condition?
A. Breast cancer
B. Vitiligo

209
C. Discoid lupus
D. Thyroid disease
E. Cataracts
►C

This patient has chronic granulomatous disease characterized by dysfunctional phagocytic cells.
The most common molecular defect in chronic granulomatous disease is a mutation in the gene
encoding the b subunit of cytochrome b. Carriers have a significant incidence of discoid lupus
erythematosus, photosensitivity, Raynaud's phenomenon, and aphthous ulcers. Carriers of ataxia
telangiectasia are at risk for breast cancer.

190
In patients with diffuse congenital hemangiomatosis, the most common site for extracutaneous
involvement is the :
A. Liver
B. Thyroid
C. Lungs
D. Colon
E. Brain
►A

Diffuse congenital hemangiomatosis is characterized by multiple hemangiomas with the liver


being the most common extracutaneous site, followed by the lungs. Liver hemangioma may be
complicated by hepatomegaly, obstructive jaundice, and portal hypertension.

191
A 5 year old boy is being seen by neurology for seizures and by ENT for a hoarse voice. He is
referred to dermatology for evaluation of verrucous nodules of the elbows and knees. While
reviewing his chart, it is noted that he has calcifications in the hippocampus seen on imaging.
What is the most likely diagnosis of this patient?
A. Lipoid proteinosis
B. Psoriasis
C. Severe combined immunodeficiency disorder
D. Poland syndrome
E. Vohwinkel syndrome
►A

Lipoid proteinosis is an autosomal recessive disorder caused by defects in the extracellular


matrix protein 1 gene. It is characterized by scars and yellow papules of the face and oropharynx,

210
yellowish papules on the eyelid margin, hoarse voice, verrucous nodules of the elbows and
knees, and calcification of the temporal lobe and hippocampus with occasional seizures.

192
A 50 year man presents with generalized metallic-grey hyperpigmentation. His past medical
history includes diabetes, hepatomegaly and arrythmias. Laboratory tests should include:
A. Copper levels
B. Lead levels
C. Iron levels
D. Arsenic levels
E. Cyanide levels
►C

Hemochromatosis is an autosomal recessive disease resulting in increased intestinal iron


absorption and iron deposition in a variety of organs. Clinical features include generalized
metallic-grey hyperpigmentation, koilonychia, sparse or absent hair, hepatomegaly, cardiac
failure/arrhythmias, insulin-dependent diabetes, hypogonadism and polyarthritis

193
A 4 year-old boy presents with 2 soft, dark-blue, compressible nodules on his extremities. His
mother has noted that these lesions have increased sweating and that they were present at birth.
No one else in the family has had similar skin lesions. What step is indicated first to help
determine the diagnosis?
A. Stool guiac
B. MRI of the abdomen
C. CBC
D. Immediate referral to a gastroenterologist
E. Biopsy of a skin lesion
►A

Blue rubber bleb nevus syndrome is described above. There are multiple venous malformations
on the extremities and trunk, often present at birth to early childhood. The number of these
lesions increase with age. The lesions may have increased sweating and can be combined with
lymphatic-venous malformations. Skin lesions can be a clue to gastrointestinal venous
malformations which can lead to secondary bleeding and anemia. The most reasonable screening
test to determine if the patient has GI hemorrhage is a stool guiac. An MRI or complete blood
count can be helpful, but are not the best test to start with. A skin biopsy is not indicated. If there
is GI blood loss, evaluation by a gastroenterologist is useful.

194

211
Patients with neurofibroma type I have a pheochromocytoma incidence of:
A. 1%
B. 5%
C. 7%
D. 9%
E. 10%
►A

Patients that have NF1 have a 1% incidence of pheochromocytoma. Other syndromes with
flushing and pheo like symptoms are von Hippel-Lindau and MEN2.

195
What is the gene defect in harlequin fetus?
A. Transglutaminase
B. Steroid sulfatse
C. ABCA12
D. ABCC6
E. None of these answers are correct
►C

Harlequin fetus is an autosomal recessive disorder. The gene defect is ABCA12.

196
A patient with a white, spongy overgrowth of the buccal mucosa that has passed in an autosomal
dominant fashion is most likely related to a mutation in which of the following?
A. Keratin 1/10
B. Keratin 6b/17
C. Keratin 4/13
D. Keratin 6a/16
E. None of these options are correct
►C

This description is most likely a white sponge nevus, an autosomal dominant defect in keratin
4/13. Keratin 1/10 is mutated in epidermolytic hyperkeratosis and Unna-Thost PPK, Keratin
6a/16 in pachyonychia congenita type I, and keratin 6b/17 in pachyonychia congenita type II.

197
Which type of epidermolysis bullosa is associated with mitten deformities of the hands?
A. Dominant dystrophic
B. Recessive dystrophic

212
C. Weber-Cockayne
D. Herlitz type
E. Generalized atrophic benign epidermolysis bullosa (GABEB)
►B

The recessive dystrophic type of epidermolysis bullosa is associated with chronic hand ulcers
resulting in scarring that causes mitten deformities. These chronic scars often result in the
formation of fatal squamous cell carcinomas.

198
A middle-aged woman complains of multiple rough lesions on her trunk. Biopsy reveals
connective tissue nevi. Because other family members have had similar lesions, genetic testing is
performed, revealing a defect in the LEMD3 gene. What is the classic bone finding in patients
with a defective LEMD3 gene?
A. Osteopoikilosis
B. Osteopathia striata
C. Tibial bowing
D. Lumbar ankylosis
E. Frequent fractures of the long bones
►A

Buschke Ollendorff syndrome is a rare genetic syndrome characterized by multiple connective


tissue nevi and osteopoikilosis. Osteopoikilosis is dysplasia of bone leading to the presence of
multiple bone islands in the skeleton. The defective gene is LEMD3. Osteopathia striata is
associated with focal dermal hypoplasia. Tibial bowing is seen in rickets. Lumbar ankylosis is
found in ankylosing spondylitis. Frequent fractures of long bones is seen in osteogenesis
imperfecta.

199
Naxos syndrome is characterized by a right sided cardiomyopathy, wooly hair, and keratoderma.
The epidermal structure defective in Naxos syndrome is:
A. Desmoglein 1
B. Desmoglein 3
C. Plakoglobin
D. Desmoplakin
E. Keratin 1/10
►C

Plakoglobin is an intracellular desmosomal component which binds desmogleins/desmocollins


on one side and to desmoplakin on the other. Desmoplakin in turn binds to the keratin

213
intermediate filaments, K1/10 in most cases. Mutation of desmoplakin leads to CarvajaL
syndrome, which is associated with a striate palmoplantar keratoderma, woolly hair and Left
sided cardiomyopathy. A simple way to remember this is the L in Carvajal cooresponds to the
Left sided cardiomyopathy vs. the right sided disease in Naxos disease.

200
Which of the following pairs of diseases are caused by mutations in DNA helicases?
A. Bloom syndrome and Rothmund-Thompson syndrome
B. Xeroderma pigmentosum and Cockayne syndrome
C. Muir-Torre syndrome and Birt-Hogg-Dube syndrome
D. Dyskeratosis congenita and Peutz-Jeghers syndrome
E. Cowden syndrome and Bannayan-Riley-Ruvalcaba syndrome
►A

Bloom Syndrome and Rothmund-Thompson syndromes are caused by autosomal recessive


mutations in DNA helicases. The common features include photodistributed poikiloderma.
Patients with Bloom syndrome also demonstrate facial dysmorphism, hypogammaglobulinemia
with recurrent respiratory and gastrointestinal tract infections, hypogonadism, leukemias and
lymphomas, gastrointestinal adenocarcinomas, and oral/esophageal SCCs. Patients with
Rothmund-Thompson syndrome develop premalignant acral keratoses, alopecia, nail dystrophy,
cataracts, hypogonadism, and occasional solid tumors.

201
Fabrys disease is a defect in the alpha galactosidase. The eye finding that looks like whorled
corneal deposits is:
A. Cornea verticillata
B. Chemosis
C. Lagophthalmos
D. Corneal ulceration
E. Conjunctival shrinkage
►A

Fabrys disease is an X linked recessive disorder. Patients can also get cornea verticillata that look
like whorled corneal deposits. They can also get spoke like lens deposits, conjunctival and retinal
tortuosity, oculomotor abnormalities.

202
A Puerto Rican woman is seen in clinic for a pruritic rash on her trunk. A punch biopsy is
performed. The biopsy site continues to bleed, with hematoma formation. The bleeding is
eventually controlled. On further exam, her skin and hair are light brown. She has a history of

214
granulomatous colitis. What it the most likely reason she had excess bleeding with a simple
procedure?
A. Her platelets lack dense bodies, causing excess bleeding
B. Her intrinsic factor is deficient
C. Her Factor VIII levels are low
D. She is congentially deficient in platelets
E. None of the answers are correct
►A

Platelets without dense bodies cause excess bleeding in Hermansky-Pudlak syndrome. Other
features of this condition include oculocutaneous albinism, ceroid lysosomal storage disease
resulting in pulmonary fibrosis, granulomatous colitis.

203
A patient that has Darier's disease has skin findings of hyperkeratotic papules in the seborrheic
areas and has nail findings with:
A. Alternating red and white longitudinal bands
B. Long nails
C. Horizontal parallel ridges
D. Longitudinal fissures
E. Corn
►A

Patient with Darier's disease have alternating red and whit longitudinal bands with a V-shaped
distal nicking, subungual hyperkeratosis and skin findings in the seborrheic areas.

204
Which of the following conditions is inherited in an X-linked recessive manner?
A. Epidermolysis bullosa simplex
B. Ichthyosis vulgaris
C. Sjogren-Larsson syndrome
D. Wiskott-Aldrich Syndrome
E. Netherton‘s Syndrome
►D

Wiskott-Aldrich Syndrome is inherited in an X-linked recessive manner. Epidermolysis bullosa


simplex and ichthyosis vulgaris are inherited in an autosomal dominant (AD) manner. Sjogren-
Larsson and Netherton‘s syndrome are inherited in an autosomal recessive manner.

205

215
Which genetic disease is characterized by round opacities seen on radiographs of the long
bones?
A. Buschke-Olendorf syndrome
B. Chondrodysplasia punctata
C. Netherton syndrome
D. Osteogenesis imperfecta
E. Focal dermal hypoplasia
►A

Buschke-Ollendorf syndrome is an autosomal dominant disorder with dermatofibrosis


lenticularis disseminata (elastomas) and osteopoikilosis, or round opacities of the bones. It is
caused by a defect in LEMD 3 (MAN1), which codes an inner nuclear membrane protein. Focal
dermal hypoplasia can cause osteopathia striata; osteogenesis imperfecta can have bone
fractures; and chondrodysplasia punctata can have stippled epiphyses.

206
Palmoplantar keratoderma with deafness is caused by a defect in which gene?
A. SLURP-1
B. Plakophilin
C. Connexin 26
D. Lysosomal papain like cysteine proteinase
E. Unknown
►C

Palmoplantar keratoderma with deafness is caused by a defect connexin 26 encoded by GJB2. A


defect in SLURP-1 causes Mal de Meleda. A defect in plakophilin causes ectodermal dysplasia
with skin fragility. A defect in cathepsin C lysososomal papain like cysteine proteinase causes
Papillon LeFevre and Haim Munk.

207
Which disease can clinically mimic pellagra but is inherited in an autosomal recessive fashion
and is due to a defect in the transport of neutral amino acids?
A. Wilsons
B. Hemochromatosis
C. Hartnup Disease
D. Fabry
E. Gaucher‖s
►C

216
The clinical manifestation of Hartnup disease is similar to that of pellagra because the resultant
defect in the transport of amino acids leads to low levels of tryptophan. Since tryptophan is
required to make nicotinic acid, pts with Hartnup disease manifest the same symptoms as niacin-
deficient patients (pellagra).

208
Patients with X-linked hypohidrotic ectodermal dysplasia have dental findings of:
A. Pegshaped/conical teeth
B. Erythrodontia
C. Hutchinson teeth
D. Enamel hypoplasia
E. Mulberry molars
►A

Patients with X-linked hypohidrotic ectodermal dysplasi have pegshaped/conical teeth. They
have problems with perspiration and thickened nails.

209
A patient Buschke-Ollendorff syndrome has osteopoikilosis and which cutaneous finding?
A. Waxy papules along the eyelids
B. Cafe au lait macules
C. Port wine stain
D. Juvenile elastoma
E. Epidermal nevi
►D

Buschke-Ollendorf syndrome is an autosomal dominant syndrome associated with increased


elastic fiber in the skin. Key features include dermatofibrosis lenticularis disseminata (also called
juvenile elastomas) and osteopoikilosis.

210
The diagnostic test for chronic granulomatous disease is:
A. Dimethylglyoxime test
B. Nitroblue tetrazolium reduction assay
C. Histamine skin test
D. Bone marrow biopsy
E. Potassium hydroxide
►B

217
Chronic granulomatous disease is characterized by a defect in the ability to kill catalase positive
organisms within phagocytic leukocytes. This results from a neutrophilic defect in the
cytochrome found in the NADPH oxidative pathway responsible for a respiratory burst. The
nitroblue tetrazolium (NBT) reduction assay demonstrates the leukocytes ability to reduce the
dye and produce a blue color change. Patients with chronic granulomatous disease are unable to
reduce the dye.

211
Homocystinuria is caused by a defect in:
A. Phenylalanine hydroxylase
B. Biotinidase
C. Holocarboxylase synthetase
D. Cystathione beta-synthetase
E. Gp91-phox
►D

Cystathione beta-synthetase is defective in homocystinuria, an autosomal recessive conditions


characterized by increased homocystine and methionine levels in blood and urine. Other findings
include a malar flush, DVTs/emboli, cardiovascular disease, livedo reticularis, leg ulcers, blonde
hair/fair complexion, downward lens dislocation, glaucoma, mental retardation, seizures,
psychiatric disorders and a marfanoid body habitus. The other enzymes are not involved in this
condition.

212
A patient presents with focal symmetric palmoplantar keratoderma, thickened, hyperkeratotic
fingernails and toenails with a "pincer" appearance and frequent staph and candida paronychial
infections, follicular hyperkeratosis of the knees and elbows and oral leukokeratosis. The patients
mother and grandfather both have similar skin findings. Which syndrome is described?
A. Jadassohn-Lewandowsky syndrome
B. Jackson-Lawler syndrome
C. Schafer-Branauer syndrome
D. Pachyonychia congenita tarda
E. None of the options are correct
►A

The description above is the classic description for pachyonychia congenita (PC) type I or
Jadassohn-Lewandowsky syndrome. Jackson-Lawler syndrome is PC type II, Schafer-Branauer
syndrome is PC type III and pachyonychia congenita tarda is PC type IV.

213

218
A child presents with sparse, short hair and sensorineural deafness. On microscopic examination
of the hair, pili torti is noted. Which of the following syndromes is the most likely diagnosis?
A. Bjornstad syndrome
B. Menkes kinky hair syndrome
C. Argininosuccinic aciduria
D. Trichothiodystrophy
E. None of the options are correct
►A

Bjornstad syndrome is the most likely diagnosis. This rare syndrome (~25 cases) is autosomal
recessive. Findings are of deafness and pili torti. The most common hair finding in Menkes
syndrome is pili torti, but it is not associated with hearing loss. Argininosuccinic aciduria is
associated with trichorrhexis nodosa and has no associated hearing loss.

214
A patient with Crowe‘s sign and an optic glioma has which of the following disorders?
A. Neurofibromatosis I
B. Neurofibromatosis II
C. Watson syndrome
D. Tuberous sclerosis
E. Lester iris syndrome
►A

The diagnostic criteria for neurofibromatosis I include meeting 2 or more of the following 7
criteria: (1) >5 café au lait macules (CALMs) that are >5mm in a prepubertal person or >15 mm
in a postpubertal person, (2) >1 neurofibroma or 1 plexiform neurofibroma, (3) axillary/inguinal
freckling (Crowe‘s sign), (4) optic glioma, (5) >1 Lisch nodule (iris hamartoma), (6) sphenoid
dysplasia, (7) 1st degree relative with neurofibromatosis I.

215
Which of the following features is not associated with Cornelia de Lange Syndrome?
A. Normal intelligence
B. Characteristic facies with downturned mouth, hirsutism, synophrys,
trichomegaly, anteverted nostrils, long philtrum and low set ears
C. Cryptorchidism
D. Fifth finger clinodactyly
E. Recurrent lung infections
►A

219
Children with Cornelia de Lange are usually severly retarded with an IQ <35. In addition to the
features listed above, other features include cutis marmorata, hypoplastic nipples and umbilicus,
low-pitched cry in infancy and congenital heart defects. While most cases are inherited in a
sporadic manner, those cases which are familial are thought to be autosomal dominant and
associated with the NIPBL (nipped-beta-like) gene. Prognosis is poor with premature death often
secondary to sspiration or recurrent pulmonary infection.

216
Which of the following is caused by a mutation in a gene that leads to defective NF-KB
activation?
A. MEN IIa
B. Piebaldism
C. Chediak-Higashi
D. Vohwinkel‘s
E. Incontinentia pigmenti
►E

Incontinential pigmenti is an X-linked dominant disorder caused by a mutation in the NEMO


gene located at Xq28. NEMO is an NF-KB modulator. A genetic defect in NEMO leads to
defective activation of NF-KB (a transcription factor). MEN IIa is caused by a mutation in the
RET proto-oncogene, piebaldism is caused by the C-kit protocogene, chediak-higashi is caused
by a mutation in lysosomal tracking, and Vohwinkel‘s is caused by a defect in GJB2 which
encodes connexin 26.

217
Crumpled ears are associated with which disorder?
A. Ehlers-Danlos Syndrome
B. Marfan syndrome
C. Congenital contractural arachnodactyly
D. Cutis laxa
E. Buschke-Ollendorf syndrome
►C

Congenital contractural arachnodactyly is an autosomal dominant disorder caused by mutations


in fibrillin 2. Affected patients have long limbs, arachnodactyly, scoliosis, and crumpled ears.

218
What gene defect would you expect to find in a child with white forelock, dystopia canthorum,
and upper limb abnormalities?
A. Pax3

220
B. MITF
C. SOX10
D. Endothelin-3
E. C-kit proto-oncogene
►A

Waardenburg's syndrome is characterized by depigmented patches, white forelock, and deafness.


Both type 1 and 3 are caused by mutations in Pax3. Type 3 is also associated with limb
abnormalities. MITF and SOX10 defects are responsible for types 2 and 4 respectively. C-kit
proto-oncogene mutatios are seen in piebaldism.

219
The prenatal diagnosis of X-linked ichthyosis is detected by the triple screen during pregnancy
which can show:
A. Low to absent estradiol levels
B. Non hydrolyzed sulfated steroids
C. Low to absent levels of hydrolyzed steroids
D. Low HCG levels
E. Increased serum cholesterol sulfate levels
►A

Prenatal diagnosis of X-linked ichthyosis is by low to absent estriol during triple screen,
presence of non hydrolyzed sulfated steroids in maternal urine. Postnatal diagnosis by serum
protein electrophoresis shows increased serum cholesterol sulfate levels.

220
Which of the following hereditary skin disorders is associated with the RAS-ERK-MAPK
pathway?
A. Costello syndrome
B. Rothmund-Thompson Syndrome
C. Carney complex
D. Tuberous Sclerosis
E. Griscelli syndrome
►A

Skin disorders associated with the RAS-ERK-MAPK pathway include: Cardio-facio-cutaneous


syndrome, Costello, LEOPARD, NF, and Noonan. Rothmund-Thompson is a RecQ DNA
helicase defect. Carney complex and Tuberous sclerosis are associated with the cAMP & AMP
activated protein kinase pathway. Griscelli syndrome involves defective vesicle
trafficking/transport.

221
221
Which of the following condition is NOT found in Von-Hippel Lindau syndrome?
A. Connective tissue nevi
B. Bilateral retinal hemangioblastomas
C. Cerebellar/CNS hemangioblastomas
D. Renal cell carcinoma
E. Pheochromocytoma
►A

Von Hippel-Lindau syndrome is characterized by all the options listed except connective tissue
nevi. Other findings include pancreatic cysts/carcinoma and cutaneous capillary malformations
of the head and neck and polycythemia.

222
The combination of painful palmoplantar keratoderma and pseudoherpetic keratitis is
characteristic of which of the following syndromes?
A. Naxos syndrome
B. Vohwinkel syndrome
C. Richner-Hanhart syndrome
D. Howel-Evans syndrome
E. Schopf-Schulz-Passarge syndrome
►C

Richner-Hanhart syndrome (tyrosenemia type II) is an autosomal recessive disorder caused by a


deficiency in hepatic tyrosine aminotransferase. This disease is characterized by painful PPK,
pseudoherpetic keratitis and blindness. Treatment is low-tyrosine/phenylalanine diet.

223
A child presenting with the scalp findings shown and a right arm hypoplasia would be diagnosed
with which of the following syndromes?
A. Adams-Oliver syndrome
B. Bart's syndrome
C. Progeria
D. Dunnigan syndrome
E. None of these options are correct
►A

Adams-Oliver syndrome is defined by aplasia cutis congenita (ACC) (shown in the image),
usually of the midline scalp with limb hypoplasia. Bart's syndrome also has ACC as a finding,

222
but it is usually present on the lower extremities and associated dominant dystrophic
epidermolysis bullosa. Progeria is a premature aging syndrome associated with a lamin-A
mutation. Dunnigan syndrome is also known as Familial partial lipodystrophy and is associated
with a mutation in the laminin A/C. Neither are associated with findings of ACC.

224
Ichthyosis linearis circumflexa is one of the findings seen in the syndrome caused by which of
the following genes?
A. SPINK5
B. SLURP1
C. GJB2
D. LMX1B
E. CYLD
►A

The SPINK5 gene encodes for LEKTI, a serine protease inhibitor important in the regulation of
proteolysis in epithelia formation and keratinocyte terminal differentiation, is mutated in
Netherton‘s Syndrome. Other findings include: trichorrhexis invaginata (bamboo, ball and socket
hair), atopic dermatitis, and anaphylaxis from food allergy. SLURP1 is mutated in Mal de
Maleda, GJB2 in Vohwinkel‘s syndrome, LMX1B in Nail-Patella syndrome, and CYLD in
Familial Cylindromiasis.

225
What is the characteristic radiographic finding in type I Gaucher disease?
A. Enchondromas
B. Osteopoikilosis
C. Melorheostosis
D. Ehrlenmeyer flask deformity
E. Supernumerary vertebrae with extra ribs
►D

The Ehrlenmeyer flask deformity is found in the femoral midshaft as well as aseptic necrosis of
the femoral head and widening of the distal femur. Endochondromas are seen in Maffucci
syndrome, Osteopoikilosis in Buschke-Ollendorf syndrome, Melorheostosis (linear hyperostosis
under affected skin) in linear scleroderma and supernumerary vertebrae with extra ribs in
incontinentia pigmenti.

226

223
A 18 yo man presents for evaluation of foot lesions. There are thick hyperkeratotic plaques
symmetrically on only the weight bearing plantar surfaces. What test(s) should this patient be
referred for?
A. Head CT
B. Thoracic CT
C. Hepatic ultrasound
D. Endoscopy
E. Knee films
►D

The patient likely has Howel-Evans Syndrome. These patients present with symmetric focal
weight bearing PPK in the second decade to adulthood. After the third decade, esophageal
carcinoma can occur. These patients should have periodic endoscopic evaluation.

227
Mutations in which of the following receptors underlie chronic mucocutaneous candidiasis
disease (CMCD)?
A. IL-12 receptor
B. IL-15 receptor
C. AIRE receptor
D. IL-17 receptor
E. IL-23 receptor
►D

Chronic mucocutaneous candidiasis disease (CMCD) is characterized by recurrent or persistent


infections of the skin, nails, and oral and genital mucosae caused by Candida albicans and, to a
lesser extent, Staphylococcus aureus, in patients with no other infectious or autoimmune
manifestations. Mutations in IL-17 receptor A (IL-17RA, autosomal recessive) and IL-17
receptor F (IL-17F, autosomal dominant) have been reported. IL-17RA deficiency is complete,
abolishing cellular responses to IL-17A and IL-17F homo- and heterodimers. By contrast, IL-
17F deficiency is partial, with mutant IL-17F-containing homo- and heterodimers displaying
impaired, but not abolished, activity.

228
An infant presents with multiple congenital hemangomas in an generalized distribution. What is
the most serious associated condition?
A. Congestive Heart Failure
B. Obstructive jaundice
C. Portal hypertension
D. All of the answers are correct

224
E. None of the answers are correct
►A

High output congestive heart failure can lead to death in these children. Obstructive jaundice and
portal hypertension both occur, but are less likely to cause death. The hemangiomas will undergo
spontaneous regression.

229
What is the genetic defect of this autosomal dominant disorder?
A. PTEN
B. PTPN11
C. Calcium ATPase 2A2
D. Calcium ATPase 2C1
E. SPINK5
►D

Hailey-Hailey is an autosomal dominant disorder that usually affects the intertriginous areas.
Clinically, there is erythema and linear fissures of the axilla and groin. On pathology, the
characteristic finding is acantholytic dyskeratosis in a "dilapidated brick wall" pattern. The gene
defect responsible is calcium ATPase 2C1.

230
A patient with a personal and family history of multiple fibrofolliculomas may have:
A. Birt-Hogg-Dube syndrome
B. Basal cell nevus syndrome
C. Muir-Torre syndrome
D. Gardner's syndrome
E. Tuberous sclerosis
►A

Fibrofolliculomas are small, benign, yellow or flesh colored papules that are usually inherited in
an autosomal dominant fashion and have a predilection for the face, neck and upper trunk. Birt-
Hogg-Dube syndrome is an autosomal dominant condition characterized by multiple
fibrofolliculomas, trichodiscomas, acrochordons, collagenomas. This condition is important to
recognize due to its association with renal cell carcinoma. In addition, lung cysts and bullous
emphysema are also features of the condition with spontaneous pneumothorax being a potential
complication. Muir-Torre syndrome is an autosomal dominant condition caused by a defect in
hMSH2 gene. This condition is associated with sebaceous neoplasms including sebaceous
carcinoma, sebaceous hyperplasia, sebaceous epithelioma, sebaceous adenoma as well as
keratoacanthomas. These can be markers for underlying malignancy in this condition; notably

225
adenocarcinoma of the colon, breast, urinary tract, lung and endometrium. Gardner syndrome is
an autosomal dominant condition caused by a defect in the APC familial adenomatous polyposis
gene in which patients have multiple hamartomatous polyps of the colon with a high rate of
malignant transformation. Cutaneous clues to the diagnosis include multiple epidermoid cysts,
fibromas, and desmoid tumors. Other manifestations include osteomas, supernumary teeth, and
congential hypertrophy of the retinal pigment epithelium. Tuberous sclerosis, (TS),also known as
Bourneville's syndrome is caused by defects in hamartin and tuberin, found on chromosome 9
and 16, respectively. Patients with TS are at increased risk of muliple neoplasms including
retinal hamartomas, angiomyolipomas, and cardiac rhabdomyomas. Cutaneous manifestations
include ash-leaf macules, shagreen patchs, cafe-au-lait macules, confetti macules, facial
angiofibromas, and periungual fibromas. As the name implies, basal cell nevus syndrome is
associated with multiple basal cell carcinomas. In addition palmoplantar pitting, multiple milia
and epidermoid cysts are seen. It is autosomal dominantly inherited and caused by a defect in the
patched gene.

231
A patient with port wine stain on a lower extremity, hemihypertrophy of the limb and lymphatic
and deep venouse insufficiency of the affected limb would be considered to have Klippel-
Trenaunay-Weber syndrome. What additional feature would need to be present to define the
patient as having Parkes-Weber syndrome?
A. Arteriovenous fistulas
B. Multiple cafe-au-lait macules
C. Macroglossia
D. Cutis marmorata
E. Distichiasis
►A

Parkes-Weber syndrome has the additional feature of arteriovenous fistulas. The remaining
features are not part of these syndromes.

232
A 20-year old male with a history of pheochromocytoma and medullary thyroid cancer presents
with mucosal papules. His overall body appearance is most likely to demonstrate:
A. Cushingoid features
B. Marfanoid features
C. Short stature
D. Lipodystrophy
E. Unilateral limb shortening
►B

226
This patient has multiple endocrine neoplasia type IIb (MEN IIb) characterized by
pheochromocytoma, thyroid cancer, and rare parathyroid carcinoma as well as mucosal
neuromas cutaneously. These patients have a marfanoid body habitus. This syndrome is caused
by the RET proto-oncogene.

233
Congenital erythropoetic porphyria has the dental findings of:
A. Erythrodontia
B. Hutchinson teeth
C. Mulberry molars
D. Hypodontia
E. Enamel hypoplasia
►A

Congenital erythropoetic porphyria has dental findings of erythrodontia. Hutchinson teeth and
mulberry molars are congenital syphilis. Hypodontia is a finding of X-linked hypohidrotic
ectodermal dysplasia.

234
In which of the following Genodermatoses would one find cutaneous hyperpigmentation, blue
lunulae and Kayser-Fleishcher rings:
A. Marfan's Disease
B. Hemochromatosis
C. Gaucher's Disease
D. Wilson's Disease
E. Osteogenesis Imperfecta
►D

In Wilson's disease (Hepatolenticular Degeneration) one will find a vague greenish discoloration
of the skin on the face, neck, and gentalia Hyperpigmentation), azure lunulae (sky-blue moons)
of the nails, and Kayser-Fleischer rings. This is due to the body retaining excessive amounts of
copper.

235
A child presents with pretibial hyperpigmentation, ataxia, decreased motor coordination,
cirrhosis, and decreased motor coordination. The physical exam which would reveal the most
specific finding for this disease is:
A. Hearing test
B. Slit-lamp eye exam
C. EKG

227
D. Colonoscopy
E. Renal ultrasound
►B

Wilson‘s disease (also known as hepatolenticular degeneration) is an autosomal recessive


disorder result in defective biliary excretion of copper, leading to copper accumulation in the
liver, brain and cornea. Clinical features include hepatomegaly, cirrhosis, ataxia, dysarthria,
decreased motor coordination, pretibial hyperpigmentation, blue lunulae, and copper deposition
in the cornea—Kayser-Fleisher ring, which can be diagnosed using a slit-lamp.

236
Which opthamologic disease is associated with this disorder? This lesion was present at birth.
A. Glaucoma
B. Ectopia lentis
C. Cataracts
D. Posterior subcasular lentiular opacity
E. Retinitis pigmentosa
►A

Sturge-Weber syndrome is a sporadic disease characterized by a capillary malformation in the


trigeminal distribution. Patients may have associated cerebral atrophy, vascular malformations of
the leptomeninges, and seizures. All patients with Sturge-Weber should be referred to the
opthamologist for glaucoma screening.

237
Patients with Chondrodysplasia punctata can have findings of stippled epiphyses on X-ray
examination. Which other x-linked dominant condition can have stippled epiphyses?
A. CHILD syndrome
B. Incontinentia Pigmenti
C. Focal Dermal Hypoplasia
D. Goltz syndrome
E. Bazex syndrome
►A

All of the syndromes listed have X-linked dominant inheritance. CHILD syndome also has
findings of stippled epiphyses. Incontinentia pigmenti is caused by defecdts in the NEMO gene.
Findings include peg/conical teeth, eye and CNS defects and alopecia. There are no bone
abnormalities. Focal Dermal Hypoplasia, otherwise known as Goltz syndrome has findings of
linear atrophy following Blaschko's lines with areas of fat herniation, mucocutaneous papillomas
and pits, alopecia, nail dystrophy, tooth abnormalities and osteopathia striata (striations of the

228
long bones). Bazex syndrome is associated with follicular atrophoderma, hypohidrosis,
hypotrichosis and multiple basal cell carcinomas. There are no bone abnormalities associated.

238
Electron microscopic examination of a hair shaft reveals a canal-like groove along the shaft of a
triangular-shaped hair. This patient has:
A. Netherton‖s syndrome
B. Menke‖s Kinky Hair syndrome
C. Spun-glass hair
D. Trichothiodystrophy
E. Bjornstad syndrome
►C

Pili trianguli et canaliculi is also known as Spun-glass hair or Uncombable Hair Syndrome.
Netherton patients have trichorexis invaginata, Menke‖s kinky hair patients have short, brittle
sparse hairs, ―tiger tail‖• hair is seen in trichothiodystrophy, and pili torti is seen in bjornstad
syndrome.

239
Which of the following is the first symptom of ataxia telangiectasias?
A. Conjunctival telangictases
B. Facial telangiectases
C. Hematologic malgignancy
D. Cerebellar ataxia
E. Breast cancer
►D

Ataxia Telangiectasia (Louis-Bar syndrome) is an autosomal recessive disorder usually caused


by mutations in the ATM gene, which is a chromosomal strand break repair enzyme. Cerebellar
ataxia is the first sign, followed by telangiectases of the conjunctiva and skin. Thymic hypoplasia
predisposes to increased infections. There is increased sensitivity to ionizing radiation resulting
in hematologic and solid tumors. Female carriers have increased risk of breast cancer.

240
The nucleotide excision DNA repair pathway is defective in which disease:
A. Bourneville's disease
B. Severe combined deficiency syndrome
C. Griscelli syndrome
D. Xeroderma pigmentosa
E. Sjogren-Larssen syndrome

229
►D

The pathogensis of Xeroderma Pigmentosum shows mutations i genes encoding DNA repair
enzymes, leading to defective DNA excision repair upon exposure to UV radiation. Severe
combined deficiency syndrome-major defect in cell-mediated and humoral immunity; most lack
antibody-dependent cellular cytotoxicity and natural killer cell function. The pathogensis for
Griscelli Syndrome is a mutation in gene encoding for myosin Va or RAB27 a. Sjogren-Larsson
Syndrome has mutations in the FALDH gene. Bourneville's syndrome (Tuberous Sclerosis)
shows a mutation in either TSC1 ancoding hamartin or TSC2 encoding tuberin.

241
A patient with myotonic dystrophy and multiple skin lesions most likely has activating mutations
in which of the following?
A. Beta-catenin
B. Desmoplakin
C. Plakoglobin
D. Desmoglein
E. Alpha 6-beta 4 integrin
►A

Myotonic dystrophy with multiple pilomatricomas is described above. Activating mutations in


Beta-catenin are found in this syndrome. The other listed options are desmosomal proteins and
are not involved in this syndrome.

242
Epidermal nevus syndromes inheritance pattern is:
A. Sporadic
B. X-linked recessive
C. X-linked dominant
D. Autosomal recessive
E. Autosomal dominant
►A

Epidermal nevus syndrome has many findings, including: sporadic inheritance, nevus unius
lateris, capillary malformations, cafe au lait macules, mantal retardation and seizures, deafness,
hemiparesis, hemihypertrophy of limbs, kyphoscoliosis and rare solid tumors. A biopsy is
helpful to rule out epidermolytic hyperkeratosis. If positive, the patient‖s offspring are at risk for
generalized epidermolytic hyperkeratosis.

243

230
A 5 month old girl presents with failure to thrive. She has had life-long atopic dermatitis treated
with topical hydrocortisone cream and has persisent hypernatremia. On your exam, she has
generalized erythema and scaling of her body and trichorrhexis invaginata on examination of
hairs from her eyebrows. Which syndrome is she most likely to have?
A. Leiner syndrome
B. Omenn Syndrome
C. Netherton Syndrome
D. Wiskott-Aldrich Syndrome
E. Severe atopic dermatitis
►C

Netherton syndrome is caused by a mutation in the SPINK5 gene, encoding LEKT1. This is a
serine protease inhibitor which is important in downregulating inflammation. Early presentation
is with failure to thrive, generalized erythema/scale, hypernatremia, and sparse hair with the
characteristic finding of trichorrhexis invaginata. Pili torti and trichorrhexis nodosa also can be
seen. Eyebrow hair is most commonly affected. Omenn syndrome is an autosomal recessive
form of severe combined immunodeficiency (SCID) with findings of failure to thrive,
erythroderma, scaling, chronic diarrhea, lymphadenopathy, and hepatosplenomegaly. Leiner
syndrome can present with failure to thrive, immunodeficiency and seborrheic dermatitis.
Wiskott-Aldrich syndrome is an x-linked recessive condition with mutations of the WAS gene.
Presentation includes atopic dermatitis, thrombocytopenia, recurrent bacterial infection,
lymphoreticular malignancy with non-Hodgkin's lymphoma being the most common, and
increased IgA, D and E. With the characteristic hair changes, atopic dermatitis alone is not the
most likely diagnosis.

244
Focal Dermal Hypoplasia (Goltz Syndrome) can differentiated from Incontinentia Pigmenti by
A. Type of inheritance
B. Presence of Linear lesions along the lines of Blaschko
C. Presence of blistering lesions
D. Hair and teeth abnormality
E. Eye and CNS abnormality
►C

Both Focal Dermal Hypoplasia (Goltz Syndrome) and Incontinentia Pigmenti are inherited as X
linked dominant (lethal in males). And both can have lesions along the lines of Blaschko with
many similarities in systemic involvement. However, Incontinentia Pigmenti is differentiated
from Focal Dermal Hypoplasia by presence of blistering lesions in addition to hyperkeratosis and
hyperpigmentation.

231
245
A patient with 20 nail dystrophy, steatocystoma multiplex and natal teeth likely has a mutation in
the genes coding for:
A. Keratins 5
B. Laminin 5
C. Plakophilin 1
D. Keratins 6b & 17
E. Keratins 6 &16
►D

Pachyonychia congenital is an autosomal dominant condition with 20 nail dystrophy. The patient
described has Type II (Jackson-Sertole) disease, which includes steatocystoma multiplex, natal
teeth, multiple cysts, and micropthalmia, and is caused by mutations in keratins 6b& 17. Type I
(Jadassohn-Lewandowsky) also includes focal symmetric PPK, follicular hyperkeratosis, oral
leukokeratoses and is caused by mutations in keratins 6 &16. Type III includes the clinical
features of type I + corneal leukokeratosis. Mutations in keratins 5&14 represents EB simplex,
Laminin 5 mutation is seen in Junctional EB, and plakophilin 1 mutation is seen in ectodermal
dysplasia with skin fragility.

246
Which of the following is caused by a defect in lysosomal transport:
A. oculocutaneous albinism I
B. oculocutaneous albinism II
C. Piebaldism
D. Chediak-higashi
E. Bloom‘s syndrome
►D

Chediak higashi is an autosomal recessive disorder caused by a mutation in the LYST gene
codes for a lysosomal tracking protein. This protein regulates microtubule mediated lysosomal
fusion. A defect in this gene leads to giant lysosomal granules seen in neutrophils (leading to
defecting phagocytosis and decreased chemotaxis), melanocytes (pigment dilution), and neurons.
OCA1 is tyrosinase negative albinism; OCA2 is tyrosinase positive albinism with a mutation in P
gene on chromosome 15.

247
Germline KILLIN methylation is associated with which of the following syndromes?
A. Bannayan-Riley-Ruvalcaba Syndrome
B. Proteus Syndrome
C. Cowden Syndrome

232
D. Birt-Hogg-Dube Syndrome
E. Galli-Galli Disease
►C

Loss-of-function mutations in phosphatase and tensin homolog gene (PTEN) mutations cause
80% of Cowden Syndrome, a rare autosomal-dominant disorder, characterized by high risks of
breast, thyroid, and other cancers. Other mechanisms of loss of function such as
hypermethylation of the KILLIN gene has been identified in Cowden Syndrome. Bannayan-
Riley-Ruvalcaba and Proteus syndrome are associated with PTEN loss-of-function mutations.
Birt-Hogg-Dube is associated with loss-of-function mutations in FLCN gene. Galli-Galli Disease
is not a malignancy-associated disease and is the result of KRT5 mutations.

248
The main cause of death in patients with dyskeratosis congenita is which of the following?
A. Oral basal cell carcinoma
B. Leukemia
C. Renal cell carcinoma
D. Pancytopenia
E. Atherosclerotic heart disease
►D

Dyskeratosis congenita is usually inherited in an X-recessive fashion due to mutations in the


dyskerin gene, which is involved in ribosomal RNA synthesis. The less common autosomal
dominant form is caused by mutations in the telomerase gene. Clinically, there is reticulated
pigmentation of skin, poikiloderma, alopecia, nail atrophy, premalignant oral leukoplakia, and
Fanconi-type pancytopenia that can result in early death. They also get severe oral SCC.

249
Pili trianguli et canaliculi is characteristic of which of the following syndromes?
A. Uncombable hair syndrome
B. Bjornstad syndrome
C. Menkes kinky hair syndrome
D. Leiners disease
E. Netherton's syndrome
►A

Uncombable hair syndrome is characterized by pili trianguli et canaliculi. On examination of the


hair, it is triangular with a canal-like groove runs along the shaft. The clinical findings are that of
blond, shiny, "spun glass" hair. It is an autosomal dominant syndrome with no known gene
locus. The remaining syndromes do not have this hair finding present.

233
250
A patient with neurofibromatosis 1 has axillary and inguinal freckling that is also known as:
A. Crowe's sign
B. Button hole sign
C. Hutchinson's sign
D. Plexiform sign
E. Neurofibromin sign
►A

Patient with NF 1 have an initial presenting sign of cafe-au-lait macules. Axillary or inguinal
freckling is known as Crowe's sign. This is pathogonomic for NF 1. Patients with NF 1 can have
a lit lam examination that will show Lisch nodules that will confirm the diagnosis.

251
Ichthyosiform erythroderma in lines of Blaschko, follicular atrophoderma, and stippled
epiphyses are characteristic of which of the following disorders?
A. Refsum syndrome
B. Chondrodysplasia punctata
C. Erythrokeratodermia variabilis
D. Netherton syndrome
E. Sjogren-Larsson syndrome
►B

Chondrodysplasia punctata is a peroxisomal disorder caused by mutations in several genes that is


associated with ichthyosiform erythroderma in lines of Blaschko, follicular atrophoderma, and
stippled epiphyses.

252
Dermatofibrosis lenticularis disseminata is seen in which of the following conditions?
A. Ehlers-Danlos syndrome
B. Marfan syndrome
C. Pseudoxanthoma elasticum
D. Focal dermal hypoplasia
E. Buschke-Ollendorf syndrome
►E

Buschke-Ollendorf syndrome is an autosomal dominant disorder characterized by


dermatofibrosis lenticularis disseminata (cutaneous elastomas distributed symmetrically over the
buttocks, trunk and proximal extremities), and osteopoikilosis (round opacities in bones).

234
253
Epistaxis in early childhood to teens followed by multiple cutaneous and gastrointestinal
telangectasias describes which of the following syndromes?
A. Hereditary Hemorrhagic Telangiectasia syndrome
B. Maffucci syndrome
C. CREST syndrome
D. Ataxia telangectasia
E. Fabry disease
►A

Hereditary Hemorrhagic Telangiectasia syndrome is described above. The first signs in over
50% of cases is epistaxis in childhood to young adulthood. Telangectiasias develop in the 30's
and 40's. Other findings include gastrointestinal telangiectasia, hepatic and pulmonary
arteriovenous malformations. The other syndromes listed can have cutaneous vascular lesions
and should be considered on the differential for hereditary hemorrhagic telangiectasia syndrome.

254
What is the genetic defect of this syndrome?
A. ABCA12 gene
B. Transglutaminase 1 (TGM1).
C. GJB2 (connexin 26) gene.
D. Steroid sulfatase (STS) gene
E. Profilaggrin gene
►A

Mutations in the ABCA12 gene cause harlequin ichthyosis. X-linked ichthyosis (XLI) caused by
a steroid sulfatase (STS) deficiency. Keratitis-ichthyosis-deafness syndrome is caused by a
mutation in the GJB2 (connexin 26) gene. Ichthyosis vulgaris is caused by a defect in
profilaggrin gene and lamellar ichthyosis involves a mutation in the gene for transglutaminase 1
(TGM1).

255
A patient with melanoma and a malignant glioma is diagnosed with Li-Fraumeni syndrome.
Which of the following tumors occurs most frequently in this disease?
A. Rhabdomyosarcoma
B. Adrenocortical carcinoma
C. Lung carcinoma
D. Breast carcinoma
E. Leukemia

235
►A

Li-Fraumeni syndrome is a familial tumor syndrome caused by mutations in the tumor


suppressor gene p53. They are at risk for a wide range of malignancies with particularly high
occurrences of soft tissue sarcomas, breast cancer, brain tumors, acute leukemia, and adrenal
cortical carcinoma. Soft tissue sarcomas are among the most common reported with this disease.

256
Findings of dysplastic nevi and melanoma inherited in an autosomal dominant fashion is linked
with which of the following?
A. Breast malignancy
B. Thyroid malignancy
C. Pancreatic malignancy
D. Renal cell carcinoma
E. Colon carcinoma
►C

Familial dysplastic nevi/melanoma syndrome is linked with increased risk of pancreatic cancers
and astrocytomas. There are no reported increased risks for patients with familial dysplastic nevi
syndrome and the other types of cancers listed.

257
Mucosal neuromas, pheochromocytoma and medullary thyroid carcinoma in a patient with a
marfanoid body habitus is associated with which of the following gene defects?
A. Menin
B. RET proto-oncogene
C. PTEN
D. BHD
E. STK11
►B

The RET proto-oncogene is mutated in Multiple Endocrine Neoplasia type II. Type IIb is
described above. Other findings include rare parathyroid abnormalities, megacolon, thickened
lips and thick, everted upper eyelids. Menin is associated with MEN type I, PTEN with Cowden
disease, BHD with Birt-Hogg-Dube syndrome and STK11 with Peutz-Jeghers syndrome.

258
Hyperextesible skin, gaping wounds, cigarette-paper scars, molluscoid pseudotumors, and
calcified subcutaneous nodules are characteristic of which syndrome?
A. Marfan syndrome

236
B. Congenital contractural arachnodactyly
C. Ehlers-Danlos Syndrome
D. Cutis laxa
E. Pseudoxanthoma elasticum
►C

The most common type of EDS is EDS type I (gravis). Cutaneous features include
hyperextensible skin, gaping wounds, cigarette-paper scars, molluscoid pseudotumors, calcified
subcutaneous nodules, and bruises. Systemic features include hypermobile joints with
dislocation, hernias, mitral valve prolapse, blue sclerae, Gorlin‖s sign (tongue reaches nose), and
absence of the lingual frenulum.

259
Which of the following syndromes is associated with cutis marmorata?
A. Netherton‘s
B. Papillon-Lefevre syndrome
C. Hemansky-Pudlak syndrome
D. Maffucci syndrome
E. Cornelia de Lange syndrome
►E

Cornelia de Lange is also known as Brachmann-de Lange syndrome. Cutaneous manifestations


include cutis marmorata, hirsutism, hypoplastic nipples and umbilicus. Patients also have small
hands and feet. They have characteristic facies which include hirsutism on the forehead,
trichomegaly, synophrys, anteverted nostrils, long philtrum, and low-set ears.

260
A patient is diagnosed with Hay Wells syndrome with wiry, sparse hair and ankyloblepharon.
The patient has partial anhidrosis, cleft lip, palate and dystrophic nails. The mutation is in the:
A. p63
B. TP73L
C. DLX3
D. PORCN
E. Arylsulfatase
►A

Patients with Hay-wells syndrome has a mutations in P63. They have wiry, sparse hair,
ankyloblepharon, PPK, partial anhidrosis, cleft lip, palate, absent, and dystrophic nails.

261

237
Spastic ditetraplegia is associated with which of the following disorders?
A. Sjogren-Larsson syndrome
B. X-linked ichthyosis
C. Lamellar ichthyosis
D. KID syndrome
E. Refsum syndrome
►A

Sjogren-Larsson syndrome is an autosomal recessive disorder caused by mutations in the fatty


aldehyde oxidoreductase/alcohol dehydrogenase gene. This disorder is characterized by
ichthyosis, spastic ditetraplegia, mental retardation, epilepsy, glistening dot retinal pigmentation,
and dental enamel dysplasia.

262
Painful crises and 'whorled' corneal opacities are seen with which of the following enzyme
abnormalities?
A. Homogentisic acid oxidase
B. Alpha-galactosidase A
C. Glucocerebrosidase
D. Iduronate sulfatase
E. Glucoronidase
►B

Painful crises and whorled corneal opacities are found in Fabry disease which is caused by a
defect in alpha-galactosidase A. The remaining conditions do not have these findings.

263
Enchondromas and chondrosarcomas are most strongly associated with which of the following
syndromes?
A. Proteus syndrome
B. Osler-Weber-Rendu syndrome
C. Maffucci syndrome
D. Nonne-Milroy disease
E. Blue rubber bleb nevus syndrome
►C

Maffucci syndrome is a sporadic condition caused by defects in the parathyroid


hormone/parathyroid hormone related protein type 1 receptor. There are venous malformations
of distal extremities, and benign enchondromas that can degenerate into chondrosarcomas.

238
264
To help diagnose trichothiodystrophy, which of the following levels are decreased in hairs of
affected individuals?
A. Arginine
B. Histidine
C. Phenylalanine
D. Glycine
E. Cysteine
►E

Cysteine and Methionine levels are decreased in hair and nails of patients with
trichothiodystrophy. Other sulfur containing amino acids include: Gluthathione, Taurine, and
Homocysteine. Testing hairs for decreased sulfur content is an indirect method of determining
this. The other listed amino acids are present in normal levels in the hair and nails of
trichothiodystrophy patients.

265
A mitten deformity is most characteristic of what inherited disease?
A. Hallopeau Siemens dystrophic epidermolysis bullosa
B. Non-hallopeau Siemens dystrophic epidermolysis bullosa
C. Herlitz type junctional epidermolysis bullosa
D. Non-herlitz type junctional epidermolysis bullosa
E. Epidermolysis bullosa simplex
►A

Hallopeau Siemens dystrophic EB is characterized by pseudosyndactyly, mitten deformities of


the hands and feet. Non Hallopeau Siemens is much less severe. Not present in junctional
epidermolysis bullosa or epidermolyisis bullosa simplex.

266
Which keratins are expressed in the suprabasal palmoplantar epidermis?
A. Keratins 1 and 9
B. Keratins 1 and 10
C. Keratins 4 and 13
D. Keratins 5 and 14
E. Keratins 8 and 18
►A

Keratins are expressed in pairs of acidic and basic keratins and are tissue- and differentiation-
specific. Keratins 1 and 9 are expressed in the suprabasal palmoplantar epidermis, keratins 1 and

239
10 in suprabasal nonpalmoplantar epidermis, keratins 2e and 10 in the granular layer, keratins 4
and 13 in mucosal epithelium, keratins 5 and 14 in the basal layer, and keratins 8 and 18 in
simple epithelium.

267
Ehlers-Danlos Syndrome with congenital adrenal hyperplasia is caused by mutations affecting
which of the following?
A. Lysyl hydroxylase
B. Collagen 5
C. Fibronectin
D. Lysyl oxidase
E. Tenascin-X
►E

Ehlers-Danlos Syndrome type with congenital adrenal hyperplasia is caused by mutations in


tenascin-X.

268
This syndrome has colobomas of the eyes, heart defects, ichthyosiform dermatosis, mental
retardation and ear defects. These are the characteristic findings of:
A. CHIME syndrome
B. Vogtg-Koyanagi-Harada syndome
C. Sturge Weber syndrome
D. Conradi-Hunermann syndrome
E. Refsum syndrome
►A

CHIME syndrome has all the findings of colobomas of the eye, heart defects, ichthyosiform
dermatosis, mental retardation and ear defects.

269
The treatment for acrodermatitis enteropathica is:
A. Zinc supplementation
B. Iron supplementation
C. Vitamin B1 supplementation
D. Vitamin B12 supplementation
E. Phlebotomy
►A

240
Acrodermatitis enteropathica is due to a defect in zinc absorption and will respond to zinc
supplementation. Iron, Vitamin B1/12 supplementation will not result in improvement in this
condition. Findings include periorificial, scalp, and acral dermatitis, scaling, vesicles/bullae,
erosions, alopecia, diarrhea and stomatitis.

270
What condition is associated with this finding of inflammatory keratotic facial papules which
may result in scarring and atrophy?
A. Chloracne
B. Systemic lupus erythematosus
C. Keratosis pilaris
D. Reiter's syndrome
E. Ulerythema ophryogenes
►E

Ulerythema ophryogenes is a rare disorder that affects children and young adults. It is
characterized by keratosis pilaris atrophicans and loss of lateral third of eyebrow.

271
Which of the following subtypes of Ehlers-Danlos Syndrome (EDS) is associated with early
demise?
A. Type 1 (Gravis)
B. Type 4 (Vascular)
C. Type 5 (X-linked)
D. Type 7 (Arthrochalasis multiplex congenita)
E. Type 10 (Fibronectin)
►B

Ehlers-Danlos Syndrome type 4 (Vascular) is caused by mutations in collagen 3 in some cases.


There is a tendency to develop arterial and visceral rupture resulting in early death.

272
The characteristic dental findings in patients with tuberous sclerosis are:
A. Peg teeth
B. Anodontia
C. Enamel pits
D. Odontogenic cysts
E. Retention of primary teeth
►C

241
Enamel pits are the characteristic dental findings in tuberous sclerosis. Peg teeth are found in
incontinentia pigmenti and anhidrotic ectodermal dysplasia. Anodontia is found in
hypomelanosis of ito and incontinentia pigmenti. Odontogenic cysts are seen in Gorlin
syndrome, and retention of primary teeth is characteristic of Job syndrome.

273
Which of the following syndromes are linked to a AKT1 gene mutation?
A. Banayan-Zonana
B. Proteus
C. Cowden
D. All of these options are correct
E. Lhermitte-Duclos disease
►B

Proteus is due to an AKT1 mutation. The remaining syndromes are due to PTEN mutations. The
PTEN gene produces a phosphatase the regulates the cell-cycle and apoptosis, therefore acting as
a tumor suppressor gene. Tissues affected by this mutation are those with increased proliferation
such as epidermis, the oral and gastrointestinal mucosa, and the thyroid and breast epithelium.
All of the syndromes listed have mutations in PTEN.

274
A mentally retarded patient is found to also have a malar rash and downward lens displacement.
Another clinical feature might include:
A. Hypertrichosis
B. Short stature
C. Multiple nevi
D. Periodontitis
E. Deep venous thromboses
►E

The above described patient has homocystinuria, an autosomal recessive disorder caused by a
deficiency in cystathionine beta-synthase, leading to an accumulation of homocysteine. Clinical
features include: malar flush, DVTs, livedo reticularis, leg ulcers, downward lens displacement,
myopia, glaucoma, sparse fine hair, mental retardation, and musculoskeletal anomalies.

275
Which of the following bony defect is found in CHILD syndrome?
A. Polyostotic fibrous dysplasia
B. Stippled epiphyses
C. Calcification of falx cerebri

242
D. Osteopoikilosis
E. Sphenoid wing dysplasia
►B

CHILD syndrome is an X-linked dominant syndrome which is lethal in males. It is caused by a


peroxisomal biogenesis disorder. It is characterized by unilateral ichthyosiform erythroderma,
limb/visceral hypoplasia, and stippled epiphyses. Stippled epiphyses can also be seen in
chondrodysplasia punctata. Polyostotic fibrous dysplasia is found in McCune-Albright
syndrome, calcification of falx cerebri in Gorlin's syndrome, osteopoikilosis in seen in Buschke-
Ollendorf syndrome.

276
Which of the following diseases is caused by a mutation in a gap junction protein?
A. Striated PPK
B. Schopf-Schulz-Passarge syndrome
C. Mal de Meleda
D. Vohwinkel syndrome (classic)
E. Vohwinkel syndrome (ichthyotic)
►D

Classic Vohwinkel syndrome is caused by mutations in connexin 26, a gap junction protein.
Ichthyotic Vohwinkel syndrome is caused by mutations in loricrin and has ichthyosis but not
deafness.

277
A patient that has the clinical triad of ichthyosis, di- or tretraplegia, mental retardation that is
caused by a deficiency in FALDH has:
A. Sjogren Larsson Syndrome
B. KID Syndrome
C. Vohwinkel Syndrome
D. Refsum Disease
E. Bjornstad's Syndrome
►A

This patient has Sjorgen-Larsson syndrome, which is autosomal recessive disorder caused by a
deficiency of FALDH. These patients have the clinical triad of congenital ichthyosis, tetraplegia,
and mental retardation. The ichthyosis is notable in that there is accentuation of the flexures and
lower abdomen.

278

243
A patient develops palmoplantar keratoderma and has esophageal cancer. They also state that
their parent had the same condition. This is most characteristic of:
A. Howel-Evans
B. Cowden's
C. Fabry's Disease
D. Castleman's tumor
E. Dowling darlings syndome
►A

Patients that have Howel-Evans syndrome are subject to squamous cell carcinoma of the
esophagus and also PPK on the hands. It is inherited in an autosomal dominant fashion.

279
A patient with thyroid carcinoma and cobblestone-like changes of the oral mucosa will also
likely have:
A. Trichoepitheliomas
B. Fibrofolliculomas
C. Tricholemmomas
D. Cylindromas
E. Syringomas
►C

The patient described may have Cowden's syndrome, an autosomal dominant condition caused
by a defect in the PTEN tumor suppressor gene. Patients with Cowden's disease are at increased
risk for thyroid and breast carcinoma. In addition, they characteristically have multiple
hamartomatous polyps of the gastrointestinal tract that are typically benign. Cutaneous features
of Cowden's syndrome which may serve as clues to the diagnosis include multiple oral
papillomas with a "cobblestone" appearance on the lips, gingival, and buccal mucosa, acral
keratotic papules on the dorsal hands and wrists, palmoplantar punctate keratoses and multiple
facial tricholemmomas. Patients with this condition need careful malignancy surveillance.

280
Which of the following is caused by a mutation in a gene which codes for steroid sulfatase?
A. Fabry disease
B. Lesch-nyhan disease
C. X-linked ichthyosis
D. Lamellar icthyosis
E. Chediak-Higashi
►C

244
The mutation in X-linked icthyosis is found in the gene for aryl sulfatase C, a steroid sulfatase.
The genetic defect in Fabry disease occurs in alpha-galactosidase A (which hydrolyzes
glycolipids and glycoproteins), the defect in lamellar icthyosis codes for transglutaminase 1 and
the defect in chediak higashi occurs in a lysosomal transport protein.

281
A patient is diagnosed with Naxos syndrome with woolly hair, diffuse palmoplantar
keratoderma, and right ventricular arrhythmogenic cardiomyopathy. The mutation is in:
A. Plakoglobin
B. DNA helicase
C. Ectodysplasin A
D. Connexin 30
E. Connexin 26
►A

Patients with Naxos syndrome has a mutation in plakoglobin which is autosomal recessive. It is
associated with woolly hair and cardiomyopathy.

282
Which is a major criterion for the diagnosis of neurofibromatosis type 1?
A. One plexiform neurofibroma
B. Scoliosis
C. Pheochromocytoma
D. Hypertension
E. Two cafe au lait macules
►A

The major diagnostic criteria are: 6 or more cafe au lait macules >5mm in prepubertal
individuals and >15mm in post pubertal individuals, 2 or more neurofibromas of any type or one
plexiform neurofibroma, axillary or inguinal freckling, 2 or more lisch nodules, sphenoid wing
dysplasia or thinning of long bone cortex with or without pseudoarthosis, first degree relative
with NF1.

283
A patient is diagnosed with progeria and has a large bald head, prominent scalp veins, and lack
of eyebrows and eyelashes. The gene mutation is the:
A. LMNA gene
B. Recq12 gene
C. PTPN11 gene
D. PRKAR1A gene

245
E. PTEN gene
►A

A patient was progeria has a mutation in the LMNA gene which encodes lamin A and C. It is
distinguished from adult progeria by early onset.

284
A 15 year old male patient is seen for evaluation of a congenital nevus. As a part of the review of
systems, it is discovered that he has recurrent joint dislocations. Multiple family members have a
similar problem. What is the defective gene in Type 3 Ehlers-Danlos syndrome?
A. Collagen 3
B. Collagen 5
C. Lysyl oxidase
D. Collagen 2
E. PLOD gene
►A

Type 3 Ehlers-Danlos syndrome, or benign hypermobile Ehlers-Danlos syndrome, is a relatively


mild form of EDS with recurrent joint dislocations. The syndrome is a result of defective
collagen 3. Defective collagen 5 is seen in EDS types 1 and 2 (gravis and mitis, respectively).
Lysyl oxidase deficiency is seen in Type 5 (x-linked). Collagen 2 mutations can lead to Type 7
(arthrochalasis multiplex congenita). The PLOD gene is involved in type 6 (ocular-scoliotic).

285
Mucosal malignancy is a complication of:
A. Oral hairy leukoplakia
B. White sponge nevus
C. Dyskeratosis congenita
D. Chronic candidiasis
E. Focal epithelial hyperplasia
►C

Dyskeratosis congenita, also called Zinsser-Engman-Cole syndrome, is an X-linked recessive


genodermatosis caused by a mutation in DKC1. DKC1 encodes for dyskerin, which helps to
maintain telomeres through the pseudouridylation of rRNA. Features of this condition include
reticulate gray-brown hyperpigmentation, dystrophic nails, alopecia and Fanconi‖s type
pancytopenia. Patients may have premalignant leukoplakia which should be followed closely.

286
Patients with homocystinuria are deficient in:

246
A. Alpha galactosidase A
B. Transglutaminase
C. Lysyl oxidase
D. Cystathionine synthetase
E. Steroid sulfatase
►D

Homocystinuria is an autosomal recessive disease caused by a deficiency of cystathionine


synthetase. Manifestations of this condition may include mental retardation, marfanoid habitus,
malar flush, ectopic lentis, embolic phenomena and leg ulcers.

287
Which of the following is caused by a defect in cathepsin C?
A. Rubenstein-Taybi
B. Bullous Icthyosis of Siemens
C. Muir-Torre
D. Papillon-Lefevre
E. Nail-Patella syndrome
►D

Papillon-Lefevre is a palmoplantar keratoderma caused by a mutation in chromosome 11q14,


leading to a defect in Cathepsin C, a lysosomal enzyme. Clinical manifestations of papillon
lefevre include sharply demarcated palmoplantar keratoderma with extension to dorsal surface
(transgrediens), spare hair, periodontitis, and pyoderma.

288
What is this syndrome which is histologically characterized by widely dispersed granular
material amidst normal fibers?
A. Ehlers Danlos Syndrome
B. Pseudoxanthoma Elasticum
C. Buschke-Ollendorf Syndrome
D. Focal Dermal Hypoplasia
E. Lipoid Proteinosis
►B

Pseudoxanthoma elasticum is genodermatosis characterized by redundant skin, angioid streaks,


yellow papules on the mucous membranes and bleeding from gastric artery. On histology,
readily apparent denerative changes of the elastic fibers are prominent, even without special
stains.

247
289
Odontogenic cysts and palmoplantar pits are seen in:
A. Gardner‖s syndrome
B. Gorlin Syndrome
C. Bloom‖s Syndrome
D. Goltz Syndrome
E. Refsum syndrome
►B

Gorlin syndrome (Basal Cell Nevus Syndrome)is an autosomal dominant disorder due to a defect
in the PTCH gene whose function normally inhibits ―SMOOTHENED‖ signaling. Odontogenic
cysts and palmoplantar pits are characteristic features, in addition to multiple basal cell
carcinomas. Other features include: frontal bossing, kyphoscoliosis, calcification of falx cerebri,
hypertelorism, ovarian fibromas and rarely mental retardation.

290
Which of the following mutations has been linked to an NF-1 like presentation?
A. SLURP-1
B. Plakophilin-1
C. SPRED-1
D. ALK-1
E. TSC-1
►C

SPRED-1 acts as a suppressor of the Ras/MAPK pathway, the same pathway involved in
neurofibromatosis-1. Likewise, defects in SPRED1 are the cause of neurofibromatosis type 1-
like syndrome, also known as Legius Syndrome. Legius Syndrome has the following
characteristics: cafe au lait spots, axillary and inguinal freckling, learning disability, and
macrocephaly; likewise patients can actually meet criteria for the clinical diagnosis of NF-1.
Although Legius Syndrome has many overlapping features with NF1, neurofibromas, lisch
nodules, and CNS tumors have not been reported. (Brems H, et al. Nat Gen 2007.)

291
A 11 year-old female patient with hypoparathyroidism is referred to your clinic secondary to
chronic mucocutanous candidiasis which is refractory to standard treatments. The patient also
has malabsorption and severe chronic diarrhea. You determine that she has autoimmune
polyendocrinopathy-candiasis-ectodermal dystrophy syndrome(APECED). What is the gene
defect for APECED?
A. XAP101 gene
B. AIRE gene

248
C. CGI-58 gene
D. ALOX gene
E. PAHX gene
►B

The gene responsible for APECED is the AIRE gene, an autoimmune regulator. The XAP101
encodes dyskerin, a defect in which causes dyskeratosis congenita. A defect in CGI-58 causes
Chanarin-Dorfman or neutral lipid storage disease with ichthyosis. Defect in the ALOX genes
cause congenital ichthyosiform erythroderma. A defect in the PAHX gene causes Refsum
syndrome.

292
An infant girl of short stature and shortened 4th and 5th metacarpals is being evaluated for
coarctation of the aorta and horseshoe kidneys. Physical examination most likely reveals:
A. Webbed neck
B. Alopecia
C. Hemangioma
D. Giant congenital melanocytic nevus
E. Arachnodactyly
►A

Turner‖s syndrome results from nondysjunction during gametogeneiss leading to the XO


genotype. Clinical features include short stature, redundant neck folds/webbed neck, multiple
pigmented nevi, low set hairline, triangular facies, low-set ears, ptosis, wide-set nipples,
shortened 4th and 5th metacarpals, hypoplasia of lymphatics, coarctation of the aorta, and
horseshoe kidneys.

293
The coast of Maine cafe au lait macule is a common finding in which of the following
syndromes?
A. Tuberous Sclerosis
B. Neurofibromatosis type I
C. McCune-Albright syndrome
D. Watson syndrome
E. Russell-Silver syndrome
►C

McCune-Albright syndrome is due to a sporadic somatic mutation in Gs subunit of adenylate


cyclase. The "coast of Maine" cafe au lait macule is a characteristic finding. Other findings in
clude polyostotic fibrous dysplasia and precocious puberty. Tuberous sclerosis,

249
Neurofibromatosis type I, Watson syndrome (pulmonic stenosis and CALM's) and Russell-Silver
syndrome (CALMs, short stature, precocious puberty, cryptorchidism and
musculoskeletal/craniofacial defects) all have CALMs as a feature, but not the jagged type seen
in McCune-Albright syndrome.

294
A patient is diagnosed with Rothmund-Thomson syndrome and has juvenile cataracts,
hypogonadism and dystrophic nails. The hair findings is/are:
A. Sparse scalp, eyebrow, and eyelash hair
B. Unruly hair
C. Uncombable hair
D. Woolly hair
E. Pili trianguli et canaliculi
►A

Rothmund-Thomson syndrome is an autosomal recessive disorder. They have a sparse scalp,


eyebrow, and eyelash hairs. Poikiloderma is seen in sun-exposed areas, dystrophic nail and
juvenile cataract.

295
In ataxia telangectasia, the ATM gene is mutated. The product of the ATM gene is an enzyme
which:
A. Responds to DNA damage by phosphorylating key DNA repair substrates
B. Binds transforming growth factor beta protein
C. Is the VEGF receptor 3
D. Participates in NF-kB activation
E. Is an inhibitor of G1 cyclin/Cdk complexes
►A

The ATM gene is a member of the phosphatidylinositol-3 family of proteins that respond to
DNA damage by phosphorylating key substrates involved in DNA repair according to OMIM.
Defects in endoglin (TGF beta3 binding protein) is deficient in Osler-Weber-Rendu syndrome.
The VEGF receptor 3 is defective in hereditary lymphedema. The NEMO gene is defective in
Incontinentia Pigmenti. Its product, NF-kB essential modulator (NEMO) is a key activator in the
NF-kB pathway. KIP2 is involved in AD Beckwith-Wiedemann syndrome and is an inhibitor of
G1 cyclin/Cdk complexes.

296
Which of the following conditions is worsened by ingestion of lithium?
A. Darier‘s Disease

250
B. Hailey-Hailey Disease
C. Haim-Munk syndrome
D. Hereditary lymphedema (Nonne-Milroy disease)
E. Epidermolytic hyperkeratosis
►A

Patients with Darier‘s disease should not be treated with lithium due to its worsening or in some
cases unmasking the disease. The mechanism for this is not known. Lithium treatment does not
worsen the other listed conditions.

297
Which of the following signs is not a criteria for the diagnosis of Neurofibromatosis type I?
A. Bilateral vestibular schwannomas
B. Axillary freckling
C. Optic gliomas
D. greater than 5 café-au-lait macules
E. Sphenoid dysplasia
►A

Bilateral vestibular schwannomas are related to type II neurofibromatosis. The remaining options
are diagnostic criteria for NF-1. Greater than 5 CALMS is the same as 6 or more CALMS as
listed in the diagnostic criteria.

298
A thirty-year-old woman presents with new progressively worsening headaches. Upon exam,
you notice multiple acral keratotic papules and papillomas of the tongue. What malignancy does
this patient need to be screened for?
A. colon cancer
B. ovarian cancer
C. breast cancer
D. basal cell carcinoma
E. melanoma
►C

This patient has Lhermitte-Duclos disease, which is a hamartomatous overgrowth of cerebellar


ganglion cells. Approximately half of patients have Cowden syndrome. Breast cancer is the
correct answer, which affects 25-35% of female patients. Patients can also develop thyroid and
genitourinary carcinoma (endometrial, urethral, renal cell, and transitional cell carcinoma of the
renal pelvis). Malignant degeneration of hamartomatous colon polyps is rare.

251
299
Multiple lipomas may be found in all of the following except?
A. Gardner's syndrome
B. Gorlin's syndrome
C. Cowden's syndrome
D. Neurofibromatosis
E. Proteus syndrome
►B

Gorlin's syndrome (Basal cell naevus syndrome) is not associated with multiple lipomas. The
PTEN associated syndromes (Cowden's, Bannayan-Riley-Ruvalcaba and Proteus syndromes) are
all associated with multiple lipomas as well as Gardner's, neurofibromatosis and MEN.

300
Which of the following syndromes is X-linked dominant?
A. Anhidrotic ectodermal dysplasia
B. Dyskeratosis congenita
C. X-linked icthyosis
D. Orofaciodigital syndrome 1
E. Menkes kinky hair syndrome
►D

Orofaciodigital sydrome 1 is an X-linked dominantly inherited disorder caused by a defect in the


CXORF5 gene. The rest of the above conditions are inherited in an x-linked recessive pattern.

301
Ichthyosis with confetti is a severe, sporadic ichthyosis caused by mutations in which gene?
A. Keratin 1
B. Keratin 2e
C. Keratin 10
D. Keratin 14
E. Keratin 15
►C

Ichthyosis with confetti, a severe, sporadic skin disease in humans, is caused by mutations in the
gene encoding keratin 10 (KRT10); all result in frameshifts into the same alternative reading
frame, producing an arginine-rich C-terminal peptide that redirects keratin 10 from the
cytokeratin filament network to the nucleolus.

302

252
This syndrome has a characteristic sensorineural hearing loss and also pili torti that makes the
hair lusterless. This patient most likely has:
A. Bjornstad syndrome
B. Argininosuccinic aciduria
C. Monilethrix
D. Uncombable hair syndrome
E. Hypohidrotic ectodermal dysplasia
►A

Bjornstad syndrome has a characteristic sensorineural deafness that is bilateral and also flattened
twisted hairs called pili torti that causes the hair to look lusterless. It can be autosomal dominant
and cases of autosomal recessive conditions have been reported

303
Homocystinuria is characterized by arterial and venous thromboses and osteoporosis and mental
retardation. It is also characterized by a downward displacement of the lens. It is also due to a
deficiency of:
A. Cystathionine beta synthase
B. Pyridoxine folate
C. Homocysteine
D. Methionine
E. Elastolysis
►A

Patients with homocystinuria have elevated serum homocysteine and methionine levels due to a
deficiency of cystathionine beta synthase.

304
A patient with hypohydrosis and hyperpyrexia, anodontia, and sparse hair has which syndrome:
A. Dyskeratosis congenita
B. Pachyonychia congenita
C. Anhidrotic ectodermal dysplasia
D. Hidrotic ectodermal dysplasia
E. Papillon Lefevre
►C

Anhidrotic ectodermal dysplasia is an x-linked recessive disorder which presents with the triad
of hypohydrosis (or anhidrosis) with hyperpyrexia, anodontia (other dental findings include peg
teeth, molars with hooked cusps) and sparse hair. Patients tend to overheat.

253
305
A patient with multiple facial trichilemmomas is at risk of which of the following cancers?
A. Basal cell carcinoma
B. Breast carcinoma
C. Acute leukemia
D. Cylindroma
E. Oral squamous cell carcinoma
►B

Cowden syndrome (multiple hamartoma syndrome) is an autosomal dominant disorder caused by


mutations in PTEN, a phosphatase that dephosphorylates tyrosine, serine, and threonine.
Clinically, there are numerous facial trichilemmomas, oral papillomas, acral keratotic papules,
sclerotic fibromas, breast fibroadenomas and adenocarcinomas, thyroid adenomas and
adenocarcinomas, and hamartomatous polyps of the gastrointestinal tract.

306
The most common cutaneous neoplasm associated with Muir Torre Syndrome is:
A. Sebaceous carcinoma
B. Sebaceous adenoma
C. Keratoacanthoma
D. Basal cell carcinoma with sebaceous differentiation
E. Squamous cell carcinomas
►B

Muir-Torre syndrome is an autosomal dominant disorder due to a DNA mismatch repair gene
(MSH2). Patients present with sebaceous tumors, with adenomas being the most common
(sebaceous carcinomas next common) and associated visceral malignancies such as colon cancer
(most common).

307
In chronic granulomatous disease, the diagnosis is made by which of the following tests?
A. Assay for fumarate hydratase
B. Nitroblue tetrazolium reduction assay
C. Assay for sphingomyelinase
D. Skin biopsy
E. Assay for glucocerebrosidase
►B

Chronic granulomatous disease is diagnosed by the nitroblue tetrazolium reduction assay. The
abnormal white blood cells cannot reduce dye due to the inability to produce the respiratory

254
burst. This is needed to kill catalase positive organisms after phagocytosis. Fumarate hydratase is
defective in familial multiple cutaneous leiomyomatosis, sphingomyelinase in Niemann-Pick
disease and glucocerebrosidase in Gaucher disease.

308
Meleda is an island off the coast of Croatia. Its inhabitants have an increased frequency of a
malodorous transgradiens palmoplantar keratoderma in a stocking-glove distribution. This
condition is autosomal recessive with a defect in:
A. SLURP1 gene
B. Keratin 1/9
C. TOC gene
D. Loricrin
E. Cathepsin C
►A

SLURP1 (Secreted LY6/UPAR-related protein 1) is defective in Mal de Meleda. Keratin 1/9 are
defective in Unna-Vorner/Thost palmoplantar keratoderma, an AD diffuse symmetric non-
transgradiens PPK. TOC gene is defective in Howel-Evans syndrome, an AD PPK associated
with focal, pressure-related, non-transgradiens PPK and esophageal cancer/oral leukoplakia.
Loricrin mutations are seen in Vohwinkel syndrome variant and symmetric progressive
erythrokeratodermia. Cathepsin C defects are seen in Haim-Munk syndrome
(PPK+periodontitis+acroosteolysis+onychogryphosis) and Papillon-Lefevre syndrome (sharply
demarcated transgradiens, stocking-glove PPK+periodontitis+dural calcifications and choroids
attachments).

309
Beare-Stevenson cutis gyrata syndrome is linked with mutations in:
A. Fibroblast growth factor receptor 2
B. BSCL2
C. LMNA
D. ATP7A
E. None of these answers are correct
►A

Beare-Stevenson cutis gyrata syndrome has been linked to mutations in fibroblast growth factor
receptor 2. This syndrome is characterized by: craniosynostosis, cutis gyrata, acanthosis
nigricans, anogenital anomalies, skin tags, prominent umbilical stump, furrowed palms and soles.
Apert syndrome is also linked to this mutation. BSCL2 is linked to Berardinelli-Seip congenital
lipodystrophy, LMNA to Familial partial lipodystrophy and ATP7A to Menkes kinky hair
syndrome.

255
310
The most common cutaneous association with monilethrix is:
A. Eczema
B. Hypopigmentation
C. Hyperpigmentation
D. Keratosis Pilaris
E. Atrophy
►D

Monilethrix is an autosomal dominant condition which, by definition, presents with ―beaded‖•


hear. Clinically, patients present with short, sparse lusterless hair. Keratosis pilaris is the most
common associated feature.

311
Menke‖s kinky hair syndrome is caused by a defect in:
A. Mitochondrial gene
B. DNA helicase
C. Gap junction protein
D. Copper Transporting ATPase
E. Proto-oncogene
►D

Menke‖s kinky hair syndrome is an x-linked recessive disorder caused by a mutation at Xq12
leading to defective intestinal copper transport.

312
A patient with colon cancer is diagnosed with Muir-Torre syndrome. Which of the following
cutaneous lesions might the patient have?
A. Keratoacanthomas
B. Seborrheic keratoses
C. Tricholemmomas
D. Arsenical keratoses
E. clear cell acanthomas
►A

Muir-Torre syndrome is an autosomal dominant disease caused by mutations in MSH2 and


MLH1, DNA mismatch repair genes. Clinically, patients have multiple sebaceous tumors
(adenomas are most common), keratoacanthomas, and are at risk for adenocarcinoma of the
colon.

256
313
Medulloblastomas are seen in which of the following syndromes?
A. Bazex syndrome
B. Familial cylindromatosis
C. Nicolau-Balus syndrome
D. Birt-Hogg-Dube syndrome
E. Nevoid basal cell carcinomas syndrome
►E

Nevoid basal cell carcinomas syndrome (Gorlin syndrome) is an autosomal dominant condition
caused by mutations in the patched gene, which is involved in the hedgehog signaling pathway.
Patients develop innumerable BCCs, palmoplantar pits, painful odontogenic jaw keratocysts,
frontal bossing, bifid ribs, and calcification of the falx cerebri. Medulloblastomas and ovarian
fibromas and fibrosarcomas are associated with this condition.

314
Cowden's disease is associated with this kind of malignancy:
A. Follicular thyroid
B. Renal cell carcinoma
C. Squamous cell carcinoma
D. Leukemia
E. Basal cell carcinoma
►A

Cowden's disease is an autosomal dominant disorder is a mutation in the PTEN gene. It is


associated with breast cancer, follicular thyroid, and colon hamartoma.

315
A 3 year-old boy presents with the findings seen in the image. He also has thrombocytopenia
with purpura and a history of recurrent pyogenic bacterial infections. What is the most likely
diagnosis in this child?
A. Chronic Granulomatous disease
B. Wiskott-Aldrich syndrome
C. Hyper-IgE syndrome
D. Severe combined immunodeficiency
E. Leiner syndrome
►B

257
The findings described are consistent with Wiskott-Aldrich syndrome. The characteristic triad
can be simplified to the 3 P's - Pruritus (atopic dermatitis), Purpura (thrombocytopenia leading to
purpura and other bleeding) and pyogenic infections. The remaining options are related to
Wiskott-Aldrich in that they all have immunodeficiency as a feature, but not the same spectrum
of disease.

316
A patient with pseudoherpetic keratitis and a painful PPK would improve with a diet low in:
A. Tyrosine/Phenylalanine
B. Glycine
C. Cytosine
D. Biotin
E. Zinc
►A

Pseudoherpetic keratitis and a painful PPK describes a patient with Richner-Hanhart syndrome
(tyrosenemia type II). Treatment is with a diet low in tyrosine and phenylalanine. A diet low in
glycine, cytosine, biotin or zinc would not be helpful in this syndrome.

317
Regarding the inheritance of Ehlers-Danlos syndrome, which subtype is inherited in an X-linked
recessive manner?
A. The type with the lysyl oxidase deficiency
B. The type with the lysyl hydroxylase deficiency
C. The type with a collagen 5 defect
D. The type with a collagen 3 defect
E. The type with a defect in procollagen aminopeptidase
►A

Lysyl oxidase deficiency is related to type V or X-linked Ehlers-Danlos syndrome (EDS). Type
IX, also has x-linked recessive inheritance with mild symptoms of EDS with occipital exostoses
and hernias. Lysyl hydroxylase deficiency is seen in type VI EDS, linked with severe
kyphoscoliosis, retinal detachment and other eye abnormalities. Collagen V deficiencies are seen
in type I (Gravis) and type II (Mitis) EDS, associated with skin elasticity, gaping wounds,
hypermobile joints, Gorlins sign, blue sclera and mitral valve prolapse.

318
Lamellar ichthyosis is caused by a defect in transglutaminase 1. It can present as a colloidion
baby at birth with subsequent large thick plates of scale especially on flexures, ectropion and

258
eclabium. If two unaffected carrier parents have a child, how likely is their child to have this
condition?
A. 25%
B. 75%
C. Only male offspring are affected
D. Only female offspring are affected
E. None of these answers are correct
►A

Lamellar ichthyosis is an autosomal recessive condition. If each parent is a heterozygous carrier,


there is a 25% chance that the child will be affected, 50% chance that the child will be a
heterozygous carrier and a 25% chance that the child will not be a carrier or affected with
lamellar ichthyosis.

319
A patient with multiple deeply pigmented papules has a skin biopsy which reveals an epitheloid
blue nevus. The next appropriate step is:
A. Reassure the patient and follow up as needed
B. Schedule prophylactic excision of the lesion
C. Begin a malignancy work-up
D. Refer to genetics
E. Order an echocardiogram
►E

Epithelioid blue nevi have been reported with and without association with cardiac myxomas as
a component of the Carney complex (NAME/LAMB syndrome). Carney complex is an
autosomal dominant disorder caused by mutations in PRKAR1A. Patients have cutaneous and
atrial myxomas, blue nevi, ephelides, adrenocortical disease, and testicular tumors.

320
A patient has sebaceous neoplasms with multiple keratoacanthomata and is diagnosed with Muir-
Torr Syndrome. The most common malignancy is:
A. Colon cancer
B. Thyroid cancer
C. Breast cancer
D. Laryngeal cancer
E. Uterine cancer
►A

259
Patient with Muir-Torre Syndrome have a high incidence of colon cancer. It is an autosomal
dominant disorder caused by the mutation in either MSH2 or MLH1 gene. The other malignancy
can be seen with this syndrome but colon is the most common.

321
Which gene is most commonly mutated in melanoma on non sun-damaged skin?
A. BRAF
B. NRAS
C. C-kit
D. CTLA-4
E. CDKN2A
►A

Regarding the genetics of melanoma, BRAF mutations are more common than NRAS mutations
on non sun-damaged skin. C-kit is associated with mucosal melanoma. Acral melanoma are
associated with c-kit more commonly than with BRAF.

322
What is the most common genetic defect associated with this syndrome?
A. Neurofibromin
B. Merlin
C. Tuberin
D. Hamartin
E. Folliculin
►A

Neurofibromatosis I occur due to a microdeletion at 17q11.2 involving the NF1 gene, which
encodes for neurofibromin. It is an autosomal dominant disorder characterized by numerous
benign tumors (neurofibromas) of the peripheral nervous system, cafe au lait macules, freckling
in the area of the armpit (crow‖s sign), two or more growths on the iris of the eye (known as
Lisch nodules or iris hamartomas), tumor on the optic nerve (optic glioma), abnormal
development of the spine (scoliosis), the temple (sphenoid) bone of the skull, or the tibia (one of
the long bones of the shin) and a first degree relative (parent, sibling, or child) with NF1. The
other proteins in the list are associated with other syndromes: In neurofibromatosis type 2, a NF2
gene mutation has been identified which encodes for a protein called Merlin, in tuberous
sclerosis two genetic mutations have been identified on two separate chromosomes namely
tuberin and hamartin, and a Folliculin mutation is seen in Birt Hogg Dube syndrome.

323
Ataxia- Telangiectasia

260
A. Is associated with significantly increased risk of ovarian cancer in carriers
B. Telangiectasias present in late adulthood
C. Carriers do not have an increased risk of malignancy
D. Is inherited in XLR (X-linked recessive)
E. First clinical sign is neurologic
►E

First sign is ataxia. Telangiectasias occur later in puberty. Carriers show an increased risk of
breast cancer.

324
Patients with Hermansky-Pudlak syndrome may experience which of the following systemic
complications as a result of their disease?
A. Arteriovenous malformations
B. Pulmomary fibrosis
C. Gastroesophageal reflux disease
D. Aortic stenosis
E. Rectal abscesses
►B

Patients with Hermansky-Pudlak syndrome are tyrosinase positive albinos. In addition to their
pigmentary dilution and increased risks for cutaneous malignancies, patients lack platelet
granules leading to impaired platelet aggregation (and therefore a bleeding diathesis). There is
also a lysosomal membrane defect which leads to the accumulation of ceroid lipofuscion in
macrophages within the lung (leading to pulmonary fibrosis), gastrointestinal tract (leading to
granulomatous colitis), and heart (cardiomyopathy).

325
Hypoplasia of the breast can be seen in which disease?
A. Anhidrotic ectodermal dysplasia
B. Maffucci syndrome
C. Congenital syphilis
D. Marfan syndrome
E. Osteogenesis imperfecta
►A

Anhidrotic ectodermal dysplasia is a X-linked recessive disease caused by mutations in


ectodysplasin, a member of the tumor necrosis family. Patients may have dry skin with
pigmentation periorbitally, hypohidrosis, sparse hair, hypo-anodontia, nail dystrophy, and frontal

261
bossing, and saddle nose deformity. In addition to abnormalities of other ectodermally derived
structures, the breast and nipple-areolar complex may be absent or hypoplastic.

326
Which of the following disorders is associated with delayed separation of the umbilical cord?
A. Leukocyte adhesion deficiency type 1 (LAD-1)
B. Immunedysregulation, polyendocrinopathy, enteropathy, x-linked (IPEX)
C. Severe combined immunodeficiency disorder(SCID)
D. X-linked agammaglobulinemia
E. Myeloperoxidase deficiency
►A

LAD-1 manifests as a B2-integrin deficiency and often times presents as at birth with a delayed
umbilical cord separation.

327
Eyelid string of pearls are seen in which of the following conditions?
A. Focal dermal hypoplasia
B. Lipoid proteinosis
C. Hutchinson-Gilford syndrome
D. Beare-Stevenson cutis gyrata syndrome
E. Gaucher‘s disease
►B

Lipoid proteinosis is an autosomal recessive condition characterized by yellow papules on the


face and oropharynx, eyelid string of pearls, hoarse voice, verrucous nodules of elbows and
knees, and bean-shaped temporal and hippocampal calcification with occasional seizures.
Histologically, there are PAS+ deposits in the affected tissue.

328
Which of the following conditions is inherited in an X-linked dominant (XD) manner?
A. Chodrodysplasia punctata
B. CHILD Syndrome
C. Focal dermal hypoplasia
D. All of the answers are correct
E. None of the answers are correct
►D

All of the syndromes listed are XD. Other XD syndromes are: Incontinentia pigmenti and Bazex
syndrome.

262
329
Nevoid basal cell syndrome is in inherited condition with basal cell carcinoma. Patients with this
syndrome have dental findings of:
A. Odontogenic keratocysts
B. Destructive periodontitis
C. Hypodontia
D. Gingival fibromas
E. Retention of primary teeth
►A

Nevoid basal cell syndrome is an inherited condition with a defect in the PTCH gene. The dental
findings are odontogenic keratocysts.

330
Which malignancy is associated with Cowden syndrome?
A. Colon cancer
B. Rhabdomyosarcoma
C. Lung cancer
D. Ovarian cancer
E. Basal cell cancer
►A

Cowden syndrome is a autosomal dominant syndome with tricholemmomas, oral mucosal


papules, acral keratotic papules, thyroid goiter, GI polyps, and fibrocystic breast disease.
Malignant associations include breast, thyroid follicular carcinoma, and colon adenocarcinoma--
malignant transformation of the polyps. They are also at increased risk for renal cell carcinoma
and endometiral carcinoma

331
A patient that has clinical signs of xeroderma pigmentosum with a history of sun sensitivity and
skin cancers plus mental retardation, dwarfism, and gonadal hypoplasia is known as:
A. De-Sanctis Cachione syndrome
B. Cockayne syndrome
C. Tay's syndrome Louis-Barr syndrome
D. Xeroderma pigmentosum plus syndrome
E. Bloch Sulzberger syndrome
►A

263
A patient that has clinical signs of xeroderma pigmentosum with mental retardation, dwarfism
and gonadal hypoplasia is known as De-Santis-Cachione syndrome. Those that have xeroderma
pigmentosum have skin cancer at a young age and is from mutations in DNA excision repair
enzymes involved in UV induced thymidine dimers.

332
Which vascular disorder is characterized by facial vascular malformation and ipsilateral
intracranial and retinal arteriovenous malformations(AVMs)?
A. Sturge-Weber syndome
B. PHACES
C. Encephalotrigeminal angiomatosis
D. Bonnet Dechaune Blanc syndrome
E. Von Lohuizen‖s disease
►D

Bonnet Bechaune Blanc syndome, also know as Wyburn-Mason syndrome, is characterized by a


facial vascular malformation and ipsilateral intracranial and retinal AVMs. Encephalotrigeminal
angiomatosis is another name for Sturge-Weber. Von Lohuizen's disease is another name for
cutis marmorata telangiectatica congenita.

333
Which of the following syndromes is associated with tricholemmomas?
A. Birt-Hogg-Dube
B. Brooke-Spiegler
C. Bannayan-Riley-Ruvacalba
D. Basex
E. Rasmussen's
►C

Tricholemmomas are seen in Bannayan-Riley-Ruvacalba syndrome. This is an autosomal


dominant condition with macrocephaly, lipomas, hemangiomas, skeletal abnormalities,
lymphangioma circupscriptum, angiokeratomas, penile lentigines, acanthosis nigricans, and
achrocrodons. There is an increased incidence of breast, thyroid, and GI cancers.
Tricholemmomas are also associated in Cowden's syndrome.

334
In a patient suspected of having multiple endocrine neoplasia type IIb, which lab test would be
appropriate?
A. Calcitonin
B. Glucagon

264
C. Parathyroid hormone
D. Calcium
E. Cortisol
►A

Multiple endocrine neoplasia (MEN) syndrome type Iib, also called multiple mucosal neuroma
syndrome is an autosomal dominant condition due to a defect in the RET protooncogene on
chromosome 10q11.2. This rare condition is associated with mucosal neuromas on the tongue
and lips, medullary thyroid carcinoma, pheochromocytoma, and gastrointestinal
ganglioneuromatosis. In addition, patients can also present with a marfanoid habitus and facial
dysmorphism. Mucosal neuromas can be a dermatologic clue to the underlying diagnosis as these
lesions appear during early childhood and present as pink, pedunculated nodules. As the major
cause of mortality in these patients is medullary thyroid cancer, which nearly all patients will
have by early adulthood, aggressive screening, with serial calcitonin level, and prophylactic
thyroidectomy are warranted.

335
Erythematous keratotic plaques of KID Syndrome most commonly occur in which location?
A. Face
B. Neck
C. Chest
D. Back
E. Abdomen
►A

The erythematous keratotic plaques of KID Syndome occur on face, extremities > trunk. KID
Syndrome is characterized by keratitis, icthyosis, and deafness. It is inherited in an autosomal
dominant fashion and caused by a mutation in the GJB2 gene which encodes connexin 26.

336
Which of the following is not classically associated with pheochromoctyoma?
A. Multiple Endocrine Neoplasia Type IIA
B. Multiple Endocrine Neoplasia Type IIB
C. Von-Hippel-Lindau Syndrome
D. Cobb Syndrome
E. Neurofibromatosis
►D

Cobb syndrome is a sporadic disease characterized by cutaneous vascular malformations


associated with malformations of the spinal cord.

265
337
A 12 year-old boy with pits on his palms and lateral fingers may have:
A. Arsenic exposure
B. A hereditary keratoderma
C. A corynebacteria infection
D. An inherited cancer syndrome
E. Secondary syphilis
►D

Basal cell nevus syndrome is an autosomal dominant disease caused by mutations in the PTCH1
gene. Clinically, patients may have numerous basal cell carcinomas, palmoplantar pits, jaw cysts,
frontal bossing, bifid ribs, calcification of falx cerebri, medulloblastoma, ovarian fibromas and
fibrosarcomas.

338
Cronkhite-Canada is associated with gastrointestinal polyposis, nail atrophy, alopecia,
generalized pigmentation of skin and melanotic macules on the fingers. Which of the following
describes its inheritance pattern?
A. Autosomal recessive
B. Autosomal dominant
C. X-linked recessive
D. X-linked dominant
E. Sporadic
►E

Cronkhite-Canada is associated with gastrointestinal polyposis, nail atrophy, alopecia,


generalized pigmentation of skin and melanotic macules on the fingers. It is inherited in a
sporadic manner.

339
Patients with junctional epidermolysis bullosa have been found to have mutations in:
A. Laminin 5
B. Bullous pemphigoid antigen 2
C. Collagen 17
D. BP180
E. All of the answers are correct
►E

266
All of the answers are correct. Laminin 5 is a protein integral in the adhesion of the dermis to the
epidermis. Also involved in junctional epidermolysis bullosa is bullous pemphigoid antigen 2,
collagen 17 and BP180, which are synonymous for the same structure.

340
Which syndrome is characterized by hyperhidrosis, lack of pain sensation, hypersalivation, and
absent fungiform papillae?
A. Turner Syndrome
B. Noonan Syndrome
C. Riley-Day
D. Rubinstein-Taybi syndrome
E. Cornelia de lange Syndrome
►C

Riley-Day syndrome is also known as Familial Dysautonomia. It is an autosomal recessive


disorder with the gene defect on the long arm of chromosome 9. Patients have unmyelinated
sensory and sympathetic neurons and autonomic dysfunction, leading to hyperhidrosis, decreased
corneal sensation and tear flow, hypersalivation, gastroesophageal reflux, decreased deep tendon
reflexes, and lack of pain sensation. They also exhibit abnormal histamine skin test.

341
Low-cystine content in hair and nails may contribute to the phenotype seen in:
A. Wilson‘s disease
B. Menke‘s Kinky Hair syndrome
C. Tay Syndrome
D. Nethertons
E. Bjornstad
►C

Tay Syndrome is also known as trichothiodystrophy, or (P)IBIDS: (photosensitivity), icthyosis,


brittle hair, intellectual impairment, decreased fertility, and short stature. Hair shaft has a
characteristic ―tiger tail‖ appearance under polarized light and the low cystine content in hair and
nails is thought to be responsible for the phenotype seen.

342
―Coast of Maine‖ cafe au lait macules are characteristic of which condition
A. Hypomelanosis of Ito
B. Carney complex
C. McCune-Albright syndrome
D. Gaucher‖s syndrome

267
E. Tuberous sclerosis
►C

McCune-Albright syndrome is a sporadic condition caused by somatic mutations in the Gs


subunit of adenylate cyclase. Key clinical features include ―coast of Maine‖ cafe au lait macules,
polyostotic fibrous dysplasia, and precocious puberty.

343
The best screening test for hemochromatosis is:
A. Ferritin
B. Hematocrit
C. Complete Blood Count
D. Transferrin
E. Copper levels
►A

The ferritin is the best screening test for hemochromatosis, an autosomal recessive disease of
increased intestinal iron absorption. The other tests are not the best method for screening.

344
A patient has Cornelia de Lange syndrome have a low set hair line, trichomegaly, hirsutism and
synophrys. The skin finding is:
A. Cutis marmorata
B. Vitiligo
C. Poikiloderma
D. Scleroderma
E. Hyperpigmentation
►A

The patient has cutis marmorata with low set hair line, trichomegaly, synophrys, hirsutism, and
heart defect. The associated gene is NIPBL and SMC1L1.

345
Which of the following syndromes is associated with markedly increased IgE levels, cold
abscesses and a characteristic coarse facies?
A. Wiskott-Aldrich syndrome
B. Chronic granulomatous disease
C. Job syndrome
D. Severe combined immunodeficiency
E. Leiner's disease

268
►C

Job syndrome or Hyper IgE syndrome is characterized by these findings. In addition, there is a
peripheral eosinophilia, eczematous dermatitis, frequent bronchitis and pneumonia, otitis media
and sinusitis. The other listed conditions are associated with immunodeficiency. Wiskott-Aldrich
can have eczematous dermatitis and all of these syndromes will have abnormal infections. They
do not have markedly increased levels of IgE like Job syndrome.

346
A 16 year-old girl presents with a family history of Gardner syndrome. Her mother is very
concerned that her daughter may have the syndrome as it runs in her family and she has many
skin complaints. What is likelihood that this girl has Gardner syndrome based on what you now
about the inheritance pattern and the fact that her father is unaffected and her mother is a
heterozygote for this condition?
A. 50%
B. 25%
C. 10%
D. 75%
E. 90%
►A

Her mother is a heterozygote for the condition and her father is homozygous as a non-carrier.
Gardner syndrome is passed in an autosomal dominant fashion. She has a 50% chance of also
being a heterozygote from her mother and a 50% chance of being completely unaffected.

347
An infant with a very hoarse cry presents to dermatology. His parents have noted that normal
scratches are slow to heal and leave broad scars. White and yellow lesions have been noted on
the lips, eyelids, and buccal mucosa. The child has had frequent upper respiratory tract
infections. What defective gene is associated with the most likely implicated disease?
A. Extracellular matrix protein-1
B. Collagen III
C. Elastin
D. Tenascin-X
E. Fibrillin-1
►A

The child described most likely has lipoid proteinosis, a rare inherited disease of connective
tissues that is inherited in an autosomal recessive manner. The implicated defective protein is

269
extracellular matrix protein-1. Collagen III and Tenascin-X defects are seen in Ehlers-Danlos
syndrome. Elastin defects are seen in cutis laxa. Fibrillin-1 defects are seen in Marfan disease.

348
Which disease is found more commonly in mothers of patients with chronic granulomatous
disease?
A. Sarcoidosis
B. Erythema nodosum
C. Churg-Straus disease
D. Wegener's disease
E. Discoid lupus erythematous
►E

Female carriers of chronic granulomatous disease have an increase incidence of discoid lupus,
infections and apthous stomatitis.

349
Menkes kinky hair syndrome is associated most commonly with which of the following hair
abnormalities?
A. Pili torti
B. Trichorrhexis nodosa
C. Trichorrhexis invaginata
D. Pili triangulati et canaliculati
E. None of these options are correct
►A

While pili torti is not exclusively found in Menkes kinky hair syndrome, this is the most
common hair abnormality found. Trichorrhexis nodosa can be also seen. Trichorrhexis
invaginata is commonly found in Netherton syndrome. Pili triangulati et canaliculati is the
finding seen in Uncombable hair syndrome.

350
This syndrome has glaucomas, choroid angiomas, anisometropic amblyopia with facial capillary
malformation and skeletal hypertrophy:
A. Sturge Weber syndrome
B. Osler Weber Rendu
C. Vogt-Koyanagi Harada syndrome
D. CHIME syndrome
E. Kid syndrome
►A

270
Sturge-weber syndrome has defects in facial capillary malformation with underlying soft tissue
and skeletal hypertorphy, ipsilateral arteriovenous malformation (AV), cerebral calcification,
hemiparesis, hemianopia, and contralateral seizures.

351
Findings of milia, cylindromas and the condition shown in the pathology image are characteristic
of which of the following syndromes?
A. Gorlin's syndrome
B. Familial cylindromatosis
C. Brook-Spiegler syndrome
D. Rasmusen syndrome
E. Rombo syndrome
►C

Brooke-Spiegler syndrome is an uncommon disease with a predisposition to develop cutaneous


adnexal neoplasms such as cylindromas, trichoepitheliomas, spiradenomas, trichoblastomas,
basal-cell carcinomas, follicular cysts, organoid nevi, and malignant transformation of pre-
existing tumors in the affected individuals.

352
Which malignancy is seen in approximately 15-20% of people with the disease characterized by
a defect in isocitrate dehydrogenase?
A. Osteosarcoma
B. Angiosarcoma
C. Chondrosarcoma
D. Rhabdomyosarcoma
E. Epitheliod sarcoma
►C

Approximately 15-20% of patients with Maffucci's syndrome and Oillier's will develop
chondrosarcoma. They are due to a defect in a isocitrate dehydrogenase 1 or 2.

353
A patient with Ref sum syndrome has ocular anomalies such as nystagmus and night blindness
has a deficiency in:
A. Phytanic acid oxidase deficiency
B. Abnormal transglutaminase
C. Abnormal steroid sulfatase
D. Fatty alcohol oxidoreductas deficiency

271
E. Defects in endoglin
►A

Patients with Refusum syndrome have a deficiency in phytanic acid oxidase deficiency. The
ocular abnormalities seen are pigmentary retinopathy, cataracts, nystagmus, and night blindness

354
A patient has many reticulated, yellow patches in the skin folds and lax skin. The associated eye
abnormality with this disease is:
A. Angioid streaks
B. Papillary edema
C. Conglobata
D. Dyschromia
E. Loss of peripheral vision
►A

The patient has pseudoxanthoma elasticum which is due to a mutation in an adenosine


triphosphate binding transporter protein. The eye abnormality seen is angioid streaks on the
retina, hemorrhages and loss of vision. They also have claudicaton and gastrointestinal
hemorrhage.

355
A deficiency in sialophorin, a surface glycoprotein, is thought to play a role in which genetic
disorder?
A. Wiskott-aldrich syndrome
B. Chediak-higashi syndrome
C. Werner syndrome
D. Bloom Syndrome
E. Cornelia de Lange syndrome
►A

Wiskott-Aldrich syndrome is an x-liked recessive disorder localized to Xp11.3. The gene


involved codes for WAS, and the protein product has been implicated in lymphocyte and
megakaryocyte signal transduction.

356
What is the most likely syndrome that this woman has in this photograph?
A. A. Klippel Trenaunay Weber
B. Filariasis
C. Neurofibromatosis

272
D. Proteus syndrome
E. Turner syndrome
►A

Klippel-Trenaunay Weber syndrome is characterized by a triad of port-wine stain, varicose


veins, and bony and soft tissue hypertrophy involving an extremity. Filariasis is an acquired
infection that leads to obstruction and scarring of the lymphatics. Neurofibromatosis is
associated with neurofibromas, cafe-au-lait macules and Lisch nodules, but not with
lymphedema and vascular malformations. Proteus syndrome is associated with vascular
abnormalities, lipomas and asymmetric limb hyperplasia, and plantar connective tissue nevi, but
usually not lymphedema. Lymphedema of the dorsal hands and feet can be seen in Turner
syndrome, but typically not this extensive.

357
Reticulate pigmentation of skin, poikiloderma, alopecia, nail atrophy, premalignant oral
leukoplakia, and a Fanconi-type pancytopenia resulting in early death in addition to posterior
fossa malformations is characteristic of which of the following syndromes.
A. Dyskeratosis congenita
B. Hoyeraal-Hreidarsson syndrome
C. Bloom syndrome
D. Cockayne syndrome
E. Wiskott-Aldrich syndrome
►B

Hoyeraal-Hreidarsson syndrome is has all of the features of dyskeratosis congenita plus posterior
fossa malformations. Bloom syndrome and Cockayne syndrome both have poikiloderma as
features, but do not include posterior fossa malformations as part of the syndrome. Wiskott-
Aldrich syndrome does not include any of these findings.

358
A patient presents with bilateral retinal hemangioblastomas and a capillary malformation on her
neck. Which gene mutation is most likely?
A. VHL
B. Endoglin
C. PTH/PTHrP type I receptor
D. VEGF receptor-3
E. MFH1
►A

273
These findings are most characteristic of Von Hippel-Lindau syndrome. VHL is a tumor
suppressor gene which is mutated in this syndrome. Other findings include: renal and pancreatic
cysts/carcinoma, pheochromocytoma, and cerebellar/other CNS hemangioblastomas. Endoglin is
defective in Osler-Weber-Rendu, PTH/PTHrP type I receptor in Maffucci syndrome, VEGF
receptor-3 in Nonne-Milroy disease (hereditary lymphedema) and MFH1 in lymphedema and
ptosis syndrome.

359
Sphenoid wing dysplasia is seen in:
A. Tuberous sclerosis
B. Tay Syndrome
C. Mafucci syndrome
D. NF-1
E. NF-2
►D

Sphenoid wing dysplasia is seen in neurofibromatosis type I. Patients with Tay syndrome have
short stature, patients with tuberous sclerosis have phalangeal cysts and periosteal thickening,
patients with Mafucci syndrome have enchondromas and short stature, and patients with NF-2 do
not have any characteristic musculoskeletal findings.

360
Dermatofibrosis lenticularis disseminata and osteopoikilosis are findings seen with mutations of
which of the following genes?
A. LEMD3
B. Fibrillin 2
C. ABCC6
D. Lysyl hydroxylase
E. Lysyl oxidase
►A

Buschke-Ollendorf syndrome is caused by a loss-of-function mutation in LEMD3.

361
Which of the following findings is characteristic of a mutation in lamin A?
A. Lipoatrophic sclerodermoid skin
B. Alopecia
C. Craniomegaly with small face
D. Severe premature atherosclerosis with early death
E. All of the answers are correct

274
►E

A mutation in Lamin A causes Progeria (Hutchinson-Gilford syndrome). Other findings include


nail atrophy and muscle/bone wasting. Presentation is in the first or second year of life. An
increased urine hyaluronic acid can be helpful in diagnosis.

362
What condition is associated with a mutation in CXCR4?
A. Ataxia telangiectasia
B. Leukocyte adhesion deficiency
C. Chronic granulomatous disease
D. SCID
E. WHIM syndrome
►E

Ataxia telangiectasia is associated with mutations in ATM gene. Leukocyte adhesion deficiency
is associated with mutations in the common chain of CD18 which is a beta-2 integrin family
member, FUCT1, or KINDLIN3. Chronic granulomatous disease is due to mutations in the
NADPH oxidase. SCID is a heterogenous group of genetic disorders due to mutations in Jak-3,
IL-7Ralpha, CD45, CD3delta/CD3episilon, RAG1/RAG2, and Artemis (DCLREC1C). WHIM
syndrome (warts, hypogammaglobulinemia, infections, myelokathexis) is due to a gain-of-
function mutation in CXCR4 which results in the retention of neutrophils in the bone marrow. It
is inherited in an autosomal-dominant fashion.

363
Premalignant leukoplakia of the oral mucosa is associated with:
A. Bloom syndrome
B. Werner Syndrome
C. Xeroderma Pigmentosum
D. Dyskeratosis Congenita
E. Rothmund-Thomson syndrome
►D

Dyskeratosis Congenita (also known as Zinsser-Engman-Cole syndrome) is thought to have two


modes of inheritance. The more common X-linked disorder is due to a mutation in the Dyskerin
gene, while the autosomal dominant form is due to a mutation in TERC, a telomerase RNA
component. Clinical features include reticulated gray-brown hyperpigmentation, paloplantar
hyperkeratosis, alopecia, onychodystrophy, premalignant leukoplakia of any mucosal surface,
and mental retardation.

275
364
What nail change is seen in patients with Mal de Meleda Syndrome?
A. Onycholysis
B. Longitudinal ridging
C. Koilonychia
D. leukonychia
E. pterygium
►C

Mal de Meleda is an autosomal recessive disease characterized by transgedient malodorous PPK,


hyperhidrosis, keratotic plaques at knees and elbows, subungual hyperkeratosis, and koilonychia.
The gene defect is SLURP 1.

365
A patient has a white forelock, depigmented patches on the central face, mid-extremities,
deafness and heterochromia irides. The patient has:
A. Piebaldism
B. Klippel Feil
C. Cornelia de Lange syndrome
D. Noonan syndrome
E. Turner syndrome
►A

A patient has Piebaldism with a mutation in c-KIT protooncogene with a white forelock. The
patient has depigmented patches on the central faces, mid-extremities, and abdomen. The patient
can have Hirschsprung, deafness and heterochromia irides.

366
On cutaneous exam, angiokeratoma corporis diffusum is characteristic of which of the following
conditions?
A. Sialodosis
B. Fucosidosis
C. Fabry disease
D. All of these options are correct
E. None of these options are correct
►D

Findings of angiokeratoma corporis diffusum are found in all three listed conditions. They
cannot by distinguished by skin exam.

276
367
A BSCL2 gene mutation with the cutaneous findings of generalized lipodystrophy, hyperlipemia,
hepatomegaly, acanthosis nigricans, elevated basal metabolic rate and non-ketotic insulin
resistant diabetes mellitus are characteristic of which of the following syndromes?
A. Berardinelli-Seip congenital lipodystrophy
B. Familial partial lipodystrophy
C. Bjornstad syndrome
D. All of the answers are correct
E. None of the answers are correct
►A

Berardinelli-Seip congenital lipodystrophy is described above. Familial partial lipodystrophy is


characterized by a defect in LMNA and has symmetric lipoatrophy of trunk and limbs with
sparing of neck, shoulders, buffalo hump area and genitalia, tuboeruptive xanthomas, acanthosis
nigricans and hypertriglyceridemia. Bjornstad syndrome is characterized by pili torti and
deafness.

368
What is the most likely nail findings in a patient who has this autosomal dominant disease with
these keratotic papules and cobblestoning of the oral mucosa?
A. Koilonychia
B. Red and white longitudinal bands
C. Melanonychia
D. Half and half nails
E. Pincer nails
►B

Darier's disease is an autosomal dominant disorder characterized by greasy hyperkeratotic


papules. The papules often coalesce into a warty plaque and have a tendency for secondary viral
or bacterial infection. The mutation is in calcium ATPase 2A2. The classic nail finding is red and
white longitudinal bands with V-shaped nicking.

369
Tyrosinase positive albinism (oculocutaneous albinism type 2) is caused by a mutation in which
of the following:
A. Tyrosinase
B. P gene
C. Tyrosinase related protein 1
D. C-kit
E. NEMO

277
►B

Oculocutaneous albinism (OCA) type 1 (Tyrosinase negative albinism) is caused by mutations in


the tyrosinase gene. OCA type 2 (tyrosinase positive albinism) is caused by mutations in the P
gene. OCA type 3 is caused by mutations in the tyrosinase related protein 1 gene. C-kit
mutations cause piebaldism and NEMO mutations cause incontinentia pigmenti.

370
Maffucci syndrome is has characteristic venous malformations of the distal extremities and
benign endochondromas which can compromise bone strength and lead to chondrosarcomas. The
defect causing this believed to be the IDH-1 or IDH-2 which is inherited in which manner?
A. Sporadic
B. Autosomal dominant
C. Autosomal recessive
D. X-linked dominant
E. X-linked recessive
►A

Maffucci syndrome is inherited in a sporadic manner, with mutations in isocitrate dehydrogenase


1 or 2 (IDH-1, IDH-2).

371
Which of the following genodermatoses has premalignant oral leukoplakia as a feature?
A. Pachyonychia congenita
B. Dyskeratosis congenita
C. Keratosis follicularis
D. Anhidrotic ectodermal dysplasia
E. Papillon-Lefevre
►B

Dyskeratosis congenita has premalignant oral leukoplakia in addition to cutaneous poikiloderma


and nail dystrophy. Pachyonychia congenita also has leukoplakia, but it is benign in nature.
Additional features of pachyonychia congenita are thickened nails with nail bed hyperkeratosis
and palmoplantar keratoderma. Keratosis follicularis, also known as Darier\'s disease, is
characterized by benign whitish papules on the oral mucosa, cutaneous keratotic papules in
seborrheic distribution, and longitudinal erythronychia with V-nicking of the distal nail plate.
Papillon-Lefevre has gingivitis, without oral leukoplakia, and acral hyperkeratosis. Anhidrotic
ectodermal dysplasia does not have leukoplakia. It consists of anhidrosis or hypohidrosis,
hypotrichosis, and anodontia.

278
372
What of the following is present in desmoplastic melanoma?
A. b-raf
B. c-myc
C. c-kit
D. Ras
E. p53
►C

C-kit staining is present in desmoplastic melanoma. It is also seen in acral melanocytic lesions.
Ras is found in all melanocytic lesions. P53 mutations are found in actinic keratoses, SCCIS, and
SCC.

373
Comma-shaped corneal opacities are characteristic of which type of ichthyosis?
A. Ichthyosis vulgaris
B. X-linked ichthyosis
C. Lamellar ichthyosis
D. Nonbullous congenital ichthyosiform erythroderma
E. Refsum syndrome
►B

X-linked ichthyosis patients have comma-shaped corneal opacities that are asymptomatic yet
highly characteristic.

374
"Mousy" odor of urine is characteristic of which of the following syndromes?
A. Phenylketonuria
B. Hunter disease
C. Alkaptonuria
D. Maple Syrup Urine disease
E. Hurler disease
►A

Phenylketonuria will cause a "mousy" odor in the urine. Patients with Alkaptonuria will have
black urine. Maple syrup urine disease will have a sweet odor. Hunter/Hurler diseases are not
associated with urinary issues.

375
Dental enamel pits are seen in which of the following conditions?

279
A. Hypomelanosis of Ito
B. Letterer-Siwe disease
C. Tuberous sclerosis
D. Jackson Sertoli syndrome
E. Hyper-IgE syndrome
►C

Tuberous sclerosis is an autosomal dominant condition caused by mutations of the TSC1


(hamartin) or TSC2 (tuberin) genes. These are tumor suppressor genes. Skin findings include
hypopigmented macules, connective tissue nevus, facial angiofibromas, periungual fibromas and
cafe au lait macules. Dental enamel pits and gingival fibromas are oral findings that are
associated with this condition. The remaining conditions do not have dental pits.

376
The finding of 'maltese crosses' in the urine is characteristic of which of the following
conditions?
A. Alkaptonuria
B. Fabry disease
C. Gaucher disease
D. Neimann-Pick disease
E. Hunter syndrome
►B

The 'maltese cross' finding in urine is characteristic of Fabry disease. Alkaptonuria will show
dark urine with a pH > 7.0. There are no urinary findings in Hunter syndrome, Gaucher or
Neimann-Pick disease.

377
An infant with doughy, redundant skin and short sparse hairs is likely to show which features on
x-ray?
A. Metaphyseal widening in the long bones
B. Sphenoid wing dysplasia
C. Periosteal thickening
D. Osteopoikilosis
E. Stippled epiphyses
►A

The patient described has Menkes Kinky Hair syndrome, an X-linked recessive disease due to a
defect in an intestinal copper transport protein. Clinical features include pili torti, short, brittle
―steel-wool‖• hair, and spare eyelashes and sparse broken eyebrows. The skin is often

280
hypopigmented with a soft, ―doughy‖• consistency and redundancy. Musculoskeletal
manifestations include metaphyseal widening with spurs in the long bones

378
Which cutaneous finding is seen in patients with phenylketonuria?
A. Angular stomatitis
B. Ichthyosis
C. Pigment dilution of hair and skin
D. Phyrnoderma
E. Erosive diaper dermatitis
►C

Phenylketonuria is an autosomal recessive condition caused by a mutation in the gene coding for
phenylalanine hydroxylase. Defect in this enzyme results in accumulation of phenylalanine and
its metabolites. Increased phenylalanine has toxic effects on the central nervous system in
addition to competitively inhibiting tyrosine in melanogenesis. Inhibition of melanogenesis
results in pigmentary dilution of the hair and skin. Other features of this condition include a
predisposition to eczema, sclerodermoid changes of the skin, urine that has a distinctive "mousy"
odor, psychomotor delay, mental retardation, seizures and hyperreflexia. A low-phenylalanine
diet instituted early on can prevent these manifestations of the disease. The morbidity of
phenylketonuria has improved since the advent of routine neonatal screening for this condition.

379
Epidermolysis bullosa simplex is caused by blistering in which structure?
A. Granular layer keratinocyte
B. Spinous layer keratinocyte
C. Basal layer keratinocyte
D. Lamina densa
E. Sublamina densa
►C

Epidermolysis bullosa simplex is caused by mutations in keratin 5 and 14 resulting in bullae


within basal cell keratinocytes.

380
A patient diagnosed with KID syndrome with keratitis-icthyosis-deafness has a mutation in:
A. Connexin 26
B. NEMO
C. DNA helicase
D. RECQL4

281
E. plakoglobin
►A

A patient with KID syndrome has an autosomal dominant disease and mutation in connexin 26.
They have keratitis, ichthyosis, deafness and alopecia.

381
Which type of epidermolysis bullosa simplex is associated with early death?
A. Weber-Cockayne
B. Generalized (Koebner)
C. Dowling-Maera
D. Ogna variant
E. Non-Herlitz variant
►C

The Dowling-Maera variant of epidermolysis bullosa simplex is associated with widespread


bullae, significant mucous membrane and laryngeal/esophageal involvement, nail dystrophy, and
early death.

382
A patient presents with several light blue cyst-like lesions on the eyelid. They consult their list of
problems and bring up plantar hyperkeratosis and dysplastic toenails. On oral exam, you note
that they have both upper and lower dentures. The patient relates that after losing their "baby
teeth", only 3 teeth grew in their place. What syndrome does this person most likely have?
A. Schopf-Schulz-Passarge
B. Gardner syndrome
C. Hypohidrotic ectodermal dysplasia
D. Cowden syndrome
E. Cronkhite-Canada
►A

Schopf-Schulz-Passarge syndrome is associated with hydrocystomas of the eyelids,


hypotrichosis (near complete loss of hair early in life), hypodontia, nail abnormalities and
multiple palmoplantar eccrine syringofibroadenomas. The other listed syndromes do not fit the
description above.

383
Patients with Russell-Silver syndrome exhibit:
A. Clinodactyly of fifth finger
B. Peg teeth

282
C. Osteopathia striata
D. Broad thumbs
E. Shortened 4th and 5th metacarpals
►A

Characteristic features of Russell-Silver include short stature, bony asymmetry, triangular facies,
clinodactyly of fifth finger, and precocious sexual development with
cryptochordism/hypospadias. Broad thumbs are seen in Rubinstein-Taybi and shortened fourth
and fifth metacarpals are seen in Turner syndrome. Osteopathia striata is characteristic of focal
dermal hypoplasia.

384
Which syndrome is characterized by broad thumbs, a large beaked nose, and capillary
malformation?
A. Klinefelter
B. Proteus syndrome
C. Bloom syndrome
D. Rubinstein-Taybi
E. Ehlers-Danlos syndrome
►D

Rubinstein-Taybi syndrome has been associated with a deletion localized to the short arm of
chromosome 16. Patients are severely retarded with strabismus, crytorchidism, and congenital
heart defects. They have a characteristic beaked nose with nasal septum below alae accompanied
by a broad nasal bridge, downslanting palpebral fissures, and broad thumbs and halluces.

385
Which finding would you expect in a child with dyskeratosis congenita?
A. Normal mucosa
B. Oral leukoplakia
C. Hirsutism
D. Normal hematologic status
E. Normal nails
►B

Mild to moderate mental retardation occurs in up to 50% of cases. A Fanconi-type pancytopenia


may occur resulting in early death. These children have alopecia, not hirsutism, along with nail
dystrophy. There is an increased risk for development of premalignant oral leukoplakia, as well
as many cancers.

283
386
A 12 year old boy presents complaining of pain and stinging of his skin within minutes of going
out to play in the park. Examination reveals rare milia of the hands and posterior neck. A
diagnosis of porphyria is made. What is the most likely defective gene in this patient?
A. Ferrochelatase
B. Uroporphyrinogen decarboxylase
C. Porphybilinogen deaminase
D. Uroporphyrinogen III synthase
E. Protoporphyrinogen oxidase
►A

Erythropoietic protoporphyria is a relatively common form of porphyria, characterized by


photosensitivity. Over time, superficial waxy scars and milia are identified in sun-exposed areas.
The defective gene is ferrochelatase. Uroporphyrinogen decarboxylase is mutated in porphyria
cutanea tarda. Porphybilinogen deaminase is defective in acute intermittent porphyria.
Uroporphyrinogen III synthase is defective in congenital erythropoietic porphyria.
Protoporphyrinogen oxidase is mutated in variegate porphyria.

284
Chapter -4-
General Dermatology
1
Which of the following is a feature of Schnitzler's Syndrome?
A. Monoclonal IgG gammopathy
B. Thromboembolic events
C. Bronchospasm
D. Hematuria
E. Sensorimotor neuropathy
►E

Schnitzler's syndrome presents as episodes of urticarial vasculitis that occur in association with a
monoclonal IgM M component. Fever, lymphadenopathy, hepatosplenomegaly, bone pain, and
sensorimotor neuropathy also occur.

2
TH2 immune responses:
A. Are associated with cell-mediated immunity
B. Produce IL-6
C. Produce IFN-gamma
D. Produce TNF-beta
E. Produce IL-2
►B

TH1 cells produce IL-2, IFN-gamma, and TNF-beta, and are associated with cell-mediated
immunity. TH2 cells produce IL-4, IL-5, IL-6, IL-10, and IL-13, and are associated with
antibody-mediated immune responses.

3
What is the treatment of choice for this condition?
A. Nystatin
B. Fluconazole
C. Better oral hygiene
D. Penicillin
E. Acyclovir
►C

285
Black tongue is associated with poor oral hygiene, the use of medications, and radiation to the
head and neck region. In many cases, simply brushing the tongue with a toothbrush or using a
commercially available tongue scraper is sufficient improve the condition.

4
The differential diagnosis for the attached image should include all of the followings except
A. Candidiasis
B. Lichen planus
C. Contact dermatitis
D. Inverse psoriasis
E. Erythrasma
►E

The image shown is inverse type of psoriasis. Although psoriatic lesions are classically
distributed on the extensor surfaces (the elbows, knees, and lumbosacral regions), lesions may
also be found in a flexural distribution with involvement of the axillae, groin, perineum, central
chest, and umbilical region. Differential diagnosis include all other choices in question. Although
erythrasma might be included in differentials in case of groin involvement only, it is unlikly for
eryhrasma to presnt on penis with well demarctaed erythamtos plaques. Erythrasma is a
superficial bacterial infection of the skin caused by C. minutissimum. It is characterized by
asymptomatic, well-demarcated, reddish brown, slightly scaly patches in the groin, axillae,
gluteal crease, or inframammary regions, and less often the interdigital spaces of the feet.
Erythrasma is frequently confused with a dermatophyte infection, it can be differentiated from
tinea infection by the characteristic coral red fluorescence seen when viewed under Wood's lamp
illumination (due to the production of porphyrins by the corynebacteria.

5
Which of the following is a progestin used in oral contraceptives with low intrinsic androgenic
properties?
A. Norgestrel
B. Novicane
C. Norgestimate
D. Levonorgestrel
E. Spironolactone
►C

Desogestril, norgestimate and gestodone are progestins with low intrinsic androgenic properties.

286
Patients that have this form of acne is seen affecting young males with suppurating nodules and
plaques that ulcerate and form blackish eschar seen on the trunk most of the time. These patients
can have leukocytosis, fever, arthralgia and lytic changes in the bone most commonly affecting
the:
A. Sternoclavicular joint and chest wall
B. Shoulder blades and sternum
C. Elbows and knees
D. Neck and sternum
E. Jaw and frontal bone
►A

Patients with acne fulminans can have severe acne lesions with lytic lesions indicative of sterile
osteomyelitis seen on x-ray and bone scans. The sternoclavicular joint and the chest wall are
most frequently affected. Treatment includes oral prednisone, antibiotics and isotretinoin.

7
Efficacy of oral contraceptives is reduced by co-administration with which of the following
drugs?
A. Minocycline
B. Doxycycline
C. Rifampin
D. TMP-SMX
E. All of these answers are correct.
►C

Rifampin is a potent hepatic microenzyme inducer and has been shown to reduce efficacy of oral
contraceptives. Anecdotal reports of decreased contraceptive efficacy on oral antibiotics have not
been confirmed in the literature. The putative mechanism is decreased absorption of hormones
secondary to altered gut microflora.

8
Secondary systemic amyloidosis:
A. Classically involves the tongue and periorbital skin
B. Involves deposition of AL protein
C. Can involve deposition of beta 2-microglobulin in the setting of rheumatoid
arthritis
D. Can be noted on biopsy of normal skin
E. Involves deposition of keratin-derived amyloid
►D

287
Secondary systemic amyloidosis presents with deposition of amyloid in the adrenals, liver,
spleen, and kidney as a result of some chronic disease, such as TB, leprosy, Hodgkin's, Behcet's,
rheumatoid arthritis, ulcerative colitis, schistosomiasis, or syphilis. The skin is not involved. AA
amyloid fibrils, derived from SAA protein (an acute phase reactant) are deposited. AA is also
seen in Muckle-Wells and familial mediterranean fever. Biopsy of normal skin may be positive
for perivascular amyloid. Dialysis-related amyloidosis occurs via deposition of beta 2-
microglobulin component altered by uremia, and resuls in carpal tunnel syndrome, bone cysts,
and spondyloarthropathy.

9
A 35 year old man with a history of celiac disease presents with a beefy, red tongue,
hyperpigmented palmar creases, and premature grey hair. Which of the following statements are
is correct?
A. The best therapy is riboflavin 5mg/day
B. This condition mimics folate deficiency
C. It is often associated with carcinoid tumors which divert tryptophan to serotonin
D. This condition can be caused by azithioprine, 5-FU, and isoniazid
E. Eating raw egg whites is a risk factor
►B

This condition is vitamin B12 deficiency and is characterized by glossitis and hyperpigmentation
in sun exposed areas and creases. Neurologic abnormaolities and megaloblastic anemia can be
seen. The symptoms can mimic folic acid deficiency. Riboflavin (B2) is associated with oral-
ocular-genital syndrome. Carcinoid tumors as well as azathioprine, 5-FU, and isoniazid are
associated with niacin deficiency. Eating raw eggs is a risk factor for biotin deficiency.

10
Which of the following HLA types is associated with early onset psoriasis?
A. HLA-B13 or HLA-B17
B. HLA-B27
C. HLA-B57, HLA-Cw6, or HLA-DR7
D. HLA-Cw2
E. HLA-DR3
►C

HLA-B57, HLA-Cw6, or HLA-DR7 are most commonly associated with early onset type I
psoriasis. The presence of HLA-B13 or B17 is associated with a 5-fold risk of developing
psoriasis and are increase in guttate and erythrodermic psoriasis. HLA-B27 may be seen in
pustular psoriasis. HLA-Cw2 is seen with late onset psoriasis, or type II. HLA-DR3 is commonly
found with subacute cutaneous lupus

288
11
Monotherapy for acne with topical antibiotics is discouraged because of:
A. Slow onset of comedolytic action
B. Potential for irritation
C. Lack of anti-inflammatory action
D. Potential for bacterial resistance
E. Poor patient compliance
►D

Topical antibiotics reduce the population of P. acnes on the skin, and thus are indirectly anti-
inflammatory. In contrast to topical retinoids, topical antibiotics are not comedolytic. They are
generally well tolerated by patients.

12
What condition does this patient have given the extent of arthritis seen in this X-ray?
A. Antiphospholipid syndrome
B. Multicentric reticulohistiocytosis
C. Alpha-1-antitrypsin syndrome
D. Eosinophilic fasciitis
E. Dermatomyositis
►B

Multicentric reticulohistiocytosis is a non-Langerhans histiocytosis associated with mutilating


arthritis. Dermatologically, patient may present with coral beading around the fingers. There is a
30% incidence of malignancy in these patients.

13
A 16 year old developmentally normal male presents to his pediatrician intermittent vague
epidodes of hand and feet paresthesias and non specific episode of GI distress. He is referred to
you to evaluate numerous punctate to 5 slightly verrucous, deep-red to blue-black papules
distributed diffusely on his trunk in a bathing suit distribution. Polarization microscopy of the
sediment of his urine demonstrates birefringent lipid globules (ie, renal tubular epithelial cells or
cell fragments with lipid inclusions) with the characteristic Maltese cross configuration. How is
this disorder inherited?
A. Autosomal dominant
B. Autosomal recessive
C. X-linked dominant
D. X-linked recessive
E. Not an inherited disorder

289
►D

Fabry's disease is inherited is x-linked recessive. Female carriers often exhibit mild forms of this
disorder. It can be establish that a patient has FD by searching for low activity of alpha-
galactosyl A in plasma, leukocytes, cultured skin fibroblasts, or dried blood spots on filter paper.
Because of the Lyon effect, enzymatic detection of carriers can be misleading; thus, specific
genetic analysis can be helpful in making the diagnosis.

14
Mutations in the MEFV gene product, pyrin, produce an autosomal recessive syndrome
characterized by recurrent fevers, peritonitis, pleuritis, arthritis and erysipelas-like erythema.
Which of the following syndromes is described above?
A. Familial Mediterranean Fever
B. PAPA syndrome
C. TNF receptor associated periodic syndrome
D. Hyper IgD syndrome
E. Familial cold autoinflammatory syndrome
►A

Familial Mediterranean Fever is described above. It is autosomal recessive with incomplete


penetrance. The main therapy is Colchicine. PAPA syndrome, TNF receptor associated periodic
syndrome, hyper-IgD syndrome and familial cold autoinflammatory syndrome are related
autoinflammatory syndromes. PAPA syndrome is Pyogenic Arthritis, Pyoderma gangrenosum
and Acne and is caused by proline serine threonine phosphatase-interacting protein (PSTPIP1) or
CD2-binding protein 1 (CD2BP1) which interact with pyrin. TNF receptor associated periodic
syndrome (TRAPS) has similar findings to FMF, but the attacks are of longer duration, is
autosomal dominant in transmission, and does not respond to colchicine. TRAPS is caused by a
mutation in the TNFRSF1A gene which results in a decrease in soluble TNF receptor. Hyper IgD
syndrome is associated with defects in the mevalonate kinase gene, which presents with a
hereditary periodic fever. Familial cold autoinflammatory syndrome is associated with cryopyrin
defects and is characterized by fever, rash, conjunctivitis and arthralgia upon exposure to cold.

15
Which of the following is important in the pathogenesis of acne vulgaris?
A. Activation of toll-like receptor-3 by P. acnes
B. Activation of toll-like receptor-2 by M. furfur
C. Activation of toll-like receptor-2 by P. acnes
D. P. acnes produces lipase which cleaves cholesterol into triglycerides
E. Demodex activates complement
►C

290
Acne vulgaris is a disease of follicular hyperkeratosis and the microcomedone is thought to be
the precursor lesion. P. acnes has lipase that cleaves triglycerides into free fatty acids. P. acnes
can activate complement and PMN chemotaxis. Toll-like receptors (TLR) recognize bacterial
patterns and P. acnes activates TLR-2.

16
A patient is diagnosed with glucagon secreting tumor of the pancreas with painful scaling
erythematous patches and plaques at pressure and friction sites. The cells that involved in this
tumor are:
A. alpha islet cells
B. beta islet cells
C. glycon cells
D. insulin cells
E. gamma islet cells
►A

The cells secreting the glucagon tumor are the alpha islet cells of the pancreas. This patient has
necrolytic migratory erythema which is associated with pancreatic glucagonoma, malignancies,
liver disease, malabsorption states, and inflammatory bowel disease.

17
A patient with holocarboxylase synthetase deficiency is most at risk for developing a deficiency
in:
A. Niacin
B. Pyridoxine
C. Biotin
D. Zinc
E. Essential fatty acids
►C

There are three autosomal recessive syndromes that can lead to biotin deficiency:
holocarboxylase synthetase deficiency, biotinidase deficiency, and an inability to transport biotin
into cells. All of these syndromes present with a dermatitis similar to that of zinc deficiency as
well as neurologic abnormalities

18
The most appropriate treatment for the condition shown is:
A. Topical clindamycin 1% lotion
B. Topical metronidazole 0.75% gel

291
C. Azelaic acid 20% cream
D. Salicylic acid 2% wash
E. Adapalene 0.1% gel
►E

This slide shows comedonal acne. Topical retinoids are the treatment of choice. Salicylic acid-
containing cleansers may be helpful adjuncts to treatment with mild comedolytic properties.

19
A patient with this cutanous finding along with peri-ocular erythema, deltoid weakness and
pulmonary disease is most likely to have which laboratory finding?
A. Anti-Jo-1 antibodies against histidyl-tRNA synthetase
B. Anti-Jo-1 antibodies against nuclear helicase
C. c-ANCA against proteinase-3
D. Lupus anticoagulant
E. Anti-histone antibodies
►A

Anti-Jo-1 antibodies against histidyl-tRNA synthetase are described in patients with


dermatomyositis and pulmonary disease. Anti-Mi-2 antibodies against nuclear helicase are
described in classic DM with a good prognosis. Patients with Wegener's disease display c-
ANCA antibodies against proteinase-3 in a cytoplasmic pattern. Antiphospholipid antibodies are
also known as anticardiolipin antibodies and lupus anticoagulant. They can cause a false positive
VDRL. Clinical features include livedo reticularis, thrombotic events and spontaneous abortions.
Anti-histone antibodies are seen in drug-induced lupus.

20
Natural infection with which of the following infectious agents has been shown to mitigate
atopic dermatitis?
A. Measles
B. S. aureus
C. Varicella
D. Rubella
E. Coxsackie virus
►A

Natural measles infection has been shown to ameliorate atopic dermatitis.

21

292
A 32 year-old pregnant woman presents for treatment comedonal and moderately inflammatory
acne. You discuss treatment options with her, and she asks about the evidence for safety in
pregnancy of various treatments. According to the FDA classifications, which of the following
treatments either shows no risk to the fetus in controlled studies (but may show risk to animals),
or shows no risk in animal studies (but no human studies have been conducted)?
A. Benzoyl peroxide
B. Trimethoprim-sulfamethoxazole
C. Topical tretinoin
D. Azeleic acid
E. Tetracycline
►D

The FDA classifies medication safety for use in pregnancy as follows: Category A: Controlled
studies in humans show no risk to fetus. Category B: Controlled human studies show no risk (but
may show risk to animals), or no risk in animal studies (but no human studies have been
conducted). Category C: Risk to human fetus cannot be ruled out, studies are lacking; animal
studies are equivocal. Category D: Controlled studies show risk, but in some instances benefits
may outweigh risks. Category X: Contraindicated in pregnancy. The question describes category
B. Azeleic acid is a category B medication. Benzoyl peroxide, topical tretinoin, and TMP/SMX
are category C medications. Tetracycline is a category D medication.

22
Kveim-Siltzback test is useful in the diagnosis of:
A. Scarlet fever
B. Sarcoidosis
C. Leprosy
D. Leishmaniasis
E. Chancroid
►B

Kveim-Siltzback test is for sarcoidosis, Montenegro-leishmanin test is for Leishmaniasis, and


lepromin test or histamine/methacholine sweat test can be used for leprosy. Kveim-Siltzback test
is done by intradermal injection of a suspension from granuloma-containing spleen, lymph node,
or other tissue from a confirmed case of sarcoidosis. A positive test is characterized by the
formation of a papule at the site of injection within 4-6 weeks which, on microscopic
examination, exhibits non-necrotizing granulomas and the absence of foreign material. This teast
is rarely done nowdays because of the difficulties involved in preparation, standardization and
validation of the test material as well as significant variation in the sensitivity and specificity of
test suspensions obtained from different sources.

293
23
Patients that have a persistent mongolian spot with indistinct borders that are persistent and or
progressive in behavior may have a sign of:
A. GM1 gangliosidosis type I
B. Handlers Disease
C. Mosaic hypopigmentation
D. Neurofibromatosis type II
E. Nevus anemicus
►A

A patient with a persistent mongolian spot with indistinct borders, persistent or progressive
behavior may have an underlying lysosomal storage disease, most commonly GM1
gangliosidosis type I and Hurler disease.

24
A patient has multiple erythematous/violaceous psoriasiform dermatitis affecting the ears, nose,
hands and feet. The patient is diagnosed with Bazex syndrome. The most appropriate course of
action is to:
A. Do an age appropriate cancer screening.
B. Use topical corticosteroids
C. To do a CT scan of the abdomen
D. To order lipase and amylase levels
E. To do a CT of the chest
►A

A patient with Bazex syndrome also known as Acrokeratosis Paraneoplastica should have an age
appropriate cancer screening. Patients can also have nail dystrophy , acquired keratoderma, and
the skin findings usually precede the underlying malignancy.

25
A patient with inflammatory bowel disease develops acute tender juicy plaques on the head and
neck, fever, and malaise. The skin lesions respond well to prednisone. Data shows increased ESR
and neutrophilic infiltrate with dermal edema on skin biopsy. Which one of the following is a
major criteria for this condition?
A. Fever and malaise
B. History of inflammatory bowel disease
C. Abrupt onset of plaques
D. Increased ESR
E. Good response to prednisone
►C

294
This condition is called Sweet's syndrome, or acute febrile neutrophilic dermatosis. Diagnosis
relies on two major and two minor criteria. Major ones include 1) an abrupt onset of juicy painful
plaques and bullae and 2) neutrophilic infiltration in the dermis on pathology. Minor criteria
include 1) presence of associated conditions ie inflammatory bowel disease, infections,
pregnancy, leukemia, etc 2) fever and malaise, 3) laboratory values ie high ESR and CRP, and 4)
excellent response to prednisone.

26
Which of the following is associated with Reiter‘s syndrome:
A. Ulcerative colitis
B. Multiple sclerosis
C. HLA-Cw6
D. Asymmetric arthritis
E. Perioral dermatitis
►D

Reiter‘s syndrome is a chronic inflammatory disease similar to psoriatic arthritis. Commonly,


patients present with either a peripheral, asymmetric arthritis, or a urethritis, or conjunctivitis.
Not all of these findings need be present. Patients are usually men with the HLA-B27 genotype.
Other clinical findings include fever, weight loss, keratitis, iritis, and cardiac disease. Skin
lesions are often found on the palms (keratoderma blenorrhagicum _ hyperkeratotic papules and
plaques) or the penis (balanitis circinata). Reiter‘s follows an infection of either the genitourinary
tract or the gastrointestinal tract.

27
Which form of sarcoidosis is associated with camptodactyly?
A. Lofgren's syndrome
B. Heerfordt's syndrome
C. Darier-Roussy
D. Mikulicz syndrome
E. Blau syndrome
►E

Camptodactyly is a flexion contracture of the 3rd through the 5th proximal interphalangeal joints
and elbows bilaterally. Blau syndrome is an autosomal dominant form of sarcodosis due to a
defect in the CARD15 gene manifesting also with arthritis, cutaneous sarcoid, uveitis, and
synovial cysts. It does not have lung or visceral involvement.

28

295
Increased chylomicrons are a feature of which type of hyperlipoproteinemia?
A. Type I
B. Type IIa
C. Type IIb
D. Type III
E. Type IV
►A

Type I (familial lipoprotein lipase deficiency or apoprotein CII deficiency) hyperlipoproteinemia


has increased chylomicrons as a feature, and presents with eruptive xanthomas and lipemia
retinalis. For the other hyperlipoproteinemias, the associated lipid abnormalities are as follows:
Type IIa -- increased LDL; Type IIb -- increased LDL and VLDL; Type III -- increased IDL;
Type IV -- increased VLDL. Type V has increased chylomicrons and increased VLDL.

29
Which systemic anesthetic can be safely used in mastocytosis?
A. Lidocaine
B. Succinylcholine
C. D-tubocurarine
D. Thiopental
E. Propofol
►E

Local lidocaine can be safely used but systemic lidocaine can precipitate anaphylaxis in patients
with mastocytosis. Other systemic analgesics that are contraindicated in mastocytosis include
succinylcholine, d-tubocurarine, thiopental, metocurine, enflurane, isoflurane, and etomidate.
Propofol, vecuronium bromide, and fentanyl are safe options for systemic anesthesia in patients
with mastocytosis.

30
A patient with spontaneous occurrence of petechiae and purpura, particularly around the eyes,
will also most likely have which of the following?
A. Diffuse scaling of the scalp
B. Acne
C. Cheilits
D. Macroglossia
E. Uveitis
►D

296
This patient has primary systemic amyloidosis. Mucocutaneous findings can be seen in up to
40% of patients. The surface of the tongue may be smooth and dry or covered with waxy papules
and nodules. Teeth indentations may be seen along the lateral borders. The most common
cutaneous signs are petechiae and purpura that occur spontaneous or after minor trauma ("pinch
purpura" about the eyelids). The most characteristic skin lesions are waxy, shiny, smooth papules
and nodules which are usually hemorrhagic or flesh-colored. Flexural areas are common sites.
These patients may also present with carpal tunnel syndrome, hepatomegaly, and edema.

31
A teenage girl presents with recurrent upper eyelid edema and upper vermillion and cutaneous
lip swelling, giving her the appearance of having a ―double upper lip.‖ She reports that the
swelling eventually self-resolves over the course of several days. Additionally, she is noted to
have an enlarged, non-tender thyroid gland upon palpation. She is most likely to suffer from
which of the following conditions?
A. Ascher syndrome
B. Hereditary angioedema
C. Sarcoidosis
D. Airborne allergic contact dermatitis
E. Granulomatous cheilitis
►A

Ascher syndrome is a rare syndrome characterized by recurrent upper (and sometimes lower)
eyelid swelling as well as upper vermillion and cutaneous lip swelling. The latter often yields the
appearance of a ―double upper lip,‖ while multiple episodes of the former results in stretching
and redundancy of the eyelid skin, causing blepharochalasis. Ascher syndrome is frequently seen
in association with a euthryoid (non-toxic) goiter, although up to 50% of patients may have no
associated thyroid abnormality. The pathogenesis of this condition remains unknown.

32
Follicular lichen planus of the skin, multifocal cicatricial alopecia of the scalp and nonscarring
alopecia of the axillary and pubic areas area characteristics of:
A. Graham-Little-Piccardi-Laassueur syndrome
B. Erosive lichen planus
C. Lichen planus pigmentosus
D. Vesiculobullous lichen planus
E. Atropic lichen planus
►A

Graham-Little-Piccardi-Lassuer syndrome has a triad of follicular LP of the skin and or scalp,


multifocal cicatricial alopecia of the scalp and non-scarring alopecia of axillary and pubic areas.

297
33
What organism caused this disease?
A. Micrococcus sedenditaris
B. Candida albicans
C. Staphylococcus aureus
D. Corynebacterium tenuis
E. Corynebacterium diptheria
►A

Pitted keratolysis is a skin disorder characterized by crateriform pitting that primarily affects the
pressure-bearing aspects of the plantar surface of the feet. It is caused by a cutaneous infection
with micrococcus sedentarius which is included in the Corynebacteria genus. These are gram-
positive, catalase-positive, aerobic or facultatively anaerobic, generally nonmotile rods.
Corynebacterium diphtheriae infection is classically characterized by a local inflammation,
usually in the upper respiratory tract, associated with toxin-mediated cardiac and neural disease.
Corynebacterium tenuis causes trichomycosis Patients typically present with yellow, black, or
red pinpoint nodules on the hair shafts in the inguinal region. Staphylococcus aureus is a
common bacterium that can result in formation of pustules, furuncles, cellulitis and abscesses.
Candida species are responsible for superficial infections such as oropharyngeal candidiasis
(thrush) and vulvovaginal candidiasis (vaginal Candidiasis) and is also occurs as an opportunistic
infection

34
A 57yo male with hx of hypothyroidism and SCC of the head and neck s/p XRT several years
ago presented with a skin eruption present on his back and upper arms for the past several
months. What is the most likely diagnosis?
A. Tinea corporis
B. Granuloma annulare
C. Metastatic squamous cell carcinoma
D. Radiation dermatitis
E. Mycosis fungoides
►B

It is most likely a granulomatous inflammatory disease with infiltrates of macrophages arranged


around focus of degenerated collagen and mucin. The etiology is unknown: metabolic
disturbances, autoimmune, allergy, and infection.

35

298
A patient has many painful oral erosions/stomatitis with many lichenoid lesions, erythema-
multiforme-like lesions and flaccid bullae. The patient is diagnosed with paraneoplastic
pemphigus and is associated with all of the following malignancy except:
A. Non-Hogkins lymphoma
B. Chronic lymphocytic leukemia
C. Thymoma
D. Castleman's tumor
E. Gastric carcinoma
►E

Patients with paraneoplastic pemphigus has painful oral lesions and polymorphous skin eruption.
The patients die from complications of underlying malignancy. Bronchiolitis obliterans has also
been reported as a complication and cause of death.

36
Which of the following is NOT associated with this disease of symmetric induration caused
mucin deposition?
A. Diabetes mellitus
B. Streptococcal infection
C. Monoclonal gammopathy
D. Hepatitis C
E. All of the answers are associated with this disease
►D

Scleredema is a type of dermal degenerating mucinosis characterized by diffuse symmetric


induration of the upper body. 3 types of scleredema have been described. The first type is seen in
children following a stretococcal infection. The second type is associated with a monoclonal
gammopathy. The third type is related to insulin dependent diabetes.

37
Reiter's syndrome is known as reactive arthritis and involves conjunctivitis, urethritis and
arthritis after a urogenital or gastrointestinal infection. The gene associated with Reiter's
syndrome is:
A. HLA-B27
B. HLA-B32
C. HLA-DQ23
D. HLA-B29
E. HLA-DQ34
►A

299
Reiter's syndrome is associated with HLA-B27 and can have a variable course often 3-12
months. There is a chronic sequelae in 15-20% of patients.

38
Patients that are diagnosed with Grover's disease have exacerbation most commonly in the:
A. Winter
B. Fall
C. Summer
D. Spring
E. Humidity
►A

Patients diagnosed with Grover's disease have exacerbation commonly in the winter time.
Authors suggest that there may be due to xerotic epidermis and impaired epidermal integrity and
decreased sweat production. It is not caused by sweating and heat and is not increased in
summer.

39
Which of the following is a feature of Sneddon's syndrome?
A. Venous thromboses
B. Hepatosplenomegaly
C. anti-Scl-70 autoantibodies
D. M. tuberculosis infection
E. C2 deficiency
►A

Sneddon's syndrome presents as livedo reticularis and livedoid vasculitis associated with
ischemic cerebrovascular lesions, hypertension, and extracerebral arterial and venous
thromboses.

40
The risk of developing systemic involvement in a patient with nodular amyloidosis is:
A. 1-4%
B. 5-10%
C. 25-30%
D. 50-60%
E. >95%
►B

300
Nodular amyloidosis is a rare type of primary cutaneous amyloidosis made up of AL protein.
The risk for systemic involvement is 7%.

41
Almost all patients with SLE have positive ANAs. A patient can have ANA-negative SLE if they
only make antibodies to:
A. ssDNA
B. Sm
C. U1RNP
D. Ro
E. dsDNA
►A

ANA-negative SLE is uncommon. The ANA has a high negative predictive value and low
positive predictive value. ANA-negative SLE can result if a patient only makes antibodies to
ssDNA, which is not detected by most tests.

42
Lichen planus pemphigoides has been associated with an antigen to which structure?
A. Type XVII collagen
B. Hemidesmosome plaque
C. Desmoglein 3
D. Desmoglein 1
E. Type VII collagen
►A

Type XVII collagen, or BPag2 (180 kD) has been associated with LP pemphigoides. The
hemisdesomosome plauqe is also known as BPag1 (230 - kD), desmoglein 1 and 3 are associated
with the Pemphigous family, and antibodies to Type VII collagen is found in Epidermolysis
Bullosa Aquisita

43
First line therapy for subcorneal pustular dermatosis is:
A. Dapsone
B. Tetracycline
C. Vitamin E
D. Topical retinoids
E. Topical steroids
►A

301
First line therapy for subcorneal pustular dermatosis or Sneddon-Wilkinson disease is dapsone
50-200mg/day. Sulfapyridine, acitretin, PUVA, NB UVB, topical and systemic steroids, vitamin
E and antibiotics have been reported to be helpful also.

44
The most common association with the generalized lesions shown is:
A. Hodgkin's disease
B. Non-Hodgkin's lymphoma
C. Granulomatous mycosis fungoides
D. HIV infection
E. Diabetes mellitus
►E

In a large retrospective study of 1383 patients, diabetes mellitus was diagnosed in 21% of
patients with generalized GA, compared with 9.7% of patients with localized GA. Othere choices
also have been reported to be associated with granuloma annulare. Dabski K, Winkelmann RK.
Generalized granuloma annulare: clinical and laboratory findings in 100 patients. J Am Acad
Dermatol. 1989;20:39-47

45
Angiolymphoid hyperplasia with eosinophilia most commonly occurs on the:
A. Ear
B. Lip
C. Dorsal hands
D. Trunk
E. Lower extremities
►A

Angiolymphoid hyperplasia with eosinophilia often presents with red to reddish-brown papules
or nodules on the head or neck. While lesions can occur on any body site, most common
locations are the ears and scalp.

46
Xanthoma striata palmaris are diagnostic of:
A. Familial hypertriglyceridemia (type IV)
B. Familial dysbetalipoproteinemia (type III)
C. Familial lipoprotein lipase deficiency (AR)
D. Familial lipoprotein lipase deficiency (AD)
E. Apoprotein CII deficiency
►B

302
Xanthoma striata palmaris are diagnostic of type III dysbetalipoproteinemia (AR; broad beta
disease). This condition also presents with palmar, planar, tendinous, tuberous, eruptive, and
intertriginous xanthomas, increased IDL, and atherosclerosis. It is associated with diabetes, gout,
and obesity.

47
Relapsing polychondritis is characterized by circulating auto-antibodies to which of the
following?
A. Collagen type IV
B. Collagen type VII
C. Collagen type II
D. Elastin
E. Chondroitin sulfate
►C

Relapsing polychondritis is a chronic, recurrent rheumatic disease characterized by cartilaginous


inflammation involving the nose, ears, and trachea. A non-erosive inflammatory arthritis may
also be present, and affected patients may suffer from neurosensory hearing loss, tinnitus, and/or
vertigo secondary to vestibular or cochlear damage. Patients may have circulating auto-
antibodies to type II collagen (found exclusively in cartilage) with circulating titers
corresponding to disease activity.

48
Which of the following medications is NOT associated with exacerbating or causing this
eruption?
A. Lithium
B. Prednisone
C. Phenytoin
D. Isoniazid
E. Propranolol
►E

Acne and acneiform eruptions can be caused or exacerbated by cortiosteroids, oral


contraceptives, androgens, ACTH, lithium, phenytoin, halogens, INH, and haloperidol.

49
What is the treatment of choice for neurotic excoriations?
A. Risperidone
B. Doxepin

303
C. Olanzapine
D. Gabapentin
E. Diphenhydramine
►B

Doxepin is the treatment of choice for patients with neurotic excoriations. It has both
antidepressant and antipruritic effects. Combined psychiatric and pharmacologic intervention is
recommended. Other agents such as risperidone, olanzapine, gabapentin, and diphenhydramine
are not drugs of choice for this disorder.

50
Diarrhea, Dementia and a photosensitive dermatitis are associated with a deficiency of which
vitamin?
A. Niacin
B. Biotin
C. Thiamine
D. Riboflavin
E. Pyridoxine
►A

Niacin (Vitamin B3) deficiency is associated with a photosensitive dermatitis, diarrhea and
dementia. The photosensitive dermatitis classically involves the face, neck and upper chest
(Casal's necklace) and forearms (as pictured). Other findings may include angular cheilitis and
thickening and hyperpigmentation of skin overlying bony prominences.

51
What complication can occur if this disease is left untreated?
A. Development of a squamous cell carcinoma
B. Testicular torsion
C. Erectile dysfunction
D. Impotence
E. Pseudo-ainhum
►A

Lichen sclerosis is most commonly located in the genitalia. Longstanding lichen sclerosis is
thought to be a risk factor for the development of a squamous cell carcinoma.

52
Concomitant discoid lupus is found in what percentage of patients with subacute cutaneous
lupus?

304
A. 5%
B. 20%
C. 40%
D. 60%
E. 80%
►B

Sub acute cutaneous lupus presents with scaly papules that evolve into either psoriasiform or
polycyclic annular lesions (more commonly). Sun-exposed surfaces of the face and neck are
commonly involves. Photo sensitivity is seen in 40%, the hard palate is involved in 40%, and
concomitant DLE is seen in 20%.

53
What is the most common digital manifestation of patients with systemic lupus erythematosus?
A. Periungual telangiectasia
B. Raynaud phenomenon
C. Splinter hemorrhages
D. Sclerodactyly
E. Digital pitting scars
►B

This is the most common digital manifestation in patients with SLE and it could present in up to
30% of the patients. It is a paroxysmal vasospam of digits in response to cold exposure or
emotional stress.

54
What is the condition which is a diagnostic cutaneous manifestation of sarcoid?
A. Discoid lupus
B. Lupus pernio
C. Lupus vulgaris
D. Granulomatous rosacea
E. Rhinoscleroma
►B

Sarcoidosis is a systemic disease charcterized by non-caseating granulomas. Organ involved


include the skin, lung, liver and eyes. Lupus pernio manifests as indurated, red-brown, swollen
plaques of the nose, lips, cheeks and ears.

55
What is the most likely diagnosis?

305
A. Dermatofibromasarcoma protuberans
B. Dermatofibroma
C. Lobomycosis
D. Blastomycosis
E. Keloid
►E

Keloids are dense overgrowths of fibrous tissue that generally form a response to tissue injury.
Lobomycosis and dermatofibromasarcoma protuberans may mimic keloids.

56
A 30 year old pregnant female presents with a lesion that has rapidly appeared. She is in her third
trimester with normal pregnancy course to date. What is the most likely diagnosis?
A. Hemangioma
B. Glomus tumor
C. Bacillary angiomatosis
D. Pyogenic granuloma
E. Inflammed wart
►D

It is one of several vascular lesions that can present on the skin during pregnancy. Other common
vascular lesions include palmar erythema, spider angiomas, varicose veins, cavernous
hemangiomas, glomus tumors, and hemorrhoids. Etiologic factors in development of vascular
lesions during pregnancy thought to be a combination of increased hormones and increased
intravascular pressure. Pyogenic granulomas may develop as a reaction to trauma. They often
regress after delivery and treatment is not usually required unless they cause excessive bleeding.

57
Nekam's Disease:
A. Is generally responsive to topical and intralesional steroids
B. Characteristically lacks scale
C. Rarely involves the buttocks
D. Presents with a reticulate pattern on the dorsal hands and feet
E. Presents with hypopigmented, atrophic lesions on the extremities
►D

Nekam's Disease (keratosis lichenoides chronica) presents with violaceous papules and nodules,
hyperpigmented and hyperkeratotic, covered with gray scales. There is often a linear and
reticulate pattern on the dorsal hands and feet, extremities and buttocks. This condition is
generally very refractory to treatment.

306
58
Acquired angioedema is characterized by:
A. Inheritance
B. Normal levels of C1
C. Self-limited course
D. Association with underlying malignancy
E. Increased C1 esterase inhibitor
►D

Hereditary angioedema is an autosomal dominant condition associated with recurrent attacks of


angioedema of the skin, respiratory, and gastrointestinal tract, without urticaria and is caused by
a functional deficiency in C1 esterase inhibitor. In contrast, acquired angioedema is caused by a
depletion in C1 esterase inhibitor. There are two forms of acquired angioedema; one results from
the production of antibodies directed against C1 esterase inhibitor, and the other is seen in
association with underlying malignancies, especially B cell lymphomas, with antibody
production against overexpressed paraproteins. Acquired angioedema can be distinguished from
hereditary angioedema by differences in serum levels of C1. While the level is markedly
decreased in acquired angioedema, a normal C1 is seen in patients with hereditary angioedema.

59
This syndrome is a variant of Dowling-Degos disease with reticulated hyperpigmentation on the
flexures and scaly erythematous papules on flexures on the trunk, AD, keratin 5 mutation. This
syndrome is called:
A. Galli-Galli Syndrome
B. Dowling Degos type I
C. Haim-Munk Syndrome
D. Dengue syndrome
E. Dowling Degos type II
►A

Gali-gali syndrome is a variant of Dowling Degos syndrome. It is characteristic of 1- to 2-mm


slightly keratotic red to dark brown papules which are focally confluent in a reticulate pattern.
The disease is also characterized by slowly progressive and disfiguring reticulate
hyperpigmentation of the flexures.

60
Acquired C1 esterase inhibitor deficiency results in:
A. A normal level of serum C1q
B. A decreased level of serum C1q

307
C. None of these answers are correct
D. A presentation of angioedema very early in life
E. A normal level of C4
►B

Acquired C1 esterase inhibitor deficiency occurs in the setting of lymphoproliferative disease,


monoclonal gammopathy of undetermined significance, or rheumatologic disease and results in a
decreased serum C1q level. Inherited C1 esterase inhibitor deficiency shows an autosomal
dominant inheritance, and earlier presentation, and a normal level of serum C1q. In the inherited
disease, the C1 esterase inhibitor may display normal levels but be functionally impaired.

61
The most common systemic manifestations of systemic sclerosis are:
A. Gastrointestinal
B. Cardiovascular
C. Pulmonary
D. Renal
E. Neurologic
►A

Esophageal dysfunction is the most systemic finding in systemic sclerosis (>90%). Dysphagia
results from decreased peristalsis and may occur before skin findings are seen. Small intestinal
involvement is also common. Pulmonary fibrosis, myocardial fibrosis (seen in 50-70%), cardiac
conduction defects, heart failure, pericarditis with effusion, and renal disease with slowly
progressive uremia may all be seen.

62
The joint most frequently affected in acne fulminans is the:
A. Elbow
B. Intervertebral
C. Distal interphalangeal joints of the hand
D. Sternoclavicular
E. Sacroiliac
►D

Acne fulminans may be associated with lytic bone changes which are indicative of a sterile
osteomyelitis. The sternoclavicular and chest wall joints are most frequently affected.

63
A person that always burns and sometimes tans has the skin phototype:

308
A. II
B. I
C. III
D. IV
E. V
►A

A person that always burns and sometimes tans is type II skin.

64
What is the most common malignancy associated with this condition in this female?
A. Brain cancer
B. Thyroid cancer
C. Rectal cancer
D. Ovarian cancer
E. Liver cancer
►D

Dermatomyositis is an autoimmune polymyositis with characteristic cutaneous findings. Patients


with dermatomyositis should be screened for a underlying malignancy. Risk factors for having
an underlying malignancy include a negative ANA, adult age, and female gender. Ovarian cancer
is one of the most frequently associated with dermatomyositis.

65
A 40 year-old man presents with a complaint of nail changes for several years. He was treated by
an outside physician with terbinafine without improvement. On further questioning, he reports
morning shoulder stiffness and back pain that lasts 1-2 hours and improves with activity. Which
of the following is true regarding his condition?
A. 50% of patients present with joint disease prior to skin involvement
B. Bony erosions are not commonly seen on radiographs
C. Cyclosporine is effective in treating the arthritis in this condition
D. A positive rheumatoid factor may be seen
E. Joint disease correlates with severity of skin involvement
►D

This patient has psoriatic nail changes and a history suggestive of psoriatic arthritis. Psoriatic
arthritis is an inflammatory arthropathy associated with psoriasis. In 80% of patients the
rheumatoid factor is negative; however a positive rheumatoid factor may sometimes be seen.
80% of patients present with skin disease first. Large eccentric erosions are classically present on

309
radiographs. Cyclosporine is not effective in treating psoriatic arthritis. Mild skin disease may be
associated with moderate-to-severe joint disease, and vice versa.

66
Which enzyme can be a useful adjunct test to diagnose zinc deficiency where the serum zinc
level is normal or near normal?
A. Zinc sulfatase
B. Alkaline phosphatase
C. AST
D. ALT
E. Creatinine kinase
►B

A low serum alkaline phosphatase, a zinc dependent enzyme, may be a valuable adjuctive test
where the serum zinc level is normal or near normal. The diagnosis of zinc deficiency should be
suspected in at-risk individuals with acral or periorificial dermatitis. It is usually confirmed by
low serum zinc levels.

67
Which antibody is 93% specific for Sjogren's syndrome?
A. anti-Ro
B. anti-La
C. anti-fodrin
D. anti-Schirmer
E. RF
►C

Sjogren's syndrome is a triad of keratoconjunctivitis sicca, xerostomia, and rheumatoid arthritis.


More than 90% of patients are women. Labial salivary gland biopsy is useful for diagnosis, and
the Schirmer test for xerostomia detects diminished glandular secretions. Laboratory findings
often include positive cryoglobulins, anti-Ro, anti-La, and RF positivity. Antibodies to fodrin are
93% specific for this diagnosis.

68
Elastophagocytosis is characteristic of which condition?
A. Sarcoidosis
B. Granuloma annulare
C. Necrobiosis lipoidica
D. Annular elastolytic giant cell granuloma
E. Palisading neutrophilic and granulomatous dermatitis

310
►D

Annular elastolytic giant cell granuloma is characterized by asymptomatic annular plaques on


sun exposed surfaces. Elastic fibers are absent in the center of lesions. The disease is seen
primarily in middle aged women.

69
Most common cause of these tender lesions on this 12-year old child is
A. Inflammatory bowel disease
B. Tuberculosis
C. beta-hemolytic streptococcal infection
D. Yersinia
E. Infectious mononucleosis
►C

The picture shows classical case of erythema nodosum (EN). Although all other choices in
question cause EN, the most common in the pediatric patient is beta-hemolytic streptococcal
infection (especially pharyngitis). The management of erythema nodosum is directed at
identification and treatment of the underlying cause. Minimal evaluation usually involves
obtaining an ASO/DNase B titer, chest radiograph, and tuberculin testing. Bed rest, with
elevation of the patient's legs, helps reduce pain and edema. When pain, inflammation, or
arthralgia is prominent, NSAIDs can be prescribed. Salicylates, colchicine, and potassium iodide
are the most commonly used alternative therapies.

70
Which of the following is true regarding relapsing polychondritis?
A. Involvement is often bilateral
B. Pathogenic antibodies have not yet been identified
C. The course is chronic, yet mortality is low
D. Both sexes are equally affected
E. Migratory arthralgias are uncommonly seen
►D

Relapsing polychondritis involves intermittent episodes of inflammation of the articular and


nonarticular cartilage, resulting in chondrolysis, dystrophy, and atrophy of the cartilage. Both
sexes are equally affected. IgG anti-type II collagen antibodies are pathogenic, with titers
corresponding to disease activity found in up to 50% of patients with relapsing polychondritis
(and in only 15% of those with RA). Involvement is often unilateral. Migratory arthralgias are
seen in 50-80%. The course is unpredictable, often chronic and variable with episodic flares.

311
Relapsing polychondritis causes death in 1/3 of patients secondary to airway collapse,
cardiovascular complications, and infection (secondary to systemic steroids).

71
The main cause of nutritional disease in developed nations is:
A. Unusual diets
B. Inflammatory bowel disease
C. Malabsorption syndromes
D. Alcoholism
E. Psychiatric illness
►D

Alcoholism is the main cause of nutritional disease in developed nations. Other conditions that
cause nutritional disease include: unusual diets, postoperative state, psychiatric illness,
inflammatory bowel disease, cystic fibrosis, surgical bowel dysfunction, and inborn errors of
metabolism.

72
This patient also has poliosis and deafness. What is the associated ophthalmologic finding?
A. Normal ophthalmologic exam
B. Retinal detachment
C. Lisch nodules
D. Corneal opacities
E. Granulomatous uveitis
►E

This patient has Vogt-Koyanagi-Haradi syndrome (VKHS). VKHS is marked by bilateral


granulomatous uveitis, vitiligo, deafness, poliosis, and alopecia. This syndrome begins with a
meningoencephalitic phase with fever, malaise, headaches, nausea, and vomiting. Varying
degrees of neurological impairment may be present. Recovery is usual complete. The second
phase is the ophthalmic-auditory phase during which uveitis appears rapidly and can results in
blindness. Treatment with systemic steroids may prevent blindness. Lisch nodules are iris
hamartomas which are seen in neurofibromatosis. Corneal opacities are seen in X-linked
ichthyosis. Retinal detachment can occur as a result of head trauma, as in shaken-baby
syndrome.

73
This tumor has associated pain, tenderness, temperature sensitivity and a reddish spot in the nail
bed:
A. Glomus tumor

312
B. Pyogenic granuloma
C. Myxoid cyst
D. Osteochondroma
E. Onychomatricoma
►A

Glomus tumors appear as one of the painful tumors that are reddish spots on the nail.

74
A patient has circular to oval erythematous plaques that are hyperkeratotic with adherent cartpet-
tack scale. This patient with discoid lupus erythematosus has a risk progression to develop SLE
that is approximately:
A. 5%
B. 15%
C. 20%
D. 50%
E. 60%
►A

Patients that have discoid lupus erythematosus have a 5% chance of developing systemic lupus
erythematosus. DLE is complicated by scarring and there is also a risk for development of
squamous cell carcinoma and basal cell carcinoma.

75
Which contact allergen is found in Krazy glue?
A. Formaldehyde
B. Lanolin alchohol
C. Ethyl cyanoacrylate
D. Octyl-dimethyl-paba
E. Mercaptobenzothiazole
►C

Ethyl cyanoacrylate is a chemical found in Krazy Glue. It can also be found in nail adhesives
causing a dermatitis on the face and fingers, nail dystrophy, and a generalized rash.
Formaldehyde is a widely used chemical found in a variety of applications and is a common
allergen. It can be found in paper, skin and hair products, cosmetics, and permanent press
textiles. Lanolin alcohol is found in wool fat, wool wax, adhesives, cosmetics and
pharmaceuticals. Octyl-dimethyl-paba is a chemical found in sunscreens and is the most
frequently used PABA group sunscreen. Mercaptobenzothiazole is a rubber accelerant and is the
most common allergen found in dermatitis to shoes. It is also found in veterinarian products such

313
as flea and tick sprays and powders. Mercapotbenzothiazole can also be found in cutting oil,
antifreeze, fungicides and photographic film emulsions.

76
A 24 year old female suffers periodic attacks of urticaria and fever. Additionally, she has
suffered from worsening deafness and mild renal failure. Her father and grandfather had similar
symptoms. What abnormally deposited protein is driving her symptoms?
A. AA
B. AL
C. B amyloid protein
D. Keratin
E. Beta 2-microglobulin
►A

Muckle Wells is an autosomal dominant disorder with periodic attacks of urticaria, fever,
deafness, and renal amyloidosis. The implicated protein is AA, which is derived from SAA, an
acute phase reactant.

77
A morbidly obese 52-year-old woman presents with dark brown velvety plaques and
acrochordons on the neck and axillae. Which of the following is most likely to be associated with
her cutaneous findings?
A. Diabetes mellitus
B. Hypertension
C. Hypercholesterolemia
D. Hypertriglyceridemia
E. Peripheral vascular disease
►A

This obese patient has acanthosis nigricans and acrochordons, both of which may be associated
with insulin resistance and diabetes mellitus. While she may very well have hypertension,
hyperlipidemia, hypercholesterolemia, and peripheral vascular disease, these are not significantly
related to her cutaneous findings. Acanthosis nigricans may also be seen in patients with gastric
adenocarcinoma.

78
In a well-designed study, the impact of psoriasis on health-related quality of life was found to be
similar to which of the following conditions?
A. Diabetes
B. Acne vulgaris

314
C. Onychomycosis
D. Seasonal allergic rhinitis
E. Hypercholesterolemia
►A

The emotional and physical burden of psoriasis is easily underestimated. Rapp et al found that
patients with psoriasis report reduced physical and mental functioning (―heath related quality of
life‖) similar to that seen in patients with cancer, hypertension, arthritis, heart disease, diabetes,
and depression.

79
A patient is on hemodialysis for end stage renal disease. After a few years of therapy, he
develops carpal tunnel syndrome, bone cysts, and spondyloarthropathy. What deposited
substance is likely causing his symptoms?
A. Beta 2-microglobulin
B. Uric acid
C. AL
D. Triglyceride
E. AA
►A

The patient described likely has dialysis associated amyloidosis. Beta 2-microglobulin is the
protein component which is involved, and can become deposited in various tissues causing,
among other thing, carpal tunnel syndrome, bone cysts, and spondyloarthropathy.

80
Scalp biopsy of a 44-year-old female with suspected discoid lupus erythematosis would likely
reveal inflammation around which portion of the hair follicle:
A. Dermal papilla
B. Hair bulb
C. Matrix
D. Isthmus
E. Infundibulum
►D

On biopsy, alopecia areata exhibits a peribulbar lymphocytic ―swarm of bees.‖ Discoid lupus
typically exhibits inflammation surrounding the isthmus, along with a perivascular dermatitis and
vacuolar interface changes. Lichen planopilaris exhibits inflammation most densely concentrated
about the infundibulum.

315
81
Human orf, also known as ecthyma contagiosum, was diagnosed in a 43 year old farmer by an
astute resident dermatologist. Patient presented with a dome shaped, firm bulla with an
umbilicated crust. Which of the following virus is responsible?
A. Pox virus
B. HHV-8
C. Parapox virus
D. HPV 5,8
E. Mycobacterium bovis
►C

Orf, caused by the parapox virus is usually contracted by direct exposure to infected sheep or
goats. Milker’s nodules are caused by a closely related virus found in cows. Both of these
viruses can be contracted by exposure to fomites (fence post, soil) containing the virus. One to
several lesions may develop usually on the hands or forearm and generally resolve without
therapy in 4-6 weeks.

82
Which HLA type is more commonly associated with this clinical finding?
A. HLA-B7
B. HLA-B15
C. HLA-B27
D. HLA-B51
E. HLA-DR4
►C

The condition shown is balnitis circinata which is part of the constellation of findings in Reiter's
syndrome in addition to arthritis, urethritis, and conjunctivitis. There is a higher incidence of this
condition people with HLA-B27.

83
What bacteria may play a role in the pathogenesis of this disease?
A. B. burgdorferi
B. H. pylori
C. E. coli
D. S. aureus
E. S. enteritidis
►A

316
Morphea is a inflammatory disease primarily of the dermis and subcutaneous fat that ultimately
leads to a scar like sclerosis. Clinically, morphea can be divided into plaque, linear, and
generalized. There is some thought that Borrelia infection may play a role in morphea. mRNA
specific for Borrelia has been found in some lesions of morphea. This association however is
controversial.

84
Which of the following is a cause of immunologic urticaria?
A. Polymyxin B
B. Amoxicillin
C. Ibuprofen
D. Opiates
E. Tartrazine
►B

Amoxicillin is in the penicillin family of antibiotics and is an immunologic cause of urticaria.


Immunologic urticaria is most commonly caused by exposure to this family and other related
beta-lactam antibiotics. Patients with reaction to penicillins have an increased risk of cross-
reacting to cephalosporins, mostly the earlier generations. The third-generation cephalosporins
are less likely to cause reactions in a penicillin allergic patient. The other listed options are
causes of non-immunologic urticaria. They alter prostaglandin metabolism which increases
degranulation of mast cells.

85
What is the most common cause of erythroderma in non-HIV patients?
A. Drugs
B. Underlying malignancy
C. Pre-existing dermatoses
D. Infection
E. Idiopathic
►C

Pre-existing dermatoses is most common cause in non-HIV patients, including atopic dermatitis,
psoriasis, seborrheic dermatitis, chronic actinic dermatitis, mycosis fungoides, pityriasis rubra
pilaris, and allergic contact dermatitis. Drugs are the second most common overall, and the most
common in HIV patients

86
Attached image can be associated with all of the followings except:
A. Smooth muscle hamartoma

317
B. Unilateral breast hypoplasia
C. Acneform lesion
D. Skeletal defect
E. Cardiac defect
►E

Becker's nevus can be occasionally associated with smooth muscle hamartoma, hypoplasia of
underlying structures, such as unilateral breast hypoplasia, unilateral or ipsilateral pectoralis
major aplasia, ipsilateral limb shortening, ipsilateral foot enlargement, spina bifida, scoliosis,
pectus carinatum, localized lipoatrophy, congenital adrenal hyperplasia, polythelia, and
accessory scrotum. In addition to acneform lesions and eczematous dermatitis. There is no
association with cardiac defect.

87
The dermatosis pictured is most commonly associated with:
A. Insulin resistance
B. Cushing's syndrome
C. Acromegaly
D. Polycystic ovarian syndrome
E. Gastric carcinoma
►A

Insulin resistance is the most common cause of acanthosis nigricans. Insulin-like growth factors,
produced by the liver in response to high levels of circulating insulin, bind epidermal growth
factor receptors to produce thickening of the epidermis and hyperkeratosis. Cushing's syndrome,
acromegaly and polycystic ovarian syndrome are associated with acanthosis nigricans, although
less common than insulin resistance. Paraneoplastic acanthosis nigricans is associated with
gastrointestinal carcinoma, classically gastric carcinoma.

88
Which antibody is specific for CREST syndrome?
A. Anti-mitochondrial
B. Anti-histone
C. Anti-ds DNA
D. Anti-nucleolar
E. Anti-centromere
►E

318
The antinuclear antibody (ANA) pattern most specific for CREST is the anti-centromere pattern.
The specificity rate is approximately 50-90% and carries a more favorable prognosis than Scl-70.
The target protein for the anti-centromere pattern is the kinetochore.

89
Patients with this HLA type, which has the most definitive association with psoriasis, have a
relative risk of having psoriasis that is 9-15 times normal.
A. HLA-B13
B. HLA-B17
C. HLA-Bw57
D. HLA-Cw6
E. HLA-B27
►D

HLA-Cw6 is the HLA type most definitively associated with psoriasis. It carries a relative risk 9-
15 times normal.

90
The protein component of primary cutaneous amyloidosis is:
A. SAA protein
B. AL protein
C. Keratin
D. Collagen
E. Bp180
►C

Primary cutaneous amyloidosis presents as either macular or lichen amyloidosis. The protein
component is keratin. Macular amyloidosis often presents over the upper back, while lichen
amyloid presents over the shins.

91
A patient recently underwent parotid surgery and now reports unilateral flushing and sweating
around mealtime. The nerve injured in this syndrome is a branch of the
A. Facial nerve
B. Maxillary nerve
C. Mandibular nerve
D. Cervical nerve
E. Frontal nerve
►C

319
This patient has Frey syndrome or auriculotemporal nerve syndrome. This is characterized by
facial flushing, sweating, or both localized to the distribution of the auriculotemporal nerve that
occurs in response to gustatory stimuli. In adults, the syndrome usually results from surgical
injury or trauma to the parotid gland.

92
Which allergen is the most likely cause of this eyelid dermatitis?
A. Mercaptobenzothiazole
B. Ethyleneurea melamine formaldehyde
C. Ethylenediamine dihydrochloride
D. Tosylonamide formaldehyde resin
E. Benzalkonium chloride
►D

Tosylonomide formaldehyde resin is found nail polish and is a common cause of eyelid
dermatitis in women.

93
This structure can function in reproduction:
A. Conidia
B. Rackets
C. Pectinate bodies
D. Favic chandeliers
E. Mycelium
►A

Conidia function as asexual organs. Favic chandeliers, pectinate bodies, racket forms and
mycelium have no reproductive capabilities.

94
Acrokeratosis paraneoplastica is most commonly associated with which of the following
malignancies:
A. Squamous cell carcinoma
B. Adenocarcinoma
C. Transitional cell carcinoma
D. T-cell lymphoma
E. Breast cancer
►A

320
Acrokeratosis paraneoplastica, also known as Bazex’s syndrome, is a rare paraneoplastic
syndrome. Clinically, it appears as symmetric, hyperkeratotic lesions on red base, in an acral
distribution, nose and helices of the ears. Most commonly associated with upper aerodigestive
tract tumors. Treatment of the tumor leads to disappearance of lesions.

95
This type of psoriasis is associated with more severe body psoriasis. The location of this type is:
A. Centrofacial psoriasis
B. Palmoplantar psoriasis
C. Scalp psoriasis
D. Nail psoriasis
E. Psoriatic arthritis
►A

Centrofacial psoriasis may be associated with more severe body psoriasis. Palmoplantar psoriasis
can be difficult to treat. Patients that have psoriatic arthritis can be candidates for biologic
therapy.

96
A young man presents with explosive onset of severe cystic acne with acute, suppurative nodules
and plaques that ulcerate and form a blackish eschar on the trunk as well as the face. Which of
the following is true regarding this entity?
A. Women are more often affected than men
B. P. acnes osteomyelitis presents with lytic changes on x-rays and bone scans
C. The sternoclavicular joint is often involved in this entity
D. Systemic corticosteroids are contraindicated given risk of sepsis
E. High-dose isotretinoin monotherapy is the treatment of choice
►C

Acne fulminans is a rare, explosive form of severe cystic acne affecting young males. Patients
may be systemically ill, with leukocytosis, fever, arthralgias, and myalgias. Lytic changes,
indicative of a sterile osteomyelitis, can be seen on x-ray and bone scans. The sternoclavicular
joint and chest wall are most frequently affected. Treatment is with oral prednisone, intralesional
steroids, antibiotics, and isotretinoin.

97
Malignancy of the aerodigestive tract is most closely associated with which paraneoplastic
dermatosis?
A. Bazex's syndrome
B. Paraneoplastic pemphigus

321
C. Sweet's syndrome
D. Acanthosis nigricans
E. Dermatomyositis
►A

Bazex's syndrome (acrokeratosis paraneoplastica) is a paraneoplastic dermatosis associated with


malignancy of the aerodigestive tract. Bazex's syndrome classically presents with violaceous
erythema involving the ears, nose, hands and feet. Lesions progress to become hyperkeratotic
and psoriasiform in appearance. The other answer choices all represent paraneoplastic
dermatoses, but none are as closely associated with aerodigestive tract malignancies as Bazex's
syndrome.

98
Which of the following are risk factor(s) for post-transplant CTCL?
A. Renal transplant
B. Liver transplant
C. Cyclosporine therapy
D. Female Sex
E. Renal transplant and Cyclosporine therapy
►E

In a recent case series and review (Arch Dermatol. 2010; 146 (5): 513-516) it was shown that the
following are associated with post-transplant CTCL: renal transplant, cyclosporine, tacrolimus,
and male sex. There have been 29 cases of post-transplant CTCL documented in the literature to
date.

99
In addition to the lesions seen on the feet of this young man he also had geographic tongue,
erythematous plaques on his penis and arthritis, what HLA type is associated with this
syndrome?
A. HLA-B27
B. HLA-B51
C. HLA-DR1
D. HLA-DR3
E. HLA-DQW2
►A

Reiter syndrome, now referred to as reactive arthritis (ReA), is a condition that most often occurs
following enteric or urogenital infections. Reactive arthritis is associated with human leukocyte
antigen (HLA) B27, although HLA-B27 is not always present in individuals who are HIV+.

322
Bacteria associated with reactive arthritis are generally enteric or venereal and include the
following: Shigella flexneri, Salmonella typhimurium, Salmonella enteritidis, Streptococcus
viridans, Mycoplasma pneumonia, Cyclospora, Chlamydia trachomatis, Yersinia enterocolitica,
and Yersinia pseudotuberculosis.

100
What is the minimum amount of time that you would advise this woman to avoid childbearing
after completing a course of isotretinoin?
A. 1 week
B. 1 month
C. 6 months
D. 1 year
E. 3 years
►B

Isotretinoin is a synthetic retinoid that is used primarily in the treatment of acne. Isotretinoin has
a half-life of approximately 20 hours. Women should be advised to avoid becoming pregnant for
at least one month.

101
Which of the following is an innate antimicrobial peptide expressed by keratinocytes in response
to injury or inflammation?
A. Human Beta defensin 1
B. Human Beta defensin 2
C. TNF-A
D. IL-4
E. IL-13
►B

Human-Beta-Defensin (HBD) and cathelicidins are innate antimicrobial peptides in human skin.
HBD-1 is constitutively expressed. HBD-2 (and the cathelicidin LL 37) is expressed in response
to skin injury and inflammation.

102
You suspect that a patient has acquired angioedema. Levels of which of the following would you
expect to be low?
A. C1q, C3
B. C1q, bradykinin
C. C4, C1q
D. bradykinin, C3

323
E. C4, C3
►C

Initial screening test in angioedema for both inherited and acquired should include C4, which is
low. C3 is normal in angioedema. C1q is low in acquired angioedema but normal in the
hereditary type. Bradykinin is elevated in both inherited and acquired angioedema.

103
Best treatment option for this stable type of vitiligo is
A. Phototherapy with narrow band UVB
B. Excimer laser
C. Oral prednisone
D. 20% monobenzyl ether of hydroquinone
E. Nitrogen mustard
►D

The picture shows generalized type of vitiligo or vitiligo universalis. Patients who have
widespread disease with only a few areas of normally pigmented skin in exposed sites can be
treated with depigmenting agents. The patients must be carefully chosen, i.e. adults who
recognize that their appearance will be altered significantly and who understand that
depigmentation requires lifelong care of the skin (sunscreens, protective clothing, etc.). The most
commonly used agent is monobenzyl ether of hydroquinone (MBEH) 20% applied twice daily to
the affected areas for 9-12 months or longer. Monobenzyl ether of hydroquinone is a potent
irritant and/or allergen, and an open use test should be performed before more widespread
application. It normally takes 1-3 months to initiate a response, and a loss of pigment can occur
at distant sites. Although depigmentation from MBEH is considered permanent, repigmentation
(especially perifollicular) can be seen following a sunburn or even intense sun exposure.
Monomethyl ether of hydroquinone in a 20% cream can be used as an alternative to MBEH. Side
effects include contact dermatitis, exogenous ochronosis and leukomelanoderma en confetti.
Phototherapy and excimer laser are not good or practical choices for this type of vitiligo.
Nitrogen mustard is not used in vitiligo.

104
Carcinoid tumors in which site do not lead to flushing or other manifestations of the carcinoid
syndrome?
A. Appendix
B. Ileum
C. Ascending colon
D. Rectum
E. Bronchus

324
►D

Carcinoid tumors are derived from enterochromaffin (Kulchitsky) cells. The appendix is the
most common site, followed by the ileum. Less than 4% of abdominal carcinoid tumors have the
carcinoid syndrome; the presence of the syndrome implies hepatic metastases, extraabdominal
carcinoid tumor, or a large enough tumor burden such that the liver cannot degrade the increased
level of hormone. Foregut tumors (bronchus, stomach, pancreas) produce histamine, cause
peptide ulcers, and produce more persistent, intense flushing, and lacrimation, sweating,
vomiting, and asthma. Midgut tumors (small-intestine to mid-colon) cause cyanotic flush,
hypotension, and bronchoconstriction more commonly. Hindgut tumors (descending colon and
rectal) do not lead to flushing or other manifestations.

105
The presentation of a foregut carcinoid tumor involves:
A. The production of serotonin, cyanotic flush, and bronchoconstriction
B. The production of histamine, intense flushing, peptic ulcer, and lacrimation
C. Cutaneous findings
D. The production of kallikrein with bronchial asthma and angioedema
E. Frequent episodes of tongue swelling and urticaria
►B

Carcinoid syndrome involves tumors derived from the enterochromaffin cells. The appendix is
the most common site. Tumors can be foregut (bronchus, stomach, or pancreas), midgut (small
intestine to mid-colon), or hindgut (descending colon and rectal). Tumors from the foregut area
produce histamine, cause peptic ulcers, and produce intense, persistent flushing and lacrimation.

106
What is the diagnosis?
A. Psoriasis
B. Lichen planus
C. Balanitis circinata
D. Candida
E. Syphilis
►C

Balanitis circinata presents as sharply demarcated, serpiginous ulcers or plaques on the penile
head. Balanitis circinata is usually seen in Reactive arthritis which has a classic triad of arthritis,
urethritis, and conjunctivitis. The syndrome typically occurs post-infection of the GI or urinary
tract.

325
107
Which of the following is true regarding neonatal lupus erythematosus?
A. Most cases involve boys
B. Lesions generally resolve spontaneously by 6 months, healing with scarring
C. Photosensitivity is generally not a feature
D. 75% of mothers have symptomatic systemic lupus erythematous at the time of
delivery
E. Congenital heart block may be the only manifestation of the disease
►E

Neonatal LE presents with annular scaling erythematous macules and plaques on the head and
extremities within the first few months of life in babies born to mothers with LE, rheumatic
diseases, or other connective tissue disorders. 50% of mothers are asymptomatic at delivery.
Lesions resolve spontaneously by 6 months, healing without scarring. Photosensitivity may be
prominent. 75% of cases involve girls. 50% have congenital heart block, which is permanent,
and may be the only manifestation of the disease. Thrombocytopenia and hepatic disease are as
frequent as cardiac disease.

108
Which of the following dermatoses occurs at the latest stage of pregnancy?
A. Darkening of nevi
B. Linea nigra
C. Melasma
D. Areolar hyperpigmentation
E. Psoriasis exacerbation
►B

Linear nigra is one the latest skin changes in pregnancy. It is a hyperpigmented, linear streak
which extends from the pubic symphysis to the xiphoid process. It usually appears from the 20th
week.

109
Which of the following is NOT true regarding primary cutaneous amyloidosis?
A. AA is not the protein component
B. Amyloid is present around blood vessels
C. AL is not the protein component
D. Notalgia paresthetica may be associated with the macular form
E. Amyloid found in benign and malignant neoplasms does not represent primary
cutaneous amyloidosis
►B

326
Keratin is the protein component in primary cutaneous amyloidosis. No amyloid is found around
blood vessels. Macular amyloidosis (which may have associated notalgia paresthetica) and lichen
amyloidosis are forms of primary cutaneous amyloidosis. Secondary cutaneous amyloidosis
presents with keratin-derived amyloid and is seen following PUVA therapy and in benign and
malignant neoplasms.

110
A 3 year-old girl has a red-yellow papule on her cheek which her mother is extremely anxious
about. Skin biopsy reveals a juvenile xanthogranuloma. Which specialist should you refer the
patient to?
A. Plastic surgeon
B. Endocrinologist
C. Neurologist
D. Ophthalmologist
E. Oncologist
►D

Juvenile xanthogranulomas are the most common form of non-Langerhans cell histiocytoses.
80% of these lesions appear within the first year of life and they present as firm, red-brown to
yellowish papule. The most common noncutaneous site is the iris, so referral to ophthalmologist
is warranted.

111
A patient presents with recurrent genital and oral ulcerations and a diagnosis of posterior uveitis.
What HLA type is associated with the diagnosis you suspect?
A. HLA-B27
B. HLA-B51
C. HLA-DR3
D. HLA-Cw6
E. HLA-DR4
►B

The patient has Behcet's disease. Behcet's disease is diagnosed based on recurrent oral ulceration
(at least 3 times in a 12 month period) plus 2 of the following: recurrent genital ulceration,
posterior uveitis, skin lesions (erythema nodosum, pseudofolliculitis, papulopustular lesions, or
acneiform nodules), and a positive pathergy test. Behcet's disease is associated with HLA-B51.

112
What marker is specific for acral melanocytic lesions?

327
A. b-raf
B. c-myc
C. c-kit
D. Ras
E. p53
►C

C-kit staining is specific for acral melanocytic lesions. Ras is found in all melanocytic lesions.
P53 mutations are found in actinic keratoses, SCCIS, and SCC.

113
The diagnosis is:
A. Psoriasis
B. Alopecia areata
C. Onychomycosis
D. Lichen planus
E. Tetracycline-induced photo-onycholysis
►A

This slide depicts nail changes consistent with psoriasis, including onycholysis, subungual debris
and pits. Pits in alopecia areata are usually more regularly spaced than in psoriasis. Fungal
culture of nail clipping for histologic evidence of onychomycosis is warranted. The main feature
of tetracycline-induced photo-onycholysis is onycholysis, rather than chronic nail bed and matrix
involvement with psoriasis.

114
One of your acne patients has been treated with doxycycline for several months and develops
culture positive gram negative folliculitis. What is the next appropriate therapy
A. Tetracycline
B. Bactrim
C. Isotretinoin
D. Ceftriaxone
E. Cefepime
►C

Gram negative folliculitis may occur after prolonged systemic antibiotic use for acne vulgaris. It
should be suspected in patients who develop a sudden acneiform eruption after having been
stable for some time. The treatment of choice is isotretinoin.

115

328
The most common associated malignant neoplasm in a nevus sebaceous of Jadassohn is:
A. Basal cell carcinoma
B. Merkel cell carcinoma
C. Squamous cell carcinoma
D. Sebaceous carcinoma
E. Microcystic adnexal tumor
►A

The most common associated malignant neoplasm is basal cell carcinoma in a nevus sebaceous
of Jadassohn.

116
Twenty-nail dystrophy, nail plate splitting and pterygium formation are nail changes seen in:
A. Lichen Planus
B. Darier's disease
C. Psoriasis
D. Scleroderma
E. Dermatomyositis
►A

Twenty-nail dystrophy, nail plate splitting and pterygium formation are nail changes associated
with Lichen Planus. Darier's disease is associated with longitudinal red and white streaks of the
nail plate and V-shaped knicking distally. Psoriasis is associated with many nail abnormalities
including pitting, onycholysis and oil-spots. Scleroderma may cause nail fold capillary dilation
and destruction while patients with dermatomyositis may exhibit nail fold telangiectasias and
frayed cuticles.

117
The most common non-specific cutaneous manifestation associated with this disease is:
A. Leukocytoclastic vasculitis
B. Ertythema multiforme
C. Acne vulgaris
D. Erythema nodosum
E. Lichen planus
►D

The most common non-specific cutaneous manifestation of sarcoidosis is erythema nodosum.


The clinical hallmark of leukocytoclastic vasculitis is palpable purpura. There are numerous
causes of erythema nodosum including drugs, infection, malignancy, and idiopathic. Lichen
planus can be associated with hepatitis C.

329
118
Inherited Quincke's edema can be detected in the first or second decade and is inherited in an
autosomal dominant pattern. The type II has a:
A. Normal amounts of dysfunctional C1 esterase inhibitor
B. Low amounts of normal C1 sterase inhibitor
C. Low C1q
D. Decreased C2 kinin levels
E. Decreased C4 levels
►A

Inherited Quincke's edema have a normal C1q levels. Type I has low amounts of normal C1
sterase inhibitors and type II has normal amounts of dysfunctional C1 esterase inhibitor.

119
An internal hordeolum can be a painful and erythematous. It is an infection to the meibomian
glands and is caused by:
A. Staphylococcus aureus
B. Actinomyces israelii
C. Borrelia burgdorferi
D. Mycobacterium tuberculosis
E. Streptococcus pyogenes
►A

Internal hordeolum are secondary infection of the Meibomian gland in the tarsal plate. The
infectious agent is Staphylococcus aureus in 90-95% of cases.

120
Pernio or Chilblains is an inflammatory skin condition which is triggered by cold, wet, non-
freezing environmental conditions. Acral skin has violaceous discoloration accompanied by
burning or itching. While avoidance and prevention is best, the most effective pharmacologic
treatment is:
A. Nifedipine
B. Nicotinamide
C. Phenoxybenzamine
D. Psoralen+UVA
E. Aspirin
►A

330
Nifedipine is effective in about 70% of patients with pernio in prevention of the development of
new skin lesions. The other options listed, other than aspirin, are anecdotally suggested to be
helpful.

121
A 45-year old woman from the United States develops erythema and swelling around her eyelids
and symmetric weakness of her shoulders and hips. What malignancy is overrepresented in
patients with this condition compared to the general population?
A. Ovarian Cancer
B. Lung Cancer
C. Leukemia
D. Uterine Cancer
E. Breast Cancer
►A

This patient has a heliotrope rash and proximal muscle weakness, consistent with the diagnosis
of dermatomyositis. 18-32% of patients with dermatomyositis will develop malignancy. The risk
of malignancy is greatest within the first 3 years of diagnosis. Ovarian cancer is overrepresented.
In Southeast Asia, nasopharyngeal cancer is overrepresented.

122
A potentially dangerous side effect of spironolactone is:
A. Hypokalemia
B. Hyperkalemia
C. Hypercalcemia
D. Hyponatremia
E. Hypernatremia
►B

Spironolactone is a potassium-sparing aldactone antagonist. Renal failure predisposes patients to


potentially dangerous hyperkalemia.

123
Probiotics, which are cultures of potentially beneficial gut microflora bacteria, have been studied
in the primary prevention of which of the following diseases?
A. Celiac disease
B. Atopic dermatitis
C. Psoriasis
D. Cutaneous T-cell lymphoma
E. Asthma

331
►B

Probiotics have been studied in the primary prevention of atopic dermatitis by Kalliomaki et al.
Lactobacillus GG cultures were given to pregnant women with a history of atopy to assess the
effect of potentially beneficial gut flora on the prevention of atopic disease in their children. The
frequency of atopic dermatitis in the children in the probiotic group was half that in the placebo
group at two years of life.

124
The most common laboratory abnormality in patients treated with isotretinoin is:
A. Decreased white blood cell count
B. Increased cholesterol
C. Elevated liver enzymes
D. Hypertriglyceridemia
E. Elevated CPK
►D

The most common laboratory abnormality seen in patients taking isotretinoin is increased
triglycerides, followed by elevation of ALT and AST.

125
The diagnosis is:
A. Onychomycosis
B. Trauma-induced nail changes
C. Chronic paronychia
D. Pseudomonal infection
E. Psoriasis arthritis
►E

This slide shows lateral onycholysis and subtle swelling of the distal interphalangeal joint (DIP),
both characteristic findings of psoriasis and psoriatic arthritis.

126
A 32 year old woman, now 12 weeks pregnant, presents to your office with pruritic scaly papules
and plaques. A biopsy reveals focal spongiosis and parakeratosis in mounds, a superficial
perivascular dermatitis, and extravasated red blood cells in the dermis. Which of the following is
true?
A. It has been associated with EBV.
B. There is often a flare post-partum and during subsequent pregnancies.
C. There is no increased incidence in immunocompromised patients.

332
D. There is an increased risk of miscarriage in mothers who developed pityriasis
rosea within the first 26 weeks of their pregnancy
E. Acyclovir may be effective in this condition
►E

The question describes a case of pityriasis rosea, which has been associated with HHV6,7. There
is an increased incidence in immunocompromised patients. There is an increased risk of
miscarriage in women who develop pityriasis rosea before 15 weeks of gestation. Acyclovir may
be effective in the treatment of pityriasis rosea, especially if treated in the first week of
presentation (JAAD 2006;54(1):82-5).

127
What is the most common paraproteinemia in scleromyxedema
A. IgA
B. IgG kappa
C. IgG lambda
D. IgM
E. It is uncommon to see paraproteinemia with scleromyxedema
►C

An abnormal paraproteinemia is found in approximately 90% of cases with scleromyxedema,


generally IgG lambda. IgA paraproteinemia can be seen in pyoderma gangrenosum and
Sneddon-Wilkinson disease. Amyloidosis and NXG can be associated with IgG kappa
paraproteinemia, and Schnitzler syndrome is associated with an IgM paraproteinemia.

128
Which of the following statements is true regarding this entity?
A. 80% of patients with early onset disease have a positive family history
B. Twin concordance (identical twins) is 55%
C. The inner root sheath and matrix of normal hair express MHC class I
D. The sisapho pattern involves the occipital scalp
E. Atopic dermatitis is a predictor of good prognosis
►B

Twin concordance (identical twins) is 55% in alopecia areata. There is a high frequency of
positive family history, especially in patients with early onset (37%). The inner root sheath and
matrix are immune privileged, not expressing MHC class I, and this privilege may collapse in
alopecia areata. The sisapho or ophiasis inversus pattern of alopecia areata is a bandlike pattern
of the fronto parietotemporal scalp. Atopic dermatitis is a predictor of poor prognosis in patients
with alopecia areata.

333
129
A patient has on exam multiple thick banded livedo patterns on the shins with painful, stellate,
necrotic and purpuric plaques on the shin. She is diagnosed with calciphylaxis. All the following
are risk for this disease except:
A. Dialysis
B. Warfarin therapy
C. Iron and vitamin D therapy
D. Liver disease
E. Smoking
►E

Most patients with calciphylaxis have chronic renal insufficiency and hyperparathyroidism. The
associated risk factors are diabetes mellitus, obestiy, dialysis, warfarin therapy, iron and vitamin
D replacement, weight loss, and liver disease.

130
What is deposited in the upper dermis in this condition?
A. AK
B. AL
C. Transthyretin
D. Beta2-microglobulin
E. AA
►A

Lichen Amyloidosis is a primary cutaneous amyloidosis. Deposits of amyloid of believed to be


derived from necrotic keratinocytes. AL deposits are seen in systemic amyloidosis which is
usually associated with a monoclonal gammopathy or myeloma and the amyloid is derived from
immunoglobulin light chains. Amyloid deposits composed of AA are seen in secondary systemic
amyloidosis such as familial Mediterranean fever and Muckle Wells syndrome, amyloid deposits
composed of transthyretin are seen in familial amyloidotic polyneuropathy and amyloid deposits
composed of Beta2-microglobulin are seen in hemodialysis patients

131
Kidney disease in Henoch-Schonlein Purpura may be predicted by:
A. Limited skin involvement
B. Spread of purpura to the upper trunk
C. Synovial involvement
D. Bullous lesions
E. Facial edema

334
►B

Henoch-Schonlein Purpura occurs mostly in children. There is an antecedent URI in 75% of


cases. HSP involves the skin, synovia, GI tract, and kidneys. Long-term morbidity results from
renal disease, which is predicted by the spread of purpura to the upper trunk. Skin lesions of
adults show blisters and necrosis.

132
Immunologic abnormalities in atopic dermatitis include:
A. Increased CD8 T-cell number and function
B. Increased secretion of IFN-gamma
C. Decreased expression of CD23 on B cells and monocytes
D. Increased secretion of IL-4
E. Accentuated DTH response
►D

Immunologic abnormalities in atopic dermatitis include increased synthesis of IgE, increased


specific IgE to multiple allergens, increased expression of CD23 (low-affinity IgE receptor) on B
cells and monocytes, increased basophil histamine release, impaired DTH response, decreased
CD8 suppressor/cytotoxic T-cell number and function, increased secretion of IL-4 and IL-5 by
TH2 cells, and decreased secretion of IFN-gamma by TH1 cells.

133
Reiter's syndrome is a chronic inflammatory disease similar to psoriasis with arthritis, urethritis,
and conjunctivitis. The most common cause of non-urethral form of Reiter's is:
A. Shigella flexneri
B. Salmonella spp
C. Yersinia spp
D. Ureaplasma urealyticum
E. Borrelia burgdorferi
►A

Reiter's syndrome is characteristics of urethritis, conjunctivitis, and arthritis. It occurs in young


men of HLA-B27 genotype that rarely occurs in women.

134
Sarcoidosis presenting with fever, cough, joint pains, hilar adenopathy and erythema nodosum is
known as:
A. Erythema contusiforme
B. Loeffler's syndrome

335
C. Lofgren's syndrome
D. Darier-Roussy sarcoid
E. Heerfordt's syndrome
►C

Sarcoidosis presenting with fever, cough, joint pains, hilar adenopathy and erythema nodosum is
known as Lofgren's syndrome. Erythema nodosum is the most common nonspecific cutaneous
finding in sarcoidosis. Lofgren's syndrome occurs frequently in Scandinavian whites and is
uncommon in American blacks.

135
Sweet's syndrome may be caused by:
A. Bleomycin
B. Cytoxan
C. Granulocyte colony stimulating factor
D. Intravenous immune globulin
E. Methotrexate
►C

Sweet's syndrome is a dramatic skin disease characterized by eruptions of tender,


pseudovesicular coalescing papules and plaques most commonly appearing on the face, neck and
upper trunk. It is seen in association with fever, leukocytosis and typically responds promptly to
systemic steroid therapy. Cutaneous pathergy has been described. The etiology of Sweet's
syndrome is unclear. A hypersensitivity reaction to a bacterial, viral or tumor antigen has been
postulated. Nearly 20% of cases are seen in association with an underlying malignancy,
particularly hematologic malignancies, with acute myelogenous leukemia being the most
common. In addition, drug-related variants of Sweet's syndrome have been described. The
following drugs have been implicated: granulocyte colony-stimulating factor (G-CSF), all-trans
retinoic acid, hydralazine, carbamazepine, levonorgestrel/ethinyl estradiol,
trimethoprim/sulfamethoxazole, and minocycline.

136
A patient has concentric erythematous rings with trailing scale on the trunk and proximal
extremities. It is associated with intense pruritus. The patient is diagnosed with erythema
gyratum repens. The most common cancer that is associated with erythema gyratum repens is:
A. Lung carcinoma
B. Pancreatic carcinoma
C. Stomach cancer
D. Colon cancer
E. Brain cancer

336
►A

Patients with erythema gyratum repens have a "wood grain appearance" and is most commonly
associated with lung carcinoma. Other cancers associated with erythema gyratum repens is
breast, cervical, bowel, and bladder cancer.

137
The diagnosis is
A. Psoriasis:
B. Alopecia areata
C. Onychomycosis
D. Lichen planus
E. Tetracycline-induced photo-onycholysis
►D

Lichen planus-related nail changes seen here include thinning of the nail plate with
onychorrhexis, ridging and pterygium formation.

138
Which of the following leukodermas has a normal number of epidermal melanocytes?
A. Albinism
B. Vitiligo
C. Piebaldism
D. Waardenburg's syndrome
E. Ziprowski-Margolis syndrome
►A

Patients with albinism have a normal number of epidermal melanocytes, however, these
melanocytes synthesize inadequate amounts of melanin. Vitiligo, piebaldism, Waardenburg's
syndrome and Ziprowski-Margolis syndrome all feature a decreased number or total absence of
epidermal melanocytes.

139
What is the most common variant of morphea in children?
A. Plaque
B. Generalized
C. Bullous
D. Deep (morphea profunda)
E. Linear
►E

337
Linear morphea is the most common presentation in children, comprising between 40% to 70%
of children with morphea. This subtype includes linear morphea of the extremity, en coup de
sabre, or progressive facial hemiatrophy, all of which may be accompanied by underlying tissue
atrophy.

140
In lichen planus pemphigoides:
A. Bullae develop characteristically in lesions of longstanding lichen planus
B. Circulating IgG antibodies react to the 230 kDa antigen within the basement
membrane zone
C. There is granular deposition of IgG and C3 at the dermoepidermal junction
D. Bullae result from intense lichenoid inflammation and extensive liquefactive
degeneration of basal keratinocytes
E. Vesicles may develop de novo on previously uninvolved skin
►E

Lichen planus pemphigoides presents with tense blisters atop lesions of LP or de novo on
uninvolved skin. It can be differentiated from bullous LP, where blisters develop in lesions of
longstanding LP as a result of intense lichenoid inflammation and extensive liquefactive
degeneration of basal keratinocytes. Histologically it resembles LP, with linear deposition of IgG
and C3 at the DE junction. Circulating IgG autoantibodies react to a 180/200 kDa antigen within
the basement membrane zone.

141
What is the most common extracutaneous manifestation of Sweet'€™s syndrome?
A. Arthralgias
B. Conjunctivitis
C. Renal involvement
D. Sterile osteomyelitis
E. Fever
►E

Extracutaneous manifestations of Sweet's syndrome occur in more than 75 percent of patients


with Sweet‖s syndrome. The most common is fever, followed by arthralgias, arthritis, or
myalgias. Other less frequent manifestations include conjunctivitis, episcleritis, oral aphthae-
like lesions, cough, dyspnea, pleuritis, and sterile multifocal osteomyelitis. Cardiac, renal,
hepatic, intestinal, or neurologic manifestations are rare.

142

338
Phrynoderma is associated with deficiency of which vitamin?
A. Vitamin A
B. Vitamin B1
C. Vitamin B6
D. Vitamin D
E. Vitamin K
►A

Phrynoderma (toad skin) presents clinically as keratotic papules on the extremities and shoulders
and results from a deficiency of Vitamin A. Vitamin A is a fat soluble vitamin. Hypovitaminosis
A is often seen in diseases such as Crohn's disease, celiac disease and cystic fibrosis which
manifest with fat malabsorbtion. Hypovitaminosis A is additionally associated with night
blindness, xeropthalmia and keratomalacia.

143
The most important mediator of retinoid activity in the skin is:
A. RAR-alpha
B. RAR-beta
C. RAR-gamma
D. RXR-alpha
E. RXR-beta
►C

Retinoid activity in humans is mediated by retinoid receptors. Two groups exist: RA receptors
(RAR) and RX receptors (RXR). Each has three receptor subtypes: alpha, beta, and gamma.
RAR-gamma is the most important mediator of retinoid activity in the skin.

144
This patient was started on isotretinoin but failed to discontinue the tetracycline. He is at risk for
what complication?
A. Acne fulminans
B. Pseudotumor cerebri
C. Diarrhea
D. Depression
E. Myositis
►B

Both isotretinoin and tetracycline are known to cause pseudotumor cerebri. In combination, the
risk is significantly elevated. Symptoms of pseudotumor cerebri include headaches, pulsatile
tinnitus, diplopia, and blurred vision.

339
145
Which of the following is true regarding syndromes associated with amyloidosis?
A. Familial mediterranean fever involves AL protein
B. MEN IIa involves AA protein
C. The syndrome that presents with renal amyloidosis, fevers, limb pains, and
deafness involves AA protein
D. Familial amyloidotic polyneuropathy type IV involves mutations in
apolipoprotein A-1
E. Familial amyloidotic polyneuropathy type III involves mutations in gelsolin
►C

Familial mediterranean fever and Muckle-Wells (renal amyloidosis, urticaria, fevers, limb pains,
and deafness) involve AA. MENA IIa involves keratin-derived amyloid. Familial amyloidotic
polyneuropathy (FAP) type III involves mutations in apolipoprotein A-1; FAP type IV involves
mutations in gelsolin.

146
Which of the following is true about nevus anemicus:
A. Usually occurs in association with vitiligo
B. Occurs more frequently in men than in women
C. Most commonly involves the upper chest
D. Results from locally decreased vascular reactivity to catecholamines
E. Contains dilated blood vessels
►C

Nevus anemicus is caused by localized hypersensitivity to catecholamines and most commonly


found in the upper chest

147
The most common location of dermatofibrosarcoma protuberans is:
A. Trunk
B. Head and neck
C. Extremities
D. None of these answers are correct
E. head and neck and extremities
►A

Dermatofibrosarcoma protuberans is typically located on the trunk. The extremities are the
second most common location for this type of neoplasm.

340
148
Which of the following is true regarding topical therapies for psoriasis?
A. Vitamin D3 analogues deactivate salicylic acid
B. Retinoids are effective in decreasing lesional erythema
C. Anthralin can cause irreversible staining of peri-lesional skin
D. Calcipotriol is deactivated by UV light
E. Anthralin inhibits PMNs and monocytes
►E

Anthralin, in addition to possessing antiproliferative activity on human keratinocytes, has strong


anti-inflammatory effects by inhibiting PMNs and monocytes. Vitamin D3 analogues are
inactivated by salicylic acid and should be used after UV light (calcipotriol absorbs UV).
Retinoids reduce scaling and plaque thickness, but do not generally decrease lesional erythema.
Anthralin can stain hair purple and cause reversible brownish discoloration of surrounding skin.

149
This woman has restricted ability to open her mouth with tight bound down skin of her hands
and pulmonary fibrosis. In addition to a positive ANA with a nucleolar pattern, what is the most
common autoimmune antibody that is associated with pulmonary fibrosis?
A. Topoisomerase I antibodies (formerly Scl-70)
B. Fibrillarin antibodies
C. Anti-U3RNP antibodies
D. Anti-PM-Scl antibodies
E. Anti-thyroglobulin antibodies
►A

This woman has scleroderma which is a systemic disease characterized by skin induration and
thickening. The cutaneous findings are accompanied by various degrees of tissue fibrosis and
chronic inflammatory infiltration in numerous visceral organs, prominent fibroproliferative
vasculopathy, and humoral and cellular immune alterations. Antinuclear antibodies are present in
about 95% of the patients, usually with a speckled or homogenous pattern. A nucleolar pattern,
although less common, is more specific for systemic sclerosis. Topoisomerase I antibodies
(formerly Scl-70) are present in approximately 30% of patients with diffuse disease (absent in
limited disease) and are associated with pulmonary fibrosis. Anticentromere antibodies are
present in about 60-90% of patients with limited disease and are rare in patients with diffuse
disease. Fibrillarin antibodies and antibodies to U3 ribonucleoprotein (RNP) may also be present
but are more common in patients with skeletal muscle involvement. Anti-U3RNP is present
mostly in patients with diffuse disease with overlap syndromes. Anti-ThRNP is present mostly in

341
limited disease and is associated with more extensive visceral disease. Anti-PM-Scl is present in
limited and overlap states and is associated with myositis and renal involvement.

150
Which of the following is true regarding acquired C1 esterase inhibitor deficiency?
A. This condition generally occurs in the first or second decade of life
B. Serum C1q is normal
C. C2 and C4 are both decreased
D. C1 esterase inhibitor may be at normal levels with functional impairment
E. Positive family history is common
►C

C1 esterase inhibitor is a protease inhibitor that inhibits the catalytic subunits of the first
components of the classical pathway. In the absence of C1 esterase inhibitor, activated C1 and
plasmin generate activated C2 kinin, which mediates angioedema. Acquired C1 esterase inhibitor
deficiency generally affects adults or elderly individuals with no family history. Serum C1q is
decreased. It occurs in the setting of lymphoproliferative disease or rheumatologic illness, where
idiotype/anti-idiotype immune complexes consume available C1q and functionally and
quantitatively lower the amounts of C1 esterase inhibitor. It can also occur in the setting of
autoimmunity directed against the C1 esterase protein. Inherited C1 esterase inhibitor deficiency
is detected in the first or second decade of life and is autosomal dominantly inherited. Serum
C1q is normal in the inherited form, but there is a defect in the synthesis and/or function of C1
esterase inhibitor. In both the inherited and acquired forms levels of C2 and C4 are decreased
because of the uncontrolled actions of C1s.

151
A 32 year-old woman presents with moderate hirsutism. She has normal menses, normal-sized
ovaries, no evidence of tumors of the adrenal or ovary, and normal adrenal function, but does
have slight elevations of plasma androstenedione and testosterone. What is the most likely
diagnosis?
A. Stein-Leventhal syndrome
B. Cushing syndrome
C. Idiopathic hirsutism
D. Occult virilizing tumor
E. Kruckenberg tumor
►C

Idiopathic hirsutism is diagnosed in women with evidence of androgen excess but with normal
menses, normal-sized ovaries, no evidence of tumors of the adrenal or ovary, and normal
androgen function. Slight elevations of plasma androstenedione and testosterone are common.

342
152
This 10 year-old girl presents to your office. These lesions have been present for months. She
denies any other systemic complaints. What will you tell her parents?
A. They should expect her to get more lesions in non sun-exposed areas
B. She is more likely to develop systemic lupus erythematosus than an adult with
these lesions
C. She is less likely than an adult to develop renal disease
D. She is more likely than an adult to develop cardiovascular disease
E. No hematologic labs are required for evaluation
►B

Discoid lupus erythematosus (DLE) presents with plaques characterized by scarring, atrophy,
follicular plugging, and scale and photosensitivity. Children presenting with DLE have a higher
incident of developing systemic lupus (SLE) than adults. Because of progression from DLE to
SLE, children should be screened and followed with antinuclear antibodies and anti- DNA
antibodies. Children and adolescents have a higher incidence of renal involvement. Treatment for
DLE includes topical steroids, oral steroids, and hydroxychloroquine.

153
A 17 year old high school cheerleader returns to clinic for follow up of her acne and
management of her isotretinoin. She denies depression or any GI complications. She is pleased
with her progress but does mention painful red bumps on her lower legs for 1 week. She
attributes these to her cheer practice. What is the most likely cause of her lower extremity
lesions.
A. Training regimen
B. trip and fall
C. Oral contraceptives
D. Allergy medication
E. Recent strep infection
►C

The patient is likely experiencing erythema nodosum (EN), a delayed hypersensitivity response
to a wide variety of eliciting factors. They consist of an eruption of erythematous, tender
nodules, typically over the anterior tibial areas. Common causes include oral contraceptives,
which is a probable choice given that the patient is likely taking them as part of her isotretinoin
treatment. Strep infection is also a common cause of EN but with no mention of symptoms is less
likely in this case. Other causes include TB, fungal infections, sarcoidosis, ulcerative colitis, and
regional enteritis.

343
154
In this patient with an autoimmune disorder, which autoantibody would be indicative of
increased risk of pulmonary disease?
A. U1RNP
B. DsDNA
C. anti-Jo-1 antibody
D. Mi-2 antibody
E. Scl-70 antibody
►C

The patient depicted has dermatomyositis. Autoantibodies to anti-Jo-1 antibody targets histidyl
transfer RNA synthetase. In dermatomyositis correlates with the development of pulmonary
disease.

155
The most common complication for sarcoidosis is:
A. Anterior uveitis
B. Glaucoma
C. Pars planitis
D. Diplopia
E. Paresis if ocular muscles
►A

The most common eye complication of sarcoidosis is anterior uveitis with mutton fat keratotic
precipitates. All the others can be a complication of sarcoidosis also.

156
Which of the following is a manifestation of psoriasis of the nail matrix?
A. Splinter hemorrhages
B. ―Oil spots‖
C. Subungual hyperkeratosis
D. Pits
E. Onycholysis
►D

Psoriatic nail changes may be of nail matrix or nail bed origin. Pits are the common finding;
splinter hemorrhages the least. Psoriatic nail changes of matrix origin include: pits (representing
focal psoriasis of the proximal matrix) and leukonychia. Psoriatic nails changes of nail bed origin
include: salmon spots, ―oil spots,‖ onycholysis, subungual hyperkeratosis, and splinter
hemorrhages.

344
157
This patient has similar lesions on his distal extremities. Which laboratory test can be done in
order to make a diagnosis?
A. Hemogram
B. Alkaline phosphatase
C. Creatinine
D. Potassium
E. TSH
►B

This patient has acrodermatitis enteropathica, a rare, inhertied disorder caused by an inability to
absorb zinc. This disease is characterized by a traid of acral dermatitis, diarrhea, and alopecia.
AE is rapidly reversed by zinc supplementation. Alkaline phosphatase is a zinc-dependent
enzyme; it is a moderately-sensitive marker for zinc deficiency (although not an early marker).

158
This variant of amyloidosis is derived from degenerated tonofilaments of keratinocytes:
A. Lichen amyloidosis
B. Nodular amyloidosis
C. Primary systemic amyloidosis
D. Dialysis-related amyloidosis
E. Secondary systemic amyloidosis
►A

Lichen amyloidosis and Macular amyloidosis are derived from degenerated tonofilaments of
keratinocytes. Primary systemic amyloidosis results from deposition of protein AL derived from
Ig Lambda light chains. Nodular amyloidosis is also associated with AL type protein. Secondary
systemic amyloidosis is associated with AA amyloid fibrils derived from SAA protein. Dialysis-
related amyloidosis is associated with beta-2-microglobulin protein deposition.

159
Solid confluent palmoplantar keratosis, salmon-orange follicular papules and diffuse
symmetrical involvement with characteristic small islands of normal skin within affected areas
are all clinical characteristics of what skin disease?
A. Lichen Sclerosis
B. Dermatomyosistis
C. Phrynoderma
D. Reiter‖s syndrome
E. Pityriasis rubra pilaris

345
►E

PRP can affect any portion of the body, but commonly affect the extensor surfaces of the
extremities as well as the sides of the neck and trunk. It may progress with an almost
erythrodermic type state with characteristic small island of normal skin within affected areas.
Phrynoderma refers to the toadskin like skin seen in some vitamin deficiencies including vitamin
A deficiency.

160
The desmoplastic trichoepithelioma is most often found in this location:
A. Face
B. Arm
C. Hands
D. Neck
E. Scalp
►A

Desmoplastic trichoepithelioma is referred to as sclerosing epithelial hamartoma. The location is


usually on the face of women

161
The mechanism of action of azithromycin is:
A. Inhibition of bacterial cell wall synthesis
B. Inhibition of RNA-dependent protein synthesis by binding to the 30s ribosomal
subunit
C. Inhibition of RNA-dependent protein synthesis by binding to the 50s ribosomal
subunit
D. Inhibition of DNA-dependent RNA polymerase
E. Inhibition of bacterial topoisomerase
►C

Azithromycin is a macrolide antibiotic. It binds the bacterial 50s ribosomal subunit and inhibits
RNA-dependent protein synthesis.

162
After being treated for several months with doxycycline, this patient develops a gram negative
folliculitis. What is the next appropriate therapy?
A. Tetracycline
B. Bactrim
C. Isotretinoin

346
D. Ceftriaxone
E. Cefepime
►C

Gram negative folliculitis may occur after prolonged antibiotic therapy for acne vulgaris. It
should be suspected in patients who are well controlled and then suddenly flare. The treatment of
choice is isotretinoin.

163
The arthritis of Behcet's disease is characteristically:
A. Symmetric, erosive polyarthritis
B. Asymmetric, erosive polyarthritis
C. Asymmetric, non-erosive polyarthritis
D. Asymmetric, erosive monoarthritis
E. Symmetric, non-erosive polyarthritis
►C

Behcet's disease is characterized by recurrent oral ulceration plus 2 of the following: recurrent
genital ulceration, eye lesions (posterior uveitis), skin lesions, positive pathergy test. Clinical
features include thrombosis of the superior vena cava, thrombophlebitis, CNS lesions that give a
picture of multiple sclerosis, and an asymmetric non-erosive polyarthritis.

164
Which of the following is most characteristic of seborrheic dermatitis?
A. Greasy, yellow, scaly plaques on the central face and chest
B. Well defined, round, scaly, pink plaques on the knees and elbows
C. Stuck on waxy papules and plaques
D. Pink papules and telangiectasias on the cheeks
E. Heliotrope violaceous periocular patches
►A

Seborrheic dermatitis is characterized by greasy, yellow, scaly plaques involving the scalp,
central face, and chest. It can be treated with medications such as ketoconazole or selenium
sulfide. Psoriasis is defined by well defined round, scaly, pink plaques on the knees and elbows.
Seborrheic keratoses are defined by stuck on waxy papules and plaques. Rosacea may be
characterized by pink papules and telangiectasias on the cheeks. Dermatomyositis is
characterized by heliotrope violaceous periocular patches.

165

347
A 35 year old man has a history of intensely pruritic papules and vesicles on the extensor
surfaces of his lower extremities. Antibodies to epidermal transglutaminase are detected. What is
this patient at risk for developing?
A. Non-Hodgkin lymphoma
B. Colon cancer
C. Lung cancer
D. CLL
E. Esophageal cancer
►A

A number of studies have indicated an increased risk of non-Hodgkins lymphoma and


enteropathy T-cell lymphoma in patients with dermatitis herpetiformis. There is also increased
prevalence of thyroid disease, type I diabetes, and other autoimmune disorders, such as vitiligo,
Addison's, and alopecia areata.

166
All of the following are true regarding Reiter'™s syndrome except:
A. The classic clinical triad is urethritis, conjunctivitis and arthritis
B. Usually occurs in young women of HLA-B27 genotype
C. May be associated with keratoderma belnnorrhagicum
D. May be associated with Chlamydia trachomatis
E. Nails may become thick and brittle with heavy subungual hyperkeratotic
deposits
►B

Reiter'™s syndrome is a chronic inflammatory disease similar to psoriasis with psoriatoc


arthritis. Reiter's syndrome usually occurs in men of HLA-B27 genotype and rarely occurs in
women. Few patients present with the classic triad, thus can be diagnosed with peripheral
arthritis >1 month and associated urethritis. Keratoderma blennorrhagicum is crusted,
hyperkeratotic papules and plaques on plantar surfaces. Painful and bloody urination and pyuria,
cystitis, prostatitis and seminal vesiculitis may occur secondary to Chlamydia trachomatis. Nail
lesions as described may occur.

167
Which of the following would be an important diagnostic sign of this non-infectious disorder?
A. Lacrimal gland enlargement
B. Periungual fibromas
C. Cafe-au-lait macules
D. Peg shaped teeth
E. Dystopia canthorum

348
►A

Sarcoidosis is a non-infectious granulomatous disorder of unclear etiology. Ocular manifestation


are relatively common in sarcoidosis and may include acute anterior uveitis (classic finding),
posterior uveitis, lacrimal gland enlargement, and conjuctival nodules.

168
A 50-year-old female with sarcoid has enlargement of the parotid, submandibular, and lacrimal
glands that is also known as:
A. Mikulicz's syndrome
B. Heerfordt-Waldenstromn syndrome
C. Darier-Roussy sarcoid
D. Lofgren's syndrome
E. Blau's syndrome
►A

This patient has Mikulicz's syndrome which is sarcoid of the parotid, submandibular and
lacrimal glands. Heerfordt-Waldenstromn syndrome demonstrates a combination of fever,
parotid enlargement, anterior uveitis, and facial nerve palsy. Darier-Roussy sarcoid is
characterized by subcutaneous nodular sarcoid on the trunk and extremities. Lofgren's syndrome
features acute sarcoid, erythema nodosum and migratory polyarthritis, fever, iritis, and bilateral
hilar adenopathy. Blau's syndrome is a rare autosomal dominant familial granulomatous
syndrome that presents with arthritis, uveitis, and skin lesions that appear as "red dots;" there is
no pulmonary involvement.

169
Which antibody is associated with a positive prognosis in dermatomyositis?
A. Anti-Jo
B. Anti-Smith
C. SRP
D. Anti-Mi
E. Aldolase
►D

Several antibodies have prognostic importance in dermatomyositis. Anti-Mi is associated with an


improved prognosis. Anti-Jo is associated with a poor prognosis and lung involvement. SRP is
also associated with a poor prognosis and heart involvement.

170

349
You are consulted to evaluate this patient with tender, warm plaques on his shins. What other
physical signs should you look for?
A. Exophthalmos
B. Uveitis
C. Tachycardia
D. Clubbing of the fingers
E. Cough
►B

Erythema Nodosum (EN) is represented by tender, warm,nodules and plaques, often but not
exclusivley located on the anterior shin. It represents a reactive panniculitis. Causes include,
infections (Streptococcal, tuberculosis, yersina, mycoplasma, campylobacter,salmonella,
histoplasmosis, blastomycosis, coccidiomycosis), drugs (sulfonamides, gold, and OCP's),
enteropathies, pregnancy, hodgkin's disease and lymphoma and sarcoidosis. Loefgren's disease is
a varient of sarcoidosis with EN, hilar adenopathy, fever, uveitis and arthritis. Differential
diagnosis includes: pretibial myxedema and erythema induratum.

171
A 16 year old developmentally normal male presents to his pediatrician intermittent vague
epidodes of hand and feet paresthesias and non specific episode of GI distress. He is referred to
you to evaluate numerous punctate to 5 slightly verrucous, deep-red to blue-black papules
distributed diffusely on his trunk in a bathing suit distribution. Polarization microscopy of the
sediment of his urine demonstrates birefringent lipid globules (ie, renal tubular epithelial cells or
cell fragments with lipid inclusions) with the characteristic Maltese cross configuration. What is
the classic ocular finding in this disorder?
A. Pseudo-herpetic corneal ulcerations
B. Brushfield spots
C. Hyperpigmentation of the retinal epithelium
D. Corneal opacities
E. Lester irides
►D

Whorled corneal opacities are classically seen in Fabry's disease. Pseudo-herpetic ulcerations are
seen in Richner-Hanhart. Brushfield spots can be seen in Down's syndrome. Congenital
hyperpigmentation of the retinal epithelium is seen in Gardner's syndrome. Lester irides are seen
in Nail-Patella syndrome.

172
A young girl presents with recurrent severe arthritis of the ankles. She also has a large ulcer on
her leg and severe acne. Which gene is mutated?

350
A. PSTPIP1
B. NOD2
C. CIAS1
D. AIRE
E. FOXP3
►A

This patient has PAPA (pyogenic arthritis-pyoderma gangrenosum-acne) syndrome. The gene
mutated is PSTPIP1, also known as CD2 binding protein 1 (CD2BP1) which encodes proline-
serine-threonine phosphatase-interacting protein 1. NOD2 is the gene involved in Blau
syndrome; CIAS1 with CINCA/Muckle-Wells/Familial cold autoinflammatory syndromes;
AIRE with APECED (autoimmune polyendocrinopathy-candidiasis-ectodermal dystrophy);
FOXP3 with IPEX (immune dysregulation, polyendocrinopathy, enteropathy, X-linked)
syndrome.

173
Which of the following is true regarding this entity?
A. The subcutaneous variant is most common in children
B. 75% of localized lesions are still present 2 years after diagnosis
C. Localized lesions often ulcerate and heal with scarring
D. The perforating variant classically involves the lower extremities
E. Diabetes is present in the majority of patients with the generalized variant
►A

Granuloma annulare is the entity pictured. Subtypes include localized GA, generalized GA,
macular GA, subcutaneous GA, and perforating GA. The subcutaneous variant is most common
in children; there is often a history of trauma to the area. Localized lesions never ulcerate and
heal without scarring; 75% of localized lesions clear within 2 years. The perforating variant
classically involves the dorsa of the hands. Diabetes is present in 20% of patients with the
generalized variant.

174
This radiograph shows a characteristic finding of severe psoriatic arthritis: bone proliferation at
the base of the distal phalanx with resorption of the tufts. What is the most common form of
psoriatic arthritis?
A. Arthritis mutilans
B. Axial
C. Symmetric polyarthritis
D. Asymmetric oligoarthritis
E. Symmetric oligoarthritis

351
►D

Approximately 70 % of psoriatic arthritis is asymmetric oligoarthritis. The slide depicts arthritis


mutilans, which occurs in about 5% of patients with psoriatic arthritis.

175
Which of the following is most associated with pruritus?
A. Purpura annularis telangiectodes
B. Gougerot–Blum syndrome (pigmented purpuric lichenoid dermatitis)
C. Lichen aureus
D. Schamberg\'s disease
E. Ducas and Kapetanakis pigmented purpura
►E

Ducas and Kapetanakis pigmented purpura presents as eczematous patches with petechiae and
hemosiderin staining. Pruritus is common, and the lesions are often more extensive than the other
pigmented purpuras. Histologically, spongiosis is noted. The other listed pigmented purpuric
eruptions tend to be asymptomatic, although lesions of Schamberg\'s disease can rarely be
associated with itch.

176
A patient has rapidly increasing seborrheic keratosis, acanthosis nigricans and tripe palms. The
most common associated malignancy with Sign of Lesser-Trelat is:
A. Gastric carcinoma
B. Lymphoma
C. Lung cancer
D. Ovarian cancer
E. Prostate cancer
►A

The Sign of Lesser-Trelat is defined as the rapidly increasing in size or number of seborrheic
keratosis. The associated malignancy is gastric and colon cancer. It may improve with treatment
of underlying malignancy.

177
Which HLA is associated with psoriatic arthritis in disequilibrium, especially if spondylitis is
present?
A. HLA-B17
B. HLA-B13
C. HLA-Bw57

352
D. HLA-Cw6
E. HLA-B27
►E

HLA-B17 is associated with an earlier onset and more serious disease of psoriasis. HLA-Cw6 is
the most definitive associated HLA type of psoriasis, with a relative risk 9-15 times normal.
HLA-B17 amd HLA-B13 are also associated with psoriasis but not psoriatic arthiritis

178
Acute hemorrhagic edema of childhood often presents initially with:
A. Facial edema
B. Laryngospasm
C. Acute abdomen
D. Hematuria
E. Hematochezia
►A

Acute hemorrhagic edema of childhood affects children and infants < 2 years of age. It presents
with painful, edematous petechiae and ecchymoses on the head and distal extremities. Facial
edema may be the initial sign. Triggering factors include infection, drugs, and immunization. It
lacks systemic features and resolves in 1-3 weeks without sequelae.

179
A 61 year old man presents with painful and pruritic unilateral vesicles on his lateral back.
Patient described pain for 1 week prior to development of the rash. Diagnosis was confirmed
with a Tzank smear. Which of the following is a frequently seen complication of this disease?
A. Blindness
B. Infertility
C. Postherpetic neuralgia
D. Renal failure
E. Hepatic insufficiency
►C

Postherpetic neuralgia (PHN) is a common complication seen in herpes zoster. Defined as the
presence of pain after skin lesions have healed. Occurs in 10-15% of patients but incidence may
increase to 50% in patients over the age of 60 who develop herpes zoster. In most cases PHN
resolves within the first 12 months but may persist for years. While generally self-limited,
antiretroviral therapy is recommended for patients over the age of 50, those who are
immunocompromised, or have ophthalmic zoster.

353
180
The most effective treatment for symptomatic ocular rosacea is:
A. Lubricant eye drops
B. Combination topical antibiotic and corticosteroid eye drops
C. Dilute baby shampoo washes
D. Doxycycline
E. Artificial tears
►D

The most definitive treatment for symptomatic ocular rosacea is an oral tetracycline.

181
A 35 year-old otherwise healthy man presents with moderate-to-severe plaque psoriasis,
improved on cyclosporine. The patient weighs 70 kilograms and is taking a dose of 300 mg/day.
His baseline creatinine was 0.8; on follow-up testing it is 1.1. All other exam and laboratory
parameters are within normal limits. The patient is pleased with his treatment and asks to
continue it. Which of the following is correct?
A. The patient has exceeded the recommeded dosage of cyclosporine
B. You offer a decrease of the cyclosporine dose to 225 mg/day and close follow-
up
C. The change in creatinine is not significant, no change is needed
D. Cyclosporine rarely has renal toxicity in young, healthy individuals; thus you
must work-up other causes of the increased creatinine
E. The cyclosporine should be stopped immediately; the patient must avoid
cyclosporine in the future
►B

Cyclosporine is highly effective in most patients with severe chronic plaque-type psoriasis.
Doses start at 2.5 to 4mg/kg/day and can go as high as 5.5mg/kg/day. Renal impairment may
occur and is often reversible. If the creatinine increases 30% or greater from baseline the dose
should be reduced by 25% and the patient followed closely.

182
Anti-Jo-1 antibodies in patients with Dermatomyositis are associated with:
A. Pulmonary fibrosis
B. Cardiac disease
C. Photosensitivity
D. Calcinosis
E. Heliotrope rash
►A

354
Anti-Jo-1 (histidyl TRNA syntetase) antibodies are seen in a minority of patients with
Dermatomyositis and are associated with pulmonary fibrosis. Anti-Jo-1 Abs are also associated
with Raynaud's and polyarthritis. Treatment of dermatomyositis may include systemic
corticosteroids, antimalarials, methotrexate, azatioprine and photoprotection.

183
Which immunoglobulins are seen in type III cryoglobulinemia?
A. Monoclonal IgG and monoclonal IgM
B. Monoclonal IgG or monoclonal IgM
C. Monoclonal IgM and polyclonal IgG
D. Polyclonal IgM and monoclonal IgG
E. Polyclonal IgM and polyclonal IgG
►E

Type III (mixed) cryoglobulinemia consists of rheumatoid factors that are polyclonal IgM and
IgG, complexed with each other or with protein. Type II (mixed) cryoglobulinemia consists of
monoclonal IgM rheumatoid factor complexed with polyclonal IgG. Type I cryoglobulinemia
consists of a single monoclonal immunoglobulin, usually IgG or IgM, usually due to an
underlying B-cell malignancy (myeloma or lymphoma).

184
Acneiform eruptions have been associated with which of the following vitamins?
A. Vitamin C
B. Vitamin E
C. Vitamin A
D. Biotin
E. Vitamin B12
►E

Vitamin B12 can cause acneiform eruptions.

185
A patient has tender erythematous plaques with fever, arthritis, conjunctivitis and oral ulcers. On
histopathology, there are prominent edema, dermal infiltrate rich with neutrophils with
leukocytoclasis. The most common cancer associated is:
A. Acute myelogenous leukemia
B. Chronic lymphocytic leukemia
C. IgM gammopathy
D. IgG gammopathy

355
E. Chronic myelogenous leukemia
►A

This patient has Sweet's syndrome or Febril neutrophilic dermatosis. The most common cancer
associated is acute myelogenous leukemia. it is also associated with lymphoma and
polycythemia vera. It is also associated with anemia, leukocytosis, neutrophilia, and elevated
ESR.

186
The best diagnosis for this congenital melanopenic lesion without extracutaneous associations is
A. Hypomelanosis of Ito
B. Segmental vitiligo
C. Ash leaf spots
D. Nevus anemicus
E. Nevus achromicus
►E

Nevus achromicus is another name for nevus depigmentosus. It usually presents at birth or
appear during early infancy as normal pigmentation increases. Most individuals will have a
solitary lesion of nevus depigmentosus, but multiple lesions and segmental forms of nevus
depigmentosus have been described. Nevus depigmentosus tends to persist lifelong, but remains
unchanged after onset. The hypopigmented white spots of tuberous sclerosis are most difficult to
distinguish from nevus depigmentosus, but lack of other cutaneous or systemic manifestaions
exclude tubrous sclerosis. Lesions of vitiligo tend to be depigmented (melanocytopenic not
melanopenic as in the question), and show a bright white coloration with Wood's lamp
examination. Nevus anemicus is a distinct vascular birthmark characterized by blanching of
cutaneous blood vessels, hence presenting as a "white" patch of skin that becomes unnoticeable
when the surrounding skin is blanched with a glass slide ("diascopy").

187
A patient presents with cutaneous lesions suggestive of sarcoidosis. You consider ordering a
serum ACE level to help with the diagnosis. What is the sensitivity and specificity of checking
an ace level in this pt?
A. Sensitivity: 60 Specificity: 80
B. Sensitivity: 90 Specificity: 60
C. Sensitivity: 95 Specificity: 80
D. Sensitivity: 60 Specificity: 60
E. Sensitivity: 90 Specificity: 25
►A

356
Serum ACE levels are positive in only approximately 60% of patients with sarcoidosis. However
the specificity is only 80%. Therefore, serum ace levels are less helpful in diagnosis (they can be
helpful in following treatment response.) For example, if your pretest clinical suspicion of
sarcoidosis is 25% (i.e. 1 of 4 likely possible etiologies based on differential diagnosis), then of a
1000 patients tested, 250 will have sarcoidosis. 160 of these will be detected by an abnormal
serum ace level, but 150 of the non-sarcoidosis patients will also have an abnormal ace level
giving you a positive predictive value of just over 50% making it a poor diagnositic test.

188
Which of the following hormones bind the androgen receptor?
A. Dehydroepiandrosterone
B. Androstenedione
C. Dihydrotestosterone
D. Dehydroepiandrosterone and dihydrotestosterone
E. Dehydroepiandrosterone, androstenedione, and dihydrotestosterone
►C

Only testosterone and dihydrotestosterone bind the androgen receptor, thus adrenal androgens
(androstenedione and dehydroepiandrosterone) virilize only in so far as they serve as precursors
for testosterone and dihydrotestosterone.

189
The changes that occur on the nose in patients with rosacea is due to:
A. Sebaceous hyperplasia
B. Sarcoid granulomas
C. Fibrous hyperplasia
D. Acanthosis
E. Tuberculoid granulomas
►A

In patients with rosacea, patients experience rhinophyma that is from the sebaceous hyperplasia.
Initially the nose becomes swollen with prominent pores followed by fibrosis in the later stages.

190
RNP antibodies include:
A. anti-SSA, anti-centromere, anti-SSB
B. anti-SSA, anti-SSB, anti-Sm, anti-U1RNP
C. anti-SSA, anti-SSB, anti-Sm, anti-dsDNA
D. ANA, anti-dsDNA
E. anti-SSA, anti-U1RNP, anti-centromere

357
►B

RNP antibodies target small ribonucleoproteins. These include SS-A (Ro), SS-B (La), Sm, and
U1RNP. The total amount of antibody has more diagnostic value than the mere presence of
antibody.

191
Eosinophilic pustular folliculitis is a noninfectious eosinophilic infiltration of hair follicles. It is
mainly seen in this ethnic group:
A. Japanese men
B. European men
C. Mediterranean men
D. African American men
E. Asian women
►A

Eosinophilic pustular folliculitis is a noninfectious eosinophilic infiltration of hair follicles. The


classic case is mainly from Japanese men. It is also immunosuppression association.

192
Which of the following is characteristic of lichen planus pigmentosus?
A. Lesions are typically hypopigmented
B. Most cases present in Caucasians
C. Oral involvement is pathognomonic
D. Can occasionally see epidermotropic T-cells
E. Trunk is typically spared
►D

Occasionally epidermotropic T-cells are seen in the lichenoid reaction and thus may raise
concern for mycosis fungoides. Lichen planus pigmentosus inversus, in particular, presents in
classic sites of mycosis fungoides, including the axilla, inguinal, and inframammary areas. The
individual lesions of lichen planus pigmentosis are typically smaller, however, than those
encountered in mycosis fungoides, thus helping with the differential. Lesions of lichen planus
pigmentosus are hyperpigmented, with most cases prsenting in skin of color. Oral involvement is
rare and the trunk can be involved.

193
Which of the following is not a common food or environmental allergen implicated in atopic
dermatitis?
A. Dermatophagoides pteronyssimus

358
B. Wheat
C. Corn
D. Eggs
E. Fish
►C

While most AD patients do not have food allergy, food allergens exacerbate AD in at least a
subset of patients, particularly infants and young children. Eggs, milk, peanuts, soybeans, tree
nuts, fish, and wheat are the most common food allergens implicated. Dust mites
(Dermatophagoides pteronyssimus) are among the environmental allergens that may exacerbate
AD.

194
Which of the following is an example of a large vessel vasculitis?
A. Takayasu arteritis
B. Wegener's granulomatosis
C. Chrug-Strauss syndrome
D. Polyarteritis nodosa
E. Henoch-Schonlein purpura
►A

Takayasu arteritis is a large vessel vasculitis that manifests as progressive granulomatous


inflammation of the aorta and its major branches. The systemic vasculitidies are classified into
three categories: large vessel, medium-sized vessel and small vessel vasculitis. Takayasu arteritis
and Giant cell (temporal) arteritis are the two large vessel vasculitidies. Polyarteritis nodosa and
Kawasaki disease are medium-sized vasculitidies. Wegener's granulomatosis, Churg-Strauss
syndrome, microscopic polyangiitis, Henoch-Schonlein purpura and cutaneous leukocytoclastic
vasculitis are examples of small vessel vasculitidies.

195
What mast cell mediators are synthesized upon exposure to a trigger, rather than preformed?
A. tryptase
B. histamine
C. serotonin
D. prostaglandin D2
E. heparin
►D

359
Mast cell mediators can be grouped into two classes: preformed and newly formed. Preformed
mediators include tryptase, histamine, serotonin, and heparin. Newly formed mediators include
prostaglandin D2, leukotriene C4, and platelet activating factor.

196
You are suspicious for a new diagnosis of lupus in a patient recently treated with penicillamine.
Which antibodies would you expect to have been induced by this medication?
A. anti-dsDNA
B. anti-histone
C. anti-SSA
D. anti-SSB
E. anti-Mi-2
►A

Penicillamine induces native systemic lupus erythematosus, associated with anti-dsDNA


antibodies, in contrast to drug-induced lupus erythematosus which has been associated with
exposure to hydralazine, procainamide, sulfonamides, penicillin, anticonvulsants, minocycline,
and INH and is associated with anti-histone antibodies.

197
A 40 year old woman presents to the clinic with multiple pustules in annular and serpiginous
patterns on the abdomen, axillae and groin. Histopathology reveals pustules below the stratum
corneum with many neutrophils and without any acantholysis. What is the diagnosis?
A. Sneddon-Wilkinson disease
B. Reiter's syndrome
C. SAPHO syndrome
D. Transient pustular melanosis
E. Pthirus pubis
►A

Sneddon- Wilkinson disease, also know as subcorneal pustular dermatosis classically presents
with what has been described in this question. SAPHO syndrome is a mnemonic which stands
for synovitis, acne conglobata or fulminans, pustular psoriasis, hyperostosis and osteitis.
Transient pustular melanosis is seen in neonates. Pthirus pubis is pubic lice.

198
What is the most common ocular finding in this patient with this granulomatous disease?
A. Blepharitis
B. Acute anterior uveitis
C. Posterior uveitis

360
D. Keratoconus
E. Cataracts
►B

Sarcoidosis is a systemic disease characterized by the formation of non-caseating granulomas.


Organs involved include the skin, eyes, lungs, liver and spleen. The most common ocular finding
is acute anterior uveitis. Other findings may include posterior uveitis, blurred vision, and
excessive lacrimation.

199
Which technique used for hemostasis may cause iatrogenic tattooing?
A. Monsel’s
B. Aluminium Cloride
C. Gel Foam
D. Cautery
E. Epinephrine
►A

Monsel’s solution is also known as ferrous subsulfate and can be a cause of iatrogenic
tattooing.

200
What is the most common infection that can cause guttate psoriasis?
A. Streptococcus pyogenes
B. Coccidiomycosis
C. Herpes
D. Mycoplasma pneumonia
E. Tuberculosis
►A

Guttate psoriasis refers to a distinctive, acute clinical presentation of an eruption characterized by


small, droplike, 1-10 mm in diameter, salmon-pink papules, usually with a fine scale. It is more
commonly seen in individuals younger than 30 years with a history of upper respiratory infection
that precedes the eruption by 2-3 weeks. The most common organism is group A beta-hemolytic
streptococci (eg, Streptococcus pyogenes). Although recurrent episodes may occur, especially
those due to pharyngeal carriage of streptococci, isolated bouts are known to occur. Guttae
psoriasis may also occur with other infections or in isolation without any identifiable infection.

201
The pigmentation shown here is most likely due to which of the following drugs?

361
A. Chloroquine
B. Amiodarone
C. Minocycline
D. Doxycycline
E. TMP-SMX
►C

This slide shows blue-gray discoloration at the anterior shins and is characteristic of minocycline
hyperpigmentation.

202
A patient presents with hemorrhagic onycholysis. The drug class most commonly associated
with this finding is:
A. Quinolone antibiotics
B. Systemic retinoids
C. Calcineurin inhibitors
D. Taxanes
E. Tetracyclines
►D

Taxane probably cause nail changes more commonly than other drugs. Cutaneous toxicity has
been reported with taxanes and includes erythema and desquamation, involving primarily the
hands. Taxanes exert their cytotoxic effect by reversibly binding the α-subunit of tubulin, thereby
inducing tubulin polymerization and inhibiting microtubule depolymerization. A balance
between polymerization and depolymerization is needed for normal microtubule function.
Taxanes disrupt this balance, leading to arrest at the G2/M phase of the cell cycle.

203
Dermatomyositis is associated with muscle weakness and arthritis. Drug induced
dermatomyositis is most commonly caused by:
A. Hydroxyurea
B. Statins
C. Cyclosporin
D. Cyclophosphamide
E. Tetracycline
►A

Drug induced dermatomyositis is the most common cause is hydroxyurea. Other common causes
are statins, cyclophosphamide, and terbinafine.

362
204
Which autoantibodies are associated with an increased risk of malignancy in dermatomyositis?
A. anti-SRP
B. anti-Mi2
C. Anti-Jo1
D. anti-155/140
E. anti-PL-7
►D

Anti-155/140 has been associated with an increased risk of malignancy in dermatomyositis


patients. Anti-SRP is associated with fulminant dermatomyositis/polymyositis and cardiac
involvement; anti-Mi2 is associated with the shawl sign, periungual telangiectasias, cuticular
overgrowth, and Gottrons papules; anti-Jo1 and anti-PL-7 are associated with antisynthetase
syndrome and interstitial lung disease.

205
The Dunnigan variant of partial lipodystrophy is caused by a mutation in which gene?
A. AGPAT2
B. Seipin
C. Zinc metalloproteinase
D. Neutrophil elastase
E. LMNA
►E

The Dunnigan variant of partial lipodystrophy is characterized by normal appearance at birth


followed by a gradual loss of subcutaneous tissue from the arms and legs around the time of
puberty. The genetic defect involves the gene encoding lamins A and C (LMNA) which are
intermediate filaments integral to the nuclear envelope. The AGPAT2 and seipin genes are
mutated in type I and type II congenital generalized lipodystrophies, respectively. The gene for
zinc metalloproteinase (ZMPSTE24) is mutated in mandibuloacral dysplasia.

206
The drug that is the most common cause for precipitating guttate psoriasis is:
A. Beta blockers
B. Hydrochlorothiazide
C. Tetracyclines
D. Penicillin
E. ACE inhibitors
►A

363
Patients that are on beta blockers can be at risk for precipitating gutate psoriasis. Streptococcus
can also precipitate gutatte psoriasis.

207
Carcinoma of which of the following has been most associated with erythema gyratum repens
A. Lung cancer
B. Breast cancer
C. Colon cancer
D. Prostate cancer
E. Upper GI tract cancer
►A

80% of cases of erythema gyratum repens have been associated with underlying malignancy.
Lung cancer is the most common neoplasm. The skin eruption preceedes the detection of
malignancy by an average of 9 months

208
Macroglossia can present in all of the following disorders except
A. Primary systemic amyloidosis
B. Down's syndrome
C. Mucopolysaccharidoses
D. Cretinism
E. Behchets disease
►E

Macroglossia is not a feature of Behchets disease. Macroglossia, a classic feature that occurs in
about 20% of patients with primary systemic amyloidosis due to deposition of amyloid in the
tongue leading to firm and enlarged tongue. These patients can also have hemorrhagic papules,
plaques and blisters on its surface. Macroglossia is seen in many diseases and syndromes
including Down's syndrome, mucopolysaccharoidoses, cretinism, hypothyroidism, lipoid
proteinosis and Beckwith-Wiederman Syndrome.

209
A patient complained of sudden appearance of multiple papules on chest and upper arms.
Preliminary report by the dermpath fellow are cystic spaces lined by two layers of cuboidal cells
and epithelial strands of similar cells. What is the most likely diagnosis?
A. Syringomas
B. Hydrocystomas
C. Acrospiromas
D. Spiradenomas

364
E. Mixed tumors
►A

The eruptive type of syringomas. In addition to the description above, some of the cysts have
small comma-like tails, which is known as a ―tadpole‖ pattern. Eruptive syringomas are
histologically identical to those on the eyelid, but appear suddenly on the neck, chest, axillae,
upper arms, and periumbilically. It usually occurs in young people. The other options do not
have an eruptive form and tend to occur in different locations of the body other than the arms and
thorax.

210
A 2 year-old boy with crusted skin papules is found to have osteolytic defects and diabetes
insipidus. These features are seen in:
A. Osteogenesis imperfecta
B. Conradi-Hunnerman disease
C. Metastatic small cell lung carcinoma
D. Hand-Schuller-Christian disease
E. Epidermal nevus syndrome
►D

Hand-Schuller-Christian disease is a chronic multifocal form of Langerhans cell


histiocytosis.70% of cases occur between the ages of 2 and 6. The four characteristic clinical
findings are bone lesions, diabetes insipidus, exophthalmus, mucocutaneous lesions. Bones
lesions are osteolytic and preferentially involve the calvarium.

211
A 20 year old woman presents with a sunken appearance to her face and trunk with normal legs.
The patient has attributed her appearance to stress from college, but is seeing you because of her
family's concern. What lab finding would you expect in this patient
A. Decreased C3 nephritic factor
B. Decreased C3
C. Decreased C1
D. Increased C1
E. Increased C4
►B

Acquired partial lipodystrophy (Barraquer-Simmons Syndrome) is characterized by an insidious


progressive loss of fat that usually begins in the face and scalp and progressed downward. Most
patients with this form of lipodystrophy have reduced levels of C3 resulting from circulation

365
polyclonal IgG called "C3 nephritic factor" which results in uncontrolled activation of C3 and
contributes to renal damage.

212
Eosinophilia, asthma, neuropathy and sinus abnormalities are associated with which of the
following systemic vasculitidies?
A. Churg-Strauss syndrome
B. Polyarteritis nodosa
C. Microscopic polyangiitis
D. Wegener's granulomatosis
E. Kawasaki disease
►A

Churg-Strauss syndrome has six diagnostic criteria as set by the American College of
Rheumatology: Blood Eosinophilia, Asthma, Neuropathy, Sinus abnormalities, Allergies and
Perivascular eosinophils. The presence of four of these six criteria yields a diagnostic sensitivity
of 85% and specificity of 99.7%.

213
Cigarette smoking has been shown to:
A. Decrease the incidence of BCC
B. Not affect the survival of surgical repairs
C. Demonstrate less solar elastosis than photoaged skin
D. Have no correlation with facial wrinkling
E. Increase cutaneous vasoconstriction
►E

Cigarette smoking accelerated photoaging. Solar elastosis in patients with significant smoking
history is present not only in the papillary dermis, but also the reticular dermis. It has also been
shown to increase the incidence of skin cancer, decrease the survival of surgical grafts, increase
the severity of wrinkling.

214
A common site for chloracne is the:
A. Occipital scalp
B. Forehead
C. Scrotum
D. Forearms
E. Shoulders
►C

366
Common sites for chloracne include the malar cheek, the post-auricular scalp and in men, the
scrotum.

215
Which of the following HLA types is associated with psoriasis and predicts earlier onset and
more severe disease?
A. HLA-B13
B. HLA-B17
C. HLA-Bw57
D. HLA-Cw6
E. HLA-B27
►B

Psoriasis has HLA associations with all of these HLA types. HLA-B17 is associated with earlier
onset and more serious disease.

216
Which of the following medications is most likely to induce a flare of this patient's disease?
A. ciprofloxacin
B. captopril
C. diphenhydramine
D. sertraline
E. acetominophen
►B

Medications that have been reported to cause flares of psoriasis include antimalarials, beta
blockers, NSAIDS, penicillin, tetracycline, ACE inhibitors, G-CSF, interferons and lithium. In
addition, withdrawal from systemic corticosteroids can induce a flare.

217
The most common internal manifestation of scleroderma is:
A. Esophageal dysmotility
B. Sepsis
C. Pulmonary hypertension
D. Acute renal failure
E. Raynaud's phenomenon
►A

367
Extracutaneous manifestations of scleroderma include esophageal dysmotility, pulmonary
fibrosis with resultant pulmonary hypertension, cardiac involvement with conduction defects,
pericarditis or heart failure, and renal involvement with acute renal failure. Esophageal
dysmotility is the most common systemic manifestation with up to 90% of systemic sclerosis
patients affected. Dysphagia can precede cutaneous involvement thereby giving rise to the term
systemic sclerosis sine scleroderma. Dysmotility is manifested by reduced peristalsis, especially
in the lower two-thirds of the esophagus. This can be demonstrated on a radiologic or
manometric study.

218
The first site in body that shows yellowish pigmentation in carotenoderma is
A. Face
B. Trunk
C. Palms and soles
D. Nails
E. Sclera
►A

Carotenoderma is yellowish discoloration of the skin secondary to carotenemia. Carotene is


excreted by sebaceous glands and in sweat, so the yellow pigmentation appears first on the face
(especially nasolabial folds and forehead) and then becomes diffusely distributed with
accentuation in palms and soles. In contrast to jaundice, carotenoderma spares mucous
membranes and sclera

219
All of the following drugs have been reported to cause rash similar to the attached image except
A. Hydroxyurea
B. D-penicillamine
C. Statins
D. Phenytoin
E. Captopril
►E

The image shows symmetrical erthematous to violaceous plaque on dorsal hands with some
periungual erythema and telangiectasia. Skin biopsy was consistent with interface dermatitis and
mucin. Many drugs have been reported to cause dermatomyositis-like picture, which include
hydroxyurea, D-penicillamine, statins, phenytoin and alfuzosin (alpha antagonist for BPH).
Captopril is not associated with DM-like rash.

220

368
Which auto-antibody correlates with this finding, shawl sign and an overall favorable prognosis
in Dermatomyositis?
A. Anti-Mi-2 antibody
B. Anti-Jo-1 antibody
C. Anti-SRB antibody
D. Anti-KU antibody
E. Anti-PL7 antibody
►A

Anti-Mi-2 antibodies correlate with shawl sign, cuticular changes (as seen in the image) and an
overall favorable prognosis. Anti-Jo-1 antibodies correlate with pulmonary fibrosis, Raynaud's
and polyarthritis. Anti-SRB antibodies correlate with cardiac disease and a poor prognosis. Anti-
KU antibodies correlate with sclerodermatomyositis. Anti-PL7 and Anti-PL12 antibodies (anti-
synthetase antibodies) also correlate with pulmonary disease.

221
Patients of which HLA type are more prone to drug-induced lupus erythematosus when exposed
to hydralazine?
A. HLA-Cw6
B. HLA-B7
C. HLA-DR3
D. HLA-DR4
E. HLA-B6
►D

Drug-induced lupus erythematous has been associated with exposure to hydralazine,


procainamide, sulfonamides, penicillin, anticonvulsants, minocycline, and INH. It generally has
a benign course. The presence of anti-histone antibodies are closely associated with symptomatic
disease. Hydralazine and procainamide are common culprits, with slow acetylators of
hydralazine (HLA-DR4) more prone.

222
Mastocytosis is a systemic disease that causes wheal upon rubbing a lesion called a positive
Darier's sign. Mastocytosis can be measured by examination of serum:
A. Tryptase
B. Kinase
C. Bilirubin
D. Histamine
E. IgE
►A

369
The most common form of cutaneous mastocytosis is urticaria pigmentosa. It is diagnosed by
measuring serum tryptase levels. The new diagnostic method is measurement of tryptase in bone-
marrow which is a new sensitive marker of the mast cell.

223
A patient has palpable purpura, Raynaud and livedo reticularis has been diagnosed with
cryoglobulinemia type I. The immunoglobulins that is associated with cryoglobulinemia type I
is:
A. Monoclonal IgM
B. Polyclonal IgM
C. Monoclonal IgG
D. Polyclonal IgG
E. Monoclonal IgA
►A

Type I cryoglobulinemia is associated with monoclonal IgM. Other associated malignancy are
multiple myeloma and Waldenstroms macroglobulinemia. The lab findings are elevated
cryoglobulins and on pathology there are eosinophilic intravascular deposits.

224
Which of the following is a function of Propionibacterium acnes (P. acnes) in contributing to the
pathogenesis of acne?
A. Downregulation of TLR-2 expression
B. Inhibition of complement
C. Activation of TLR-2
D. Inhibition of Il-1
E. Reduction in free fatty acid levels
►C

P. acnes flourishes in the presence of sebum and makes an enzymatic lipase which cleaves
triglycerides into free fatty acids. Free fatty acids serve as an indirect measure of P. acnes
populations on the skin. P. acnes itself is also pro-inflammatory; it can activate complement, as
well as neutrophil chemotaxis and activity. Il-1, a pro-inflammatory cytokine, may promote
follicular plugging and microcomedo formation. TLR-2 activation by P. acnes leads to
stimulation of pro-inflammatory cytokines.

225
The treatment of choice for imaging a ateriovenous malformation is:
A. Arteriography

370
B. Doppler
C. Annual Xray
D. Multidetetor Xray
E. 3D MRI venography
►A

Arteriography is the technique of choice to evaluate arteriovenous malformations. Doppler


should be done at age 1 and annual xray after the age of 2. Multi-detector CT and fast 3D MRI
venography can also be done.

226
Which of the following statements comparing inflammatory lesions from patients with psoriasis
and patients with atopic dermatitis is true?
A. Human beta-defensin 2 (HBD-2) and LL 37 levels are significantly lower in
psoriasis lesions compared to atopic dermatitis lesions
B. Expression of HBD-2 and LL 37 is negligible in psoriatic lesions
C. HBD-2 and LL 37 are significantly elevated in atopic dermatitis lesions
D. Expression of HBD-2 and LL 37 in psoriasis is similar to normal skin, whereas
it is decreased in atopic dermatitis lesions
E. HBD-2 and LL 37 are significantly lower in atopic dermatitis lesions compared
to psoriasis lesions
►E

Ong et al found a deficiency of HBD-2 and LL 37, which are innate antimicrobial peptides, in
lesions from patients with atopic dermatitis compared to lesions from patients with psoriasis. IL-
4 and IL-13, which are Th2-type cytokines, were found to inhibit expression of HBD-2 and LL
37. Decreased expression of HBD-2 and LL 37 may account for the increased susceptibility to S.
aureus infection in patients with atopic dermatitis.

227
A patient presents to you with purpura after minor trauma. In addition, macroglossia with teeth
indentations. Which amyloid protein is associated with her condition?
A. AB2M
B. AA
C. AL
D. Altered keratinAB
E. AB
►C

371
AL protein is associated with primary systemic amyloid. AA is associated with secondary
systemic amyloid. AB2M is associated with hemodialysis. AB is associated with alzheimer‘s
disease. Altered keratin is associated with lichen and macular amyloid.

228
You prescribe oral erythromycin to a 35 year-old woman. Co-administration of which of the
following medications could lead to potential adverse outcomes?
A. Oral contraceptives
B. Warfarin
C. Carbamazepine
D. Methylprednisolone
E. Warfarin, carbamazepine, or methylprednisolone
►E

Erythromycin inhibits the hepatic cytochrome P450 system and can increase serum levels and
potential toxicities of carbamazepine, theophylline, warfarin, digoxin, and methylprednisolone.

229
Which special stain is utilized to confirm the diagnosis of Langerhans cell histiocytosis?
A. H&E sufficient for diagnosis
B. C-kit
C. CD20
D. CD1a
E. Chloroacetate esterase
►D

Langerhan cell histiocytosis includes the diseases Letterer-Siwe, Hand-Schuller-Christian,


Eosinophilic Granuloma, and Hashimoto-Pritzker. Cells stain positively with CD1a, S100,
langerin, and peanut agglutinin. Cells may also be indentified by the presence of Birbeck
granules.

230
Which of the following statements is true regarding Morbihan's Disease?
A. It is often misdiagnosed as cellulitis
B. It presents with blepharitis, conjunctivitis, iritis, and keratitis
C. Histopathology reveals perifollicular and perivascular noncaseating epithelioid
granulomas
D. It occurs around the mouth and/or nose and eyes and may be triggered by topical
steroid use

372
E. It presents with large coalescent nodules and confluent draining sinuses
occupying most of the face
►A

Morbihan's Disease, also known as rosacea lymphedema or persistent edema of rosacea, presents
with hard, nonpitting edema. It is often misdiagnosed as cellulitis.

231
Which of the following medications is most likely to induce or exacerbate psoriasis in your
patient.
A. Hydrochlorothiazide
B. Lisinopril
C. Amlodipine
D. Metoprolol
E. Diltiazem
►D

Several drugs have been incriminated as inducers of psoriasis, in particular lithium, B-Blockers,
antimalarials, and interferon. More recent additions include terbinafine, calcium channel
blockers (nicardipine, nifedipine, nisoldipine, verapamil, and diltiazem), captopril, glyburide,
and lipi-lowering drugs such as gemfibrozil.

232
Topical calcineurin inhibitors may offer several benefits over topical steroids in treating the
scaly, lichenified periorbital plaques in this patient with atopic dermatitis, including:
A. More cost effective
B. Quicker onset of action
C. Decreased irritant potential
D. Absence of reports of cutaneous atrophy
E. Once daily application
►D

Tacrolimus and pimecrolimus are non-steroid anti-inflammatory drugs and are not associated
with side effects reported with use of topical corticosteroids, including atrophy, striae formation,
telangiectasias, cataracts, and HPA axis suppression.

233
A patient has multiple skin colored papule that when lanced, express clear fluid. On histology,
the cyst is lined by eosinophilic cuticle with a thin squamous epithelium and mature sebaceous
glands attached to the wall. The most likely defect is:

373
A. Keratin 17
B. Keratin 1
C. Keratin 5
D. Keratin 7
E. Keratin 20
►A

The patient has steatocystoma multiplex that is linked to the defect in keratin 17, 10. The lesions
usually occur in the second and third decades of life. It also occurs sporadically and is inherited
in the benign form.

234
A pregnant woman presents with the following condition which is bothersome to her. Which of
the following is the most appropriate treatment option based on FDA classifications of
medication in pregnancy?
A. Topical erythromycin/benzoyl peroxide gel
B. Topical tretinoin
C. Tazarotene 0.1% cream
D. Azelaic acid 20% cream
E. Bactrim DS
►D

This patient has mild-moderate inflammatory acne. Azelaic acid is the only medication listed
which falls under category B. The others listed are category C, except for tazarotene, which is
category X

235
Most often, xanthelasma is associated with:
A. An IgM paraprotein
B. An IgG paraprotein
C. No associated disease
D. Type I hyperlipoproteinemia
E. Type III hyperlipoproteinemia
►C

Xanthelasma are the most common type of xanthoma. They are usually present without any other
disease, but may occur concomitantly with other xanthomas, and can occur in types II and III
hyperlipoproteinemias (familial hypercholesterolemia, common hypercholesterolemia, and
familial dysbetalipoproteinemia). They are also common among women with biliary or hepatic
disoders, and are also seen in myxedema, diabetes, and phytosterolemia.

374
236
A 55 year-old male presents with a indurated yellow-brown plaque near the eye. Work-up
reveals:
A. IgG monoclonal gammopathy
B. IgG polyclonal gammopathy
C. IgA monoclonal gammopathy
D. IgA polyclonal gammopathy
E. no paraproteinemia seen in 50% of cases
►A

IgG monoclonal gammopathy is seen in over 80% of cases of necrobiotic xanthogranuloma.


Pathology reveals palisading granulomas with necrobiosis and cholesterol clefts. Recurrence
rates as high as 42% have been reported after surgical excision.

237
Which of the following cytokines is most characteristic of atopic dermatitis in its chronic phase?
A. Interferon-gamma
B. IL-4
C. IL-5
D. IL-10
E. IL-13
►A

While acute atopic dermatitis is a TH2 state, chronic eczematous AD is most often a TH1
cytokine environment with interferon-gamma being the best choice. Initial lesions TH2
dominant, cytokines IL-4,5,10,13. IL-4/5 produce elevated IgE and eosinophilia, IL-10 inhibits
delayed-type hypersensitivity. IL-4 downregulates IFN-gamma.

238
Lacrimal gland involvement is characteristically seen in patients with:
A. Lymphoma
B. Sarcoid
C. Rosacea
D. Syphilis
E. Pemphigus
►B

375
Sarcoid is granulomatous autoimmune condition characterized by the formation of non-caseating
granulomas which may affect any organ system. Lacrimal gland involvement occurs in 15-28%
of patients. Manifestation of lacrimal gland involvement includes bilateral painless swelling.

239
This 35 year-old man presents with the lesions shown. He was treated elsewhere for a different
skin condition. Biopsy of these lesions is likely to show:
A. Increased staining on Fontana Masson but not Perls stain
B. Increased staining on Perls stain but not Fontana Masson
C. Increased staining on both Fontana Masson and Perls stain
D. Increased melanin at the basal layer and within macrophages only
E. Fibrosis and increased mucin deposition
►C

The patient has Type 2 minocycline-associated hyperpigmentation. Three types of minocycline-


associated hyperpigmentation are generally described. The first is blue-black discoloration
appearing in areas of prior skin injury, such as acne scars. The second type is a blue-gray
discoloration, often on the lower anterior legs and forearms. The third type is the least common,
and is characterized by muddy brown discoloration of sun-exposed areas. The first two types
show staining for both iron and melanin (Fontana Masson stains melanin black; Perls stains iron
[hemosiderin] blue). The third type shows increased melanin at the basal layer and within
macrophages.

240
The National Psoriasis Foundation Consensus recommends that patients with latent TB should be
treated with TB within:
A. 1-2 months of starting anti-TNF therapy
B. 3-4 months of starting anti-TNF therapy
C. 5-6 months of starting anti-TNF therapy
D. 6-8 months of starting anti-TNF therapy
E. 8-12 months of starting anti-TNF therapy
►A

The National Psoriasis Foundation Consensus states that patients with a positive TB. Treat latent
TB prior to starting therapy. After 1-2 months of appropriate TB treatment, psoriasis therapy
may begin is necessary.

241

376
This woman is being treated for corticosteroid-induced rosacea with topical metronidazole and
an oral tetracycline. She is very concerned about the redness of her face. Judicious use of what
color concealer can reduce the appearance of redness on the skin?
A. Yellow
B. Lavender
C. Pink
D. Bronze
E. Green
►E

This slide shows a woman with rosacea. Green and red are on opposite sides of the color wheel
and thus can ―cancel‖ each other out. Green concealer can neutralize redness on the skin.

242
Menopausal flushing can be effectively treated with:
A. Nadolol
B. Tryptophan
C. Clonidine
D. Danazol
E. Tamoxifen
►C

Menopausal flushing occurs at menopause or perimenstrual when estrogen levels are low.
Pharmacologic menopause may be caused by drugs, including danazol, tamoxifen, clomiphene
citrate, decapeptyl, leuprolide, and 4-hydroxyandrostenedione. Treatment is with oral estrogen
replacement or clonidine hydrochloride 0.05mg bid. Nadolol is an effective treatment of
emotional flushing.

243
Mycosis fungoides is characterized with atypical lymphocytes in the epidermis. The marker for
more advanced mycosis fungoides is:
A. CA27.29
B. CA30.32
C. CA40.42
D. CA50.52
E. CA55.57
►A

377
Mycosis fungoides is the most common form of cutaneous T cell lymphoma. It can look like
chronic eczema and can be histologically very difficult to distinguish from mycosis fungoides.
CA27.29 may be a marker for advanced mycosis fungoides.

244
A child has a cutaneous ossification disorder characterized by endochondral bone formation,
deafness, baldness, and mental retardation. Which form of osteoma cutis does this patient have?
A. Fibrodysplasia ossificans progressiva
B. Progressive osseous heteroplasia
C. Plate-like osteoma cutis
D. Albright's hereditary osteodystrophy
E. Calcinosis cutis
►A

This patient has fibrodysplasia ossificans progressiva characterized by endochondral bone


formation (the other types have intramembranous bone formation), noggin gene defects and
other systemic symptoms as listed. This is a progressive and potentially fatal condition.
Progressive osseous heteroplasia is also progressive, seen in mostly females, and demonstrates
increased alkaline phosphatase, LDH and CPK with normal calcium, phosphate, and PTH. Plate-
like osteoma cutis is limited and seen in children and newborns. Albright's hereditary
osteodystrophy is due to a mutation in GNAS-1. This disorder is characterized by a lack of
responsiveness to parathyroid hormone, resulting in low serum calcium, high serum phosphate,
and appropriately high serum parathyroid hormone. Individuals with Albright's hereditary
osteodystrophy have short stature, characteristically shortened fourth and fifth metacarpals,
rounded facies, and often mild mental retardation. Calcinosis cutis is not a form of osteoma cutis.

245
Which of the following drugs is bactericidal?
A. Clindamycin
B. Tetracycline
C. Trimethoprim
D. Penicillin
E. Chloramphenicol
►D

Other bactericidal drugs include bacitracin, monobactams, quinolones, vancomycin, and


polymyxin B. Clindamycin, tetracycline, trimethoprim, and chloramphenicol and bacteriostatic.

246

378
A healthy 50 year old man presents to your office with 3 red-brown indurated plaques on his
forehead. Biopsy reveals a focal leukocytoclastic vasculitis, with a diffuse infiltration of
neutrophils, eosinophils, and lymphocytes. Which of the following is a potential treatment option
for this patient?
A. Q-switched alexandrite laser
B. Pulse dye laser
C. Q-switched ruby laser
D. Nd:YAG laser
E. Frequency-doubled Nd:YAG laser
►B

The question stem describes a case of granuloma faciale. This condition presents with red-brown
papules and plaques on the face, most commonly in middle-aged Caucasian males. While
traditional treatment options include intralesional steroids with or without cryotherapy, pulse dye
laser (595 nm) represents another treatment option for this condition, with successful clearance
in a number of case reports.

247
Patients with psoriasis treated with cyclosporine should be monitored for:
A. Sicca symptoms
B. Hypermagnesemia
C. Alopecia
D. Acute interstitial pneumonitis
E. Hyperkalemia
►E

Well-documented adverse effects and toxicities of cyclosporine include renal impairment,


hypertension, elevated triglycerides, hyperkalemia, hypomagnesemia, hepatotoxicity,
hypertrichosis, and long-term increased risk of malignancy.

248
The constellation of erythema nodosum, bilateral hilar lymphadenopathy, uveitis, fever, and
arthritis in patients with sarcoidosis is known as:
A. Loeffler's syndrome
B. Schnitzler's syndrome
C. Lofgren's syndrome
D. Heerfordt's syndrome
E. Mikulicz's syndrome
►C

379
The presence of EN, bilateral hilar lymphadenopathy, uveitis, fever, and arthritis in patients with
sarcoidosis is known as Lofgren's syndrome. Heerfordt's syndrome, also known as uveoparotid
fever, frequently occurs in patients with sarcoidosis of the central nervous syndrome. It consists
of uveitis, facial nerve palsy, fever, and parotid gland involvement. Milulicz's syndrome is
bilateral sarcoidosis of the parotid, submandibular, sublingual, and lacrimal glands. Loeffler's
syndrome is characterized by a patchy infiltrate in the lungs and eosinophilia of the blood and
sputum. Loeffler's syndrome can be with creeping eruption (larva migrans). Schnitzler's
syndrome is a rare disorder of chronic urticaria, fever, disabling bone pain, hyperostosis,
increased erythrocyte sedimentation rate, and monoclonal IgM gammopathy. Pruritus is not a
feature of Schnitzler syndrome.

249
A patient has gout found on the helices of the ear diagnosed by the dermatopathologist. Fixation
in this allows visualization of urate crystals:
A. Carnoy's fluid
B. Formaldehyde
C. Michele's media
D. Normal saline
E. Water
►A

Absolute ethanol or Carnoy's fluid allows visualization of urate crystals. The crystals are brown
and refractile with polarized light.

250
What characteristic body would be found on histopathology of this disease?
A. Kamino
B. Civatte
C. Henderson-Patterson
D. Donovan
E. Dutcher
►B

Hypertrophic lichen planus is a type of lichen planus characterized by hyperkeratosis. They are
usually pruritic and often found on the extremities. Civatte bodies are degenerated keratinocytes
often seen in lichen planus.

251
First line treatment of chromoblastomycosis includes surgery and a medication with which
mechanism of action?

380
A. Inhibits squalene epoxidase
B. Inhibits 14-alpha-demethylase
C. Disrupts microtubule mitotic spindle formation
D. Inhibits sythesis of beta-1,3-diglucan and disrupts cell walls
E. Inhibits fungal cytochrome P-450 mediated 14 alpha-lanosterol demethylation
►B

The correct answer is surgery and itraconazole, which inhibits 14-alpha-demethylase.


Terbinafine inhibits squalene epoxidase. Griseofulvin disrupts microtubule mitotic spindle
formation. Caspofungin inhibits synthesis of beta-1,3-diglucan. Foscarnet inhibits fungal
cytochrome P-450 mediated 14 alpha-lanosterol demethylation.

252
Immunohistochemistry of this disease would likely show:
A. Perivascular IgA
B. IgM staining of colloid bodies
C. Linear band of IgA at the basement membrane
D. IgM intercellularly
E. Granular C3 at the basement membrane
►B

Colloid bodies are apoptotic cell remnants in the papillary dermis. In lichen planus, the colloid
bodies will often stain positively with IgM.

253
A pregnant woman in her third trimester presents with nonpruritic erythematous plaques and
pustules in the intertriginous regions, the trunk, and the extremities as well as systemic
symptoms of malaise and fever. Which complication is most associated with this condition?
A. Premature labor
B. Placental insufficiency
C. No risk to fetus or mother
D. Small for gestational age
E. Microcephaly
►B

This patient has pustular psoriasis of pregnancy which is also called "impetigo herpetiformis." It
is characterized by red plaques with a peripheral ring of pustules that are distributed
symmetrically in flexural areas and trunk and extremities. Patients can have elevated ESR and
leukocytosis as well as hypocalcemia. There is a risk of fetal morbidity and mortality secondary
to placental insufficiency and maternal mortality secondary to cardiac or renal failure. Premature

381
labor may be associated with cholestasis or pregnancy. Patients with herpes gestationis may have
a risk of premature labor and small for gestational age. Microcephaly may occur with isotretinoin
taken during pregnancy.

254
Patients that are on hemodialysis can develop amyloidosis due to altered uremia. Dialysis related
amyloidosis has this kind of fibril protein:
A. Beta 2-microglobulin
B. AL
C. AA
D. Altered keratin
E. B amyloid protein
►A

Dialysis related amyloidosis is due to beta 2-microglobulin. The amyloid fibril is altered by
uremia and causes carpal tunnel, bone cyst, and spondyloarthropathy.

255
What is the eponym used for this inherited, congenital condition?
A. Meige disease
B. Milroy disease
C. Proteus syndrome
D. Klippel-Trenaunay syndrome
E. Stuart-Treves syndrome
►B

Milroy disease in an autosomal dominantly inherited form of primary lymphedema. Unlike


Meige disease, Milroy disease is congenital and due to a failure of the development of lymphatic
vessels in utero.

256
A 52 year-old man presents with large comedones as well as inflammatory papules, pustules, and
cysts on the malar cheeks, postauricular scalp, and scrotum. Which of the following could be a
cause of this presentation?
A. Erlotinib chemotherapy
B. A pituitary adenoma
C. 2,3,7,8 tetrachlorobenzodioxin exposure
D. PTEN mutations
E. Isotretinoin overdose
►C

382
This patient has a presentation consistent with industrial acne. Dioxin (2,3,7,8
tetrachlorobenzodioxin) is a well-known, potent trigger of this acneiform eruption. Overall,
insoluble cutting oils are the most frequent cause of industrial acne.

257
What is the major constituent of sebaceous gland lipid?
A. Triglyceride
B. Wax esters
C. Squalene
D. Free cholesterol
E. Cholesterol esters
►A

Triglyceride is the major constituent of sebaceous lipid, accounting for over 50% of the lipid.
Wax esters constitute about 25% of sebaceous lipid and Squalene accounts for about 15%. The
remainder is free cholesterol and cholesterol esters.

258
Patients that have vascular malformations can experience localized intravascular coagulation.
One of the risk factors are:
A. Muscle involvement
B. Gorham-Stout syndrome
C. Smaller surface area
D. Skeletal hypertrophy
E. Grouped superficial vesicles and hyperkeratotitc plaques
►A

Patients that have vascular malformations can experience localized intravascular coagulation.
Less then 85% will experience this with segmental pattern. The risk factors are large surface
areas, muscle involvement, palpable pheboliths. The treatment is low molecular weight heparin.

259
Which cellular organelle is affected in the partial lipodystrophies?
A. Nucleolus
B. Rough endoplasmic reticulum
C. Golgi apparatus
D. Cell membrane
E. Nuclear lamina
►E

383
Both the congenital (Dunnigan - Type 1) and acquired (Barraquer-Simons) lipodystrophies are
caused by mutations involving the nuclear lamina of the cell. Dunnigan lipodystrophy is
characterized by a mutation in LMNA and Barraquer-Simons is caused by a LMNB2 mutation.

260
Which of the following is a manifestation of psoriasis of the nail matrix?
A. Splinter hemorrhages
B. Oil spots
C. Subungal hyperkeratosis
D. Onycholysis
E. Pits
►E

Psoriatic nail changes may be of nail matrix or bed in origen. Pits are the most common findings,
while splinter hemorrhages are the least. Psoriatic nail changes involving the matrix include pits
(representing focal psoriasis of the proximal nail matrix) and leukonychia.

261
Acute hemorrhagic edema of childhood is distinguished from Henoch-Schonlein Purpura based
on:
A. The presence of pupura on the upper trunk
B. The lack of an antecedent infection
C. The involvement of the synovia
D. The neurologic complications
E. The lack of systemic features
►E

Acute hemorrhagic edema of childhood affects children and infants < 2 years of age. It presents
with painful, edematous petechiae and ecchymoses on the head and distal extremities. Facial
edema may be the initial sign. Triggering factors include infection, drugs, and immunization. It
lacks the systemic features of HSP, and resolves in 1-3 weeks without sequelae. HSP occurs
mostly in children. There is an antecedent URI in 75% of cases. HSP involves the skin, synovia,
GI tract, and kidneys. Long-term morbidity results from renal disease, which is predicted by the
spread of purpura to the upper trunk.

262
A patient begins to lose the fat in her face and upper torso acutely after a viral illness. Which
internal organ may be affected by this disease?
A. Lungs

384
B. CNS
C. Bone marrow
D. Kidney
E. Heart
►D

Barraquer-Simons syndrome, acquired partial lipodystrophy, presents typically in females and is


secondary to a mutation in LMNB2. It is often preceded by a viral illness. More than one third of
patients will have glomerulonephritis, which may lead to chronic renal sequelae.

263
What is the recommended allowance of daily vitamin D intake for a 35-year-old female?
A. 200 IU
B. 400 IU
C. 600 IU
D. 800 IU
E. 1000 IU
►C

Recommended dietary allowance of vitamin D as established by the Institute of Medicine in


2010 is 400 international units for infants 0 to 12 months, 600 IU for ages 1 to 70 years, and 800
IU for ages 71 and above. Additionally, recommended allowance for pregnant/lactating women
is 600 IU.

264
Acute intermittent porphyria has an enzyme defect:
A. Porphobilinogen deaminase
B. Uroporphyrinogen III synthase
C. Coproporphyrinogen oxidase
D. Uroporphyrinogen decarboxylase
E. Protoporphyrinogen oxidase
►A

Acute intermittent porphyria has a defect in porphobilinogen deaminase. Acute intermittent


porphyria (AIP) is a rare autosomal dominant metabolic disorder affecting the production of
heme, the oxygen-binding prosthetic group of hemoglobin.

265
Which of the following viruses is associated the most with lichen planus?
A. Hepatitis C virus

385
B. Hepatitis B virus
C. Human immunodeficiency virus
D. Hepatitis A virus
E. Epstein-barr virus
►A

Lichen planus classically involves the wrists, ankles, and oral mucosa. It is characterized by
violaceous polygonal, purple, pruritic papules. It has a strong association with hepatitis C virus.
Many other associations exist, including other viruses and medications, but hepatitis C is the best
choice.

266
A 22 year-old woman notes irregular menses and acne that has been refractory to several over-
the-counter and prescription regimens. Appropriate initial work-up includes
A. Serum prolactin levels
B. 24-hour urine cortisol
C. Serum free and total testosterone, LH, FSH, androstenedione
D. Serum free and total testosterone, LH, FSH, DHEA-S
E. Serum free and total testosterone, LH, FSH, 17-OH, hydroxylase
►D

The history and clinical appearance of this patient suggest a possible hormonal basis to her acne.
The work-up in answer d assesses for adrenal hormonal production and screens for PCOS.
Androstenedione is of ovarian origin.

267
What is the treatment of choice for this patient who developed acne fulminans one month after
starting isotretinoin therapy?
A. Add oral prednisone
B. Increase isotretinoin dose
C. Add doxycycline
D. Acne surgery
E. Check lipid levels
►A

Acne fulminans is an explosive form of acne in which patients may develop systemic symptoms,
ulcerated nodules on the face and trunk. Acne fulminans may be triggered by initiation of
isotretinoin therapy. The treatment of choice is systemic steroids at a dosage of 1 mg/kg/day
tapered over the course of 6 weeks.

386
268
A patient with a recent diagnosis of tuberculosis on treatment presents with a new photosensitive
eruption on his face, neck and upper chest. In addition, he has progressive diarrhea and
depression. What is the etiology of his symptoms?
A. Vitamin B12 deficiency
B. Drug induced pellagra
C. Drug induced riboflavin deficiency
D. Carcinoid syndrome
E. Photoallergic contact dermatitis
►B

This patient has pellagra secondary to isoniazid treatment. He has the photosensitive eruption
and €"Casal'€™s necklace"€•, in addition to diarrhea and depression. Other symptoms
include the 3 D€'s: dermatitis, diarrhea, dementia. Other potential medications that may cause
this constellations of symptoms include azathioprine and 5-FU.

269
Pyoderma gangrenosum is most commonly reported with which malignancy?
A. Leukemia
B. Medullary thyroid carcinoma
C. Lung carcinoma
D. Breast carcinoma
E. Prostate carcinoma
►A

Pyoderma gangrenosum, (PG), is an uncommon, ulcerative inflammatory skin condition


characterized by boggy ulcerations with undermined borders preceded by the breakdown of a
painful nodule or pustule. These lesions characteristically enlarge progressively over time and
display marked tenderness. The diagnosis of PG is a diagnosis of exclusion and infectious
etiologies should be excluded. Pyoderma gangrenosum can arise in the absence of an underlying
disorder or it can be seen in association with underlying systemic conditions. These conditions
include inflammatory bowel disease such as ulcerative colitis and Crohn's disease, polyarthritis,
chronic active hepatitis and Behcet's disease. Pyoderma gangrenosum has also been associated
with an underlying paraproteinemia, mostly of the IgA type, although IgM and IgG types have
also been described. In addition, some patients have myeloma at presentation or develop it
subsequent to their diagnosis of PG. Pyoderma gangrenosum has also been described in
association with myelodysplasia, and in acute myeloblastic, myelomonocytic, and chronic
myeloid leukemia. Treatment of pyoderma gangrenosum should include therapy directed at the
underlying systemic disorder.

387
270
A patient has a rash from one of her cosmetic products. The most common cosmetic preservative
to cause contact dermatitis is:
A. Quaternium 15
B. Thimerosal
C. Balsam of Peru
D. Diethyl methylene
E. Benzene
►A

The most common allergen in cosmetic products is quaternium 15. It is a preservative in personal
care products and cosmetics.

271
Treatment of Raynaud's phenomenon includes:
A. Nifedipine
B. Metoprolol
C. UVB
D. Tetracyline
E. Simvastatin
►A

Treatment of Raynaud's includes calcium channel blockers such as nifedipine 30-6-mg/d,


antiplatelet aggregation drugs (such as aspirin or dipyridamole), pentoxyphilline 400mg BID-
TID and D-penicillamine. Losartan 50mg/d may reduce frequency and severity of Raynaud's.

272
A patient with renal amyloidosis, urticaria, fevers, limb pains, and deafness with familial
Mediterranean fever has:
A. Muckle-Wells
B. MEN IIa
C. MEN IIIa
D. FAP I
E. FAP III
►A

Patients with Muckle-Wells has amyloid AA fibrils. It is an autosomal dominant condition


characterized by deafness, hives, and mutation in the CIAS1 gene.

273

388
A patient has non-tender reddish brown nodular lesions located on the dorsal hands and nail
folds. Patients also have symmetric arthritis of interphalangeal and temporomandibular joints and
is diagnosed with multicentric reticulohistiocytosis. The % of patient that develop malignancy
is:
A. 10%
B. 20%
C. 30%
D. 40%
E. 50%
►B

20% of patients that are diagnosed with multicentric reticulohistiocytosis have a risk of
developing a malignancy but there is no predominant type. Pathology shows nodular infiltrate
composed of multinucleated oncocytic giant cells with eosinophilic cytoplasm.

274
A patient with eruptive vellus hairs seen in steatocystoma multiplex and pachyonychia congenita
type 2 has a defect in :
A. Keratin 17
B. Keratin 18
C. Keratin 19
D. Keratin 20
E. Keratin 21
►A

A patient with this condition has a defect in keratin 17. These conditions can be seen together
since they share the same defect in the genotype. Both pachyonychia congenital type 2 and
steatocystoma multiplex have a defect in keratin 17.

275
A patient has shiny, firm translucent papules around the mouth, nose and eyes. There is also
pinch purpura and glossitis. The patient has amyloidosis has an association with:
A. Multiple myeloma
B. T cell Lymphoma
C. B cell Lymphoma
D. Leukemia
E. Chronic lymphocytic leukemia
►A

389
Patients with amyloidosis has an association malignancy with multiple myeloma and plasma cell
dyscrasia. The pathology is eosinophilic, amorphic fissured globules in the dermis. The amyloid
is composed of immunoglobulin lambda light chains.

276
Which of the following is true regarding this diagnosis?
A. The protein deposited is derived from Ig light chains, kappa subtype
B. Skin is involved in <10% of cases
C. A different protein is found in skin lesions associated with a plasmacytoma
D. Bullae, when present, are subepidermal
E. Factor V deficiencies are commonly associated with this entity
►D

Primary sytemic amyloidosis involves the skin in 40% of cases. The tongue, heart, and GI tract
are commonly involved. The protein AL is derived from Ig light chains (lambda subtype); AL is
also found in nodular or tumefactive cutaneous amyloidosis produced by a plasmacytoma.
Glossitis is common, and may lead to dysphagia; the lateral aspects of the tongue often show
indentations from teeth. Purpuric lesions result from amyloid infiltration of blood vessels, and
occur after trauma (pinch purpura). Bullous amyloidosis presents with tense, hemorrhagic bullae
at areas of trauma; lesions are subepidermal. Arthropathies of small joints, enlarged deltoids
(shoulder pad sign), factor IX and X deficiencies, cardiac arrythmias, and CHF may all result.

277
The primary component of human sebum is:
A. Triglycerides
B. Free fatty acids
C. Ceramides
D. Cholesterol
E. Squalene
►A

Human sebum is rich in triglycerides. The lipase made by P. acnes cleaves triglycerides to free
fatty acids.

278
What is the most likely diagnosis?
A. Voight line
B. Linea nigra
C. Fuchter line
D. Phytophotodermatitis

390
E. Incontinentia pigmenti
►B

Linea nigra is a hyperpigmented, linear patch that often becomes apparent during pregnancy. It is
thought to be secondary to hormonal fluctuations and usually spontaneously resolves after
pregnancy. Voight and Fuchter lines delineate a vestigial hyperpigmentation of the dorsal aspect
of organisms.

279
Patients with Dermatitis Herpetiformis are most likely to have:
A. Antibodies to BPAg2
B. Antibodies to transglutaminase 3
C. Mutations in transglutaminase I
D. Mutations in laminin 5
E. Mutations in plectin
►B

In dermatitis herpetiformis, antibodies are found to transglutaminase 3, and the direct


immunofluorescent studies show granular IgA and C3 in the dermal papillae. Antibodies to
BPag2 are found in bullous pemphigoid. Mutations in plectin are found in EBS with muscular
dystrophy. Mutations in laminin 5 are found in patients with JEB,Herlitz type. Mutations in
transglutaminase I are found in pateints with lamellar ichthyosis and non bullous congenital
ichthyosiform erythroderma.

280
Phrynoderma can be seen in all of the following nutritional deficiency except
A. Vitamin A
B. Vitamin B
C. Vitamin C
D. Vitamin D
E. Vitamin E
►D

Phrynoderma or ―toad skin‖• is typically associated with vitamin A deficiency. These keratotic
follicular papules often first develop on anterolateral thighs and posterolateral upper arms then
spread to extremities, shoulders, abdomen and back. Although phrynoderma is originally
reported in association with vitamin A deficiency, it can also be observed with defeciencies in B-
complex vitamins and vitamins C and E, in addition to essential fatty acid deficiency.

281

391
The first step in the formation of a closed comedone is:
A. Obstruction of the pilosebaceous duct
B. Increased sebum production
C. Colonization by P. acnes
D. Increased hormonal stimulation of the pilosebaceous unit
E. Clogging of the pore with make-up
►A

The first step in the formation of a microcomedone is the obstruction of the pilosebaceous unit
by keratinocytes. After that has occured, increased sebum and P. acnes can contribute to the
further formation of the comedone.

282
Sturge-Weber syndrome is a vascular malformation. It can occur in the first year of life and more
common in patients with bilateral port wine stains. The most common neurologic abnormality
is:
A. Seizures
B. Migraines
C. Glaucoma
D. Tram track calcification
E. Tunnel vision
►A

Patients with Sturge-Weber have seizures as a common neurologic abnormality. They can have
glaucoma in 2/3 of patients at birth. Tram track calcifications don't appear on XR before two
years old.

283
This syndrome is characterized with fever, erythema nodosum, bilateral hilar adenopathy, and
arthralgias:
A. Lofgren syndrom
B. Heerfordt syndrome
C. Richner Hanhart syndrome
D. Wilson's syndrme
E. Carney syndrome
►A

Lofgren syndrome is characterized by fever, erythema nodosum, bilateral hilar adenopathy, and
arthralgias. It is associated with anterior uveitis in 6% of patients.

392
284
A 25 year-old previously healthy man presents with the skin findings shown, urethritis, and one
month of peripheral arthritis. Which of the following is true regarding this condition?
A. Females and males are equally affected
B. A chronic deforming arthritis occurs in 20%
C. TNF-alpha inhibiting agents have no role in the treatment of this condition
D. Patients must have urethritis, conjunctivitis, and arthritis for diagnosis
E. Chlamydia cervicitis is not associated with this condition
►B

The patient has Reiter's syndrome. Reiter's syndrome is a chronic inflammatory disease similar
to psoriasis with psoriatic arthritis, and is thought to be a variant form. The classic triad consists
of urethritis, conjunctivitis, and arthritis. Few patients present with the classic triad, and thus the
syndrome can be diagnosed with peripheral arthritis >1 month duration and associated urethritis
(or cervicitis). It occurs in young men with the HLA-B27 genotype and rarely occurs in women.
Skin findings include keratoderma blennorrhagicum and circinate balanitis (in men), as well as
oral erosions, severe stomatitis, and nail changes. The course of disease is marked by
exacerbation and remission; a chronic deforming arthritis occurs in 20%. Treatment includes
topical steroids, NSAIDs, methotrexate, acitretin, cyclosporine, and TNF-inhibiting biologics,
such as etanercept.

285
What is the most photosensitive form of all cutaneous lupus subtypes?
A. Acute cutaneous lupus
B. Subacute cutaneous lupus
C. Classic discoid lupus
D. Lupus tumidus
E. Chiblain
►D

It is a rare entity. Patients complaint of edematous and erythematous plaques, usually on the
trunk. The lesions generally respond to antimalarials.

286
Sneddon-Wilkinson Disease:
A. Most often occurs in elderly women
B. Rarely involves intertriginous areas
C. May occur in association with an IgG monoclonal gammopathy
D. Can be treated with narrow band UVB
E. Is usually an acute, self-limited condition

393
►D

Sneddon-Wilkinson disease, or subcorneal pustular dermatosis, presents with superficial pustules


in annular and serpiginous patterns in the axillae, groin, and abdomen. Middle-aged women are
most often affected. This condition rarely occurs in association with an IgA monoclonal
gammopathy. It is a chronic condition, possibly related to psoriasis, with remissions of variable
duration. Treatments including dapsone, acitretin, and narrow band UVB.

287
This 57-year old male complains of weakness when climbing the stairs and this clinical
presentation. Which cytokine has been implicated in the etiology of this condition?
A. IL-1
B. Il-10
C. IL-5
D. Interferon-gamma
E. Tumor necrosis factor alpha
►E

Dermatomyositis is an autoimmune condition which presents with typical skin findings and
muscle weakness. Age-appropriate screening should be done for internal malignancy as there is a
higher incidence of cancer in these patients. Polymorphisms of tumor necrosis factor-alpha have
been implicated in the etiology. Interferon alpha has also been implicated.

288
Patients with Gottrons papules and periungual telangiectasias will likely have a positive
serologic test to which antibody?
A. Anti-SM
B. Scl-70
C. Anti-Ro
D. Anti-centromere
E. Anti- Jo-1
►E

Gottron's papules and periungual telangiectasias (Samitz's sign) are among the cutaneous
features of dermatomyositis, which also include heliotrope rash, photosensitive poikiloderma of
the upper back (shawl sign). Anti-Jo-1 antibody is present in 20-30% of these patients. The target
antigen is anti-histidyl-tRNA synthetase (Jo-1). The presence of this antibody corresponds to the
development of pulmonary disease.

289

394
There are many HLA types that are associated with psoriasis. The HLA association that has a
relative risk that is 9-15 times the normal limit is:
A. HLA-Cw6
B. HLA-B13
C. HLA-Bw57
D. HLA-B17
E. HLA-B20
►A

HLA-Cw6 has a relative risk of 9-15 times the normal risk for psoriasis. HLA B-17 is associated
with early onset and more serious disease.

290
An elderly gentleman with rheumatoid arthritis has a 2-month history of recurrent painful, red,
swollen ears and hearing loss. Physical exam is notable for sparing of the earlobes. You suspect
he has:
A. Chondrodermatitis nodularis helices
B. Recurrent otitis externa
C. Relapsing polychondritis
D. Systemic lupus erthematosus
E. Severe seborrheic dermatitis
►C

Relapsing polychondritis is a rare disease manifested by recurring inflammation of cartilaginous


tissue. Antibodies to type II collagen are thought to be pathogenic in this disease. Clinically,
patients have auricular chondritis and arthritis. The chondritis is limited to the cartilaginous
portion of the external ears. Involvement of the cartilaginous portions of the eye, respiratory
tract, the inner ear, and the cardiovascular system has been reported as well. A significant portion
of patients with relapsing polychondritis have an associated rheumatic or autoimmune disease.

291
Industrial acne can be caused by insoluble oils seen on the scalp, malar cheeks and scrotum. All
of the following have been known to cause acneiform eruptions except:
A. Dioxin
B. Halogen bromide
C. Iodide
D. Androgenic hormones
E. Tazarotene
►E

395
Dioxin, halogen bromide, iodide, androgenic hormones such as testosterone can cause acneiform
eruption. Tazarotene is a topical retinoid used to treat acne.

292
A farmer develops a rash in photodistributed areas due to a crop that he has been handling. The
most common cause is:
A. Celery
B. Apples
C. Lemons
D. Lime
E. Tomatoes
►A

The celery is a common photosensitizing agent. It can cause both a phototoxic and photoallergic
reactions. Celery and parsnip and weeds also can cause these reactions.

293
Which of following medications may increase the likelihood of pseudotumor cerebri in patients
taking oral isotretinoin?
A. Spironolactone
B. Dapsone
C. Amoxicillin
D. Rifampin
E. Tetracycline
►E

Pseudotumor cerebri, or benign intracranial hypertension is more common in adolescent and


young adult women, but can occur in children. Medicines reported to be associated with the
condition include vitamin A analogues, tetracyclines, steroids (especially in withdrawal),
nalidixic acid, sulphonamides, lithium, thyroxine, growth hormone, amiodarone and tamoxifen.
It most often presents with headache (90% of cases), pulsatile in quality. Less frequent
symptoms are visual disturbances and pulsatile tinnitus. Pseudotumor can also be completely
asymptomatic. The mechanism is not fully understood but current opinion favors impaired
reabsorption of cerebrospinal fluid. Of the medicines associated with the condition, minocycline
is most frequently reported in the literature followed by tetracycline and doxycycline.
Isotretinion has been repored to cause it and it is possible that the incidence of pseudotumor may
increase if two or more drugs which might cause it are used together. For this reason
tetracyclines should not be prescribed concomitantly with oral retinoids.

294

396
A patient has a contact allergy from her contact lens and ophthalmic solution. The allergen that is
most likely the culprit is:
A. Thimerosal
B. PPD
C. Benzene
D. Tuliposidase A
E. Formaldehyde
►A

Contact lens and ophthalmic solutions have the allergen thimerosal.

295
The nail changes seen in the capillary nail folds from the autoimmune connective tissue disorders
such as dermatomyositis and scleroderma are:
A. roughness, hemorrhages, necrosis of the cuticles
B. vascular areas with condensed capillary areas
C. normal density
D. tortuous and dilated
E. increased capillary density
►A

Patients that have dermatomyositis and scleroderma have roughness, hemorrhages, and necrosis
of the cuticles. Patients that have scleroderma have reduced capillary density and avascular areas
which alternate with dilated capillary loupes. Systemic lupus erythematosus reveals a normal
density of capillaries which are tortuous and dilated.

296
The most common cause of mortality in patients with limited systemic sclerosis is:
A. Myocardial infarction
B. Pulmonary fibrosis
C. Pulmonary hypertension
D. Renal failure
E. Sepsis
►C

The most common cause of morbidity and mortality in patients with limited systemic sclerosis is
pulmonary hypertension. Pulmonary fibrosis is more often associated with diffuse systemic
sclerosis. Mortality from renal failure has been significantly reduced by the use of ACE
inhibitors.

397
297
A patient presents to the ER with skin pain and large sheets of epidermal detachment on greater
than 10% of his body surface area after starting bactrim. No purpuric macules or target lesions
are noted. Which of the following can help predict mortality in this patient?
A. glucose greater than 150 mg/dL
B. age over 40 years
C. heart rate greater than 100
D. history of diabetes
E. BUN greater than 15 mg/dL
►B

The patient described has TEN. A diagnosis of TEN can be made if the patient presents with
epidermal detachment of >30% of the body surface with widespread purpuric macules or flat
atypical targets, or if there are large sheets of epidermal detachment involving >10% of the body
surface without purpuric macules or target lesions. The SCORTEN scoring system was
developed to assess severity of illness and predict mortality in TEN, and is meant to be
calculated within the first 24 hours after admission and again on day three. The score is the sum
of the following clinical variables: (1) age over 40 years; (2) heart rate >120 beats per minute;
(3) the presence of cancer or hematologic malignancy; (4) epidermal detachment involving body
surface area >10% on day one; (5) blood urea nitrogen >28 mg/dL (10 mmol/L); (6) glucose
>252 mg/dL (14 mmol/L); and (7) bicarbonate >20 mEq/L. The mortality increases sharply with
each additional point, with a score of 5 or greater having a 90% mortality.

298
Which organic system is least likely to be involved in the attached image?
A. Liver
B. Spleen
C. Genitourinary
D. Gastrointestinal tract
E. Bone marrow
►C

Attached image shows mastocytosis. The disease presents classically with tan-like or red brown
macules or papules. There are different forms of cutaneous mastocytosis, most common form is
urticaria pigmentosa which can appear in both children and adults and diffuse cutaneous
mastocytosis which occur almost exclusively in infants, solitary mastocytoma and telangiectasia
macularis eruptiva perstans. Systemic involvement occurs most commonly in adults and bone
marrow,lymph nodes, liver, spleen and GIT are among the most common involved systems
whereas genitourinary and neuroendocrine systems are least likely to be involved.

398
299
Which lab abnormality is most associated with sarcoidosis?
A. Hypercalcemia
B. Hyperkalemia
C. Hypernatremia
D. Hyperchloremia
E. Hyperlipidemia
►A

Up to 10 % of patients with sarcoidosis may have hypercalcemia. Resultant hypercalciuria and


nephrolithiasis may lead to renal failure. The other listed lab abnormalities are not typically
associated with sarcoidosis.

300
Which of the following MOST favors a diagnosis of Sweet‘s over atypical (bullous) pyoderma
gangrenosum?
A. Recurrence of lesions
B. Presence of lesions on the H/N and hands
C. Association with infections
D. Presence of constitutional symptoms
E. Presence of hemorrhagic bulla and ulcerations
►D

Both Sweet‘s syndrome and atypical pyoderma gangrenosum fall within the category of
neutrophilic dermatoses with bullous and ulcerated lesions. Both favor the head and neck and
extremities. Both can recur, and both demonstrate a heavy neutrophilic infiltrate on
histopathology. Both are associated with infections and hematologic disorders, though atypical
PG is more likely associated with hematologic malignancy. The presence of constitutional
symptoms such as fever, arthralgia, and arthritis are strongly associated with Sweet‘s and can
assist in differentiation between these entities.

301
The most common associated disorder in a patient with elastosis perforans serpiginosa is:
A. Marfan syndrome
B. Down syndrome
C. Osteogenesis imperfecta
D. Ehlers-Danlos syndrome
E. Rothmund-Thomsom syndrome
►B

399
About 1/3 of case of elastosis perforans serpiginosa occur in patients with other concomitant
disorders. All of the above can occur in patients with EPS; however the most common is Down
syndrome. One mnemonic to remember the associated disorders is: PROMEDA. This stands for
PXE, Rothmund-Thomson, Osteogenesis imperfecta, Marfan syndrome, Ehlers-Danlos
syndrome, Down syndrome, acrogeria.

302
What autoimmune disease is associated with a PRP-like eruption?
A. Systemic lupus
B. Rheumatoid arthritis
C. Dermatomyositis
D. Scleroderma
E. Pemphigus erythematosus
►C

The Wong type of dermatomyositis is characterized by erythematous, hyperkeratotic, follicular


papules . It can have a limited or generalized distribution. It can evolve into full-blown
dermatomyositis with classic cutaneous features. This rare variant bears a striking clinical
resemblance to PRP. It can be distinguished on skin biopsy. It has been reported in both adults
and children. Some reports suggest the follicular papules are confined to the skin overlying the
extensor knees and elbows and this variant may be more common in Asian patients.

303
A patient with chronic lymphedema has swelling and verrucous changes with hypertrophic
fibrosis. Patient also has mossy leg and lymphangitis. This patient most likely has:
A. Elephantiasis nostras verrucosa
B. Acanthosis nigricans
C. Palmoplantar keratoderma
D. Psoriasis
E. Seborrheic keratosis
►A

This patient most likely has elephatiasis nostra, which is a complication where there is swelling
and verrucous changes to the skin. The other choices do not describe the changes that you would
see. Psoriasis are silvery scales and PPK are thickened heels. Acanthosis nigracans are velvety
changes that occur on the axillary region and the neck.

304
Which of the following treatments for acne inhibits RNA-dependent protein synthesis by binding
to the 50s ribosomal subunit?

400
A. Erythromycin
B. Tetracycline
C. Trimethoprim-sulfamethoxazle
D. Benzoyl peroxide
E. Azeleic acid
►A

Macrolide antibiotics inhibit RNA-dependent protein synthesis by binding to the 50s ribosomal
subunit, and include erythromycin and azithromycin. Tetracyclines inhibit RNA-dependent
protein synthesis by binding to the bacterial 30s ribosomal subunit. TMP-SMX inhibits bacterial
folic acid synthesis. Benzoyl peroxide is a bactericidal agent with direct oxidizing effects.
Azeleic acid is a dicarboxylic acid that inhibits tyrosinase; the mechanism of action against P.
acnes is not completely understood.

305
A patient is found to have an allergy to a substance in wrinkle resistant clothing. The allergen is
most likely:
A. Formadehyde
B. Thimerosal
C. Tuliposidase A
D. PPD
E. Benzene
►A

The most common allergen found in wrinkle free clothing is formaldehyde.

306
In the setting of the lupus erythematosus-associated complement deficiency syndrome, which of
the following would most often be observed?
A. low C3 and C4
B. low C2 and C3
C. low C2 and C4
D. low C2 only
E. low C3 only
►C

In the complement deficiency syndrome, low C2 and C4 are most commonly seen.
Photosensitivity, annular SCLE lesions, and Ro antibody formation are commonly observed.

307

401
What is the treatment of choice for this condition:
A. Topical antifungal medication
B. High potency topical steroid
C. Topical antibiotic cream
D. Oral antibiotic
E. Excision
►B

The condition depicted is lichen sclerosus et atrophicus which most commonly affects the genital
region. High potency topical steroids are the treatment of choice.

308
What is the most common paraproteinemia in scleromyxedema?
A. IgA
B. IgG kappa
C. IgG lambda
D. IgM
E. It is extremely uncommon to see a paraproteinemia with scleromyxedema
►C

An abnormal paraproteinemia is found in 90% of cases of scleromyxedema, usually IgG lamda.

309
Steven Johnson syndrome and TEN is a hypersensitivity syndrome most often caused by drugs,
infections and rarely cancers. The most common cause of SJS/TEN is:
A. Allopurinol
B. Bactrim
C. Prednisone
D. Hydrocholorothiazide
E. Carbamazepine
►A

SJS and TEN is a hypersensitivity syndrome characterized by epidermal necrosis. The most
common cause is allopurinol.

310
A patient has a contact allergy to an ingredient found in permanent hair dyes. The most likely
allergen is:
A. PPD
B. Tuliposidase A

402
C. Thimerosal
D. Fomaldehyde
E. Parabens
►A

Patients that are allergic to permanent hair dyes are allergic to PPD. The contact dermatitis can
present as a erythematous rash on the hands or on the frontal scalp.

311
The protein component of nodular amyloidosis is:
A. SAA protein
B. AL protein
C. Keratin
D. Collagen
E. Bp180
►B

Nodular amyloidosis presents as single or multiple nodules, often on the extremities. The
primary protein component is of the AL type.

312
Neutrophilic dermatoses en plaque is often associated with which of the following?
A. IgA monoclonal gammopathy and a benign course
B. IgA monoclonal gammopathy and a malignant course
C. Myeloma and B lymphomas and a benign course
D. IgG monoclonal gammopathy and a benign course
E. IgG monoclonal gammopathy and a malignant course
►A

Neutrophilic dermatoses en plaque are well defined, sharply demarcated intensely red plaques.
Myeloma and B cell lymphomas are rarely associated. These resolve with treatment of the
gammopathy.

313
An overweight, post-menopausal woman presents with intense pain in the fatty deposits on her
knees, thighs and hips. She also describes swelling of her hands and feet, associated fatigue, and
history of depression. On clinical exam the pain in the adipose tissue appears out of proportion to
the clinical findings. The diagnosis is:
A. Angiolipomas
B. Adiposis dolorosa

403
C. Congenital lipodystrophy
D. Lipodermatosclerosis
E. Erythema nodosum
►B

Adiposis dolorosa (Dercum\'s disease) is a disorder that usually occurs in obese women 40 to 60
years of age, and it is characterized by pain in adipose tissue that appears to be out of proportion
to the physical findings. The pain can be localized to multiple, painful lipomas on lower
extremities and knees. This condition is accompanied by swelling of different areas of the body,
such as hand and feet, which may be transient. Patients typically have subjective fatigue or
confusion, and may have history of depression or emotional instability. The cause of adiposis
dolorosa is not known. The cause of the pain is speculated to be due to the pressure on nerves by
the adipose tissue deposits. Therapeutic treatments are not very effective, and can involved
procedures such as liposuction, systemic corticosteroids, pregabalin, lidocaine, and psychiatric
care.

314
An 8 year old boy with seasonal allergies presents with red brown macules on the trunk present
for several months. Parent mentions that when these lesions are scratched they form an irritated,
red wheal. Which of the following sign is likely described?
A. Asboe-Hansen sign
B. Homan’s sign
C. Darier’s sign
D. Auspitz sign
E. Hutchinson sign
►C

Darier’s sign is seen in patients with urticarial pigmentosa, a common form of mastocytosis.
Erythema and wheals are commonly elicited with stroking or rubbing secondary to mast cell
degranulation. Symptoms can range from very mild (flushing, hives, no treatment needed) to
life-threatening (vascular collapse). Asboe Hansen, commonly seen in pemphigus vulgaris, refers
to extension of a bulla to adjacent unblistered skin when pressure is put on top of a bulla.

315
The most common cause of allergic contact dermatitis overall in clinical studies is:
A. Nickel
B. Jewelry
C. Dimethylglyoxime
D. Orange Pekoe tea
E. Epoxy Resin

404
►A

The most common cause of allergic contact dermatitis is nickel and jewelry is also another
common allergen. Dimethylglyoxime test is used to detect nickel. If you rub on the item the
solution turns color from pink to reddish that can indicate a positive reaction.

316
Pyostomatitis vegetans is characteristically associated with which systemic disease?
A. Pemphigus vulgaris
B. Ulcerative colitis
C. Lichen planus
D. Lymphoma
E. Rheumatoid arthritis
►B

Pyostomatitis vegetans is a pustular, vegetative variant of pyoderma gangrenosum, found in the


oral mucous membranes. It is most frequently associated with inflammatory bowel disease.

317
Bullous lupus erythematosus is most commonly associated with antibodies to:
A. Type IV collagen
B. Laminin 5
C. Desmoglein I
D. Plectin
E. Type VII collagen
►E

Type VII collagen is found in the sublamina densa and patients with bullous lupus erythematosus
have been found to have antibodies to this protein. Bullous LE and EBA share antibasement
membrane zone antibodies of identical specificity (type VII collagen) as well as clinical and
histologic overlap. A differentiating feature between bullous LE and EBA is that bullous LE has
a dramatic response to dapsone.

318
There are four types of kaposi sarcoma and can be associated with HIV. The viral strain
associated with classic AIDS-associated kaposi sarcoma is:
A. HHV-8
B. HHV-6
C. HHV-5
D. HHV-4

405
E. HHV-3
►A

Patients with kaposi sarcoma associated with AIDs can have kaposi sarcoma associated with
HHV8. It can present as a erythematous purple plaque.

319
Eyebrow growth:
A. Is androgen-dependent in men but not in women
B. Is regulated by dehydroepiandrosterone but not androstenedione
C. Is not androgen-dependent
D. Requires conversion of testosterone to dihydrotestosterone in the hair follicle
E. Is regulated only by testosterone and dihydrotestosterone
►C

Eyebrows, eyelashes, and vellus hair are not androgen-dependent, thus there is no difference
between these areas of hair growth in men and women.

320
A 48 year old woman was recently diagnosed with dermatomyositis. Which examination would
be most important in a work up for malignancy?
A. Breast exam
B. Chest xray
C. Thyroid exam
D. Pelvic exam
E. Stool guiac
►D

Dermatomyositis is an idiopathic inflammatory disease with myositis and characteristic


cutaneous manifestations. There is an increased incidence of malignancy in these patients which
may precede, occur with or follow the diagnosis of dermatomyositis. The most common form of
malignancy in adult women is ovarian cancer. Other malignancies that have been associated
include testicular cancer, gastrointestinal, lung and nasopharyngeal carcinomas.

321
Which of the following is the most common type of melanoma found in black patients?
A. Acral lentiginous melanoma
B. Superficial spreading melanoma
C. Nodular melanoma
D. Amelanotic melanoma

406
E. Lentigo maligna melanoma
►A

Overall, superficial spreading melanomas are most frequently seen. However, acral lentiginous
melanoma is most frequently seen in patients with darker skin types. It is seen on the palms,
soles, and nail units.

322
A patient presents with lesion that can be herniated into the skin on palpation. What is thought to
be important in the pathogenesis of this condition?
A. Calcification of the elastic fibers
B. Formation of excessive collagen
C. Focal loss of elastic tissue
D. Atrophy of the epidermis
E. Inflammation of the panniculus
►C

Anetoderma is a benign condition caused by focal loss of elastic tissue. Primary or idiopathic
anetoderma originates from previously healthy skin with unknown pathogenesis. Various ocular,
bony, cardiac, and other abnormalities have been reported with primary anetoderma. Secondary
anetoderma occurs after the resolution of an inflammatory disease of the skin.

323
This type of psoriasis is the acute variant with a fever that last for several days with eruption of
sterile pustules 2-3 mm in diameter paralleling the fever:
A. Von Zumbusch type
B. Psoriatic erythroderma
C. Eruptive psoriasis
D. Localized pustular psoriasis
E. Plaque type psoriasis
►A

Von Zumbusch type is also known as generalized pustular psoriasis. The sterile pustules is on
trunk, extremities including the nail beds, palms, and soles. Pustules arise in highly erythematous
skin.

324
What type of collagen is overrepresents in this lesion?
A. Collagen I
B. Collagen III

407
C. Collagen IV
D. Collagen V
E. Collagen VII
►B

Keloids are dense overgrowths of fibrous tissue that usually develop after injury to the skin.
Collagen III is overrepresented in these keloids.

325
P. acnes activation of which of the following may stimulate a pro-inflammatory cytokine
cascade?
A. TLR-2
B. TLR-5
C. IL-1
D. IL-8
E. TNF-000
►A

Toll-like receptors are a large group of receptors that recognize a variety of bacterial motifs. P.
acnes has been shown to activate TLR-2, leading to signal transduction and production of pro-
inflammatory cytokines.

326
Necrobiotic xanthogranuloma is most commonly associated with which of the following:
A. IgA monoclonal gammopathy
B. IgM monoclonal gammopathy
C. IgG lambda monoclonal gammopathy
D. IgG kappa monoclonal gammopathy
E. IgG/IgA polyclonal gammopathy
►D

Necrobiotic xanthogranuloma is associated with a monoclonal IgG kappa paraproteinemia in


approximately 80% of cases. It is less commonly associated with an IgA gammopathy. Myeloma
or myelodysplastic syndromes may resultantly occur.

327
What is the most common presentation of psoriatic arthritis?
A. Asymmetric oligo- or polyarthritis
B. Symmetric polyarthritis
C. Spondylitis (axial)

408
D. Distal interphalangeal joint (DIP) disease
E. Arthritis mutilans
►A

Psoriatic arthritis affects 20-40% of patients with psoriasis. Most (~80%) of psoriatic arthritis
patients are rheumatoid factor negative. All of the answer choices represent possible
presentations of psoriatic arthritis, but asymmetric oligo- or polyarthritis is the most common
presentation.

328
What disease is found to be associated with retention of perifollicular pigmentation?
A. Dermatomyositis
B. Secondary syphilis
C. Scleroderma
D. Discoid lupus
E. Sarcoidosis
►C

Scleroderma is a systemic disease which may affect almost any organ in the body. Cutaneous
findings are characterized by symmetric, thickening of the skin. Dyspigmentation may occur
with a "salt and pepper" appearance due to perifollicular retention of pigmentation with
depigmentation of the interfollicular skin.

329
A positive ANA with a speckled staining pattern correlates with staining of what cellular
component?
A. Ribonucleoproteins
B. Nucleolar RNA
C. Native DNA
D. Kinetochore
E. Histones
►A

A speckled ANA pattern correlates with staining of ribonucleoproteins and is seen in Mixed
Connective Tissue Disease (MCTD), Systemic Lupus Erythematosus (SLE), systemic sclerosis
and Sjogren's syndrome. A nucleolar staining pattern correlates with staining of nucleolar RNA.
Centromere staining correlates with staining of kinetochore. A peripheral staining pattern
correlates with staining of native DNA and a homogeneous staining pattern correlates with
staining of native DNA and histones.

409
330
Which of the following is true regarding this entity?
A. Extragenital lesions commonly involve the torso and are usually pruritic
B. Childhood onset is reported in ~5% of cases
C. 20% of both men and women with this diagnosis have at least one autoimmune
disease
D. Girls outnumber boys 2:1
E. Genital disease represents 50% of childhood cases
►C

The patient pictured has lichen sclerosus. This likely represents an autoimmune phenomenon, as
20% of both men and women have at least one autoimmune disease (vitiligo, alopecia areata, or
thyroid disease), and a larger proportion have circulating antibodies. Extragenital lesions most
frequently involve the torso and are usually asymptomatic. Childhood onset occurs in 10-15% of
cases, and girls outnumber boys 10:1. Genital disease represents 90% of childhood lichen
sclerosus.

331
Which of the following is not associated with this condition?
A. Poor oral hygeine
B. Antibiotic use
C. Aging
D. HIV
E. Smoking
►C

Black hairy tongue is a common reactive condition due to the enlongation of filiform papillae.
The most common etiologies include poor oral hygeine, use of oral antibiotics, HIV infection,
use of tobacco, radiation, and alcohol.

332
The substance that is responsible for the yellow tattoo color is:
A. Cadmium sulfide
B. Cobalt aluminate
C. Ferric oxide
D. Chromic oxide
E. Cinnabar
►A

Cadmium sulfide is responsible for the yellow color in tattoos.

410
333
Androstenedione is produced by:
A. The adrenals
B. The ovaries
C. The adrenals and ovaries
D. Extraglandular conversion
E. The adrenals, ovaries, and extraglandular conversion
►C

Androstenedione is produced by the adrenals and the ovaries. Testosterone is produced by the
adrenals, ovaries, and by extraglandular conversion of androstenedione and
dehydroepiandrosterone.

334
A patient's biopsy is found to have curled hair shafts with hyperkeratotic plugs. Which of the
following is not an associated feature of this condition
A. Hypochondriasis
B. Gingivitis
C. Delayed wound healing
D. Nail changes
E. Tender nodules on the lower extremities
►D

The patient's biopsy demonstrates the corkscrew hairs of scurvy, or vitamin C deficiency, which
is characterized by the 4 "H"s: hemorrhagic signs (tender nodules on the lower extremities),
hyperkeratosis of hair follicle, hypochondriasis, and hematology abnormalities

335
Which of the following is true regarding atopic dermatitis?
A. Over 80% of affected individuals present in the first year of life
B. Natural measles infection has been noted to improve atopic dermatitis
C. Most children develop worsening of atopic dermatitis if they develop asthma
later in childhood
D. Staphylococcus aureus is found in ~25% of atopic dermatitis skin lesions
E. Increased expression of cathelicidins such as LL 37 has been noted in atopic
dermatitis
►B

411
Natural measles infection has been noted to improve atopic dermatitis. Atopic dermatitis
typically begins in infancy, with ~50% in the first year of life and an additional 30% between 1
and 5 years. Most children with AD eventually develop allergic rhinitis or asthma later in
childhood. Many outgrow AD as respiratory allergy develops. S. aureus is found in over 90% of
AD skin lesions. Decreased expression of innate antimicrobial peptides, such as human beta-
defensin and cathelicidins, such as LL 37, may explain the increased susceptibility to
colonization and skin infection with S. aureus in patients with atopic dermatitis.

336
Which of the following is NOT a feature associated with this condition?
A. Accelerated blanch response
B. White dermatographism
C. Anterior subcapsular cataracts
D. Keratoconus
E. Pityriasis alba
►A

The child pictured has atopic dermatitis. Features associated with atopic dermatitis include
pityriasis alba, white dermatographism and delayed blanch reponse, anterior subcapsular
cataracts, and keratoconus.

337
Ulcerative colitis and crohn'€™s disease are most commonly associated with which of the
following:
A. Pyoderma gangrenosum
B. Erythema elevatum diutinum
C. Sweet'€™s syndrome
D. IgA pemphigus
E. Sneddon-Wilkinson's disease
►C

Sweet's syndrome is also associated with acute myelogenous leukemia, colony-stimulating


factor, all-trans retinoic acid and tetracyclines. Flu-like symptoms, high-grade fever, malaise and
peripheral neutrophils accompany the cutaneous lesions. Steroids and dapsone are the treatments
of choice.

338
What is the most common site of extracutaneous involvement in this non-infectious disease?
A. Lungs
B. Eyes

412
C. Bone Marrow
D. Liver
E. Nasal mucosa
►A

Sarcoidosis is a non-infectious granulomatous disorder of unclear etiology. It has a predilection


for the lungs (90%), lymph nodes (75-90%), eyes (25%), nasal mucosa (20%), bone marrow (25-
40%), and liver (15-40%).

339
Which of the following is associated with this scarring condition?
A. Thyroid disease
B. Acne conglobata
C. Sarcoidosis
D. Systemic lupus erythematosus
E. Lichen planus
►B

Dissecting cellulitis is part of the follicular occlusion tetrad which also includes acne conglobata,
hidradenitis suppurtiva, and pilonidal cysts.

340
Which of the following is a clinical manifestation of neonatal lupus erythematosus:
A. Renal disease
B. Cerebral vascular accident
C. Arthritis
D. Thrombocytopenia
E. Neutrophilia
►D

Neonatal lupus involves annular scaling macules and plaques appearing within the first few
months of life in babies born to mothers with LE or other autoimmune connective tissue
diseases. In addition to the rash, babies display congenital heart block, hepatic disease, and
thrombocytopenia.

341
A 42 year-old woman has the findings pictured, as well as erythematous patches on the upper
chest and shoulders. No Raynaud's, polyarthritis, pulmonary, or cardiac involvement is noted.
Which antibody might you expect to be present in this patient?
A. Anti-SRB

413
B. Anti-Jo-1
C. Anti-Mi-2
D. Anti-Ku
E. Anti-La
►C

This patient has dermatomyositis. Anti-Mi-2 antibodies in DM correlate with the presence of a
shawl sign, cuticular changes, and good prognosis. Anti-Jo-1 antibodies correlate with
pulmonary fibrosis, Raynaud's, and polyarthritis. Anti-SRP antibodies correlate with cardiac
disease and poor prognosis. Anti-Ku antibodies correlate with sclerodermatomyositis. Anti-La
antibodies correlate with Sjogren's syndrome.

342
A patient develops pulmonary fibrosis, Raynaud's phenomenon and skin changes similar to
systemic sclerosis. These changes resolve following discontinuation of which of the following
medications?
A. Bleomycin
B. Penicillamine
C. Hydralazine
D. Minocycline
E. Isoniazide
►A

The changes described above are similar to findings of scleroderma. Patients receiving
bleomycin can develop a reversible syndrome similar to scleroderma. The other four listed
options are related to drug-induced lupus erythematosus and are not linked to scleroderma-like
syndromes.

343
Periorbital purpura is a characteristic cutaneous manifestation of:
A. Sarcoidosis
B. Amyloidosis
C. Dermatomyositis
D. Lymphoma
E. Thrombocytopenia
►B

Primary systemic amyloidosis, often myeloma associated, is characterized by the deposition of


amyloid fibrils derived from immunoglobulin light chains. 40% of patients manifest with

414
cutaneous findings including petechiae, purpura, waxy translucent papules on the eyelids, diffuse
alopecia, scleroderma and nail dystrophy.

344
In cryosurgery, the temperature needed for destruction of keratinocytes is?
A. 0-10 degrees C
B. -20- -30 degrees C
C. -5 degrees C
D. -50 degrees C
E. -196 degrees C
►B

Cryosurgery is the application of low temperature to produce local tissue destruction. -20- -30
degrees C is needed to destroy keratinocytes. -4- -7 degrees are needed to destroy melanocytes.
Temperatures of -50 are recommended to treat malignant lesions.

345
The condition shown here may be exacerbated by the inappropriate use of which of the
following?
A. Comedogenic make-up foundation
B. Sunscreens
C. Erythromycin 2% ointment
D. Topical corticosteroids
E. Mineral oil-containing moisturizers
►D

This slide shows perioral dermatitis, characterized by small, erythematous papules in a perioral,
often periorificial, distribution. Topical corticosteroids can exacerbate or trigger perioral
dermatitis. Note the absence of comedones, which distinguishes rosacea and its subsets from
acne.

346
A 48 year-old man with a long history of alcohol abuse and dependence presents with angular
cheilitis, atrophic glossitis, a scrotal dermatitis that spares the midline and extends to the thighs,
photophobia, and blepharitis. Which nutritional deficiency do you suspect?
A. Vitamin B12
B. Vitamin C
C. Pyridoxine
D. Vitamin B2
E. Iron

415
►D

The patient presents with classic findings of the oral-ocular-genital syndrome seen in the setting
of vitamin B2 (riboflavin) deficiency. This deficiency occurs most often in alcoholics. It
characteristically presents with angular cheilitis, atrophic glossitis (magenta), a seborrheic-like
dermatitis around the nose, genital dermatitis (scrotal dermatitis sparing the midline and
extending to the thighs), photophobia, and blepharitis.

347
The hyperproliferative epithelium of this mature psoriasis plaque is associated with increased
expression of which keratin(s)?
A. K1, K10
B. K5, K14
C. K6, K16
D. K17
E. K2e
►C

Expression of keratins K6 and K16 is upregulated in hyperprolferative psoriasis plaques.

348
Folliculotropic CTCL presents most commonly on the head and neck. The treatment of choice is
:
A. PUVA
B. Narrow band UVB
C. Methotrexate
D. Mycophenolate mofetil
E. Cyclosporin
►A

The most common location for folliculotropic CTCL is head/neck. It can present as a infiltrative
plaque, acneiform lesions or keratosis pilaris like lesions and is usually very pruritic. The
treatment of choice is PUVA.

349
A 3 month old presents with a diffuse vesiculobullous rash with copper colored macules on the
palms and soles. You ascertain from the history that his mother had a nonpainful erosion on her
labia during pregnancy which spontaneously resolved. What other symptoms would you expect
this infant to have?
A. Pseudoparalysis of Parrot

416
B. Higoumenakis sign
C. Clutton joints
D. Mulberry molars
E. Saddle nose deformity
►A

Pseudoparalysis of Parrot is a sign of early congenital syphilis. Early congenital syphilis occurs
before 2 years of age, whereas late congenital syphilis generally occurs after 2 years of age.
Higoumenakis sign, clutton joints, mulberry molars, and saddle nose deformity are all signs of
late congenital syphilis.

350
The "€œfollicular occlusion "triad" consists of:
A. Acne conglobata, hidradenitis suppurativa, kerion
B. Acne conglobata, chloracne, pyoderma faciale
C. Acne fulminans, hidradenitis suppurativa, dissecting cellulites of the scalp
D. Acne comedonicus, hidradenitis suppurativa, dissecting cellulites of the scalp
E. Acne conglobata, hidradenitis suppurativa, dissecting cellulites of the scalp
►E

Acne conglobata is a sever variant of acne vulgaris characterized by large, often multiple
comedones, abscesses with sinus formation and inflammatory nodules. Hidradenitis suppurativa
is a chronic condition characterized by swollen, painful, inflamed lesions in the axillae, groin,
and other parts of the body that contain apocrine glands. Dissecting cellulites of the scalp, also
known as perifolliculitis capitis abscedens et Suffodiens of Hoffman, consists of deep
inflammatory boggy nodules +/- sinus tracts on the occipital scalp, most commonly in African
American males.

351
ACE inhibitors cause angioedema via stimulation of?
A. Bradykinins
B. Histamine
C. Prostaglandins
D. Leukotrienes
E. Complement
►A

Bradykinins are responsible for angiotensin converting enzyme inhibitor induced angioedema.

352

417
Teenage boys with severe, eruptive cystic acne and fever, leukocytosis, myalgias and other
constitutional symptoms can have which of the following bony change?
A. Dimpling above the 5th MCP
B. Osteolytic lesions of the clavicle
C. Osteopoikilosis
D. Stippled epiphyses
E. Jaw cysts
►B

The syndrome described is that of acne fulminans. Osteolytic bone lesions may accompany this
syndrome, most commonly of the clavicle. Dimpling above the 5th MCP is a feature of
Albright‘s Hereditary Osteodystrophy. Osteopoikilosis is a feature of Buschke-Ollendorf
syndrome. Stippled epiphyses occurs in chondrodysplasia punctata. Jaw cysts are a feature of
Gorlin‘s syndrome.

353
MAGIC syndrome involves:
A. Relapsing polychondritis
B. Acne conglobata
C. Livedo reticularis
D. Lipodystrophy
E. Psoriatic arthritis
►A

MAGIC syndrome is a combination of Behçets disease and relapsing polychondritis. Patients


show mouth and genital ulcers with inflamed cartilage.

354
The most common benign neoplasm in a nevus sebaceous of Jadassohn is:
A. Syringocystadenoma papilliferum
B. Trichoblastoma
C. Fibroepithelioma
D. Sebaceous adenoma
E. Sebaceoma
►B

The most common benign neoplasm is Trichoblastoma.

355
What is the diagnosis of this lesion on the penile shaft?

418
A. Sclerosing lymphangitis
B. Loaloa
C. Varicose vein
D. Syphilis
E. Lymphogranuloma venereum
►A

Sclerosing lymphangitis is a self-limited, cord-like structure on the penile shaft, which typically
occurs 24-48 hours after vigorous sexual intercourse.

356
Sarcoidosis presenting as uveitis, facial nerve palsy, fever and parotid gland swelling is referred
to as:
A. Heerfordt's syndrome
B. Lofgren's syndrome
C. Lupus pernio
D. Darier-Roussy disease
E. Schaumann syndrome
►A

Heerfordt's syndrome is the name given to sarcoidosis presenting with uveitis, facial nerve palsy,
fever and parotid gland swelling. Lofgren's syndrome is an acute presentation of sarcoidosis that
presents wth fever, arthritis, erythema nodosum and hilar adenopathy. Darier-Roussy disease is
sarcoidosis presenting as painless firm subcutaneous nodules. Lupus pernio refers to sarcoidosis
presenting as papulonodules and plaques involving the nose (especially the alar rim), ears and
cheeks. Schaumann syndrome was a distractor as Shaumann bodies are seen in sarcoidosis on
histopathology.

357
A 35-year-old man presents with scattered infiltrative cutaneous plaques, chronic fever, parotid
gland enlargement, and a facial nerve palsy. What other finding is most likely on physical
exam?
A. Anterior uveitis
B. Periungual telangiectasia
C. Nail pits
D. Non-scarring alopecia
E. Condyloma lata
►A

419
Heerfordt-Waldenstrom Syndrome is a form of sarcoidosis consisting of fever, parotid
enlargement, facial nerve palsy, and anterior uveitis. Periungual telangiectasias are commonly
seen in collage vascular diseases. Nail pits are commonly seen in psoriasis and alopecia areata.
Sarcoidosis involvement of the scalp often results in a scarring alopecia. Condyloma lata is seen
in secondary syphilis.

358
A young man treated with minocycline developed blue-black discoloration with acne scars at his
cheeks. A Perls stain would show:
A. Increased melanin at the basal layer of the epidermis
B. Black staining granules within macrophages
C. Blue staining granules within macrophages
D. Muddy brown pigment granules
E. Giant melanosomes
►C

Perls stain stains hemosiderin (iron) blue. Fontana Masson stains melanin black. Minocycline
hyperpigmentation often shows positive staining for both iron and melanin.

359
Leukokeratosis nicotina palati:
A. Is not attributed to smoking
B. Represents a premalignant condition
C. Responds to intralesional steroids
D. Represents inflamed minor salivary glands
E. Resolves with tetracycline therapy
►D

Leukokeratosis nicotina palati, also called smoker�s palate, is a complication of tobacco use,
especially pipe smokers. Lesions develop as a result of inflamed minor salivary glands and
manifest as uniform keratosis of the hard palate with multiple red, umbilicated papules.

360
Mutations in which of the following genes has been implicated in the pathogenesis of
disseminated superficial actinic porokeratosis?
A. BRAF
B. K-RAS
C. PTCH
D. PTEN
E. SART3

420
►E

Disseminated superficial actinic porokeratosis (DSAP) is a disorder characterized by numerous


keratotic macules and papules often localized in a photodistribution. Clinically, they are
distinguished by a rim of keratotic scale and often may have an atropic appearance centrally.
Histologically, they often display a lichenoid infiltrate flanked by characteristic cornoid lamella
(inward-bending tiers of parakeratosis with underlying hypogranulosis and dyskeratosis).
Recently, mutations in SART3 (squamous cell carcinoma antigen recognized by T-cells 3) have
been implicated in a Taiwainese patient cohort affected by DSAP. Of note, while all varieties of
porokeratoses have the potential for malignant degeneration and development of squamous cell
carcinoma, lesions of DSAP have the lowest risk.

361
Patients with xanthoma disseminaturn may demonstrate:
A. Elevated serum beta lipoproteins
B. Sparing of the oral mucosa
C. Involvement of the pituitary gland
D. Sparing of the flexural areas
E. Increased risk of malignant degeneration
►C

Xanthoma disseminatum is a rare mucocutaneous disease of discrete yellowish-red to brown


papules in the axillary and inguinal folds. Patient are generally normolipidemic. Involvement of
the pituitary gland may occur, leading to diabetes insipidus.

362
The "flag sign" is seen in which of the following conditions?
A. psoriasis
B. sarcoidosis
C. hidradenitis suppurativa
D. kwashiorkor
E. telogen effluvium
►D

The "flag sign" is a striking physical finding most readily seen in long and dark hair. Hair that
has grown during periods of inadequate nutrition is pale so alternating bands of light and dark
can be seen along a single strand (reflecting periods of adequate and inadequate nutrition). The
"flag sign" can be seen with other nutritional disorders as well.

363

421
Pilomatricomas has been associated with which of the following conditions?
A. Acute myelogenous leukemia
B. Multiple sclerosis
C. Crohns Disease
D. Myotonic dystrophy
E. Pneumothorax
►D

Answer D. Pilomatricomas are benign tumors that are derived from hair matrix cells. This tumor
presents more frequently in children as a solitary asymptomatic purple red papule or nodule
usually on the head and neck. Mutations in beta catenin is associated with the development of
these tumors. Multiple pilomatricomas may be a marker for the development of myotonic
dystrophy. This rare disorder is AD, and is associated with difficulty relaxing muscles after
contraction. In a review of patients with this disorder, patients tended to have mulitple
pilomatricomas. Pilomatricomas can also be found with Turner syndrome, Rubinstein-Taybi, and
Churg Strauss syndromes.

364
Spindle cell lipoma is commonly found on the:
A. Head
B. Lower extremities
C. Buttocks
D. Breast
E. Posterior shoulder
►E

Spindle cell lipoma is a solitary benign tumor seen in adult male patients, and is most often
located on the posterior shoulder and neck regions.The tumor histologically consists of mature
collagen, adpose tissue, spindle cells, and mast cells. Treatment is with local excision.

365
What name is commonly used to describe this condition of nodular elastosis with cysts and
comedones?
A. Actinic granuloma
B. Chlorance
C. Cutis rhomboidalis nuchae
D. Colloid milium
E. Favre-Racouchot
►E

422
Favre-Racouchot syndrome is a condition that generally affects elderly white males with history
of chronic sun exposure and smoking. It is characterized by multiple open comedones on the
temples and forehead with a background of actinincally damaged skin.

366
Patients with chronic idiopathic urticaria should avoid:
A. Latex products
B. Alcohol
C. Nickel sulfate
D. �-blockers
E. aspirin
►E

Chronic idiopathic urticaria is defined by the presence of urticaria of unknown etiology lasting
greater than 6 weeks. Patient with chronic idiopathic urticaria should avoid aspirin as it
aggravates urticaria in about 30% of patients.

367
Which statement best describes the mechanism of action of spironolactone?
A. Non-steroid inhibitor of DHT binding to the androgen receptor
B. Progestin
C. GnRH agonist
D. Mineralocorticoid
E. Steroid inhibitor of androgen receptor and androgen biosynthesis
►E

Spironolactone is a steroid molecule. It acts as an anti-androgen by blocking the androgen


receptor and inhibiting androgen synthesis.

368
A 35 year old man with plaque psoriasis well-controlled with twice daily application of
calcipotriene 0.005% cream noticed a flare of his psoriasis shortly after he started using a
prescription moisturizer lotion for psoriasis. Which is the most likely explanation for the
apparent decreased efficacy of calcipotriene?
A. The moisturizer impaired penetration of the calcipotriene
B. The moisturizer diluted the calcipotriene
C. The two events are unrelated
D. The prescription moisturizer likely contains lactic acid.
E. The decreased efficacy is due to tachyphylaxis
►D

423
The active ingredient in Dovonex is easily inactivated, particularly by acidic compounds like
salicylic acid and lactic acid.

369
All of the following are consider intermediate filament except:
A. Microtubules
B. Keratins
C. Vimentin
D. Desmin
E. Peripherin
►A

Intermediate filaments are composed of keratins, vimentin, desmin, peripherin, neurofilaments,


nuclear laminins, and nestin. These are part of cytoskeletal elements.

370
An atypical finding in patients with Lofgren syndrome is?
A. Fever
B. Arthritis
C. Subcutaneous Tender Nodules
D. Acute Iritis
E. Beaded papules on nasal rim
►E

Lofgren syndrome is a form of acute, self resolving, sarcoidosis manifested with hilar
adenopathy, fever, migrating polyarthritis, acute iritis, and erythema nodosum. In general these
patients do not have other manifestations of cutaneous sarcoidosis. Papules along the nasal rim
are suggestive of Lupus Pernio sarcoidosis and suggest a worse prognosis with bronchial
involvement.

371
A patient has erythema, vesicles, and erosions in periorificial, and acral flexural distribution.
Adult onset diabetes, glucose intolerance, and weight loss is associated with these symptoms.
The % of patients that have metastatic disease by diagnosis is:
A. 10%
B. 25%
C. 50%
D. 75%
E. 90%

424
►D

Patients that are diagnosed with necrolytic migratory erythema have a 75% chance of having
metastatic disease by the time of diagnosis. Lesions have a circinate pattern due to peripheral
spread. Patient experience adult onset diabetes, glucose intolerance and weight loss.

372
A 9-year-old boy presents with calcinosis cutis, periorbital violaceous erythema, and periungual
telangiectasias. Which substance is most likely to be elevated in this patient?
A. Fodrin
B. Fibrillarin
C. Thrombospondin-1
D. Complement
E. Triglycerides
►C

This patient has juvenile dermatomyositis which can feature a heliotrope rash, periungual
telangiectasias, psoriasiform dermatitis, calcinosis and lipodystrophy. Thrombospondin-1 is a
mediator of angiogeneis that is increased in patients with juvenile dermatomyositis. Systemic
sclerosis can demonstrate antibodies to fibrillarin.

373
This is used to describe the blanching seen around psoriatic plaques secondary to decrease in
prostagladin, PGE2:
A. Woronoff Ring
B. Auspitz sign
C. Koebnerization
D. Silver sign
E. Ashoe sign
►A

Woronoff ring is an area of blanching around psoriatic plaques secondary to decrease in


prostagladin, PGE2. Koebnerization is seen in 20% of patients. Auspitz sign is bleeding upon
removal of scale.

374
What type of amyloid is deposited into the skin of this pruritic disorder?
A. Amyloid AA
B. Amyloid AL
C. Keratin derived

425
D. Beta-2 microglobulin
E. Transthyretin
►C

Lichen amyloidosis is a pruritic eruption that often occurs in areas of chronic rubbing. Clinically,
it has a rippled appearance. Keratin derived amyloid is the type deposited in the skin.

375
The most common associated malignancy with dermatomyositis is:
A. Colon carcinoma
B. Hepatocellular carcinoma
C. Renal cell carcinoma
D. Esophageal carcinoma
E. Ovarian carcinoma
►E

Patients with dermatomyositis (DM) may have cancer that precedes, occurs simultaneously as, or
follows the diagnosis of DM. Ovarian cancer is overrepresented in this population.

376
What is the most common cause of death in malignant atrophic papulosis Degos disease?
A. Cutaneous infection
B. Cerebrovascular accident
C. Myocardial ischemia
D. Bowel infarction
E. Pulmonary hemorrhage
►D

Degos disease is an obliterative arteritis clinically presenting with pink-red papules that later
develop atrophic, porcelain-white centers. Death is usually secondary to Infarction of the bowel
and perforation. Less commonly, death results from cerebral infarction.

377
A patient presents requesting removal of a tattoo. Examination reveals a tattoo with red, orange,
yellow, and purple pigment. Which laser would be appropriate to treat this tattoo?
A. Q switched Nd:YAG (532nm)
B. Q switched alexandrite (755nm)
C. Q switched ruby (694nm)
D. Nd:Yag (1064nm)
E. Alexandrite (755nm)

426
►A

The Q switched Nd:YAG can also be used to treat tan pigment. QS alexandrite and QS ruby can
remove green pigment. QS Nd:YAG is a good choice for patients with darker skin color.

378
A patient has a rapid onset of hyperpigmented velvety plaques involving the intertriginous areas.
The most likely associated cancer with this disease is:
A. Gastric carcinoma
B. Lung carcinoma
C. Breast carcinoma
D. Lymphoma
E. Mycosis fungoides
►A

A patient with acanthosis nigricans can also have florid oral papillomatosis. The most commonly
seen cancer is gastric carcinoma. Other cancers associated are lung, breast, uterine, ovarian,
lymphoma and mycosis fungoides. It is associated with an increase in growth hormone.

379
Which of the following is associated with dermatitis herpetiformis?
A. Gluten-sensitive enteropathy
B. Inflammatory bowel disease
C. Autoimmune hepatitis
D. Herpes labialis
E. Rheumatoid arthritis
►A

Gluten-sensitive enteropathy or Celiac disease is demonstrated on small bowel biopsy of all


patients with dermatitis herpetiformis. However, most of these patients do not demonstrate
symptoms of gastrointestinal disease. Dermatitis herpetiformis (DH) is not associated with
inflammatory bowel disease, autoimmune hepatitis, herpes labialis, or rheumatoid arthritis.

380
Which of the following disorders is primarily associated with a monoclonal gammopathy of the
IgA type?
A. Scleromyxedema
B. Scleredema
C. Necrobiotic xanthogranuloma
D. Schnitzler's syndrome

427
E. Pyoderma gangrenosusm
►E

Pyoderma gangrenosum is an destructive, inflammatory disease of the skin characterized by a


painful nodule or pustule that later forms a progressively enlarging ulcer. On pathology, there is
a neutrophilic infiltrate with leukocytoclasia. Diseases that have been associated with pyoderma
gangrenosum include inflammatory bowel disease, arthritis, an IgA monoclonal gammopathy
(primarily), and myelodysplasia.

381
Melanocytes can be found in all of the following except:
A. Nevus depigmentosa
B. Tyrosinase positive albinism
C. Nevus anemicus
D. Vitiligo
E. Postinflammatory hypopigmentation
►D

Vitiligo is an acquired disease in which there is total loss of pigment. The central process in
vitiligo is the destruction of melanocytes. With silver stains or dopa reaction, well established
lesions of vitiligo are completely devoid of melanocytes.

382
A 28 year old previously healthy man presents with thick crusting/hyperkeratosis of the palms
and soles, urethritis, and one month of peripheral arthritis. What of the following is true
regarding his condition
A. Females and males are equally affected
B. A chronic deforming arthritis occurs in 20%
C. TNF-alpha inhibiting agents have no role in the treatment of this condition
D. Patients must have urethritis, conjunctivitis, and arthritis for diagnosis
E. Clamydia cervicitis is not associated with this condition
►B

The patient has Reiter's syndrome. Reiter's syndrome is a chronic inflammatory disease similar
to psoriasis with psoriatic arthritis, and is though to be a variant form. The classic triad includes
urethritis, conjunctivitis, and arthritis. Few patients present with the classic triad, and thus the
syndrome can be diagnosed with peripheral arthritis for greater then one month duration and
rarely occurs in women. Skin findings include keratoderma blennorrhagicum and circinate
balanitis (in men), as well as oral erosions, sever stomatitis, and nail changes. Chronic deforming

428
arthritis occurs in 20% of patients. Treatments include topical steroids, NSAIDs, methotrexate,
acitretin, cyclosporin, and TNF-inhibiting biologics such as etanercept.

383
A 48 year old African American woman presents with 1-3 cm red-brown plaques on the
extremities, fevers, bilateral knee pain, and tender nodules on bilateral shins. What is her
presentation most consistent with?
A. Lofgren‘s syndrome
B. Rheumatoid arthritis
C. Grave‘s disease
D. Psoriatic arthritis
E. Gout
►A

This patient has Lofgren‘s syndrome, a variant of sarcoidosis, with erythema nodosum, hilar
adenopathy, fever, and arthritis. Rheumatoid arthritis, psoriatic arthritis, and gout result in joint
pain, but are not consistent with this patient‘s presentation.

384
This syndrome has characteristics of fever, uveitis, parotid enlargement, and facial nerve palsy:
A. Heerfordt syndrome
B. Lofgren syndrome
C. Sjogrens syndrome
D. Richer Hanjart syndrome
E. Wilson's syndrome
►A

Heerfordt syndrome is also known as uveoparotid fever. It is characterized by fever, uveitis,


which may precede parotid enlargement and facial nerve palsy.

385
Juvenile pityriasis rubra pilaris accounts for 10% of all cases occurs between 5 and 10 years old.
It often follows an acute infection. This is the cardinal feature of juvenile pityriasis rubra pilaris:
A. Palmoplantar hyperkeratosis
B. Erythroderma
C. Hyperkeratotic flexures
D. Ichthyosis
E. Plantar dermatosis
►A

429
Palmoplantar hyperkeratosis is a cardinal feature of juvenile pityriasis rubra pilaris. It occurs in
the age of 5-10 years of age and can follow after an infection.

386
A patient is diagnosed with carcinoid syndrome with facial flushing, diarrhea and intermittent
bronchospasm. The urine test is:
A. 5-hydroxyindole-acetic acid
B. Gastric acid
C. Amylase
D. Lipase
E. Serotonin
►A

Patients with carcinoid syndrome will develop telangiectasia, pellagra like or sclerodermoid like
eruption. The neoplasm originated in the GI tract and 5-hydroxyindole-acetic acid (5-HIAA)
levels would be elevated. Surgical treatment is the therapy.

387
Which of the following forms of angioedema is mediated by kinin release?
A. Hereditary angioedema
B. Acute allergic angioedema
C. Angioedema associated with urticarial vasculitis
D. NSAID angioedema
E. Infectious angioedema
►A

Types of angioedema mediated through kinin release include hereditary angioedema, acquired
C1 inhibitor deficiency, and ACE inhibitor associated angioedema. These conditions are not
associated with concurrent urticaria.

388
Which of the following is correct about eosinophilic folliculitis?
A. painful
B. only seen in adults
C. classified as an AIDS-defining illness
D. more common in females
E. associated with P. acnes infection of hair follicles
►C

430
Eosinophilic pustular folliculitis is a non-infectious eosinophilic infiltrate of the hair follicle. It is
characterized by intense pruritus. The 3 variants of eosinophilic folliculitis include classic
eosinophilic pustular folliculitis, immunosuppression-associated (mostly HIV-related), and
infancy-associated eosinophilic folliculitis. The classic type (Ofuji′s disease) is more common in
Japanese men. The male-to-female ratio of eosinophilic folliculitis is 5:1. It is associated with
immunosuppresion and has been classified as an AIDS defining illness. Eosinophilic pustular
folliculitis should be viewed as a possible cutaneous sign of immunosuppression.

389
Which of the following medications would likely exacerbate your patient's psoriasis?
A. Hydrochlorothiazide
B. Lisinopril
C. Amlodipine
D. Metroprolol
E. Diltiazem
►D

Several drugs have been incriminated as inducers of psoriasis, in particular and most notably
lithium, beta-blockers, antimalarials, and interferon. More recent additions include terbinafine,
calcium channel blockers, captopril, glyburide, and lipid-lowering drugs such as gemfibrozil

390
A newborn is exposed to the cold and a lacy reticulated pattern is revealed. The most common
abnormality with cutis marmorata telangiectatica congenita is:
A. Limb atrophy
B. Limb hypertrophy
C. Bone resorption
D. AV malformation
E. Leg shortening
►A

Cutis marmorata telangietatica is seen in newborn infants when exposed to cold weather with a
reticulated pattern. It represents a vasomotor response and decreases with age.

391
A 45-year-old woman develops bilateral poikilodermatous patches without atrophy on her lateral
thighs. This is most likely a manifestation of what disease?
A. Lupus Erythematosus
B. Dermatomyositis
C. Mycosis Fungoides

431
D. Hepatitis C
E. Diabetes Mellitus
►B

Poikilodermatous changes on the lateral thigh represent the "holster sign"ۥ seen in
dermatomyositis. The poikilodermatous patches seen in dermatomyositis are often symmetric
macular violaceous erythemas with dyspigmentation. Other characteristic poikilodermatous
manifestations of dermatomyositis include the "V-sign" on the anterior neck and upper chest and
the "shawl sign"€™ on the posterior neck, upper back, and the posterior shoulders. Lesions
of Mycosis Fungoides typically have some component of epidermal atrophy.

392
You prescribe doxycyline to a 22 year-old woman with acne. Your patient takes an oral
contraceptive to prevent pregnancy. Your patient read in a magazine that the doxycycline may
decrease the efficacy of her contraceptive, and asks you about this. Which of the following
antibiotics has been definitively shown to reduce contraceptive efficacy?
A. Tetracycline
B. Minocycline
C. Azithromycin
D. TMP-SMX
E. Rifampin
►E

Theoretically, decreased enterohepatic absorption of hormones due to altered gut flora could
decrease contraceptive efficacy. However, this theory has not been borne out in studies. Only
rifampin, which is a potent hepatic microenzyme inducer, has been definitively shown to reduce
contraceptive efficacy.

393
What mast cell mediators are preformed, rather than synthesized upon exposure to a trigger?
A. tryptase
B. leukotriene C4
C. thromboxane
D. prostaglandin D2
E. platelet activating factor
►A

Mast cell mediators can be grouped into two classes: preformed and newly formed. Preformed
mediators include tryptase, histamine, serotonin, and heparin. Newly formed mediators include
prostaglandin D2, leukotriene C4, and platelet activating factor.

432
394
Behcet's disease is associated with which HLA type?
A. HLA-B51
B. HLA-B17
C. HLA-B13
D. HLA-DR4
E. HLA-Cw6
►A

Behcet's disease is associated with HLA-B51. HLA-Cw6 is the HLA type most closely
associated with psoriasis. HLA-B13 and HLA-B17 are also both associated with psoriasis; HLA-
B17 is assiated with earlier onset and more serious psoriasis.

395
A patient presents with recurrent crops of papules that ulcerate and then spontaneously heal.
What immunohistochemical stain would be helpful in making the diagnosis?
A. CD 4
B. CD 20
C. CD 30
D. CD 56
E. CD 68
►C

Lymphomatoid papulosis Lymphomatoid papulosis (LyP; Macaulay disease) is a chronic


lymphoproliferative disease of the skin characterized by recurrent crops of papules that may
ulcerate. The papules heal spontaneously over a period of 1-2 months and may leave a depressed
scars. The T-cells in this disorder typically stain positively for CD 30.

396
The causative organism for mucocutaneous leishmaniasis is:
A. Leishmania aethiopica
B. Leishmania infantum
C. Leishmania brasiliensis
D. Leishmania tropica
E. Leishmania major
►C

Mucocutaneous disease is most commonly caused by New World species, although Old World L
aethiopica has been reported to cause this syndrome. Infection by Leishmania viannia

433
braziliensis may lead to mucosal involvement in up to 10% of infections depending on the region
in which it was acquired. Initial infection is characterized by a persistent cutaneous lesion that
eventually heals, although as many as 30% of patients report no prior evidence of leishmaniasis.
Several years later, oral and respiratory mucosal involvement occurs, causing inflammation and
mutilation of the nose, mouth, oropharynx, and trachea.

397
Histologic examination of this chronic pruritic plaque on the back, which of the following stains
would NOT be positive?
A. Von Kossa
B. Cotton dyes
C. Crystal violet
D. PAS
E. Thioflavin T
►A

Macular amyloid is a form of keratin derived amyloid, which is typically located between the
shoulder blades. Many stains are used to identify amyloid in the skin including Congo red, cotton
dyes, crystal violet, PAS and thioflavin T.

398
What is the best muscle to biopsy in dermatomyositis?
A. triceps
B. biceps
C. quadriceps
D. gluteus maximus
E. deltoid
►A

The triceps muscle is involved early in the disease; therefore, it has the highest yield in a muscle
biopsy looking for the changes seen in dermatomyositis. Surgeons have traditionally biopsied the
deltoid mulscle, but it is not involved until late in the disease.

399
What systemic condition is often associated with this disease?
A. Diabetes mellitus
B. Tobacco use
C. Alopecia areata
D. Coronary artery disease
E. Hepatitis C

434
►E

Oral lichen planus is often found in patients with hepatitis C. The association between cutaneous
lichen planus and hepatitis C is not clear.

400
A 62 year-old man with chronic renal failure on hemodialysis presents with carpal tunnel
syndrome, bone cysts, and spondyloarthropathy. A diagnosis of amyloidosis is suspected. Which
of the following is true regarding his most likely diagnosis?
A. AA amyloid is likely causative
B. AL amyloid is likely causative
C. beta 2-microglobulin is likely causative
D. keratin-derived amyloid is likely causative
E. Amyloid P component will not be found in affected tissues
►C

The patient likely has dialysis-related amyloidosis. In this condition, beta 2-microglobulin is the
protein component altered by uremia. Carpal tunnel syndrome, bone cysts, and
spondyloarthropathy commonly result. Amyloid P component and ground substance are found in
all forms of amyloidosis; the protein-derived amyloid fibers differ among the various forms.

401
A teenager comes into your office requesting laser hair removal of her significant facial hair.
Before you agree to treat her, you order which of the following laboratory analyses?
A. FSH and LH
B. Glucose and hemoglobin A1C
C. Ferritin and TIBC
D. Testosterone and DHEA-S
E. TSH and T3
►D

Hisrutism describes excessive terminal hair growth in areas on women that are normally found
only in post-pubescent males (such as beard, chest, inner thigh). Hypertrichosis refers to
excessive hair density or length. Hirsutism is under the influence of androgen stimulation.
Testosterone and DHEA-S can be used to detect excessive adrenal or ovarian androgen
production. Other clinical manifestations of androgen excess in women are acne and virilization.

402
Presence of which of the following autoantibodies is diagnostic of SLE and not reported in
patients with other connective tissue diseases?

435
A. anti-U1RNP
B. anti-dsDNA
C. anti-Ro
D. anti-La
E. anti-Sm
►E

Anti-Sm is diagnostic of SLE and not reported in patients with other connective tissue diseases.
It is found in 15-40% of patients with SLE. Most patients with anti-Sm also have antibodies to
U1RNP, but the converse is not true. anti-U1RNP is found in 100% of patients with MCTD and
in 30% of patients with SLE (the majority of patients with positive U1RNP have SLE rather than
MCTD). Anti-dsDNA correlates with renal involvement in SLE. Anti-Ro and anti-La antibodies
are found in LE and Sjogren's, and strongly associated with photosensitivity.

403
Which of the following disorders is most strongly associated with a monoclonal gammopathy of
the IgG-lambda type?
A. Scleredema
B. POEMS syndrome
C. Necrobiotic xanthogranuloma
D. EED
E. Scleromyxedema
►E

All of the above conditions are assocaited with some type of monoclonal gammopathy.
Scleromyxedema is associated with an IgG-lambda monoclonal gammopathy. Scleredema and
NXG are most often associated with IgG-kappa, and both POEMS syndrome and EED are
usually associated with IgA monoclonal gammopathies.

404
A young woman presents with tender, erythematous nodules an the anterior lower extremities.
Which of the following would not be an appropriate test to consider?
A. TSH
B. ESR
C. ASO
D. Fungal cultures
E. PPD
►A

436
Erythema nodosum can be triggered by several medical conditions, including drugs, IBD,
infections (strep, hepatitis, TB, fungus), sarcoidosis, and malginancy. Thyroid disease is not a
known trigger for erythema nodosum

405
A concerned 23 year old male presents to clinic with multiple verrucoid lesions on his penis. He
is anxious to have them removed. You explain that these are commonly treated cryosurgically.
What is the boiling point of liquid nitrogen in Celsius?
A. -89.5
B. -40.8
C. 3.8
D. -195.8
E. -78.5
►D

The boiling point of liquid nitrogen is -195.8. Nitrous oxide is -89.5. Carbon dioxide is -78.5.
Dichlorotrifluoroethane (Freon 114) is 3.8. Chlorodifluoromethane (Freon 22) is 40.8. Colder
cryogen is needed for adequate deep destruction, making liquid nitrogen the only reliable agent
of the five listed. The Freons, solid CO2 and nitrous oxide are used only for topical anesthesia
and superficial destruction.

406
Syndrome that is associated with disease shown in image is known as
A. SAPHO syndrome
B. Follicular occlusion tetrad syndrome
C. LEOPARD syndrome
D. NAME syndrome
E. PAPA syndrome
►E

PAPA syndrome ( Pyogenic Arthritis, Pyoderma gangerosum and Acne) The image shows
classic ulcerative pyoderma gangrenosum with undermined violaceous gray border. Re-
epithelialization occurs from the margins and the ulcer heal usually with atrophic cribriform
pigmented scar. Although the classic morphologic clinical presentation of pyoderma
gangrenosum is an ulceration, there are several variants (bullous, pustular, and superficial
granulomatous) which differ by their clinical presentation, location, and associated diseases.

407
Increased LDL and VLDL is seen in which type of hyperlipoproteinemia?
A. Type IIb

437
B. Type I
C. Type III
D. Type IV
E. Type V
►A

Type IIb hyperlipoproteinemia is autosomal dominant and features increased LDL and VLDL.
Associated clinical findings include xanthelasma, planar, tendinous and tuberous xanthomas.
Type I hyperlipoproteinemia is autosomal recessive and manifests with increased chylomicrons.
Clinically, patients develop eruptive xanthomas and lipemia retinalis. Type IIa
hyperlipoproteinemia is autosomal dominant. Patients have increased LDL and may develop
tendinous, tuberous, planar or eruptive xanthomas in addition to xanthelasma and arcus juvenilis.
Type III hyperlipoproteinemia is autosomal recessive and features increased IDL. Clinically,
these patients are prone to develop palmar, planar, tendinous, tuberous and eruptive xanthomas.
Type IV hyperlipoproteinemia is autosomal dominant and manifests with increased VLDL.
Patients develop eruptive, tendinous and tuberous xanthomas. Type V hyperlipoproteinemia is
autosomal dominant. Patients have increased chylomicrons and VLDL and may develop eruptive
xanthomas and lipemia retinalis.

408
The most common finding in patients with systemic sclerosis is:
A. Migratory polyarthritis
B. Esophageal dysfunction
C. Pulmonary fibrosis
D. Renal disease
E. Cardiac conduction defects
►B

Esophageal dysfunction is the most common systemic finding in systemic sclerosis. A migratory
polyarthritis is the first manifestation of disease in many. Pulmonary fibrosis, renal disease and
cardiac conduction defects are all manifestations but not the most common.

409
A pregnant woman presents with mild inflammatory acne which is very bothersome to her.
Which of the following is the most appropriate treatment option based on FDA classifications of
medication in pregnancy?
A. Topical erythromycin/benzoyl peroxide gel
B. Topical tretinoin
C. Tazarotene 0.1% cream
D. Azelaic acid 20% cream

438
E. Bactrim
►D

Azeleic acid is category B in pregnancy. The others are category C (benzoyl peroxide, topical
tretinoin, and bactrim) or category X (tazarotene). The categories for safety of drugs in
pregnancies are as follows: Category A: controlled studies in humans show no risk to fetus.
Category B: controlled human studies show no risk to fetus but may show risk to animals, or no
risk in animal studies but no human studies conducted. Category C: risk to human fetus cannot
be ruled out, studies are lacking; animal studies equivocal. Category D: controlled studies show
risk to human fetus, benefits may sometimes outweigh risk. Cateogory X: contraindicated in
pregnancy.

410
Child abuse is often incorrectly suspected when a young girl presents with which cutaneous
finding?
A. Molluscum contagiosum
B. Lichen sclerosus et atrophicus
C. Condyloma accuminata
D. Ecchymoses on face and hands
E. Genital herpes simplex
►B

Lichen sclerosus et atrophicus is an inflammatory disease of the skin that most often occurs oin
the genital region. Clinically, lichen sclerosus may appear as a hypopigmented, well demarcated
patch with a crinkled appearance. It is often pruritic and if left untreated may result in
progressive scarring. The treatment of choice is ultra potent topical steroids.

411
The majority of patients with positive U1RNP autoantibodies have which diagnosis?
A. Mixed connective tissue disorder (MCTD)
B. Rheumatoid arthritis
C. Systemic lupus erythematosus
D. Sjogren's syndrome
E. Dermatomyositis
►C

Anti-U1RNP is found in 100% of patients with MCTD and in 30% of patients with SLE; patients
with SLE and anti-U1RNP also have other positive serologies. The majority of patients with
positive U1RNP have SLE rather than MCTD. The presence of U1RNP autoantibodies is
associated with sclerodactyly, Raynaud's, and esophageal dysmotility.

439
412
A 69 year old man with a history of multiple myeloma develops a deposition disorder of his
heart, tongue, GI tract, and skin. What form of amyloid is most likely present?
A. AL
B. AA
C. Beta 2-microglobulin
D. Keratin
E. Colloid
►A

Primary systemic amyloidosis involves the deposition of protein AL, which is derived from Ig
light chains. AL is also found in nodular amyloidosis produced by a plasmacytoma. 40% of
patients with primary systemic amyloidosis have skin involvement.

413
A premature infant who is being weaned off breast milk develops vesicobullous and eczematous
skin lesions and diarrhea. Which of the following is not another classic precipitant for this
condition
A. Parenteral nutrition
B. Stress (i.e. infection)
C. Diets with mainly cereal grains
D. Liver disease
E. Alcoholism
►D

Zinc deficiency can be seen in premature or term infants being weaned off breast milk, which is
usually high in zinc content, as well as in parenteral nutrition use, alcoholism because of poor
nutritional intake, malabsorption, IBD, diets high in grains containing phytate which binds zinc,
and metabolic stress

414
Necrobiosis is characteristic of which two entities below?
A. Sarcoid and granuloma annulare
B. Sarcoid and necrobiosis lipoidica
C. Sarcoid and annular elastolytic giant cell granuloma
D. Granuloma annulare and necrobiosis lipoidica
E. Granuloma annulare and cutaneous Crohn's disease
►D

440
Necrobiosis is seen in GA and NLD as well as rheumatoid nodules, interstitial granulomatous
dermatitis and palisading neutrophilic and granulomatous dermatitis. Mucin is also present in
GA. NLD shows giant cells, extracellular lipid, and vascular changes.

415
Which of the following medications is not associated with exacerbating this condition?
A. Lithium
B. Prednisone
C. Phenytoin
D. Isoniazid
E. Propranolol
►B

Acne or acneiform erruptions can be caused or exacerbated by corticosteroids, oral


contraceptives, androgens, ACTH, lithium, phenytoin, INH, and haloperidol

416
Which medication used to treat pruritus works by depleting substance P from nerve fibers?
A. Capsaicin
B. Doxepin
C. Diphenhydramine
D. Pramoxine
E. Gabapentin
►A

Capsaicin is an antipruritic and analgesic that works by depleting substance P. Repeated use
leads to decreased heat, pain, and itch sensations. Doxepin is a topical tricyclic antidepressant.
Diphenhydramine is an antihistamine. Pramoxine is a topical anesthetic. Gabapentin, commonly
used to treat notalgia paresthetica, has an unknown mechanism of action.

417
Drug induced psoriasis can be from steroid withdrawal. Other medications known to cause drug
induced psoriasis are all of the following except:
A. Lithium
B. Beta-blockers
C. Interferons
D. ACE inhibitors
E. Hydrocholorthiazide
►E

441
Lithium, beta-blockers, interferons, ACE inhibitors, and granulocyte-colony stimulating factors
have all been known to cause exacerbation of drug induced psoriasis.

418
Which of the following is true regarding the components of amyloid?
A. Ground substance is present only in secondary cutaneous or tumor-associated
amyloidosis
B. The amyloid P component is present in all forms
C. Protein AA is present in primary systemic amyloidosis
D. The amyloid present in secondary systemic amyloid does not lose its
birefringence after treatment with potassium permanganate
E. Protein AL loses its biregringence after treatment with potassium permanganate
►B

All forms of amyloidosis contain amyloid P component and ground substance. The protein-
derived amyloid fibers differ among various forms of amyloid. Protein AL is found in primary
systemic amyloidosis. Protein AA is found in secondary systemic amyloidosis. Secondary
systemic amyloid (AA) loses its birefringence after treatment with potassium permanganate, but
amyloid in primary and localized forms do not.

419
Which of the following receptor is the most important mediator of retinoid activity in the skin?
A. RAR-gamma
B. RXR-alpha
C. RXR-gamma
D. RAR-beta
E. RXR-beta
►A

Retinoid activity is mediated by retinoid receptors. Two groups exist: RA receptors (RAR) and
RX receptors (RXR). Each has three receptor subtypes: alpha, beta, gamma. RAR-gamma is the
most important mediator of retinoid activity in the skin.

420
The lesions shown here are most likely to:
A. Occur in infants and elderly patients
B. Occur as an adverse reaction to medication
C. Be refractory to treatment
D. Follow a streptococcal infection
E. Erupt after the appearance of a ―herald patch‖

442
►D

This slide shows guttate psoriasis.

421
Cutaneous manifestations of vitamin D deficiency include:
A. Alopecia
B. Follicular hyperkeratosis
C. Edema
D. Angular cheilitis
E. Atrophic glossitis
►A

Alopecia is the only cutaneous manifestation of vitamin D deficiency. Edema is the only
cutaneous manifestation of vitamine B1 (thiamine) deficiency (Beriberi). Follicular
hyperkeratosis is seen in hypervitaminosis A. Angular cheilitis is seen in vitamin B2 (riboflavin),
B6 (pyridoxine), folic acid, niacin (vitamin B3 or nicotinic acid), biotin, zinc, and iron
deficiency. Atrophic glossitis is observed in vitamin B2 (riboflavin), vitamin B6 (pyridoxine),
vitamin B12, folic acid, and iron deficiency (among others).

422
A 35 year-old woman presents with predominantly lower facial inflammatory acne, hirsutism,
and irregular menses. She has failed multiple conventional treatments. Laboratory work-up
reveals a very high level of DHEA-S. You are most concerned about:
A. Congenital adrenal hyperplasia
B. Adrenal tumor
C. Polycystic ovary syndrome
D. Toxicity from prolonged use of spironolactone
E. Progestin excess from oral contraceptives
►B

A patient whose acne fails to respond to conventional therapy, whose acne flares cyclically, with
hirsutism, alopecia, or irregular menses warrants an endocrine work-up, including free and total
testosterone, LH, FSH, and DHEA-S. Although rare, very high levels of DHEA-S may suggest
an adrenal adrogen-secreting tumor.

423
A patient has allergic contact dermatitis to paraphenylenediamine. Which of the following
allergens may show a potential cross- reaction?
A. Adhesive

443
B. Sulfa drugs
C. Lidocaine
D. Fragrance
E. Nickel
►B

Paraphenylenediamine (PPD) may cross react with a number of substances, including azo dyes,
benzocaine, procaine, sulfa drugs, and para-aminobenzoic acid (PABA). PPD is often found in
dark hair dyes and in modified henna tattoos. Fragrance, nickel, lidocaine, and adhesive do not
show cross-reaction with PPD.

444
Chapter -5-
Dermatopathology
1
This tumor is best visualized using PAS stains with and without diastase what is it?
A. Clear cell acanthoma
B. Seborrheic keratosis
C. Bowen's disease
D. Tricholemmoma
E. Inverted follicular keratosis
►A

Clear cell acanthoma is composed of pale staining keratinocytes that have increased glycogen
content. The increased glycogen in this tumor is due to a defect in phosphorylase. Histologically
the keratinocytes are mildly enlarged and pale and distinctly separated from the surrounding
epidermis. The epidermis appears focally expanded by an acanthotic plate like growth that spares
the follicular epithelium. Also associated with this tumor are PMNs that extend into the
epidermis. There may be edema of the papillary dermal and some telangiectasis. PAS stain with
and without diastase highlights the abundant glycogen.

2
This slide represents which type of reaction?
A. Arthropod assault
B. Foreign body reaction
C. Phototoxic reaction
D. Infection
E. Collagen vascular disease
►A

An arthropod bite reaction will show superficial and deep eosinophils with associated dermal
edema.

3
On histology, this type of dermatofibrosarcoma protuberans is associated with recurrence and is
regarded as a poor prognosis. It is characterized by a typical herringbone morphology of more
closely packed spindled cells with increased mitosis. This best describes:
A. Bednar tumor
B. Fibrosarcomatous tumor
C. Giant cell fibroblastoma
D. Fibroblastic tumor

445
E. Angiodermatofibrosarcoma tumor
►B

This best describes the fibrosarcomatous type of dermatofibrosarcoma protuberans. It has a high
rate of recurrence and is associated with a poor prognosis. They are also CD34+ and wide
excision is recommended.

4
This is associated with which condition?
A. ochranosis
B. IgA monoclonal gammopathy
C. vasculitis
D. IgG monoclonal gammopathy
E. sepsis
►D

Necrobiotic xanthogranuloma is associated with IgG monoclonal gammopathy which can


progress to myeloma. Patients present with red-yellow plaques with telangiectasias and atrophic
scars. On histology, there are palisaded granulomas around degenerated pink collagen with
cholesterol and fibrin. There is no mucin present.

5
Which of the following lesions demonstrates a pseudo-Darier‘s sign?
A. Mastocytoma
B. Spitz nevus
C. Smooth muscle hamartoma
D. Pilomatricoma
E. Bullous pemphigoid
►C

Smooth muscle hamartomas are benign tumors which arise from smooth muscle of the dermis.
Pseudo-Darier's sign may be elicited due to transient piloerection after rubbing. Histologically,
red-orange bundles and fascicles are present with blunt-ended nuclei.

6
Multiple dermatofibromas are seen in:
A. Cowden's
B. Lobomycosis
C. Incontinentia pigmenti
D. Lupus erythematosus

446
E. Reticulohistiocytosis
►D

Multiple dermatofibromas are seen in lupus erythematosus and immunosuppression/HIV.

7
Sebaceous carcinomas commonly arise on the eyelid. A biopsy of an ocular sebaceous carcinoma
typically reveals:
A. uniform, eosinophilic cells with rare atypia
B. pagetoid cells in the epidermis
C. negative staining with Androgen Receptor (AR)
D. positive staining with Ber-Ep4
E. may be associated with overexpression of MLH-1 and MSH-2
►B

Periocular sebaceous carcinomas are histologically composed of lobules of cell that extend deep
in the dermis and subcutaneous tissue. The cells are pleomorphic, with vacuolated cytoplasm
with moderate atypia. A characteristic feature in ocular tumors is the pagetoid spread of the
tumor in the overlying epidermis. Immunostaining for the androgen receptor has been reported as
a method to determine sebaceous differentiation, and it is helpful in diagnosing poorly
differentiated sebaceous carcinomas, which may lack staining for EMA and other markers. Basal
cell carcinomas, nodular-type are typically positive for Ber-Ep4 while sebaceous tumors are
almost always negative for this marker. Sebaceous carcinomas may be associated with Muir-
Torre syndrome, which is characterized with loss of expression of MLH-1 and MSH-2

8
What diagnosis is shown here?
A. Lipodermatosclerosis
B. Erythema nodosum
C. Pancreatic panniculitis
D. Lupus profundus
E. Erythema induratum
►B

On histology, erythema nodosum is characterized by a septal panniculitis with septal thickening


and granulomatous inflammation.

9
Positive staining with Gross cystic disease fluid protein 15 (GCDFP-15) suggests:
A. An apocrine origin

447
B. An eccrine origin
C. A sebaceous origin
D. A follicular origin
E. None of these answers are correct
►A

Gross cystic disease fluid protein-15 (GCDFP-15) is a commonly used apocrine marker.

10
Granular cell tumors are derived from:
A. Connective tissue
B. Smooth muscle
C. Vascular tissue
D. Neural tissue
E. Adipose tissue
►D

40% of granular cell tumors occur on the tongue. They appear well-circumscribed, raised, firm
nodules. Histologically, the cells are plump and polygonal arranged in nests and cords. Cells are
filled with fine granules representing lysozymes. The tumors are neurally derived and stain with
S-100 and PAS.

11
Which theory best explains the pathogenesis of this disorder?
A. Malignant transformation
B. Autoimmune reaction
C. Infection
D. Reaction to a medication
E. Clonal disorder of keratinization
►E

Porokeratosis is believed to be a clonal disorder of keratinization.

12
Langerhans cells express or are characterized by all of the following except:
A. Chromagranin
B. HLA-DR
C. CD1a
D. Birbeck granules
E. S-100

448
►A

Chromagranin stain neuroendocrine cells, Merkel cellcarcinomas and eccrine glands. They do
not stain Langerhans cells.

13
During an examination, what history would be useful for diagnosing this lesion?
A. History of melanoma
B. History of intestinal polyps
C. History of seizures
D. History of smoking
E. History of dental restorations
►E

An amalgam tattoo develops secondary to traumatic implantation of dental amalgam into


adjacent oral mucosa. Histology reveals the presence of black or brown fine granules in the
dermis.

14
Which test is the first to become reactive?
A. RPR
B. FTA-ABS
C. VDRL
D. ELISA
E. Lumbar puncture
►D

In syphilis, the first test to become reactive is the ELISA, therefore it is the test of choice for
early primary syphilis and congenital disease. It is less useful in late disease due to decreased
IgM. FTA-ABS is the most sensitive with late primary syphilis.

15
What is the most likely diagnosis?
A. Wood splinter
B. suture
C. silicone
D. gel foam
E. Hyaluronic acid
►A

449
Wood splinter is brown with rectangular cells. Suture is typcially a circle filled with many
smaller circles. Silicone is a collection of vacoules. Gel foam is an irregularly shaped purple
mass. Hyaluronic acids are just like mucin wispy and purple collections.

16
These lesions typically occur on which part of the body?
A. Face
B. Hands
C. Mucosa
D. Legs
E. Scalp
►A

Hidrocystomas are cysts lined by a double layer of epithelial cells. They most commonly occur
on the face but can occur in other areas as well.

17
These cells would likely stain with which stain?
A. Von kossa
B. ER+
C. verhoeff von gieson
D. oil red o
E. ulex europeus agglutinin 1
►B

Metastatic breast cancer can be ER + and PR + just like the primary tumor. Von Kossa stains
calcium, verhoeff von gieson stains elastin, oil red o stains fat and ulex europeus agglutinin 1
stains endothelial cells and keratinocytes and is positive in angiosarcoma and kaposi sarcoma.

18
The green color in chloroma is secondary to:
A. Stromelysin
B. Chloracetate
C. Fumarase
D. Myeloperoxidase
E. Alkaline phosphatase
►D

Chloromas are greenish tumor grossly secondary to involvement of the skin in acute granulocytic
leukemia. The green color is secondary to myeloperoxidase.

450
19
On histology of a eccrine ductal carcinoma there are ducts and nest of epithelial cells associated
with a dense fibrous stroma. There is variable nuclear pleomorphism, mitotic activity, and also
positive for CK7 and also:
A. Estrogen and progesterone receptor positive
B. CK20 positive
C. CEA positive
D. p53 positive
E. Keratin positive
►A

In an eccrine ductal carcinoma, the estrogen and progesterone receptor are positive. They are
often positive for CK7, negative CK20, S100 and GCDFP-15 positive.

20
This neoplasm derives from which structure?
A. Apocrine gland
B. Hair follicle
C. Ecrrine duct
D. Subcutaneous fat
E. Muscle
►A

Adenoid cystic carcinoma is considered a malignant apocrine tumor. It appears most commonly
in the oral cavity and has a very unpredictable course.

21
Which gene mutation and/or amplifications are more commonly found in this type of melanoma
and mucosal sites than in melanomas on intermittently sun-exposed sites
A. NRAS
B. KIT
C. GNAQ
D. HRAS
E. PTEN
►B

KIT mutations and/or amplifications are more commonly found in melanomas located on acral
and mucosal sites than in melanomas on intermittently sun-exposed sites.

451
22
Histopathologically, a swiss cheese type appearance is characteristic to which of the following?
A. Silica
B. Tattoos
C. Liquid silicone
D. Sea urchin spines
E. Zirconium
►C

Parafinoma and silicone granuloma both have a swiss cheese type appearance in the dermis to
subcutaneous fat. The remaining options typically appear sarcoidal.

23
In this disorder which typically occurs on the head and neck, the cells would demonstrate which
staining pattern?
A. CD1a + and S100 +
B. CD1a- and S100 -
C. CD1a+ and S100-
D. CD1a- and S100+
E. CD68-
►B

Benign cephalic histiocytosis is a benign eruption that typically occurs on the face of children. It
is a non-Langerhans cell histiocytosis; hence, it is CD1a and S100 negative but is CD68 positive.

24
The predominant location of the cleft in linear IgA is:
A. Dermal
B. Basement membrane zone
C. Basal keratinocytes
D. Suprabasal
E. Subcorneal/granular
►B

Linear IgA bullous dermatosis is an autoimmune disorder that presents with tense blisters along
red annular rings (like a string of pearls). Antibodies are found in the lamina lucida against LAD-
1 antigen in anchoring filaments. The cleft in linear IgA would be found in the basement
membrane zone/subepidermal with direct immunostaining IgA in a linear pattern at the dermal-
epidermal junction.

452
25
Which of the following is sometimes used to stain BCC during Mohs micrographic surgery?
A. Carcinoembryonic Antigen
B. Factor 13A
C. Glial Fibrillary Acidic Protein
D. Desmin
E. Toluidine Blue
►E

The correct answer is toluidine blue. Toluidine blue staining shows dark blue BCC tumor cells
and a magenta-colored stroma around the tumor cells. CEA is used to stain adenocarcinoma,
extramammary Paget's disease, and eccrine neoplasms. Desmin stains muscle. GFAP stains
neurologic cells including astrocytes and Schwann cells. Factor 13A is used to stain
dermatofibroma.

26
This lesion represents a congenital anomaly of which embryonic structure?
A. Third branchial arch
B. Third branchial cleft
C. Fourth branchial arch
D. First branchial arch
E. First branchial cleft
►D

An accessory tragus represents a congenital anomaly of the first branchial arch. They may be
associated with an increased risk of hearing impairment. Histologic features include numerous
vellus hair follicles, subcutaneous tissue, and often, a central cartilaginous core.

27
All of the following are true regarding smooth muscle hamartomas except:
A. Hyperpigmentation
B. Transient piloerection
C. Hypertrichosis
D. Association with ―Michelin tire baby‖ in females
E. Becker‘s nevus syndrome is an example
►D

Smooth muscle hamartomas of the skin may develop from three locations: the arrector pili
muscles, blood vessel walls, and genital/areolar skin. The have been associated with

453
hyperpigmentation, tranisent piloerection ("pseudo-Darier's sign") and hypertrichosis. It is
thought to be the underlying lesion in "Michelin tire baby" syndrome in boys, not girls.

28
The type of necrosis shown in the center of this lesion is which of the following?
A. Fat necrosis
B. Liquefactive necrosis
C. Fibrinoid necrosis
D. Caseous necrosis
E. Coagulative necrosis
►D

Granulomas with central caseous necrosis is a classic histopathologic finding for LMDF.

29
This lesion is characterized by dermal deposition of which material?
A. Amyloid
B. Mucin
C. Fibrin
D. Uric acid
E. Lipid
►D

Gouty tophi are formed by the deposition of monosodium urate crystals. Fibrin is found in
rheumatoid nodules, and mucin is deposited in lesions of granuloma annulare.

30
This neoplasm demonstrates which of the following characteristics?
A. EBV+
B. CD20+, often MUM1/bcl-2+
C. HHV8+
D. CD20-
E. CD4+
►B

B Cell lymphoma, leg type is characterized by a diffuse dermal infiltration of CD20+ cells which
are also usually positive for MUM1/IRF4 and bcl-2.

31
The mutation in the syndrome that is most associated with this lesion is which of the following?

454
A. PTCH
B. APC
C. MSH2
D. PTEN
E. STK11
►D

Cowden's syndrome is a mutation in PTEN and is associated with tricholemmomas and breast,
thyroid and GI carcinoma. They also get acral keratoses and palmar pits and lipomas.

32
The special stain that can show maturation of dermal component and decreases expression with
descent into the dermis in a normal melanocytic lesion is:
A. HMB45
B. Ki67
C. Melanin A
D. MART-1
E. S100
►A

In a normal proliferating melanocytic nevus, HMB45 shows maturation of the dermal component
with decrease expression with descent into the dermis. Ki67 shows low proliferation in the
dermis.

33
Which of the following stains with Ulex europeus agglutinin I?
A. Smooth muscle
B. Eccrine glands
C. Endothelial cells
D. Macrophages
E. Melanocytes
►C

Ulex europeus agglutinin I is a stain which identifies endothelial cells, keratinocytes,


angiosarcomes and Kaposi's sarcoma.

34
This is a culture of a fungus what is it?
A. Rhizopus
B. Phaeohyphomycosis

455
C. Aspergillosis
D. Mucor
E. Absidia
►A

Mucormycosis: Rhizopus the roots are directly under sporangia. This is # 1 cause of localized
and systemic disease due to mucormycosis. Absidia the roots are on either side or the sporangia.
Causes localized infections such as in burns, under dressings and also seen in IV drug users.
Mucor the roots are absent. Mucor more frequently causes disseminated lesions such as
rhinocerebral infections in diabetics with ketoacidosis or in patients with leukemia or
neutropenia. These organisms result in a necrotizing cellulitis or a plaque or nodule with a
necrotic eschar. The organisms invade the wall of blood vessels resulting in thrombosis and
infarction. The organism in tissue displays irregular branching at 90 degree (right angle) and the
hyphae are broad ribbon-like with no septae. The organism can be seen on H & E, but better with
GMS

35
A skin biopsy shows numerous fibroblasts with fibrosis and thickening of the dermis. There is
sparse mucin deposition and on low power the biopsy appears square. Which of the following
paraproteins would you expect to find in this patient?
A. IgG lambda
B. IgA
C. IgM
D. IgG kappa
E. IgA gamma
►A

The description of the biopsy above is that of scleromyxedema. These patients have an
associated IgG lambda paraprotein. Other IgG subtypes may occur but lambda is most common.
Scleromyxedema is a subset of lichen myxedematosus (papular mucinosis). It will present with
coalescent erythematous to yellow papules and plaques. The most common location is the face,
but will occur in many other locations. The surrounding skin is usually sclerodermoid in
appearance. (Bolognia, p648-9)

36
In children with pernio, what may be associated?
A. cryoglobulins
B. diabetes
C. lupus
D. calcinosis cutis

456
E. osteoma cutis
►A

In kids, pernio can be associated with cryoglobulins and cold agglutinins. It is triggered by cold
and wet which results in acral violaceous color with burning and itching.

37
This lesion is part of the constellation of findings in which genodermatosis?
A. Neurofibromatosis I
B. Darier's disease
C. Brooke-Spiegler syndrome
D. Cowden's disease
E. Birt-Hogg-Dube syndrome
►E

Birt-Hogg-Dube syndrome is caused by a mutation of the FLCN gene which encodes a protein
called folliculin. The disorder is characterized by the presence of fibrofolliculomas,
trichodiscomas, and acrochordons.

38
Which form of calcinosis cutis would describe calcinosis scroti?
A. Idiopathic
B. Metastatic
C. Dystrophic
D. Iatrogenic
E. Autoimmune
►A

Localized calcium deposition frequently occurs idiopathically on the scrotum, penis, or vulva.
Metastatic calcification occurs in the setting of abnormal calcium or phosphorus metabolism.
Dystrophic calcification occurs in areas of trauma or other damage to tissue. Iatrogenic
calcification occurs secondary to a treatment or procedure.

39
This is a plakin:
A. BPAg1
B. Plakoglobin
C. Plakophilin
D. Desmocollin
E. Beta-catenin

457
►A

Desmoplakins include desmoplakin 1, BPAg1, envoplakin, and periplakin.

40
Hypopigmented Mycosis Fungoide:
A. Occurs in older patient populations rather than Classical MF
B. Occurs in women more commonly than men
C. Is more likely to occur on the face
D. Does not differ from classic MF in terms of those affected
E. Is more likely to occur in patients with skin types V and VI.
►E

Hypopigmented MF occurs more commonly in younger patients and patients with darker skin
types.

41
The inclusions in infantile digital fibromatosis stain for trichrome and:
A. Phosphotungstic acid hematoxylin
B. Osmium tetroxide
C. Thioflavin T
D. Bodian
E. Pentahydroxy flavanol
►A

Osmium tetroxide stains fat. Thioflavin T stains amyloid. The Bodian stain is for nerves.
Pentahydroxy flavanol is a fluorescent stain for calcium.

42
Which of the following body contains calcium:
A. Negri body
B. Lipschutz body
C. Michaelis-Gutman body
D. Guarnieri body
E. Negri body and Michaelis-Gutman body
►C

The Michaelis-Gutman body is a concentrically laminated spherical inclusion that contains


calcium that is seen within macrophages in malakoplakia. Other inclusion bodies that contain
calcium are Schaumann bodies and psammoma bodies. The Negri body is seen in rabies. The

458
Lipschutz body is an intranuclear inclusion seen in herpes. It is considered synonymous with the
Cowdry A body. The Guarnieri body is seen in smallpox.

43
A 22-year-old man with a diagnosis of Noonan‘s syndrome presents with a slowly enlarging
nodularity on the tongue. Biopsy is most likely to reveal what finding:
A. Psammoma bodies
B. Pustulo-ovoid bodies
C. Russel bodies
D. Verocay bodies
E. Weibel-Palade bodies
►B

The most likely diagnosis is a granular cell tumor, which most commonly arises on the tongue
and is associated with Noonan‘s syndrome. Biopsy would reveal intracellular granules along
with pustule-ovoid bodies representing phagolysosomes. Psammoma bodies are concentrically
laminated calcified bodies seen in meningioma, ovarian, and thyroid neoplasms. Russel bodies
are immunoglobulin inclusions in plasma cells often seen in Rhinoscleroma. Verocay bodies are
palisading nuclei arranged in rows with peripheral eosinophilic cytoplasm characteristic of
Schwannomas. Weibel-Palade bodies are organelles that are seen on macroscopy of endothelial
cells.

44
Which of the following would suggest a diagnosis of malignant lymphoma, B-cell type over
pseudolymphoma, B-cell type?
A. Mixed-cell infiltrate
B. Mitotic figures only in germinal centers
C. Histoicytic giant cells episodically
D. Larger lymphocytes predominate
E. Prominant vasculature with thick-walled blood vessels lined by plump
endothelial cells
►D

Histological findings in pseudolymphoma, B-cell type include a mixed-cell infiltrate (important


diagnostic criterion), small lymphocytes (that predominate), the presence of histiocytes and giant
cells, mitotic figures limited to the germinal centers and prominant vasculature.

45
The mutation that most likely resulted in this lesion is which of the following?
A. PTCH

459
B. CDKN2A
C. VEGF
D. P53
E. RET
►D

P53 is the most common mutation in squamous cell carcinoma. This SCC is located on the lip.
PTCH is mutated in BCCs and CDKN2A is mutated in familial melanomas.

46
This tumor was excised from the scalp of an older woman, what is this neoplasm?
A. Cylindroma
B. Sebaceous carcinoma
C. Pilomatricoma
D. Basal cell carcinoma
E. Spiradenoma
►A

Cylindroma: Most often seen in middle age females on the scalp. Multiple lesions can be
associated with trichoepitheliomas, spiradenoma and adenomas of parotid gland in Brooke
Spieler syndrome. Histologically this is a poorly circumscribed dermal tumor with multiple small
irregularly shaped basophilic islands often likened to puzzle pieces composed of a peripheral
layer of smaller darker cells, with cells that have increased eosinophilic cytoplasm centrally.
Each island is surrounded by a thick eosinophilic cuticle (type IV and VII collagen), with similar
eosinophilic hyaline droplets seen within the island.

47
On which anatomic location does this neoplasm typically occur?
A. Scalp
B. Lower legs
C. Arms
D. Oral mucosa
E. Trunk
►B

Clear cell acanthomas tend to occur on the lower legs of middle-aged or and elderly individuals.

48
A patient with this neoplasm should be evaluated for loss of which genes?
A. APC

460
B. MSH2,MLH1, MSH6, PMS2
C. PTEN
D. STK11
E. MEN1
►B

The image is a sebaceous carcinoma. Sebaceous carcinomas are associated with muir torre
syndrome which is due to a loss of mismatch repair genes MSH2, MLH1, MSH6 and PMS2. The
biopsy tissue blocks can be evaluated for the loss of these genes.

49
A too superficial biopsy of this neoplasm could be misdiagnosed as which lesion?
A. Desmplastic trichoepithelioma
B. Cylindroma
C. Spiradenoma
D. Syringoma
E. Chondroid syringoma
►D

Superficial biopsies of microcystic adnexal carcinoma are sometimes misdiagnosed as


syringomas. Microcystic adnexal carcinomas demonstrate ductal structures and cords in a fibrous
stroma. They extend deep into the dermis and even subcutis.

50
What type of nevus is demonstrated in this image?
A. Spitz
B. Blue
C. Dysplastic
D. Halo
E. Congenital
►A

Spitz nevi typically occur as pink or skin-colored nodules in children although they can occur at
any age. They are composed of epithelioid or spindle cells. They demonstrate symmetry and
downward maturation of melanocytes.

51
This neoplasm occurs most frequently in what anatomic location?
A. Leg
B. Mucosa

461
C. Acral
D. Head and neck
E. Trunk
►D

Desmoplastic melanoma is spindle cell variant of malignant melanoma. It typically occurs in the
head and neck region in older individuals. It demonstrates a high recurrence rate but lower rates
of metastasis.

52
This lesion represents a congenital anomaly of which embryonic structure?
A. Third branchial arch
B. Third branchial cleft
C. Fourth branchial arch
D. First branchial arch
E. First branchial cleft
►D

An accessory tragus represents a congenital anomaly of the first branchial arch. They may be
associated with an increased risk of hearing impairment. Histologic features include numerous
vellus hair follicles, subcutaneous tissue, and often, a central cartilaginous core.

53
On histology, there are large amounts of mucin deposited within reticular dermis, resulting in
separation of collagen bundles. Clinically is can be seen in the lower legs associated with Grave's
disease. This best describes:
A. Pretibial myxedema
B. Scleredema
C. Follicular mucinosis
D. Lichen myxedematosus
E. Scleromyxedema
►A

This best describes pretibial myxedema and can be associated with Grave's disease. The skin is
diffusely doughy with skin thickening with nonpitting indurated plaques and nodules seen
bilaterally. The skin surface is shiny and sometimes seen a peau d'orange appearance.

54
What is the diagnosis?
A. Kaposi sarcoma

462
B. Hemangiopericytoma
C. Angiosarcoma
D. Kaposiform hemangioendothelioma
E. Tufted angioma
►B

The adult type hemangioperictyoma is characterized by the presence of tightly packed cellular
areas surrounding staghorn vessels.

55
The organisms highlighted by this GMS stain are?
A. Blastomycosis
B. Coccidioidomycosis
C. Mucormycosis
D. Candida albicans
E. Histoplasmosis capsulatum
►A

Blastomycosis: Gilchrist disease, Bird droppings or spores on wood along Mississippi basin,
incubation 33-44 days. Most infections develop in previously healthy people. In patients with
HIV, 46% will get CNS symptoms, which is 5-10x greater than non-HIV pts. 4 forms,
pulmonary, disseminated (CNS, bone, skin and GU) with secondary cutaneous lesions and
primary inoculation (rare-1-2 weeks after exposure-sporotrichoid pattern- mixed infiltrate
numerous budding organisms). Lesions more typically are verrucous or ulcerated plaques in
males on exposed skin-face. Histology: Pseudoepitheliomatosis hyperplasia overlying acute
dermal inflammation usually only a few organisms in center of abscess with thick walled 7-15
microns, single broad based bud, has a WI-1 adhesion protein and are best seen with PAS or
GMS.

56
This lesion is composed of which material?
A. Collagen
B. Adnexal tissue
C. Muscle
D. Brown fat
E. Neural tissue
►D

A hibernoma is thought to derive from brown fat in the body.

463
57
The predominant location of the cleft in transient neonatal pustular melanosis is:
A. Dermal
B. Basement membrane zone
C. Basal keratinocytes
D. Suprabasal
E. Subcorneal/granular
►E

Transient neonatal pustular melanosis is an idiopathic pustular eruption of newborns, mostly on


the chest, that heals with hyperpigmentation. It is most common on pigmented individuals.
Histologically it presents as subcorneal pustules with eosinophils and neutrophils.

58
Degeneration of cartilage in chondrodermatitis nodularis helices affects which type of collagen?
A. Type I collagen
B. Type II collagen
C. Type III collagen
D. Type IV collagen
E. Type VII collagen
►B

Chondrodermatitis nodularis helices presents as a tender nodule usually on the helix or antihelix
of the ear. It is thought to be a result of vascular insufficiency. Degeneration of the cartilage is
seen on histopathology which is primarily composed of type II collagen.

59
What is the best diagnosis for this vesiculobullous disorder?
A. Bullous Pemphigoid
B. Porphyria Cutanea Tarda
C. Pemphigus vulgaris
D. Dermatitis Herpetiformis
E. Arthropod Bite
►A

Bullous Pemphigoid: Most common subepidermal blistering disease that frequently occurs on
abdomen, groin, flexor surface of arms and legs of the elderly. In men an association with HLA-
DQ7 has been seen. Often eosinophilia and elevated IgE is seen in the serum. The pathogenesis
of this blistering disorder is due to binding of antibodies, IgG1 and 4 or IgE and rarely IgA, to
BPAg I (230kd) and less commonly BP Ag II (180kd) in the hemidesmosome. Histologically

464
there is a unilocular subepidermal blister with eosinophils. Early lesions may show eosinophilic
spongiosis with papillary dermal edema. Within the dermis there is a lymphocytic eosinophilic
infiltrate that is both perivascular and interstitial. DIF shows linear deposits of C3> IgG along
the DEJ. Salt split skin show localization of immunoreactants in the vesicle roof. Circulating
anti-BMZ antibodies are seen in 60-80% of patients but do not correlate to disease activity.

60
Paraproteinemia is associated with all except:
A. Scleromyxedema
B. Necrobiotic xanthogranuloma
C. Scleredema
D. Sclerosing panniculitis
E. Plane xanthoma
►D

Sclerosing panniculitis (lipodermatosclerosis) displays characteristic changes in the fat


(lipomembranous change); it is not associated with paraproteinemia. Generalized plane
xanthomas, scleromyxedema, necrobiotic xanthogranuloma, scleredema, erythema elevatum
diutinum, xanthoma disseminatum, and pyoderma gangrenosum have all been associated with a
paraproteinemia.

61
Lipomembranous change is seen in:
A. Hibernoma
B. Cystic sebaceous adenoma
C. Sebaceous carcinoma
D. Mucocele
E. Sclerosing panniculitis
►E

Lipomembranous change is a non-specific histologic pattern that is most commonly seen in


lipodermatosclerosis, which is also known as sclerosing panniculitis; this condition may be
secondary to venous stasis.

62
Which of the following histologic features is seen in aging skin?
A. Thickened dermal-epidermal junction
B. Increased mast cells
C. Increased number of terminal hairs
D. Fewer Langerhans cells

465
E. Increased sebum production
►D

Histologic features of aging epidermis include flattened dermo-epidermal junction, occasional


nuclear atypia, decrease in the number of melanocytes and Langerhans cells. Changes that are
present in the dermis include atrophy, decrease in fibroblasts, mast cells and blood vessels.

63
This is considered to be the juvenile counterpart of DFSP:
A. Juvenile hyaline fibromatosis
B. Giant cell fibroblastoma
C. Myxofibrosarcoma
D. Myofibromatosis
E. Plexiform fibrous histiocytoma
►B

An entity called giant cell fibroblastoma is CD34-positive, is mostly seen in male children on the
neck/trunk, and is thought to be a juvenile counterpart of DFSP.

64
What is the diagnosis?
A. Sarcoidosis
B. Tuberculosis
C. Rosacea
D. Leprosy
E. Granulomatous leishmaniasis
►E

Long-standing lesions of chronic cutaneous leishmaniasis will often show granulomatous


inflammation with scarce organisms.

65
What type of lesion is shown here?
A. Chondroid syringoma
B. Hidradenoma
C. Trichoblastoma
D. Cylindroma
E. Spiradenoma
►E

466
On histology, a spiradenoma is a dermal nodule composed of basaloid cells in a trabecular
pattern. Two cell types are usually present: small basaloid cells and larger cells with pale nuclei.

66
A patient has a yellow to pink papule with epitheloid melanocytic cells with nests that are large
and associated with retraction artifact. The nest and cell sizes diminish with depth. Also,
eosinophilic cytoplasmic extracellular globules are seen in the junctional region and are called:
A. Kamino bodies
B. Cytoid bodies
C. Nuclear bodies
D. Spitz bodies
E. Von Kossa bodies
►A

The above lesions describes a spitz nevus. They occur most commonly in children and seen on
the face, trunk, or extremities. An atypical spitz nevus is regarded as having an uncertain
malignant potential. Kamino bodies are seen in Spitz nevus that are eosinophilic cytoplasmic
extracellular globules in the junctional region.

67
A patient with a blue-red discoloration of the nail plate reports that the same finger becomes very
tender when exposed to the cold. You suspect a:
A. Periungual verruca
B. Glomus tumor
C. Pterygium
D. Mucous cyst
E. Pyogenic granuloma
►B

Glomus tumors are tumors of the arterio-venous anastamosis of the digital dermis. They occur
most frequently in the nail bed. The commonly have a bluish-red discoloration and may be
tender or painful with exposure to heat or cold.

68
Which of the following is characteristic of PLEVA?
A. Alternating orthokeratosis and parakeratosis
B. Red cell extravasation
C. Dermal mucin deposition
D. Fibrinoid necrosis of medium sized vessels
E. Numerous eosinophils and neutrophils

467
►B

Pityriasis lichenoides et varioliformis acuta has characteristic findings. Parakeratosis, spongiosis


with vacuolar alteration, papillary dermal edema, wedge shaped infiltrate, and extravasated
RBC's can all be seen. Occasionally, the infiltrate can include atypical lymphoid cells.
Eosinophils and neutrophils are more frequently seen in LyP.

69
What HPV strain causes this lesion?
A. HPV 2
B. HPV 6
C. HPV 5
D. HPV 7
E. HPV 1
►E

Myrimecial warts are endophytic growths on the soles of the feet. They are caused by HPV 1.
There are numerous eosinophilic inclusions within the keratinocytes.

70
Eccrine glands are found on the:
A. Labia majora
B. Labia minora
C. Glans
D. Prepuce
E. Vermilion
►A

Eccrine glands are present all over the body except on the vermilion of lips, glans, labia minora,
nail beds, and inner prepuce.

71
There is a red-yellow, smooth firm lobulated papule that is associated with Muir-Torre syndrome
on the face of a patient. On histology, there are lobulated sebaceous tumors often with a
connection to the epidermis. There are mature sebocytes (>50%) within a predominantly
basaloid cell population. This describes a:
A. Sebaceoma
B. Sebaceous adenoma
C. Sebaceous carcinoma
D. Sebaceous hyperplasia

468
E. Basal cell carcinoma with sebaceous differentiation
►A

This describes a sebaceoma and is located on the head and neck on the face and scalp. It is
associated with Muir-Torre syndrome with a mutation in the MSH2 and MLH1. It is a benign
tumor and surgical excision is curative with little chance of recurrence.

72
This neoplasm tends to occur most frequently on which anatomic region?
A. Sun-exposed areas of the head and neck
B. Genital region
C. Acral sites
D. Oral mucosa
E. Extremities
►A

Lentigo maligna tends to occur most frequently in the head and neck region in sun-exposed
areas.

73
Which of the following is true regarding pilomatricomas?
A. Calcification is rarely seen
B. Poorly demarcated tumor
C. Comprised of shadow cells and basophilic cells
D. Mutations found in the APC gene
E. Frequent malignant degeneration
►C

Pilomatricomas are well-defined lobular tumor in dermis or subcutis comprised of germinative-


matrical cells that are basaloid and transition to "ghost" or "shadow" cells. Calcifiation or
ossification are frequently seen. Mutations are found in beta catenin.

74
Comma-shaped bodies are seen in:
A. Benign cephalic histiocytosis
B. Malakoplakia
C. Sarcoidosis
D. Lipoid proteinosis
E. Gaucher‖s
►A

469
Comma-shaped bodies and worm-shaped bodies can be seen in a variety of histiocytoses and are
non-specific. The most common association is with benign cephalic histiocytosis.

75
The most common cause of a longitudinal groove of the nail plate is:
A. Myxoid cyst
B. Verruca vulgaris
C. Psoriasis
D. Onychomycosis
E. Paronychia
►A

A myxoid cyst is not a true cyst as it is devoid of cyst lining. They usually occur in an acral
location and are the most common cause of a longitudinal groove of a nail plate.

76
What neoplasm is pictured here?
A. Trichoadenoma
B. Tumor of the follicular infundibulum
C. Cutaneous lymphadenoma
D. Desmoplastic trichoepithelioma
E. Syringoma
►D

Desmoplastic trichoepitheliomas are composed of small cords and islands of basaloid cells
within a fibrous stroma. Keratinous cysts are often present.

77
Which type is most common in children?
A. Type 1
B. Type 4
C. Type 3
D. Type 2
E. Type 5
►B

In PRP, type 4, circumscribed juvenile is the most common form in kids. Type 1 is classic adult.
Type 2 is atyical, chronic adult. Type 3 is classic juvenile, type 5 is atypical juvenile.

470
78
What is your best diagnosis?
A. Lichen Trichophyticus
B. Secondary Lues
C. Lichen Nitidus
D. Scabies
E. Acral Hyperkeratosis
►C

A chronic inflammatory disease characterized by shiny, flat-topped, usually flesh-coloured


micropapules no larger than the head of a pin. Lesions are localized in the early stages, found
chiefly on the lower abdomen, penis, and inner surface of the thighs. Distribution may become
generalized as the disease progresses.

79
A patient presents with a brown patch that has darker macules within it and this pathology. What
is the best diagnosis?
A. Nevus spilus
B. linchen nitidus
C. lichen striatus
D. nevus comedonicus
E. agminated nevus
►A

Nevus spilus is a tan macule/patch with darker smaller maclues scattered within it. On pathology,
you see an increased number of melanocytes in a lentigenous pattern along the DEJ with
increased nests in focal areas. In an agminanated nevus there are groups of brown macules on
normal skin. There is no background brown macule differentiating it from nevus spilus.

80
Calcium can be seen this stain:
A. Von Kossa
B. Mucicarmine
C. Grocott
D. Warthin Starry
E. Melanin A
►A

Calcium is stained with Von Kossa and it stains it black. Mucicarmine stains mucin red. Grocott
stains fungus cell walls black and Warthin-Starry stains spirochetes black.

471
81
On histology this disease has mucin deposition within the external root sheath and sebaceous
gland. The mucin distends into the intracellular spaces within follicular epithelium forming
cystic cavities. The following stains will be positive except for:
A. CD3
B. CD4
C. CD7
D. CD8
E. CD9
►E

This patient has follicular mucinosis. This is benign and seen in children and is a nonscarring
alopecia and may be pruritic or tender seen on the face and scalp. All the following stains will be
positive CD3, CD4, CD7, CD8.

82
This eruption responds very well to which medication?
A. Dapsone
B. Gluten free diet
C. Terbinafine
D. Fluconazole
E. Vancomycin
►A

Bullous systemic lupus erythematosus responds very well to dapsone therapy.

83
What is this neoplasm?
A. Sclerotic fibroma
B. Angiofibroma
C. Keloid
D. Neurofibroma
E. Dermatofibroma
►A

Sclerotic Fibroma (circumscribed storiform collagenoma): This can occur as an isolated lesion n
the head or neck or upper extremities, but multiple lesions are associated with Cowden's disease
which is an AD disorder linked to a mutation of PTEN on chromosome 10p. Histology: Dome
shaped papule with an overlying attenuated epidermis. A well-circumscribed nodule composed

472
of thickened homogenous collagen that is arranged in whorls with a thumbprint or grains of
wood appearance is seen in the dermis. Overall the lesion exhibits low cellularity and the elastic
fibers are absent. The tumor stains positively with vimentin, Factor XIIIA and focally for CD34.

84
This biopsy was obtained from a Filipino woman who lives in Southern California, what is this
organism?
A. Coccidioidomycosis
B. Rhinosporidium
C. Prototheca wickerhamii
D. Histoplasmosis capsulatum
E. Cryptococcus neoformans
►A

Coccidioidomycosis: Found in nature in the soil of desert areas of the South West, also known as
―Valley Fever‖. Usually presents as a self-limited lung infection, but in less 1% of people
dissemination can occur. The risk is highest in immunocompromised people, and there is a
higher risk in Mexicans, Filipinos, African Americans, American Indians, and in pregnancy.
Systemic illness can be associated with eosinophilia, arthritis and rarely hypercalcemia.
Disseminated cutaneous lesions present as papulopustules on nasolabial fold or verrucous plaque
on face. Approximately 20% of patients with pulmonary infections develop a hypersensitivity
reaction to the infection resulting in erythema nodosum. Primary inoculation is rare and usually
occurs in farmers and can present with a sporotrichoid pattern. Histology: Pseudoepitheliomatous
hyperplasia of the epidermis overlying a suppurative granulomatous dermatitis that often
contains numerous eosinophils in the inflammatory infiltrate. Within the granulomas are thin
walled sporangia measuring 10-80 microns that often contain multiple endospores. The organism
can usually be easily seen on H & E, but use of PAS and Congo red stains highlights the spores.
The classic appearance on culture is alternating light and dark chains of organisms with a box
car-like appearance.

85
Which HLA genotype is strongly associated with this disorder?
A. HLA-DR3
B. HLA-DRB1
C. HLA-B27
D. HLA-DQ2
E. HLA-DQB1
►D

473
90% of patients with dermatitis herpetiformis and celiac disease have the HLA class II DQ2
genotype.

86
This disorder has an association with which infection?
A. HIV
B. Syphilis
C. Hepatitis C
D. Borreliosis
E. Group A Strep
►C

Lichen planus has been associated with hepatitis C virus infection.

87
What material is present in the dermis in this image?
A. Hyaluronic acid filler
B. Cartilage
C. Bone
D. Calcium
E. Urate
►D

This image demonstrates calcinosis in the dermis.

88
This condition frequently occurs on which part of the body?
A. Abdomen
B. Forearms
C. Scalp
D. Neck
E. Finger
►B

Actinic purpura typically occurs in older individuals on sun-damaged, traumatized skin such as
the forearms.

89
What is a test that can be used to help differentiate primary extramammary Paget's and pagetoid
metastasis to the skin from underlying colon cancer?

474
A. Extramammary Paget's disease is carcinoembryonic antigen (CEA) positive.
B. Extramammary Paget's disease is carcinoembryonic antigen (CEA) negative.
C. Extramammary Paget's disease is gross cystic disease fluid protein-15 positive.
D. Extramammary Paget's disease is gross cystic disease fluid protein-15 negative.
E. Extramammary Paget's disease is EMA positive.
►C

Extramammary Paget's disease is gross cystic disease fluid protein-15 positive, whereas pagetoid
metastasis to the skin from underlying colon cancer is not. Extramammary Paget's disease also is
CK7 positive. Pagetoid cells may often be stained positive with both CEA and EMA.

90
Which of the following stains Natural Killer Cells?
A. CD1a
B. CD11
C. CD30
D. CD56
E. CD68
►D

CD56 stain Natural Killer Cells and angiocentric lymphomas.

91
What neoplasm is shown here?
A. Malignant melanoma
B. Blue nevus
C. Bednar tumor
D. Desmoplastic melanoma
E. Atypical fibroxanthoma
►C

A Bednar tumor is a dermatofibrosarcoma protuberans that contains melanin-producing cells.

92
What is the diagnosis of this neoplasm in infants?
A. Infantile hemangioperictyoma
B. Tufted angioma
C. Kaposiform hemangioendothelioma
D. Dermatomyofibroma
E. Giant cell fibroblastoma

475
►A

Infantile hemangiopericytomas are benign lesions. They are multilobulate, with perivascular and
intravascular tumors outside the main tumor mass.

93
What substance is found histologically in talon noir to be the source of the pigment seen
clinically?
A. Hemoglobin
B. Hemosiderin
C. Melanin
D. Carbon
E. Keratin
►A

Talon noir is caused by trauma and hemorrhage trapped in the stratum corneum. It can clinically
appear pigmented and is often biopsied to rule out melanoma. Because phagocytosis of
extravascular RBCs and subsequent degradation of hemoglobin to hemosiderin does not occur in
the stratum corneum, traditional iron stains do not work and histochemical stains must be
directed toward hemoglobin.

94
Which medication is very frequently associated with this eruption?
A. Vancomycin
B. Furosemide
C. Warfarin
D. Methotrexate
E. Isotretinoin
►B

Furosemide is commonly associated with drug-induced bullous pemphigoid. Other responsible


agents include penicillin and its derivatives and sulfasalazine.

95
A male patient presents with scattered pink-purple papules on the lower legs. He is otherwise
healthy. What is your diangosis based on this path?
A. pigmented purpuric lichenoid dermatosis of gougeroit and blum
B. mastocytosis
C. kaposi sarcoma
D. lichen nitidus

476
E. mycosis fungoides
►A

Pigmented purpuric lichenoid dermatosis of gougeroid and blum is easily identified on pathology
by a lichenoid infiltrate with deep melanophages and a few necrotic keratinoctytes. It can look
similar to EM but is more localized and the melanophages are much deeper in the pigmented
purpuric dermatosis. Kaposi sarcoma is on the differential clinically, however on path it is not a
lichenoid infiltrate and therefore is easily ruled out.

96
This was excised from the face, what is this neoplasm ?
A. Syringoma
B. Trichoadenoma
C. Basal cell carcinoma
D. Desmoplastic trichoepithelioma
E. Microcystic adenocarcinoma
►A

Syringomas on the eyelids, cheeks, chests and can be eruptive and increased numbers in Down's
syndrome and a clear cell change has been associated with diabetes. Histologically this is a
dermal tumor consisting of eccrine ducts, lined by two cell layers sometimes creating a tad pole
appearance, there can also be small basaloid islands or strands of cells. Within the lumens of the
ducts is eosinophilic material, the lumens are CEA+. The tumor is located superficially and fails
to display perineural extension as is seen in MAC. The glands are associated with a fibrous
sclerotic stroma, usually no (or minimal) keratin cysts or foreign body granulomas as seen in
desmoplastic trichoepithelioma. Enlargement of the glandular cells more of a clear or vacuolated
cytoplasm can be seen in patients with diabetes.

97
CK20 stains this tumor in what type of pattern?
A. Perinuclear dot
B. homogeneous
C. speckeled
D. double dot
E. centromere
►A

Merkel cell carcionma is stained by CK20 in a perinuclear dot pattern.

98

477
In a halo nevus, there is diffuse lymphohistiocytic infiltrate with melanocytes. There are dermal
melanophages and mild dermal fibrosis. The marker used to label the lymphocytes but not
melanocytes is:
A. Ki-67
B. Melanin A
C. S100
D. CD1A
E. MART-1
►A

Ki-67 labels the lymphocytes but not the melanocytes in a halo nevus. It usually presents with a
brown papule with a white depigmented rim and the back is the most common site.

99
This is an image of a very early lesion for the patient who presented covered in blisters. What is
your most likely diagnosis?
A. Urticarial BP
B. pemphigus vulgaris
C. pemphigus vegetans
D. pemphigus foliaceus
E. fogo sevalgum
►A

There is a dense infiltrate of eosinophils and there appears to be the beginning of a subepidermal
split as seen in bullous pemphigoid. The remainder of the choices are all suprabasal splits.

100
Stromelysin 3 is a negative marker for which of the following?
A. Dermatofibroma
B. Basal cell carcinoma
C. Dermatofibromasarcoma protuberans
D. Squamous cell carcinoma
E. Breast carcinoma
►C

Stromelysin 3 is a negative marker which helps to distinguish dermatofibrosarcoma protuberans


from dermatofibromas. Stromelysin 3 is a metalloproteinase which is expressed tissue
remodeling. In a study performed by Cribier et.al. 100% of dermatofibromas stained positive
Stromelysin 3 (ST3) is a member of the metalloproteinase family, which is expressed in tissue
remodeling processes such as scarring, embryogenesis, or tumoral invasion.

478
101
What is the diagnosis?
A. Lichen planus-like keratosis
B. Malignant melanoma
C. Syphilis
D. Spitz nevus
E. Halo nevus
►E

A halo nevus is characterized by a symmetric proliferation of nevus cells in the dermis,


surrounded by an intense lymphocytic infiltrate.

102
What infectious agent is most likely responsible for a reaction of fibrin and antibodies which
help to prevent phagocytosis?
A. Nocardia
B. Ricketsii species
C. Actinomycosis
D. Candida albicans
E. Anthrax
►C

Hoeppli-Splendore reaction is characterized histologically by intensely eosinophilic material


consisting of fibrin and antibodies. Causes of the phenomenon include Actinomycosis israelii,
Staph aureus, Proteus, Pseudomonas and E. coli.

103
An increased number of miniaturized hairs are seen in:
A. Lichen planopilaris
B. Alopecia areata
C. Trichotillomania
D. Telogen effluvium
E. Lichen planopilaris and alopecia areata
►B

In alopecia areata, especially early stages, increased miniaturized catagen hairs can be seen in
addition to the finding of peribulbar lymphocytes resembling a ―swarm of bees.‖ Lichen
planopilaris is a scarring alopecia in which vertical tracts of fibrosis are seen in place of follicles.
Trichotillomania displays follicular plugging, trichomalacia, pigmented casts, hemorrhage, and

479
increased catagen hairs on biopsy. Telogen effluvium is characterized by an increased number of
telogen hairs.

104
Which genetic abnormality has been found in the cells of this neoplasm?
A. t(17;22)
B. BRAF V600E mutation
C. NRAS mutation
D. t(9;22)
E. KIT mutation
►A

Dermatofibrosarcoma protuberans has been associated with reciprocal translocations of


chromosomes 17 and 22, t(17;22). This rearrangement fuses the collagen type I alpha 1
(COL1A1) and the PDGF-beta chain (PDGFB) genes.

105
This lesion typically occurs on which anatomic location?
A. Sole of foot
B. Leg
C. Trunk
D. Scalp or upper half of the face
E. Arm
►D

Idiopathic cutaneous cases of angiosarcoma most frequently occur in the head and neck region,
specifically the scalp and upper half of the face in elderly individuals. On histology, vascular
channels are lined by atypical endothelial cells.

106
What syndrome is associated with multiples of these tumors?
A. Rubenstein Taybi
B. Rothmund thompson
C. neurofibromatosis
D. Rombo syndrome
E. sturge weber
►A

480
Rubenstein taybi is associated with multiple pilomatrichomas. It is due to a sporadic mutation in
the CREB binding protein which is responsible for the coactivation of cAMP regulated gene
expression. They also get broad thumbs, cryptorchidism and congenital heart defects.

107
Which of the following sets of special immunohistochemical stains would help differentiate an
atypical fibroxanthoma (AFX) from a malignant fibrous histiocytoma (MFH)?
A. CD74 and CD99
B. CD34 and Stromelysin-3
C. HMB45 and p75NPR
D. CK20 and TTF-1
E. CK20 and GCDFP-15
►A

CD74 (LN2) and CD99 help differentiate an AFX from an MFH where an AFX is CD74-
,CD99+ and an MFH stains weakly from CD99 and is CD74 negative. CD34 and Stromelysin-3
differentiate dermatofibromas and DFSPs. HMB45 and p75NPR are stains helping to
differentiate melanomas from desmoplastic melanomas. CK20 and TTF-1 staining differentiates
a merkel cell carcinoma and metastatic small cell lung carcinoma. CK20 and GCDFP-15 staining
helps distinguish primary and secondary (assoc with underlying neoplasm) Paget's disease.

108
What is the diagnosis?
A. Pigmented purpuric dermatosis
B. Livedoid vasculopathy
C. Hematoma
D. Leukocytoclastic vasculitis
E. Thrombotic vasculopathy
►C

A hematoma is defined by the presence of extravasated erythrocytes.

109
Weibel-Palade bodies are seen in:
A. Endothelial cells
B. Spitz Nevi
C. Cells infected with MCV
D. Plasmacytoid Cells
E. Malakoplakia
►A

481
Weibel-Palade bodies are seen in endothelial cells and are therefore found in vascular lesions.
Kamino bodies are found in Spitz nevi. Henderson Patterson bodies are seen in molluscum.
Dutcher bodies are intranuclear inclusions seen in plasmacytoid cells. Michaelis Gutmann bodies
are partially digested bacteria seen in malakoplakia.

110
Subcutaneous panniculitis-like T-cell lymphoma with an indolent course is positive for which of
the following?
A. CD4
B. CD8
C. CD10
D. CD41
E. CD57
►B

There seems to be two subsets of subcutaneous panniculitis-like T-cell lymphoma. One has an
indolent course, and is often CD8-positive and is positive for the alpha-beta T-cell receptor. The
other has an aggressive course, sometimes with evidence of systemic hemophagocytosis and
high mortality. This subset is CD56-positive and positive for the gamma-delta T-cell receptor.

111
What diagnosis does this image show?
A. Langerhans cell histioctyosis
B. Malignant melanoma
C. Cutaneous T-cell lymphoma
D. Merkel cell carcinoma
E. Metastatic breast cancer
►A

On histology, langerhans cell histiocytosis shows sheets of large, ovoid cells with an eosinophilic
cytoplasm and reniform nucleus.

112
Caterpillar bodies are seen in:
A. Lipoid proteinosis
B. Amyloidosis
C. Porphyria cutanea tarda
D. Mucocele
E. Dyskeratosis congenital

482
►C

Caterpillar bodies are thought to be type IV collagen.

113
All have been associated with increased risk of breast cancer except:
A. Cowden‘s
B. Multicentric reticulohistiocytosis
C. Peutz-Jeghers
D. Birt-Hogg-Dube
E. ataxia telangiectasia
►D

Birt-Hogg-Dube is associated with renal cancer and thyroid cancer. Female carriers of a mutated
ATM (homozygous mutations ATM cause ataxia telangiectasia) have an increased risk of breast
cancer.

114
This eruption is mediated by antibodies to which structure?
A. Desmoglein 1
B. Desmoplakin
C. Collagen VII
D. BP180
E. Desmoglein 3
►D

Bullous pemphigoid is most often associated with antibodies to BP180.

115
Histologic examination of cutaneous metastases from breast cancer is likely to demonstrate:
A. Dense lymphocytic infiltration
B. Tumors cells perivascularly
C. Eosinophilia
D. Tumor cells in the lymphatics
E. Band-like dermal infiltrate
►D

Histologic appearance of metastatic breast carcinoma to the skin typically shows tumor cells in
linear arrangement in so-called "Indian-filing" pattern. These tend to occur in the dermis and

483
subcutaneous lymphatics. Extensive lymphatic dissemination may be caused by retrograde
lymphatic spread. The tumor cells are large, pleomorphic with hyperchromatic nuclei.

116
What material is deposited extravascularly in the skin in this disorder?
A. Uric acid
B. Mucin
C. Amyloid
D. Lipid
E. Pigment
►D

Eruptive xanthomas occurs in the setting of elevated plasma chylomicrons. Histology reveals the
extracellular deposition of eosinophilic, lace-like lipid material between the collagen bundles and
inflammatory cells including neutrophils, lymphocytes and histiocytes. Foam cells may
occasionally be sparse or absent.

117
Which marker helps to differentiate extranodal NK-T cell lymphoma from cutaneous gamma
delta lymphoma?
A. CD56
B. MUM-1
C. EBV status
D. CD43
E. bcl-2
►C

Both cutaneous gamma delta lymphoma and extranodal NK-T cell lymphoma are CD56+.
However, only extranodal NK-T cell lymphoma is EBV + while cutaneous gamma delta
lymphoma is EBV negative. Both these lymphomas have a 5-year survival rate that approaches
0%.

118
Birt-Hogg-Dube has been associated with increased risk of renal cancer and:
A. Spontaneous pneumothorax
B. Hepatic cysts
C. Adrenal hemorrhage
D. Cirrhosis
E. Gastrointestinal polyps
►A

484
The gene defect in Birt-Hogg-Dube is folliculin.

119
This lesion typically occurs in which population?
A. Children on the legs
B. Middle-aged on the face
C. All ages on the trunk
D. Elderly on the head and neck
E. Infants on the trunk
►D

Atypical fibroxanthomas develop in elderly patients primarily on the highly sun-damaged skin of
the head and neck. The prognosis is usually very good despite the pleomorphic histology which
is characterized by very bizarre, atypical spindle-shaped and anaplastic cells with numerous
mitoses.

120
Which of the following is characteristic of pleomorphic lipoma?
A. Foreign body giant cells
B. Floret giant cells
C. Frequent mitoses
D. Exocytosis of lymphocytes
E. Virchow bodies
►B

Pleomorphic lipomas characteristically have a mixture of variably sized fat cells with a varying
number of pleomorphic enlarged cells. These cells contain nuclei arranged in a circumferential
pattern that has been termed floret cells. Rare lipoblasts are found. Focal collections of
lymphocytes and plasma cells within the tumor are seen.

121
CK20 is useful in the diagnosis of:
A. Dermatofibrosarcoma
B. Dermatofibroma
C. Epitheloid sarcoma
D. Merkel cell carcinoma
E. Plasmacytoma
►D

485
CK20 is useful in diagnosis of Merkel Cell Ca.
Dermatofibrosarcoma would stain CD34+.
Dermatofibroma would stain CD68 +.
Plasmacytoma would stain CD138 + and CEDa.
Epithelioid sarcoma would stain CD34+.

122
Goblet cells are seen in:
A. Cutaneous ciliated cyst
B. Endometriosis
C. Dermoid cyst
D. Bronchogenic cyst
E. Steatocystoma
►D

Bronchogenic cysts have a pseudostratified cuboidal or columnar lining that is ciliated; goblet
cells are found in the lining as well.

123
Which of the following is characteristic of lichen planus histopathologically?
A. Wedge-shaped parakeratosis
B. Irregular acanthosis
C. Absent interface changes
D. Parakeratosis
E. Absent granular layer
►B

Histopathology of lichen planus: Acanthosis with wedge-shaped hypergranulosis, irregular


(sawtooth acanthosis), hyperkeratosis without significant parakeratosis, and basal vacuolization
(interface changes).

124
What material is found inside multinucleated giant cells in this condition?
A. Calcium
B. Red blood cells
C. White blood cells
D. Collagen fibers
E. Elastic fibers
►E

486
Annular elastolytic giant cell granuloma presents as multiple papules and annular plaques. On
histology, there is granulomatous inflammation with multinucleated giant cells demonstrating
elastophagocytosis.

125
What infection does this patient have?
A. Cryptococcosis
B. Blastomycosis
C. Coccidioidomycosis
D. Histoplasmosis
E. Leprosy
►C

Lesions of cutaneous coccidioidomycosis demonstrate non-caseating granulomas in the dermis,


often with overlying pseudoepitheliomatous hyperplasia. Thick walled spherules of C. immitis
are found within multinucleated giant cells and granulomas.

126
A 42-year-old multiparous female develops hyperkeratotic papules involving the lower
abdomen. As expected, biopsy is consistent with a perforating disorder. Histopathologic findings
could be accentuated with what stain?
A. Fontana-Masson
B. Fite
C. von Kossa
D. Colloidal iron
E. Osmium tetroxide
►C

Perforating periumbilical calcific elastosis is most commonly seen in multiparous African


American females. Biopsy findings are similar to those of pseudoxanthoma elasticum, with
calcified elastic fibers extruding through the epidermis. Staining with von Kossa may be used to
accentuate these findings.

127
What is the best treatment option?
A. Surgical excision
B. Incision and drainiage
C. observation
D. topical steroids
E. intralesional steroids

487
►A

The treatment of choice for a mucocele is surgical excision. You can see the salivary glands in
the image so you know you are in the mouth; there is also a cavity filled with salivary fluid.

128
Merkel cell carcinomas are seen in older patients and present with a skin-colored to
erythematous or violaceous papule in a sun exposed distribution. The cytokeratin 20 is positive
in :
A. 90% of cases
B. 70% of cases
C. 50% of cases
D. 30% of cases
E. 20% of cases
►A

Patient that are diagnosed with Merkel cell carcinoma will test positive for cytokeratin 20 in 90%
of the time with perinuclear dot pattern. It also tests positive in CAM5.2, NSE +, chromogranin,
and synaptophysin+. It is also thyroidd transcription factor-1 negative and s-100 negative.

129
Multiple spiradenomas are seen in this syndrome:
A. Alagille
B. Brooke-Spiegler
C. Nicolau-Balus‘
D. Schopf‘s
E. Lhermitte-Duclos
►B

Brooke-Spiegler syndrome combines multiple cylindromas and multiple trichoepitheliomas, and


sometimes multiple spiradenomas can be seen as well.
Alagille syndrome associates arteriohepatic dysplasia and nevus comedonicus.
Nicolau-Balus‘ syndrome describes the constellation of multiple eruptive syringomas, milia, and
atrophoderma vermiculata.
Schopf‘s syndrome combines multiple apocrine hidrocystomas, palmar-plantar keratoderma,
teeth abnormalities (hypodontia), and onychodystrophy.
Lhermitte-Duclos disease may be a manifestation of Cowden‘s. In Lhermitte-Duclos, there is a
proliferation in the cerebellum (dysplastic gangliocytoma) with macrocephaly.

130

488
These lesions occur predominantly on which anatomic site?
A. Arms
B. Legs
C. Acral
D. Cephalic
E. Trunk
►D

Arteriovenous malformations demonstrate direct communication between arteries and veins.


They occur most frequently in the cephalic region of the body.

131
Treatment of postmenopausal women with systemic conjugated estrogens has demonstrated:
A. Increased cellular atypia
B. Decreased mitotic activity of keratinocytes
C. Increased dermal collagen content
D. Decreased sebaceous gland activity
E. Increased skin laxity
►C

Systemic use of conjugated estrogens in postmenopausal women increases total skin thickness,
dermal collagen content and mitotic activity of keratinocytes. In addition, there is a reduction of
dry skin, slackness and increased hydration.

132
The cells in this neoplasm have which staining pattern?
A. MITF+
B. S100+ CD1a-
C. S100+ CD1a+
D. S100- CD1a+
E. S100- CD1a-
►C

Langerhans cells demonstrate positive staining with both S100 and CD1a.

133
What diagnosis is pictured here?
A. Lichen planus
B. Syphilis
C. Discoid lupus

489
D. Erythema multiforme
E. Fixed drug
►C

Discoid lupus is characterized on histology by vacuolar degeneration of the basal layer of the
epidermis, necrotic keratinocytes, basement membrane thickening, hyperkeratosis, follicular
plugging,dermal mucin, and a lichenoid perivascular and periadnexal infiltrate.

134
This patient presents with scatterd wheals. What is the most likely diagnosis?
A. Urticaria
B. Psoriasis
C. PRP
D. Sweets syndrome
E. contact dermatitis
►A

A patient with wheals most likely has urticaria. When neutrophils are seen in the vessels with
significant dermal edema, urticaria must be ruled out. Psoriasis, contact deramtitis and PRP are
papulosquamous diesease. Sweets would have a dense neutrophilic infiltrate .

135
Immunohistochemical staining with neuron-specific enolase is positive in:
A. Anaplastic large cell lymphoma
B. Malignant firbroushistiocytoma
C. Cutaneous T cell lymphoma
D. Merkel cell carcinoma
E. Sebaceous carcinoma
►D

Neuron-specific enolase is a cytoplasmic product produced by Schwann cells and neurons. This
enzyme is present in neuroendocrine cells, neurons and tumors derived from them. Positive
staining for neuron-specific enolase is found in Merkel cell carcinomas, carcinoid tumors, and
malignant melanoma.

136
Both mesenchymal and epithelial elements may be found in which of the following tumors?
A. Cylindroma
B. Microcystic adnexal carcinoma
C. Chondroid syringoma

490
D. Bednar tumor
E. Folliculosebaceous cystic hamartoma
►C

Cutaneous mixed tumor, also known as Chondroid syringoma, represents an acquired hamartoma
with folliculosebaceous-apocrine differentiation that has been generally interpreted as a form of
adnexal adenoma (neoplasm). It has both a mesenchymal and epithelial component.

137
These lesions are common in the genodermatosis caused by which gene mutation?
A. PTEN
B. BRAF
C. NFKB
D. CYLD
E. ABCC6
►D

Brooke-Spiegler syndrome is caused by a mutation in the CYLD gene. It is characterized by the


presence of spiradenomas, cylindromas, and trichoepitheliomas.

138
This disorder most frequently occurs in which anatomic region?
A. Scalp
B. Genital region
C. Extremities
D. Trunk
E. Face
►B

Lichen sclerosus et atrophicus most commonly occurs in the genital region.

139
What is this condition?
A. Gouty Tophus
B. Sarcoid
C. Keratin granuloma
D. Rheumatoid Nodule
E. Granuloma annulare
►A

491
Gouty Tophi occur due to an accumulation of monosodium urate in the tissues. Deposits in the
dermis act as a foreign body and granulomas form around these deposits. Unless the tissue is
fixed with alcohol (Carnoy's fixative), the yellow brown crystals of gout are not well visualized
on H and E. When the tissue is submitted in formalin the tophus appears pink and amorphous.

140
This type of neurofibroma is known to be pathognomonic for neurofibromatosis type I and is
composed of neurofibroma with hypertrophied nerves in a plexiform patter (the gross correlate
of this plexiform nature is described as a "bag of worms" appearance:
A. Plexiform variant
B. Diffuse neurofibroma
C. Wagner Meissner variant
D. Myxoid variant
E. Unencapsulated
►A

The plexiform variant is composed of neurofibroma with hypertrophied nerves in a plexiform


pattern. This is pathognomonic of neurofibromatosis type I. Diffuse neurofibroma is a variant
seen in a proportion of neurofibromatosis type I patients. It extends into the subcutaneous tissue
with honey combing this variant has typical Wagner-Meissner bodies that resemble Meissner's
corpuscles.

141
A 9 year-old girl presents for evaluation of alopecia. Physical examination reveals a bizarrely
shaped patch of hair loss in the vertex area, with broken hairs of varying lengths. Biopsy is most
likely to demonstrate:
A. Lymphoid inflammation at the level of the hair bulb
B. Mucin within follicular epithelium
C. Fungal spores within hair shafts
D. Traumatized hair follicles with perifollicular hemorrhage, empty follicles, and
deformed hair shafts
E. Suppurative folliculitis
►D

Biopsy of trichotillomania, characterized by an abnormal urge to pull out the hair, reveals
traumatized hair follicles with perifollicular hemorrhage, empty anagen follicles, many catagen
hairs, deformed hair shafts (known as trichomalacia), and melanin casts within the follicular
canal. Lymphoid inflammation at the level of the hair bulb is a feature of alopecia areata. Mucin
within follicular epithelium is characteristic of follicular mucinosis. Tinea capitis demonstrates
fungal spores within hair shafts. Suppurative folliculitis may be seen in folliculitis decalvans.

492
142
Which of the following immunohistochemical stains can help distinguish basal cell carcinomas
and trichoepitheliomas from microcystic adnexal carcinomas?
A. Pro-collagen 1
B. CD34
C. Peanut agglutinin
D. TTF-1
E. Ber-Ep4
►E

Ber-Ep4 helps distinguish between BCCs/Trichoepitheliomas and microcystic adnexal


carcinomas. Morpheaform BCC and desmoplastic trichoepithelioma are Ber-Ep4 positive where
as microcystic adnexal carcinomas (MAC) are Ber-Ep4 negative. Other markers that help
distinguish between BCCs and trichoepitheliomas are peanut agglutinin and CD34. Peanut
agglutinin is positive in BCC and negative in trichoepithelioma vs CD34 which is negative in
BCC and positive in the peritumoral fibroblasts of trichoepitheliomas.

143
The predominant location of the cleft in erythema toxicorum neonatorum is:
A. Dermal
B. Basement membrane zone
C. Basal keratinocytes
D. Suprabasal
E. Subcorneal/granular
►E

Erythema toxicorum neonatorum is an idiopathic vesiculopustular eruption on the face and trunk
of neonates that often resolves in a few weeks. Histologically it presents with subcorneal
eosinophil rich pustules.

144
Biopsy of a mucosal neuroma from a patient with MEN IIb looks histologically like a:
A. Neurilemmoma
B. Neurofibroma
C. Palisaded encapsulated neuroma
D. Traumatic neuroma
E. Neurothekeoma
►C

493
Mucosal neuromas in MEN IIb often look histologically like PENs. Occasionally mucosal
neuromas display thickened nerves/perineurium.

145
This lesion represents a congenital anomaly of which embryonic structure?
A. Third branchial arch
B. Third branchial cleft
C. Fourth branchial arch
D. First branchial arch
E. First branchial cleft
►D

An accessory tragus represents a congenital anomaly of the first branchial arch. They may be
associated with an increased risk of hearing impairment. Histologic features include numerous
vellus hair follicles, subcutaneous tissue, and often, a central cartilaginous core.

146
Multiple pilomatricomas are seen in:
A. Myotonic dystrophy
B. Cowden's
C. Turner's
D. Gorlin's
E. Myotonic dystrophy and Turner's
►E

Multiple pilomatricomas are seen in Rubinstein-Taybi, Gardner's (cyst-like pilomatricomas),


myotonic dystrophy, Turner's, sarcoidosis, sternal cleft and coagulation defects.

147
This form of subcutaneous sarcoid is known as which of the following?
A. Heerfordt Waldenstrom syndrome
B. lofgren syndrome
C. Darier-Roussy
D. blau syndrome
E. mikulicz syndrome
►C

Darier-Roussy is the subcutaneos form of sarcoid.

148

494
A healthy 6 month old girl has a subcutaneous nodule above her right eyebrow. A skin biopsy
demonstrates a cystic lesion with adnexal structures in the wall. Your diagnosis is:
A. Steatocystoma
B. Pilar cyst
C. Nevus sebaceous
D. Epidermal inclusion cyst
E. Dermoid cyst
►E

Dermoid cysts present along lines of embryonic closure. The are most commonly found on the
head (around the eyes) and the neck. They are lined by an epidermis that contains various
epidermal appendages that are usually fully matured.

149
Clear cell syringomas are associated with:
A. Malignancy
B. Sarcoidosis
C. Argyria
D. Diabetes
E. Lichen myxedematosis
►D

Clear cell syringomas are associated with diabetes. Syringomas are associated with Down‖s
syndrome.

150
The cytoplasmic granules seen in granular cell tumor are:
A. Phagolysosomes
B. Ribosomes
C. Mitochondria
D. Intermediate filaments
E. Vacuoles
►A

The granules in granular cell tumor are phagolysosomes. The granularity of the granular cell
layer in epidermodysplasia verruciformis may be secondary to increased ribosomes.
Mitochondria fill the cells in hibernoma.

151
Silver preferentially deposits in:

495
A. Eccrine glands
B. Apocrine glands
C. Hair follicles
D. Fat
E. Eccrine glands and apocrine glands
►A

Argyria can look like normal skin if you do not notice the silver (black) deposits in the eccrine
glands in the deep dermis.

152
On histology there is acanthosis, hyperkeratosis, vaculoar interface change and apoptotic
keratinocytes. There is also papillary dermal edema with endothelial swelling and mixed
inflammatory infiltrate. These features are seen in:
A. Erythema multiforme
B. Toxic epidermal necrolysis
C. Lupus erythematosus
D. Discoid lupus
E. Graft versus host disease
►A

This is the description of erythema multiforme. It is a hypersensitivity reaction with


characteristic targetoid skin lesions and mucosal involvement. Etiologies include herpes simplex,
Mycoplasma infection and drugs.

153
What is the clinical sign associated with this lesion?
A. Frank sign
B. hutchinsons sign
C. asboe hansen sign
D. Samitz sign
E. Darier's sign
►E

The image is a mastocytoma. Darier's sign is when a mastocytoma urticates after being rubbed.
Hutchinson's sign is when melanoma of the nail involves the cuticle. Asboe hansen sign is when
pressur eis plased perpendicular to a blister and it extens. Samitz sign is ragged/frayed cuticles in
dermatomyositis. Frank sign is bilateral diagnonal ear lobe crease associated with CAD,
especially in men less than 40 years of age.

496
154
This biopsy was obtained from an immunosuppressed patient with multiple bruise like lesions on
arms and legs. What is this neoplasm?
A. Kaposi's sarcoma
B. Lobular hemangioma
C. Targetoid hemosiderotic hemangioma
D. Tufted hemangioma
E. Angiosarcoma
►A

Kaposi's Sarcoma: The vascular proliferation in all subtypes of Kaposi's sarcoma are associated
with an infection with HHV-8. Histologically early on there is a subtle proliferation of irregular
angulated slit like blood vessels in dermis that dissects the collagen. When these new blood
vessels encase preexisting blood vessels it's called a promontory sign. In the stroma there is a
lymphoplasmacytic infiltrate, extravasated red blood cells and hemosiderin.

155
Methyl-green pyronin stains RNA what color?
A. Pink
B. Green
C. Blue
D. Purple
E. Black
►A

Methyl-green pyronin stains RNA pink and stain DNA green.

156
What is the best diagnosis?
A. Keloid
B. Ochronosis
C. Lichen Nitidus
D. Compound Spitz Nevus
E. Dermatitis Herpetiformis
►A

Distinguishing hypertrophic scar (HS) from keloid histopathologically is sometimes difficult


because thickened hyalinized collagen (keloidal collagen), the hallmark of keloid, is not always
detectable and [alpha]-smooth muscle actin ([alpha]-SMA), a differentiating marker of HS, is
variably expressed in both forms of scar.

497
157
This is associated with MEN IIa:
A. Neurothekeoma
B. Macular amyloidosis
C. Malignant peripheral nerve sheath tumor
D. Chondroid syringoma
E. Mucocele
►B

Macular amyloidosis is associated with MEN IIa.

158
The predominant cleft in dermatitis herpetiformis is:
A. Dermal
B. Basement membrane zone
C. Basal keratinocytes
D. Suprabasal
E. Subcorneal/granular
►B

Dermatitis herpetiformis or Duhring's disease, presents with very pruritic vesicles symmetrically
on extensor surfaces. On histology it presents as suprapapillary vesicles with mostly neutrophils
and inflammatory destruction of the basement membrane zone. Direct immunoflourescence
shows granular deposition of IgA in the dermal papillae and along the basement membrane zone.
The cleft in dermatitis herpetiformis is most commonly found in the basement membrane
zone/subepidermal. The antigen is transglutaminase.

159
Where does this lesion typically occur on the body?
A. Anywhere along the milk lines
B. Preauricular area
C. On the neck
D. On the hand
E. On the leg
►A

supernumerary or accessory nipple can occur anywhere along the milk lines. The histologic
features include epidermal acanthosis, smooth muscle bundles, pilosebaceous units, and
sometimes mammary glands.

498
160
Which disease frequently demonstrates this finding?
A. Morphea
B. Atopic dermatitis
C. Langerhans cell histiocytosis
D. Rosacea
E. Dermatomyositis
►E

Calcinosis cutis is a common occurrence in dermatomyositis, especially the juvenile form of the
disease.

161
Which of the following shows granular deposition of IgA in the dermal papillae and along the
basement membrane zone on direct immunoflourescence:
A. IgA pemphigus
B. Bullous pemphigoid
C. Linear IgA dermatosis
D. Dematitis herpetiformis
E. Herpes gestationalis
►D

Dermatitis herpetiformis or Duhring's disease, presents with very pruritic vesicles symmetrically
on extensor surfaces. On histology it presents as suprapapillary vesicles with mostly neutrophils
and inflammatory destruction of the basement membrane zone. Direct immunoflourescence
shows granular deposition of IgA in the dermal papillae and along the basement membrane zone.

162
A 26-year-old man with Basal Cell Nevus syndrome is diagnosed with a meningioma. Histologic
findings are most likely to reveal:
A. Psammoma bodies
B. Pustulo-ovoid bodies
C. Russel bodies
D. Verocay bodies
E. Weibel-Palade bodies
►A

Patients with Basal Cell Nevus syndrome are predisposed to developing meduloblastomas and
meningiomas, both of which can rarely present with cutaneous involvement. Psammoma bodies

499
are concentrically laminated calcified bodies seen in meningioma, along with ovarian and
thyroid neoplasms. Pustolo-ovoid bodies are seen in granular cell tumors. Russel bodies are
immunoglobulin inclusions in plasma cells often seen in Rhinoscleroma. Verocay bodies are
palisading nuclei arranged in rows with peripheral eosinophilic cytoplasm characteristic of
Schwannomas. Weibel-Palade bodies are organelles that are seen on macroscopy of endothelial
cells.

163
A 44-year old man presents with a beefy red plaque involving the right penile shaft, along with
suppurative lymphadenopathy. Biopsy would reveal which of the following histologic findings:
A. Asteroid Bodies
B. Caterpillar Bodies
C. Cowdry Type A Bodies
D. Donovan Bodies
E. Dutcher Bodies
►D

Granuloma Inguinale is caused by Klebsiella granulomatis. This infection presents as a chronic


suppurative infection of the genital region. Biopsy reveals Donovan bodies, which are
intrahistiocyte inclusions comprised of the implicated organisms. These stain positively with
both Warthin-Starry stain and Giemsa.

164
Which of the following stains would be helpful in the diagnosis of cryptococcus?
A. Colloidal iron
B. Oil red O
C. Mucicarmine
D. Giemsa
E. Verhoeff von Gieson
►C

The yeast cytoplasm of Crytococcosis stains with PAS and methanamine silver while the the
capsule stains with Alcain blue and mucicarmine. Colloidal iron stains mucin; Oil red O stains
fat; Giemsa stains mast cells and leishmaniasis; Verhoeff von Gieson stains elastic tissue.

165
Which of the following is characteristic of Birt-Hogg-Dube Syndrome?
A. Autosomal recessive mode of inheritance
B. Multiple trichoepitheliomas
C. Caused by mutation in hamartin

500
D. Multiple trichodiscomas
E. Colon cancer common
►D

Birt-Hogg-Dube Syndrome (BHD) characterized by multiple small harartomas of mesodermal


component of hair discs, which were indentified as trichodiscromas. It is an autosomal dominant
disease caused by mutations in folliculin. Patients with multiple fibrofolliculomas may also have
acrochordons, collagenomas, lipomas, and/or oral fibromas. BHDS recently has been reported in
association with various types of renal tumors, such as oncocytoma and a variant of papillary
renal cell carcinoma. There are no trichoepitheliomas in BHD syndrome.

166
This tumor was excised from a verrucous yellow plaque on the scalp what is it?
A. Syringocystadenoma papilliferum
B. Eccrine acrospiroma
C. Hidradenoma Papilliferum
D. Tubular apocrine adenoma
E. Syringofibroadenoma
►A

Syringocystadenoma Papilliferum: 33% arise in association with a nevus sebaceous, 10% may
coexist with a BCC or trichoblastoma. A subset of cases has a deletion of 9q22 PTCH gene and
9q21 (p16). Histologically they present as surface invagination of glandular cords composed of
one to two layers of cuboidal cells associated with a fibrovascular stroma with numerous plasma
cells. Cystic spaces may form within which are free floating islands which are peripherally lined
by cuboidal cells with an inner core composed of loose connective tissue, blood vessels and
numerous plasma cells.

167
Atypical keratinocytes are noted on this biopsy. What HPV type is likely associated?
A. HPV 16
B. HPV 2
C. HPV 1
D. HVP 4
E. HPV 7
►A

HPV 16 is an oncogenic strain of HPV which can trigger development of SCC within a wart.

168

501
What is the diagnosis?
A. silicone
B. monsels
C. aluminum chloride
D. gel foam
E. tattoo
►E

Tattoo is a pigment deposited deep in the dermis, and is whatever color the tatto itself is. Gel
foam is purple; silicone is a group of small bubbles; and monsels is more of a brown color and is
localized.

169
Which marker stains melanosomes?
A. S-100
B. Mart-1/Melan-A
C. MITF
D. HMB-45
E. Fontana-Masson
►D

HMB-45 is a melanosome immunostain targeting glycoprotein.


S-100 is a melanocyte immunostain, but it has low specificity also staining Langerhans cells,
smooth and skeletal muscle, adipocytes, and eccrine coils.
MITF is a melanocyte nuclear stain.
Fontana-Masson is a melanin stain.

170
Which of the following stains is specific for melanin?
A. S-100
B. HMB-45
C. Fontana-Masson
D. MART-1
E. Melan-A
►C

Fontana-Masson is a silver stain and stains melanin black.


S-100, HMB 45, MART-1 (Melan-A) are melanocyte stains. S-100 also decorates Langerhans
cells, acrosyringium, and neural crest-derived cells.

502
171
Mulberry cells contain increased:
A. Phagolysosomes
B. Mitochondria
C. Golgi
D. Ribosomes
E. Phagolysosomes and mitochondria
►B

Hibernomas commonly arise in the neck, axillae, and posterior shoulder. The cells are
multivacuolated and resemble mulberries; the cells are filled with mitochondria, as are the cells
in normal brown fat.

172
Which of the following characteristics describes this lesion?
A. Metastatic upon presentation
B. Benign
C. Low recurrence rate, high risk of metastasis
D. High recurrence rate, low risk of metastasis
E. Low recurrence rate, low risk of metastasis
►E

Atypical fibrous histiocytomas are also known as dermatofibromas with monster cells. These
tumors are uncommon and are characterized by a low recurrence rate and a rare risk of distant
metastases.

173
Leukocytoclastic vasculitis can be seen in which condition?
A. Rheumatoid neutrophilic dermatosis
B. Erythema elevatum diutinum
C. Urticaria
D. Granuloma inguinale
E. Sweet's Syndrome
►B

Rheumatoid neutrophilic dermatitis is seen in the setting of severe rheumatoid arthritis. The
pathogenesis is not understood, but in the few reports of this rare dermatosis, a true vasculitis has
not been seen. Sweet's syndrome also does not show evidence of a true vasculitis. Erythema
elevatum diutinum histologically shows a leukocytoclastic vaculitis, but with prominent

503
interstitial neutrophils. Other conditions with evidence of LCV include Henoch-Schonlein
purpura, granuloma faciale, urticarial vasculitis, and occasionally serum sickness.

174
On histology, there is a follicular based epithelial cystic invagination with differentiation towards
the outer root sheath forming eosinophilic cellar bodies. Central invagination contains lamellated
keratinous material. This best describes:
A. Pilar shealth acanthoma
B. Tumor of follicular infundibulum
C. Desmoplastic trichilemmoma
D. Basaloid follicular hamartoma
E. Trichoblastoma
►A

This describes a pilar sheath acanthoma and is a benign follicular based, comedo-like epithelial
proliferation with differentiation toward the outer root sheath. It presents as a skin colored papule
with central depressed pore.

175
What is the most likely diagnosis?
A. Verrucous carcinoma
B. Keratoacanthoma
C. Basal cell carcinoma
D. bowen's disease
E. pilomatrichoma
►A

Verrucous carcinoma is a large nedophytic condylomatous lesion. It is giant, well differentiated


and has a pushing margin.

176
What is the most likely diagnosis?
A. tinea versicolor
B. tinea nigra
C. candida
D. sporothrixosis
E. chromomycosis
►A

504
Tinea veriscolor has the typical spaghetti and meatballs as seen here, the hypae and spores. Tinea
nigra is pigmented.

177
What is this neoplasm?
A. Poroma
B. Eccrine acrospiroma
C. Sebaceous adenoma
D. Basal Cell Carcinoma
E. Trichoblastoma
►A

Poroma: Palmar/plantar skin, trunk and lower extremities. Named based upon location, If purely
intraepidermal called a hidroacanthoma simplex, if only in the dermis it is a dermal duct tumor,
but more commonly seen both in the epidermis and dermis and then it is given general
classification as poroma. Associated with Schopf-Schultz-Passarge syndrome and Clouston's
syndrome. Histologically composed of a proliferation of small uniform cuboidal basaloid cells
that are PAS+DS (i.e. contains glycogen). The cells are smaller than neighboring keratinocytes.
The tumors can be pigmented and there can be necrosis en mass. The latter finding is an
exception to the general rule that benign tumors do not show necrosis en mass.

178
Histologically, adenoma sebaceum represent which of the following lesions?
A. Neurofibromas
B. Angiofibromas
C. Collagenomas
D. Angiokeratomas
E. Smooth muscle hamartomas
►B

Adenoma sebaceum, fibrous papules and pearly penile papules all have similar features
histologically, presenting as angiofibromas. Features include atrophic epidermis with patchy
melanocytic hyperplasia and hyperkeratosis, vertically oriented collagen, increased fibroblasts
and blood vessels.

179
This lesion is defined by the presence of what material in the dermis?
A. Amiodarone
B. Ink
C. Iron

505
D. Melanin
E. Carbon
►D

A blue nevus is characterized by the presence of dendritic melanocytes and melanophages.

180
The predominant location of the cleft in cicatricial pemphigoid is:
A. Dermal
B. Basment membrane zone
C. Basal keratinocytes
D. Supra basal
E. Subcorneal/granular
►B

Cicatricial pemphigoid is an autoimmune blistering disease that presents with ulcers, blisters and
erosions of mucosal surfaces, especially the eyes and mouth. The cleft in cicatricial pemphigoid
is found in the basement membrane zone/subepidermal as the antigens are usually
BPAg2,laminin 5 and alpha-6-beta-4 integrin. Direct immunoflourescence is identical to that of
bullous pemphigoid showing linear IgG and complement deposits in the basement membrane
zone.

181
Mantle cell lymphoma is characteristically positive for which of the following?
A. CD10
B. CD23
C. CD138
D. Bcl-1
E. bcl-6
►D

Bcl-1 (Cyclin D1) is a marker for mantle cell lymphoma. CD10, bcl-6, and bcl-2 are markers for
follicular cell lymphoma. Bcl-2 also stains normal T cells. CD23 is a marker for CLL/SLL and is
negative in mantle cell lymphoma. CD138 is a marker for plasma cells.

182
What is this neoplasm?
A. Pilomatricoma
B. Proliferating pilar tumor
C. Trichoepithelioma

506
D. Basal cell carcinoma
E. Eccrine acrospiroma
►A

Pilomatrixoma (Calcifying epithelioma of Malherbe): Found on the head, neck and upper
extremities in the first 2 decades of life. Typically solitary but multiple pilomatricomas arise in
several syndromes. Even in isolation these tumors may arise due to activating mutations in the
beta-catenin gene. Histologically: This tumor consists of two major cell types plus an
intermediate or transitional cell type. Initially the tumor is more cystic with the cells at the
periphery of the tumor that are more basophilic with indistinct cell borders and little cytoplasm.
The cells have hyperchromatic nuclei and often normal mitoses can be appreciated. Centrally
there are eosinophilic ―ghost or shadow‖• cells which are cells that have undergone terminal
differentiation. These cells have more distinct borders, increased cytoplasm but only a ghost of a
nucleus. Then there are cells that reside somewhere in between these two cell types. The
proportion of these cells varies depending on the stage of the lesion; i.e younger lesions have
more basophilic cells and appear more cystic, older lesions have a greater component of ghost
cells and up to 20% of lesions on removal are completely composed of ghost cells.

183
This lesion derives from which structure?
A. Adnexae
B. Keratinocyte
C. Neural tissue
D. Vasculature
E. Smooth muscle
►A

A spiradenoma is thought to derive from the adnexae.

184
This patient presented with a papulosquamous eruption involving his palms, soles and trunk. He
notes that three weeks ago he had a small lesion on his penis that healed within a week. What is
your most likely diagnosis?
A. Syphilis
B. haemophilus ducryi
C. LGV
D. HSV
E. psoriasis
►A

507
Secondary syphilis presents with a papulosquamous eruption that includes the palms and soles.
They may or may not remember the penile chancre as it is painless. Both haemophilus ducryi
and HSV are painful.

185
The presence of which cell type confirms this diagnosis?
A. Lipidized histiocyte
B. Mast cell
C. Multinucleate keratinocyte
D. CD30+
E. Langhans giant cell
►C

Herpes infection is confirmed by the presence of multinucleate keratinocytes which often have
intranuclear inclusion bodies.

186
On histology there is acanthosis, papillomatosis with occasional neutrophils in the stratum
corneum. There is alternation of hyperkeratosis alternates with parakeratosis. There is a lack of
granular layer in the areas of parakeratosis and is most likely:
A. Inflammatory linear verrucous epidermal nevus
B. Psoriasis
C. Pityriasis rubra pilaris
D. Lichen simplex chronicus
E. Ritter's syndrome
►A

This is ILVEN seen unilateral with inflamed papules and plaques in a linear distribution. It
presents during infancy and childhood but can arise in adults. It follows the line of Blaschko and
is located on the extremities and is usually pruritic.

187
Diffuse staining with this marker is suggestive of BCC rather than trichoepithelioma:
A. CAM 5.2
B. Bcl-2
C. PTAH
D. CEA
E. Cytokeratin 8
►B

508
Trichoepitheliomas stain with bcl-2 on the periphery of individual tumor islands.

188
A lichenoid infiltrate that surrounds eccrine glands is seen in:
A. Lichenoid drug rection
B. Lichen striatus
C. Lichen planus
D. Lichenoid purpura
E. Lichen planopilaris
►B

Lichen striatus is an uncommon inflammatory dermatitis seen most commonly in children aged 5
to 15. It presents unilaterally along Blaschko's lines as raised, slightly scaly, erythematous
papules, which are often pruritic. These lesions typically regress spontaneously within a year.
The histopathologic features of lichen striatus include a superficial perivascular inflammatory
lymphohistiocytic infiltrate with rare plasma cells and eosinophils. There is a focal lichenoid
infiltrate in the papillary dermis with basilar vacuolar alteration and necrotic keratinocytes.
Spongiosis with exocytosis of lymphocytes can be seen in the epidermis. A specific and
distinctive feature of lichen striatus is the presence of an inflammatory infiltrate that surrounds
hair follicles and eccrine glands.

189
This infection is caused by contact with which exposure?
A. Seagulls
B. Pigeon droppings
C. Soil containing bird and bat droppings
D. Sandfly
E. Dolphins
►D

Leishmaniasis is a protozoal infection caused bya bite from a sandfly.

190
What is the most likely diagnosis?
A. telangiectasia
B. angiofibroma
C. microvenular hemangioma
D. angiosarcoma
E. kaposi sarcoma
►A

509
This is a superficial proliferation of thin walled vessels most consistent with a telangiectasia. An
angiofibroma would have a fibrotic stroma, and a microvenular hemangioma would have vessels
proliferating deep. Angiosarcoma would have jagged vessels with atypical endothelial cells.
Kaposi sarcoma would have many more vessels and extravasated RBCs.

191
This lesion behaves in which manner?
A. High rate of distant metastases
B. Low rate of distant metastases
C. High recurrence rate
D. Benign
E. Highly aggressive
►B

Atypical fibroxanthomas have low recurrence rates and low risk of distant metastases.

192
The most likely diagnosis of this image is which of the following?
A. lipodermatosclerosis
B. sclerema neonatorum
C. erythema nodosum
D. lupus profundus
E. subcutaneous fat necrosis of the newborn
►E

Subcutaneous fat necrosis of the newborn is easily recognized by the intense inflamation,
radially arranged eosinophilic crystals in the fat cells, and the foreign body giant cells, in
comparison to sclerema neonatorum, where there is no inflammation as the patient is too sick to
mount a response.

193
This eruption responds very well to which medication?
A. Dapsone
B. Gluten free diet
C. Terbinafine
D. Fluconazole
E. Vancomycin
►A

510
Bullous systemic lupus erythematosus responds very well to dapsone therapy.

194
When this lesion occurs in multiples, which disorder should be suspected?
A. Rombo syndrome
B. Multiple Familial Trichoepithelioma
C. Goltz
D. Brooke-Spiegler syndrome
E. Carney complex
►D

Patients with Brooke-Spiegler syndrome often have multiple spiradenomas, cylindromas, and
trichoepitheliomas.

195
A deposition of what substances leads to these deposits?
A. Homogentisic acid
B. Amyloid
C. Fungal organisms
D. Colloid
E. Tattoo
►A

Ochronosis: can be either endogenous or exogenous. The endogenous form is due to an AR


defect in homogentisic acid oxidase, which prevents tyrosine and phenylalanine from being
degraded beyond homogentisic acid. The exogenous form of ochronosis, on the other hand,
occurs from topical hydroquinone, mercury resorcinol, phenol, picric acid or benzene, or
systemic quinine or chloroquine which all inhibits homogentisic oxidase and leads to a focal
accumulation of homogentisic acid. Within the superficial dermis there are irregular shaped
elongated yellow deposits which have been described as as bananas in the dermis that occur due
to deposition of homogentisic acid on the collagen.

196
Which of the following can be used to stain amyloid?
A. PAS
B. Eosin
C. Giemsa
D. Aldehyde fuchsin
E. Crystal violet
►E

511
Stains for amyloidosis: Congo Red, Thioflavin T, Crystal Violet, Methyl violet, Pagoda red no.
9, PAS + diastase, amyloid P-component antibody

197
Acantholysis is not a prominent histopathologic feature of which disease?
A. Dermatitis herpetiformis
B. Darier's disease
C. Grover's disease
D. Pemphigus vulgaris
E. Hailey-Hailey diesease
►A

Dermatitis herpetiformis is a bullous disease that on histology shows neutrophils in the dermal
papillae. On direct immunoflourescence IgA is seen in a granular depostion pattern. Acantholysis
is seen histologicaly in Darier's disease, Grover's disease, Pemphigus vulgaris, Hailey-Hailey
disease and warty dyskeratoma.

198
What structure does this tumor most commonly invade?
A. blood vessels
B. hair follicle
C. nerves
D. sebaceous gland
E. arrector pili muscle
►C

Microcystic adnexal carcinomas typically have perineural involvement. They must be


distinguished from desmoplastic trichoepitheliomas, syringomas and morpheaform BCCs.

199
The promontory sign is seen in:
A. Tufted angioma
B. Glomeruloid hemangioma
C. Spindle cell hemangioendothelioma
D. Acroangiodermatitis of Mali
E. Kaposi‘s sarcoma
►E

512
The promontory sign refers to the formation of new vessels around existing vessels and adnexal
structures. This is seen in Kaposi‘s.

200
Which of the following is the most likely diagnosis of this image?
A. Lipoma
B. Merkel cell carcinoma
C. Metastatic adenocarcinoma
D. Melanoma
E. Myxoid neurothekeoma
►C

In this image, the lymphatics are filled with large atypical cells trying to form glands making
metastatic adenocarcionma the most likely diagnosis. Merkel cell carcinoma is a sheet of blue
cells with salt and pepper nuclei. A lipoma is a collection of normal fat. Melanoma would have
nests of atypical melanocytes. A myxoid neurothekeoma would be a loose collection of cells in
thekes with a cellular stroma and the cells do not attempt to form glands.

201
This image depicts a histologic variant of which structure?
A. Fibrokeratoma
B. Fibrous papule
C. Schwannoma
D. Xanthoma
E. Granuloma annulare
►B

This image demonstrates a granular cell fibrous papule. Other histologic variants of fibrous
papules include hypercellular, clear-cell, pleomorphic, pigmented, and inflammatory types.

202
On histology this disease has mucin deposition within the external root sheath and sebaceous
gland. The mucin distends into the intracellular spaces within follicular epithelium forming
cystic cavities. The following stains will be positive except for:
A. CD3
B. CD4
C. CD7
D. CD8
E. CD9
►E

513
This patient has follicular mucinosis. This is benign and seen in children and is a nonscarring
alopecia and may be pruritic or tender seen on the face and scalp. All the following stains will be
positive CD3, CD4, CD7, CD8.

203
What underlying disease does this patient have?
A. Wiskott-Aldrich syndrome
B. PHACES
C. Osteogenesis imperfecta
D. Darier's disease
E. Vitiligo
►C

Elastosis perforans serpiginosa is associated with an underlying condition in up to a third of


cases. Some of these conditions include osteogenesis imperfecta, Ehlers-Danlos syndrome,
Marfan's syndrome, acrogeria, scleroderma, and Down syndrome.

204
What is the diagnosis?
A. Marjolin's ulcer
B. Lichen planus
C. Basal cell carcinoma
D. Lichen amyloid
E. Dermatofibroma
►A

Marjolin's ulcer is an aggressive ulcerated squamous cell carcionoma that arises due to chronic
inflammation. The image shows the invasive atypical keratinocytes. In lichen amyloid there are
pink amorphous deposits that do not have nuclei.

205
Subcutaneous fat necrosis of the newborn has been associated with:
A. Hypocalcemia
B. Hypercalcemia
C. Hypokalemia
D. Hyperkalemia
E. Hyponatremia
►B

514
Hypercalcemia has been noted in some cases of subcutaneous fat necrosis of the newborn.

206
All are sebaceous glands except:
A. Tyson
B. Moll
C. Mongomery tubercle
D. Fordyce
E. Meibomian
►B

Moll‘s glands are modified apocrine glands located on the eyelid. Sebaceous glands in certain
locations have an associated name (Tyson‘s on the penis, Montgomery tubercle on the areola,
Meibomian on the eyelid, and Fordyce on the vermilion/oral mucosa).

207
What is the most likely diagnosis for this single leison?
A. wart
B. hailey hailey
C. dariers
D. pemphigus vegetans
E. Warty dyskeratoma
►E

Warty dyskeratoma is a cup-shaped invagination with acanthlytic and dyskeratosis. Both hailey
hailey and darier's do not have this architecture. Hailey hailey has more acantholysis, dariers has
more dyskeratosis, and both are larger plaques. It would be unusal to have one single leison of
hailey hailey, dairers or pemphigus vegetans.

208
This eruption is mediated by autoantibodies to which antigen?
A. Desmoplakin
B. Desmoglein 1
C. BP230
D. Collagen VII
E. BP180
►D

Bullous systemic lupus erythematosus is mediate by antibodies to collagen VII. Histology is


similar to that of linear IgA bullous dermatosis and dermatitis herpetiformis.

515
209
Using the salt-split skin technique with direct immunofluorescence, epidermolysis bullosa
acquisita will show linear deposition of complement in what position?
A. Roof of the split
B. Roof and floor of the split
C. Floor of the split
D. Neither the roof or floor of the split since IgA is the most common reactant
E. None of the above since the pattern is not linear
►C

Epidermolysis bullosa acquisita (EBA) is a bullous disease of adults in which minor trauma
(usually on the hands and feet) leads to blisters that heal with scaring. On histology there is
classically a noninflammatory subepidermal split. The blister will immunostain with IgG on the
floor of salt-split skin, as the antigen is type VII collagen.

210
Which of the following drugs is most likely to cause the reaction shown?
A. Metformin
B. Warfarin
C. Glyburide
D. Ipilimumab
E. Dabigatran
►A

Metformin, B-blockers, penicillamine, ACH inhibitors, Calcium channel blockers, quinine,


NSAIDS, Gold, Allopurinol, HCTZ, fenofibrate, Antimalarials and demethylchlorotetracycline
can all cause lichenoid drug reactions. Glyburide is a sulfonylurea that has been implicated in
drug induced psoriasis. Warfarin and dabigatran are both anticoagulants.

211
Which class of medication is associated with the development of this neoplasm?
A. ACE inhibitors
B. Beta blockers
C. Protease inhibitors
D. TNF alpha inhibitors
E. Antifungals
►C

516
Protease inhibitors have been associated with a lipodystrophy syndrome in some patients with
HIV, including the development of angiolipomas. Histologic features of an angiolipoma include
normal subcutaneous fat with the presence of thrombosed blood vessels.

212
A 64-year old woman presents with scalloped erosions in the dermatome of the first branch of
the trigeminal nerve. Biopsy would reveal which of the following histologic findings:
A. Asteroid Bodies
B. Caterpillar Bodies
C. Cowdry Type A Bodies
D. Donovan Bodies
E. Dutcher Bodies
►C

Varicella zoster virus is the causative agent in chicken pox and shingles, where recrudescence
from latency in the dorsal root ganglion causes a dermatomal distribution of painful vesicles and
erosions. Biopsy of herpes viruses reveals cells with multinucleation, margination of the
chromatin, and Cowdry Type A bodies, which are intranuclear eosinophilic amorphous bodies
surrounded by a clear halo.

213
What is the diagnosis?
A. Syphilis
B. Psoriasis
C. PRP
D. Lichen planus
E. PLEVA
►A

The diagnosis is syphilis. Anytime a lichenoid infiltrate plus psoriasiform hyperplasia is present,
syphilis must be ruled out. There are typically numerous plasma cells in the infiltrate along with
long skinny rete.

214
Clinically, a nondescript hyperkeratotic papule on the ulnar side of the base of the fifth finger is
most likely:
A. Acquired digital fibrokeratoma
B. Accessory digit
C. Cutaneous horn
D. Digital fibromatosis

517
E. Glomus tumor
►B

Accessory digits (supernumerary digits) are usually found at the base of the fifth finger, often
bilaterally.

215
Granular cell tumors will stain positive for all of the following markers except:
A. S100
B. PAS
C. PTAH
D. HMB-45
E. Neuron specific enolase
►D

Granular cell tumors are of neural origin, with the granules representing an accumulation of
lysosomes in the cytoplasm. Lesions present on the tongue in 25% of cases. The granules stain
positive with periodic acid-Schiff (PAS) staining and are resistant to diastase. Being of neural
origin, lesions also stain with S100 and neuron specific enolase. The cells would not react with
melanocyte markers, such as HMB-45.

216
A 44-year old woman presents to clinic two weeks after injuring her finger while pruning her
rose garden. Exam reveals a suppurative nodule on her right index finger with lymphangitic
spread involving her forearm. Biopsy exhibits a dense granulomatous infiltrate and which of the
following histologic findings:
A. Asteroid Bodies
B. Caterpillar Bodies
C. Cowdry Type A Bodies
D. Donovan Bodies
E. Dutcher Bodies
►A

This is a classic presentation of sporotrichosis, which presents as lymphangitic spread after


penetrating trauma. Other organisms with true lymphangitic spread include leishmania, nocardia,
and atypical mycobacteria. Biopsy of sporotrichosis reveals granulomatous inflammation and
asteroid bodies, along with cigar bodies of budding yeast cells.

217
Which immunohistochemical stain would be positive in eosinophilic granuloma?

518
A. HMB-45
B. Cytokeratin 20
C. Congo red
D. Mucin
E. CD1a
►E

Eosinophilic granuloma is a form of Langerhans Cell Histocytosis (LCH), previously called


Histiocytosis X. Eosinophilic granuloma is a localized, benign form which is more common in
males and generally affects the bones. All forms of LCH are characterized by the infiltration of
Langerhans cells on pathology, which stains for S-100, CD1a and contain cytoplasmic birbeck
granules.

218
Scalp biopsy of a 14-year-old female with suspected alopecia areata would likely reveal
inflammation around which portion of the hair follicle:
A. Dermal papilla
B. Hair bulb
C. Inner root sheath
D. Isthmus
E. Infundibulum
►B

On biopsy, alopecia areata exhibits a peribulbar lymphocytic ―swarm of bees‖. Discoid lupus
typically exhibits inflammation surrounding the isthmus, along with a perivascular dermatitis and
vacuolar interface changes. Lichen planopilaris exhibits inflammation most densely concentrated
about the infundibulum.

219
The predominant location of the cleft in porphyria cutanea tarda is:
A. Dermal
B. Basement membrane zone
C. Basal keratinocytes
D. Subrabasal
E. Subcorneal/granular
►B

Porphyria cutanea tarda (PCT) is the most common porhyria. It is due to a deficiency of
uroprophyrin decarboxylase. Adults present with sun induced vesicles, papules, crusts and milia
in areas of scaring. On histology the cleft in porphyria cutanea tarda is subepidermal or in the

519
basement membrane zone with festooning of dermal papillae into the blister. Hyalinized material
around blood vessels in the papillary dermis may be seen, as well as caterpillar bodies
(eosinophilic segemented basement membrane) in the roof of the blister. PCT is associated with
hepatitis C, alcohol abuse and liver disease.

220
What is the most common location of a verruciform xanthoma?
A. Mouth
B. Lower leg
C. Face
D. Hand
E. Thigh
►A

The oral mucosa is the most common location of a verruciform xanthoma. It is also seen in
CHILD syndrome, EBA, GVHD, and lymphedema syndromes.

221
What is this neoplasm?
A. Trichoadenoma
B. Trichoblastoma
C. Trichoepithelioma
D. Basal cell carcinoma
E. Nevus comedonicus
►A

Trichoadenoma: A nodule on the face or buttock that is slightly depressed. Typically confined to
the upper dermis and composed of multiple milia or infundibular-like cysts that have a squamous
epithelial lining associated with a granular layer and central flakey keratin in the lumen. The
stroma can be sclerotic. The lesion is composed primary of cysts with a few thin strands of
basaloid cells; if basaloid strands predominate with only a few cysts then lesion is a
trichoepithelioma.

222
Psoriasis has been shown to be genetically linked to:
A. Pityriasis rosea
B. Clear cell acanthomas
C. Epidermodysplasia verruciformis
D. Verruciform xanthomas
E. Poromas

520
►C

Mutations have been found in the EVER1 and EVER2 genes in epidermodysplasia
verruciformis; these genes are on 17q25, the location of the PSORS2 gene (one of the genes
implicated in the inheritance of psoriasis).

223
In addition to psoriasis, all of the following dermatoses demonstrate regular psoriasiform
hyperplasia except:
A. Lichen simplex chronicus
B. Pityriasis rubra pilaris
C. Acanthosis nigricans
D. Reiter‘s syndrome
E. Inflammatory linear verrucous epidermal nevus
►A

Lichen simplex chronicus develops in areas of chronically rubbed skin. On histopathology, there
is hyperkeratosis with areas of parakeratosis, hypergranulosis, slight spongiosis, and a sparse
superficial perivascular infiltrate. The acanthosis in LSC is irregular. The other choices all have
regular psoriasiform hyperplasia.

224
A pregnant patient presents complaining of a worsening skin eruption for the duration of the past
three years. What is the most likely diagnosis?
A. Seborrheic dermatitis
B. Tinea versicolor
C. Tinea corporis
D. Prurigo of pregnancy
E. Pityriasis rosea
►B

Tinea versicolor presents as hyper or hypopigmented coalescing scaly macules on the trunk and
upper extremities. It is more common during the summer and favors oily areas of skin. Mild
pruritus may be present. It is caused by Malassezia furfur. Pityrosporum orbiculare is the yeast
phase of the organism. This patient had tinea versicolor for several years and apparently got
worse during her pregnancy. A study in Italy, revealed the frequency of tinea versicolor during
pregnancy (5.7%) does not seem different from that reported in general population living in
temperate climates (2-5%). However, higher degree of colonization by Malassezia resulted at the
end of pregnancy and postpartum.

521
225
Eosinophilia-Myalgia syndrome is caused by:
A. Norwegian salt-petter
B. Unadultered Spanish grapeseed oil
C. Pb intoxication
D. L-Tryptophan
E. Excessive anaerobic exercise
►D

The eosinophilia myalgia syndrome is characterized by marked peripheral eosinophilia with a


clinical spectrum of signs and symptoms, including generalized myalgias, pneumonitis,
myocarditis, neuropathy, encephalopathy and fibrosis. Many patients progress to a clinical
picture clinically indistinguishable from eosinophilic fasciitis. The disease is caused by the
ingestion of certain lots of L-tryptophan.

226
Which of the following can be seen in dermatofibromas?
A. Hypopigmentation of the basal layer
B. Infiltration into the fat with a honeycomb pattern
C. Hypoplasia of the epidermis
D. S-100 positivity
E. Vimentin positivity
►E

Dermatofibromas have characteristic features that can include collagen trapping, hyperplasia of
epidermis, hyperpigmentation of basal layer, dermal spindle cells, and whorls of fibrous tissue
with keloidal collagen. It stains with factor XIIIa(+) but not MAC 387 (-), S-100(-), or CD34 (-).
DFSP infiltrates the fat with a honeycomb pattern.

227
Verruciform xanthoma is seen most commonly on:
A. Head and neck
B. Distal extremities
C. Oral mucosae and genital areas
D. Mucosal surfaces and trunk/proximal extremities
E. Nail bed and periungual areas
►C

522
Verruciform xanthoma is an uncommon lesion that usually occurs on the oral mucosa of middle-
aged persons or on the scrotum of middle-aged to elderly Japanese men. The most common site
for verruciform xanthoma is the oral mucosa.

228
Which type of lymphoma with an excellent prognosis is shown here?
A. Follicuar
B. Marginal zone
C. Diffuse large B-cell
D. Cutaneous T-cell
E. NK/T-cell
►A

Follicular lymphoma demonstrates a nodular or diffuse infiltrate involving the dermis and
sometimes the subcutis. There may be a follicular, follicular and diffuse, or a diffuse pattern of
atypical B-cells.

229
Which stain would best elucidate this disease?
A. Alcian blue
B. Fontana-Masson
C. Silver
D. Prussian blue
E. Fite
►E

A Fite stain would highlight the acid fast bacilli M. leprae.

230
This was a deep seated large tumor in the thigh, what is it?
A. Liposarcoma
B. Nodular fascitis
C. Hibernoma
D. Spindle cell lipoma
E. Pleomorphic lipoma
►A

A liposarcoma is one of the most common sarcomas to occur in adults, 50-70yos. Those that
occur in resectable regions (like the thigh) have a good prognosis; those that occur in
retroperitoneum, mediastinum or spermatic cord recur repeatedly and have a risk to

523
dedifferentiate with a mortality approaching 80% over the following 10-20 years. It has a
characteristic supernumerary ring with amplification of 12q14 MDM2 region. Histologically The
adipocytes vary in size throughout the tumor. Pleomorphism is variable depending upon the
degree of differentiation that the tumor displays. The nuclei of the more well-differentiated
adipocytes, as well as, the spindled cells are hyperchromatic with nuclear atypia. The presence of
lipoblasts, i.e. either mono-vacuolated signet ring type of lipoblast as is more commonly seen in
myxoid liposarcoma and/or multi-vacuolated lipoblasts with central nuclei scalloped by
vacuoles, are characteristic of a liposarcoma, but are not necessary to make the diagnosis. There
is often a plexiform proliferation of blood vessels, with an appearance that has been likened to
crow's feet or chicken wire. The stroma can be loose and delicate, fibrous or myxoid. Thick
ropey fibrous septae are common in liposarcoma and in lipoblastoma.

231
An obese 15-year-old boy is diagnosed with acanthosis nigricans. The presence of this disorder
correlates best with elevated plasma:
A. Glucagon
B. Calcitonin
C. Insulin
D. Glucose
E. Calcium
►C

Acanthosis nigricans correlates most closely with elevated insulin resistance. Acanthosis
nigricans may also present in association with internal malignancies, most commonly gastric
cancer. Additionally, certain drugs, classically niacinamie, may induce this disorder.

232
A 54-year-old woman presents to clinic with a tender lesion involving the index finger that is
worsened by cold. Biopsy findings reveal a glomus tumor. Electron microscopy of these tumors
characteristically exhibits:
A. Comma-shaped bodies
B. Pustulo-ovoid bodies
C. Russel bodies
D. Verocay bodies
E. Weibel-Palade bodies
►E

Weibel-Palade bodies are organelles that are seen on macroscopy of endothelial cells. These are
increased in glomus tumors. Pustolo-ovoid bodies are seen in granular cell tumors. Verocay
bodies are palisading nuclei arranged in rows with peripheral eosinophilic cytoplasm

524
characteristic of Schwannomas. Russel bodies are immunoglobulin inclusions in plasma cells.
Comma-shaped bodies may be seen on electron microscopic analysis of benign cephalic
histiocytosis.

233
All of the following are true of reticulohistiocytoma except:
A. Rare occurrence in children
B. Giant cells with ―ground-glass‖ cytoplasm
C. Association with arthritis
D. Immunostaining is positive for OKM1
E. Trauma is precipitating factor
►C

Reticulohistiocytomas, also called giant cell reticulohistiocytomas, occur almost exclusively in


adults. They are generally solitary, and unlike the multicentric type, are not associated with
mutilating arthritis or predisposition for malignancy.

234
On histology this disease has mucin deposition within the external root sheath and sebaceous
gland. The mucin distends into the intracellular spaces within follicular epithelium forming
cystic cavities. The following stains will be positive except for:
A. CD3
B. CD4
C. CD7
D. CD8
E. CD9
►E

This patient has follicular mucinosis. This is benign and seen in children and is a nonscarring
alopecia and may be pruritic or tender seen on the face and scalp. All the following stains will be
positive CD3, CD4, CD7, CD8.

235
Apocrine hidrocystoma may be associated with which of the following syndromes?
A. Downs syndrome
B. Schopf-Schulz-Passarge
C. Buschke-Ollendorf syndrome
D. Cowden‖s Disease
E. Osteogensis Imperfecta
►B

525
Schopf-Schulz-Passarge is a syndrome of ectodermal dysplasia which presents with keratosis
palmoplantaris with hypodontia, hypotrichosis, and cysts of the eyelids. The other answer
choices have not been associated with apocrine hidrocystomas.

236
What is the best diagnosis?
A. Psoriasiform
B. Dermatitis Herpetiformis
C. Angiolymphoid Hyperplasia with Eosinophilia
D. Lichen Nitidus
E. Ochronosis
►C

Angiolymphoid Hyperplasia with Eosinophilia(EPITHELIOID HEMANGIOMA) consists of


solitary or multiple benign cutaneous nodules comprised of immature and mature vascular
structures intermingled with endothelial cells and a varied infiltrate of eosinophils, histiocytes,
lymphocytes, and mast cells.

237
This characteristic change of the fat is characteristic of which panniculitis?
A. Subcutaneous panniculitis like T cell lymphoma
B. Lupus panniculitis
C. Sclerosing panniculitis
D. Pancreratic panniculitis
E. Calciphylaxis
►B

Lupus panniculitis is characterized by hyalinization of the fat with a lymphoplasmocytic


infiltrate, as depicted here. It is a lobular panniculitis without vasculitis. The other answer
choices are other lobular panniculitidies. Subcutaneous panniculitis like t cell lymphoma has
atypical cells rimming the fat lobules. Sclerosing panniculitis is lipodermatosclerosis which has
lipomembranous change. Pancreatic panniculitis has liquefactive fat necrosis and saponificatoin
of the fat. Calciphylaxis has calium deposites in the walls of tbe blood vessels.

238
What medication causes the following lesion to erupt?
A. Metformin
B. coumadin
C. ace inhibitors

526
D. Indinavir
E. azathioprine
►D

Indinavir, capecitabine, systemic retinoids, hydroxyurea and GMCSF cause pyogenic


granulomas to erupt.

239
Psoriasis vulgaris may demonstrate all except one of the following histopathologic changes.
A. Munro microabscesses
B. Clubbing of the rete ridges
C. Spongiform pustules of Kogoj
D. Hypergranulosis
E. Suprapapillary thinning
►D

Histopathological features of psoriasis: Hyperkeratosis with confluent parakertosis associated


with a thin to absent granular layer, Uniform elongation of rete ridges and they are expanded at
their tips (Clubbing of rete ridges), papillary dermal edema with dilated toruous capillaries,
thinned suprapapillary plates, neutrophils within stratum corneum (Munro's micorabscesses),
neutrophils within the spinous layer (spongiform pustule of Kogoj), and superficial perivascular
lymphocytic infiltrate.

240
You examine a biopsy that is square on low power. The clinical history reports an IgG
paraproteinemia and the presence of a "doughnut sign". Which of the following conditions would
you think of without looking on higher power?
A. Scleromyxedema
B. Scleroderma
C. Scleredema
D. Localized Morphea
E. Pretibial myxedema
►A

The doughnut sign is seen on proximal interphalangeal joints where a central depression
surrounded by an elevated rim can be seen (Bolognia p649). This is a feature of the skin
thickening seen in Scleromyxedema. The other options will have a square appearance on low
power microscopic examination and scleredema can have an IgG paraprotein. Other microscopic
findings in scleromyxedema include incrased fibroblasts with fibrosis/increased collagen and

527
thickening of the dermis. Mucin may be scant or absent. This bx is more cellular than pretibial
myxedema.

241
What is the best treatment for this eruption?
A. Prednisone
B. Gluten-free diet
C. Dapsone
D. Rituximab
E. Mycophenolate mofetil
►B

The most effective way to treat dermatitis herpetiformis is to follow a gluten-free diet.

242
This lesion is associated with what syndrome?
A. nicolau balus syndrome
B. cowdens
C. muir torre
D. gardner
E. schopf schultz passarge
►A

Nicolau balus syndrome is associated with syringomas as depicted here on path, plus milia and
atrophoderma vermiculatum. Cowdens is associated with tricholemomas; muir torre is associated
with Kas and sebaceous neoplasms; and gardner syndrome is assoicated with EICs and colon
cancer. Schopf schultz passarge is associated with eccrine syringofiboradenomas.

243
What is this neoplasm?
A. Hibernoma
B. Malakoplakia
C. Myospherulosis
D. Liposarcoma
E. Pleomorphic lipoma
►A

Hibernoma: This is a rare neoplasm that typically occurs in 30-40 year old males as a slowly
enlarging warm mass on scapular region, trunk, axilla or thigh. There are also myxoid & spindle
cell variants that occur on the posterior neck/shoulder. The lesion enhances with contrast on CT

528
and MRI fails to reveal fat septations which differs for that seen with other lipomas.
Abnormalities in 11q13 and loss of MEN1 gene on 10q22 have been noted. Histologically: The
lesion is composed of large polygonal adipocytes that have with multiple vacuoles and an
eosinophilic granular cytoplasm with a central nucleus and prominent nucleolus. These cells are
referred to as ―Mulberry cells‖•. Admixed among these larger adipocytes are smaller cells with
a granular cytoplasm, mature white fat and some times spindle cells.

244
A 30-year old male presents with clustered, painful plaques on his shoulder. Histopathology
showed interlacing bundles of cells with eosinophilic cytoplasm and no mitoses. Desmin and
SMA stains were positive. What gene defect is associated with the development of these
lesions?
A. Fumarate Hydratase
B. Endoglin
C. ERCC8
D. RECQL2
E. GJB3
►A

The patient has multiple cutaneous leiomyomas. Hereditary leiomyomatosis is an inherited


defect of fumarate hydratase. These patients develop multiple cutaneous leiomyomas and have
an increased incidence of renal cell carcinoma. Female patients will usually also develop
significant uterine leiomyomatosis resulting in hysterectomy.

245
Treatment of a patient with chronic venous ulceration is best achieved by which modality?
A. endoveous laser ablation
B. ambulatory phlebectomy
C. sclerotherapy
D. observation
E. surgical excision and debridement
►A

Venous stasis ulcers often occur due to an incompetent superficial venous system which creates
pooling of blood in the ankles. Treatment of the atypical greater saphenous vein with
endovenous laser ablation, removes the atypical veins allowing the blood to return to the heart in
the deep system, relieveing the pressure in the venous system and thus allows the venous
ulceration to heal.

246

529
Scalp biopsy of a 54-year-old female with suspected lichen planopilaris would likely reveal
inflammation around which portion of the hair follicle:
A. Dermal papilla
B. Hair bulb
C. Matrix
D. Isthmus
E. Infundibulum
►E

On biopsy, alopecia areata exhibits a peribulbar lymphocytic ―swarm of bees.‖ Discoid lupus
typically exhibits inflammation surrounding the isthmus, along with a perivascular dermatitis and
vacuolar interface changes. Lichen planopilaris exhibits inflammation most densely concentrated
about the infundibulum.

247
Elastosis perforans serpiginosa is associated with all except:
A. Rothmund-Thompson
B. Scleredema
C. Ehlers-Danlos
D. Acrogeria
E. Down‘s
►B

Elastosis perforans serpiginosa is associated with Down‘s syndrome, Ehlers-Danlos type IV,
Osteogenesis imperfecta, Rothmund Thompson, Marfan‘s, Werner‘s, acrogeria, and
penicillamine therapy.

248
Which of the following regarding stains is true?
A. A Verhoeff-van Gieson stain is used to stain elastic fibers red.
B. A methanamine silver stain is used to identify bacteria.
C. A fite stain is used to identify spirochetes.
D. A Von Kossa stain is used to identify calcium.
E. A giesma stain is used to identify eosinophil granules.
►D

Stains/Application/Result Van Gieson - Elastic fibers - Black Methanamine-silver - Fungi,


parasites - Black AFB/Fite - Acid-fast bacilli - Red Von Kossa - Calcium - Black Giesma - Mast
cells - Metachromatically purple

530
249
This is an H and E stained slide, what is the organism?
A. Exophiala jeanselmei
B. Chromomycosis
C. Nocardia
D. Zygomycosis
E. Mucormycosis
►A

Phaeohyphomycotic cyst: Typically the result of an opportunist infection caused by being


impaled by a splinter contaminated with a dematiaceous or pigmented fungus. Most commonly
seen on distal extremities and results in a dermal abscess with fibrous tissue and granulomas
within which are pigmented hyphae and yeast. The most common organism is Exophiala
jeanselmei (yellow-brown septae hypha) and Wangiella dermatitidis. Histology: Circumscribed
cyst with a fibrous wall within which is a chronic granulomatous reaction or dermal abscess, a
splinter is sometimes seen along with the brown filamentous hyphae and yeast.
250
Which stain would confirm this diagnosis?
A. D2-40
B. CD68
C. CD34
D. S100
E. HHV8
►E

Kaposi sarcoma is positive for HHV8.

251
On histology this disease has mucin deposition within the external root sheath and sebaceous
gland. The mucin distends into the intracellular spaces within follicular epithelium forming
cystic cavities. The following stains will be positive except for:
A. CD3
B. CD4
C. CD7
D. CD8
E. CD9
►E

531
This patient has follicular mucinosis. This is benign and seen in children and is a nonscarring
alopecia and may be pruritic or tender seen on the face and scalp. All the following stains will be
positive CD3, CD4, CD7, CD8.

252
Osteoclast-like giant cells are characteristic of:
A. Giant cell fibroblastoma
B. Reticulohistiocytoma
C. Necrobiosis lipoidica diabeticorum
D. Giant cell tumor of the tendon sheath
E. Necrobiotic xanthogranuloma
►D

Osteoclast-like giant cells are seen in giant cell tumor of the tendon sheath. Osteoclast-like giant
cells have eosinophilic cytoplasm with haphazardly arranged nuclei. Reticulohistiocytoma is
characterized by giant cells with an oncocytic glassy pink cytoplasm.

253
What is the diagnosis?
A. Melanoma
B. Halo nevus
C. Lichen planus
D. Lichen nitidus
E. Lichenoid keratosis
►E

A licheniod keratosis is a benign lesion with a localized lichenoid infiltrate. There are no nests of
melanocytes present, making a halo nevus unlikely. There is no junctional component or dermal
component of melanocytes, making melanoma highly unlikley. Lichen nitidus is the classic "ball
and claw" with the rete grasping lymphocytes and a few histiocytes.

254
Supporting evidence for the diagnosis of mycosis fungoides is CD4+ lymphocytes with loss of
CD7 as well as loss of
A. CD27
B. CD68
C. CD5
D. CD20
E. CD30
►C

532
CD5 as well as CD7 are sometimes lost on the surface of epidermotropic T cells in mycosis
fungoides. CD2, CD3, and CD5 are T cell markers. CD20 is a B cell marker. CD30 is positive in
anaplastic large cell lymphoma cells, Hodgkins lymphoma, and lymphomatoid papulosis.
Reactive infiltrates can also have some CD30-positive cells. CD68 marks histocytes.

255
This lesion is often found in which anatomic location?
A. Skin folds
B. Acral sites
C. Scalp
D. Mucosa
E. Trunk
►A

This image demonstrates an infarcted skin tag. Skin tags are common in skin folds.

256
What is your best diagnosis?
A. Sebaceous Glands
B. Sebaceum
C. Lichen Planus
D. Atopic Eczema
E. Angiofibroma
►E

Angiofibroma is a small reddish spot or bump that consists of fibrous tissue and blood vessels.
They are most commonly found around the nose, cheeks, and chin, often combining to form a
distinctive butterfly-shaped pattern. Previously known as adenoma sebaceum.

257
This material is formed during which stage of wound healing?
A. Inflammatory
B. Proliferative
C. Remodeling
D. Contraction
E. Vasoconstrictive
►B

Granulation tissue forms during the proliferative phase of wound healing.

533
258
Which of the following histological features would be most helpful in differentiating lichenoid
drug eruption from lichen planus?
A. Civatte bodies
B. Parakeratosis and eosinophils
C. Squamatization of the basal layer
D. Presence of pruritus
E. Band-like infiltrate with ―Saw-tooth‖ rete ridges
►B

Lichenoid drug eruptions share clinical and histopathologic features with lichen planus.
Sometimes differentiation is not possible; however, eosinophil, parakeratosis, and a deeper
perivascular infiltrate is more suggestive of lichenoid drug. Implicated medications include
captopril, penicillamine, and chloroquine.

259
What are these organisms?
A. Leishmania
B. Trichosporon beigelii
C. Sporothrix schenckii
D. Penicillium marneffei
E. Histoplasmosis capsulatum
►A

Leishmaniasis: Three main types: cutaneous (L tropica and mexicana), mucocutaneous (L.
brasiliensis) and visceral/kala-azar (L donovani). Old world Leishmania (L tropica-major, minor
and aethiopica) transmitted by Phlebotomus sandfly, new world (L mexicana, brasiliensis)
transmitted by Lutzomyia sandfly. The promastigote lies in the sandfly is transmitted to humans
and the amastigote infects macrophages. Macrophages become engorged with the organisms, but
there is abnormal intracellular killing. Histology: Epidermis may display PEH and there is a
dense dermal infiltrate consisting of foamy histiocytes, Leishman-Donovan bodies, with
intracellular amastigotes along with epithelioid monocytes, giant cells, plasma cells and
lymphocytes, variable eosinophils and PMNs. Organisms stain better with Giemsa, have a
paranuclear kinetoplast and typically are located at periphery of macrophage, i.e. the Marquee
sign. Can also see organism on skin slit smear. Older lesions show decreased organisms and
tuberculoid granulomas may form.

260

534
On histology there is a proliferation of basaloid cells with larger pale cells in a jigsaw pattern.
There is an eosinophilic cuticle surrounding the lobules. This describes a:
A. Cylindroma
B. Eccrine spiradenoma
C. Mixed tumor
D. Nodular hidradenoma
E. Eccrine syringofibroadenoma
►A

This describes a cylindroma. This is a benign sweat tumor linked with CYLD gene. It presents as
a skin colored to erythematous bluish rubbery nodule. Located on the head and neck and on the
scalp. It is also known as a turban tumor.

261
All of the following markers can be helpful in differentiating basal cell carcinoma from
trichoepithelioma except
A. CD10
B. CD31
C. CD34
D. Bcl-2
E. Stromelysin-3
►B

The histopathologic discrimination of trichoepithelioma and BCC often presents a diagnostic


challenge. The distinction is of clinical significance due to the differences in prognosis and
treatment of these tumors. Thus, a large number of ancillary laboratory techniques have been
investigated as an aid in this differential, include CD10, CD34, BCL-2, Stromelysin-3 and Ki-67.
CD31 is a vascular marker that stains positive in angiosarcoma and Kaposi sarcoma.

262
Multiple clear cell acanthomas are associated with:
A. Ichthyosis
B. Cowden's
C. Immunosuppression
D. Gastrointestinal polyps
E. Breast cancer
►A

Clear cell acanthoma is associated with ichthyosis.

535
263
Which gene mutation and/or amplifications are more commonly found in this type of melanoma
and mucosal sites than in melanomas on intermittently sun-exposed sites?
A. NRAS
B. KIT
C. GNAQ
D. HRAS
E. PTEN
►B

KIT mutations and/or amplifications are more commonly found in melanomas located on acral
and mucosal sites than in melanomas on intermittently sun-exposed sites.

264
A 32-year-old woman presents to clinic with hypertrophic plaques on the external nares. Biopsy
revealed findings consistent with Rhinoscleroma. Findings are most likely to include:
A. Psammoma bodies
B. Pustulo-ovoid bodies
C. Russel bodies
D. Verocay bodies
E. Weibel-Palade bodies
►C

Rhinoscleroma is caused by Klebsiella rhinoscleromatis. Biopsy findings include Mikulicz cells,


which are foamy macrophages containing bacteria, and Russel bodies. Russel bodies are
immunoglobulin inclusions in plasma cells. Pustolo-ovoid bodies are seen in granular cell
tumors. Verocay bodies are palisading nuclei arranged in rows with peripheral eosinophilic
cytoplasm characteristic of Schwannomas. Weibel-Palade bodies are organelles that are seen on
macroscopy of endothelial cells.

265
Which of the following is true of Rosai-Dorfman disease?
A. S-100 negative, CD1a positive, not characterized by emperipolesis
B. S-100 positive, CD1a positive, not characterized by emperipolesis
C. S-100 positive, CD1a negative, characterized by emperipolesis
D. S-100 positive, CD1a positive, characterized by emperipolesis
E. S-100 negative, CD1a negative, characterized by emperipolesis
►C

536
Rosai-Dofrman disease (also known as sinus histiocytosis with massive lymphadenopathy) is
characterized by large "fluffy" histiocytes that are S-100 positive but CD1a negative,
multinucleated giant cells, plasma cells, aggregates of lymphocytes, and emperipolesis. There is
debate as to whether the condition is related to herpes type 6.

266
What type of lymphoma is shown here?
A. Follicle center cell
B. Intravascular large B-cell
C. Marginal zone
D. Cutaneous T-cell
E. NK/T-cell
►B

Intravascular B-cell lymphoma is characterized by the presence of large, atypical cells partially
or fully occluding blood vessels in the dermis and subcutis. The prognosis is very poor.

267
What is the diagnosis?
A. Squamous cell carcinoma
B. Trichilemmoma
C. Poroma
D. Pilar sheath acanthoma
E. Inverted follicular keratosis
►E

An inverted follicular keratosis is a benign tumor of the follicular infundibulum. On histology,


they demonstrate tumor lobules composed of central squamous cells and peripheral basaloid
cells. Squamous eddies are commonly noted.

268
What exongenous material is shown here?
A. Calcium hydroxylapatite
B. Hyaluronic acid filler
C. Suture
D. Gelfoam
E. Poly-L-lactic acid
►D

537
Gelfoam is present in this tissue specimen. Gelfoam has a characteristic appearance with slightly
basophilic honeycomb or collapsed net-like deposits.

269
Of the patients that present with discoid lupus, how many will progress to systemic lupus
erythematosus?
A. 5%
B. 25%
C. 50%
D. 75%
E. 100%
►A

Of the patients that present with DLE, 5% of them will prgress to SLE. However, of the patients
that present with SLE, 25% will get DLE.

270
What is this lesion?
A. Hidradenoma
B. Spiradenoma
C. Dermal duct tumor
D. Trichoepithelioma
E. Chondroid syringoma
►E

Chondroid syringomas (mixed tumors) are well-circumscribed dermal nodules demonstrating


epithelial cords and ductal structures in a myxoid, chondroid and fibrous stroma.

271
This eruption is often associated with which disease?
A. Celiac disease
B. Inflammatory bowel disease
C. Chronic myelogenous leukemia
D. HIV
E. Rosacea
►C

Acute febrile neutrophilic dermatosis (Sweet syndrome) is associated with certain medications,
inflammatory bowel disease, malignancy, certain infections and can also be idiopathic. Bullous
lesions are more common in patients with underlying malignancies.

538
272
What is the neoplasm in this biopsy?
A. Chondroid syringoma
B. Papillary eccrine adenoma
C. Tubular apocrine adenoma
D. Eccrine acrospiroma
E. Trichoblastoma
►A

Chondroid Syringoma: Most commonly occurs in middle age males on the head and neck.
Histologically this is a well circumscribed dermal tumor composed of dilated ducts, hyaline or
plasmacytoid cells (myoepithelial cells), keratinous cysts, +/- cells that show hair matricial
differentiation occasionally with some areas of mature fat. All elements arise within a
homogenous bluish chondromyxoid matrix (+ Type II collagen, + acid MPS stains with Alcian
blue/green (stains at high and low PH) consistent with chondroitin sulfate). Focal calcification
may occur.

273
A patient presents with a few of these lesions and some hyperkeratotic papuleson his anterior
shins. What diagnositic test should the patient undergo?
A. Colonoscopy
B. cystoscopy
C. bronchoscopy
D. cortisone stimulation
E. otoscopy
►A

The image is a sebaceous adenoma, and the question describes keratoacanthomas on the anterior
shin, thus this patient should undergo colonoscopy to evaluate for colon cancer. Given this
constellation of symptoms one must evaluate for muir torre syndrome.

274
This lesion most commonly metastasizes to which location?
A. Scalp
B. Arm
C. Leg
D. Back
E. Nose
►A

539
Renal cell carcioma most commonly metastasizes to the scalp.

275
This clinically raspberry-like papilloma occurs in which syndrome?
A. Goltz
B. Rothmund-Thomson
C. Incontinentia pigmenti
D. Hermansky Pudlak
E. CHILD
►A

Goltz syndrome is characterized by distinct raspberry-like papillomas at junctions of mucosa and


skin. Other findings include the following: skin atrophy, ulcerations, hernialike outpouchings of
fatty tissue, syndromic facies, and lobster claw deformity.

276
Which of the following regarding necrobiotic xanthogranuloma and normolipemic plane
xanthomas is FALSE:
A. Both can have an associate paraproteinemia
B. Both commonly have normal serum lipids
C. Both are most commonly distributed on the upper body
D. Normolipemic plane xanthomas lack the induration and ulceration commonly
seen in necrobiotic xanthogranuloma
E. Multiple myeloma is seen in 80% of necrobiotic xanthogranuloma but is rarely
seen in normolipemic plane xanthomas
►E

Necrobiotic xanthogranulomas are closely related to normolipemic plane xanthomas: both have a
yellow hue, associated paraproteinemia, normal serum lipids, upper body distribution. However,
normolipemic plane xanthomas have a much stronger association with multiple myeloma, their
plaques lack induration and they rarely ulcerate. Treatment is usually directed at the
paraproteinemia.

277
This lesion would most likely stain positive for which of the following?
A. von Kossa
B. VVG
C. mucicarmine
D. oil red o

540
E. mart-1
►A

This is scrotal calcinosis. The calcium component would stain strongly with von Kossa. VVG
stains elastin, mucicarmine stains mucin pink, oil red o stains fat and mart 1 stains melanocytes.

278
This tumor is vimentin+ and cytokeratin+:
A. Angiosarcoma
B. Neurothekeoma
C. Nodular fasciitis
D. Dermatofibroma
E. Epithelioid sarcoma
►E

Characteristic immunohistochemistry of epithelioid sarcoma is vimentin- and cytokeratin-


positivity.

279
Where does this lesion typically occur on the body?
A. Anywhere along the milk lines
B. Preauricular area
C. On the neck
D. On the hand
E. On the leg
►A

supernumerary or accessory nipple can occur anywhere along the milk lines. The histologic
features include epidermal acanthosis, smooth muscle bundles, pilosebaceous units, and
sometimes mammary glands.

280
Which stain would confirm this diagnosis?
A. D2-40
B. CD68
C. CD34
D. S100
E. HHV8
►E

541
Kaposi sarcoma is positive for HHV8.

281
Which antibody is responsible for this disease?
A. IgG to BP180
B. IgA to BPA2
C. IgA to desmosomal components of the skin
D. IgG to Dsg1
E. IgG to collagen VII
►C

IgA pemphigus is an intraepidermal vesiculopustular disorder caused by IgA autoantibodies to


desmosomal components of the epidermis, desmogleins, or desmocollins.

282
Which gene mutation and/or amplifications are more commonly found in this type of melanoma
and mucosal sites than in melanomas on intermittently sun-exposed sites
A. NRAS
B. KIT
C. GNAQ
D. HRAS
E. PTEN
►B

KIT mutations and/or amplifications are more commonly found in melanomas located on acral
and mucosal sites than in melanomas on intermittently sun-exposed sites.

283
Which disorder is shown here?
A. Contact dermatitis
B. Incontinentia pigmenti
C. Angiolymphoid hyperplasia with eosinophilia
D. Granuloma faciale
E. Arthropod assault
►D

On histology, granuloma faciale is characterized by a mixed inflammatory cell infiltrate


separated from the epidermis by a Grenz zone.

284

542
The organism on this slide stained with Warthin-Starry is responsible for what disease?
A. Bacillary Angiomatosis
B. Cellulitis
C. Madura foot
D. Pyoderma gangrenosum
E. Pyogenic granuloma
►A

Bacillary (epithelioid) angiomatosis is caused by Bartonella Henselae or Bartonella quintana


(trench fever- from body louse). Clinically resembles PG but multiple lesions and in a HIV+
patient, usually with history of exposure to cats. Histologically there is dome shaped lesion with
an epidermal collarette with a proliferation of reactive plump blood vessels similar to a PG, but
the lesion is less lobular and there is PMNs and dust throughout the lesion along with
extracellular clumps of purplish gram -ve bacilli. The organism is + for Warthin-Starry, which
highlights the organisms in a characteristic Chinese letter configuration.

285
Which of the following is true regarding S-100 Protein?
A. S-100 protein is an basic protein that binds Ca2+ and Zn2+
B. It is not soluble in 100% ammonium sulfate at neutral pH
C. Can be detected in melanocytes and in Schwann cells
D. It is not useful in diagnosing of spindle cell melanoma & desmoplastic
melanoma
E. It is not useful in diagnosing poorly differentiated cutaneous metastases
►C

S-100 protein is an acidic protein that binds Ca2+ and Zn2+. At a neutral pH, it is soluble in
100% ammonium sulfate. It is useful in diagnosing both spindle cell melanomas, desmoplastic
melanomas, as well as poorly differentiated cutaneous metastases. It also stains positively in
neurofibromas and schwannomas.

286
This type of lupus completely spares the epidermis.
A. acute cutaneous lupus
B. discoid lupus
C. chronic cutaneous lupus
D. chilblains lupus
E. tumid lupus
►E

543
Tumid lupus completely lacks any interface dermatitis and the epidermis is completely spared. It
is the most photosensitive of all the lupus types. There is lots of mucin on biopsy.

287
The deficiency in familial multiple cutaneous leiomyomatosis is:
A. Arginase
B. Adenosine deaminase
C. Endoglin
D. MC1R
E. Fumarate hydratase
►E

Fumarate hydratase is deficient in familial multiple cutaneous leiomyomatosis. This same


enzyme is deficient in familial uterine leiomyomatosis associated with renal cell cancer.
Arginase is deficient in arginemia. Adenosine deaminase is deficient in autosomal recessive
severe combined immunodeficiency disease (SCID). Endoglin is deficient in some patients with
Osler-Weber-Rendu. MC1R (melanocortin 1 receptor) shows sequence variation in people with
red hair.

288
Which of the following does not stain for mucopolysaccharides (mucin)?
A. Periodic-acid Schiff (PAS)
B. Trichome
C. Alcian blue
D. Colloidal iron
E. Mucicarmine
►B

Trichome does not stain acid mucopolysaccharides. Trichome stains collagen blue or green and
muscle red depending on reagents used. PAS stains mucopolysaccharides red, alcian blue stains
mucopolysaccharides blue, colloidal iron stains mucin blue and mucicarmine stains mucin red.

289
A male patient presents with hypopigmented macules on his upper back that are asymptomatic
and more noticeable in the summer. What is the most likely diagnosis?
A. pityriasis rosea
B. vitiligo
C. tinea versicolor
D. eczema
E. waardenberg syndrome

544
►C

Tinea versicolor often is asymptomatic and more noticeable in the summer as patients often tan
and the areas with the tinea versicolor do not tan as azeleic acid is secreted by the malassezia and
melanin production is suppressed.

290
These lymphocytes are positive for what stain?
A. CD8
B. CD4
C. CD68
D. CD56
E. CD20
►A

The lymphocytes in PLEVA are CD8 positive. It is an acute reaction with parakeratosis,
dyskeratotic keratinocytes, vacuolar change and a v shaped infiltrate.

291
This disorder is sometimes asscociated with what other symptoms?
A. Fissured tongue and facial nerve palsy
B. Shortness of breath
C. Angiomas
D. Atopic dermatitis
E. Neuromas
►A

Melkersson-Rosenthal syndrome is traditionally characterized by the triad of granulomatous


cheilitis, facial nerve palsy, and fissured tongue.

292
In this disorder, colloid bodies are composed of which material?
A. Fibrin
B. T-cells
C. Apoptotic keratinocytes, IgM, and complement
D. Langerhans cells
E. Basement membrane
►C

Colloid bodies are composed of apoptotic keratinocytes, IgM, and complement.

545
293
This biopsy was obtained from an annular pigmented plaque on the trunk, what is this
neoplasm?
A. Targetoid hemosiderotic hemangioma
B. Glomeruloid hemangioma
C. Angiosarcoma
D. Kaposi's sarcoma
E. Tufted hemangioma
►A

Targetoid Hemosiderotic Hemangioma: Occurs most commonly on young adult males on the
trunk and named for its clinical appearance. The lesion typically appears as a plaque with
concentric rings, there is violaceous central region surrounded by a ring of pallor which is then
surrounded by an erythematous or brown rim. Histologically in the superficial dermis there are
dilated blood vessels, some of which have plump hobnailed endothelial cells. In the deeper
dermis the vessels have a narrower lumen and dissect between the collagen bundles. Often
within the lesion there are extravasated red blood cells and hemosiderin.

294
The pigment deposits in ochronosis are accentuated with:
A. Cresyl violet
B. Methyl-green pyronin
C. Silver nitrate
D. Bodian
E. Cresyl violet and silver nitrate
►A

Cresyl violet stains the pigment deposits in ochronosis black. Methylene blue will also stain the
pigment black. This pigment does not stain with silver nitrate. The Bodian stain is for nerves.
Methyl-green pyronin stains RNA and DNA.

295
This disorder is characterized by what process?
A. Vascular damage caused by infiltration of neutrophils
B. Vascular fragility
C. Large vessel involvement
D. Delayed-type hypersensitivity reaction
E. The presence of amyloid
►A

546
Leukocytoclastic vasculitis is characterized by vascular damage caused by the infiltration of
neutrophils.

296
62-year old female with history of acute myeloid leukemia presents with multiple edematous,
erythematous papules after starting G-CSF.
A. Sweet's syndrome
B. Leukocytoclastic vasculitis
C. Bowel bypass dermatosis
D. Polymorphous light eruption
E. Erythema multiforme
►A

Sweet's syndrome, or acute febrile neutrophilic dermatoses, is often associated with AML and G-
CSF. Histologically, there is marked dermal edema with a prominent infiltrate composed of
neutrophils with leukocytoclasia. There is an absence of extensive vascular damage.

297
The predominant location of the cleft in acropustulosis of infancy is:
A. Dermal
B. Basment mebrane zone
C. Basal keratinocytes
D. Suprabasal
E. Subcorneal/granular
►E

Acropustulosis of infancy presents as idiopathic pustules on acral skin. Diagnosis is made only
after other causes of pustules have been ruled out, and it usually resolves in a few years. The
cleft in acropustulosis of infancy is subcorneal/granular with neutrophils.

298
Where does this lesion typically occur on the body?
A. Anywhere along the milk lines
B. Preauricular area
C. On the neck
D. On the hand
E. On the leg
►A

547
supernumerary or accessory nipple can occur anywhere along the milk lines. The histologic
features include epidermal acanthosis, smooth muscle bundles, pilosebaceous units, and
sometimes mammary glands.

299
Guarnieri bodies can be found associated with which of the following viral infections?
A. Variola
B. HHV6
C. HHV8
D. Rubeola
E. Rubella
►A

Guarnieri bodies are eosinophilic intracytoplasmic inclusions associated with variola infection
(smallpox), however they can also be seen in infections with other pox viridae, including
vaccinia, cowpox, and parapox. These inclusions represent aggregates of viral particles.

300
This malignancy resembles which cutaneous tumor?
A. Malignant melanoma
B. Leiomyosarcoma
C. Extramammary Paget's
D. Cutaneous T-cell lymphoma
E. Squamous cell carcinoma
►A

Clear cell sarcoma was historically termed "malignant melanoma of the soft parts." It is an
aggressive soft tissue tumor with a melanocytic phenotype but distinct genetic findings.

301
What type of nevus is represented in this image?
A. Blue
B. Spitz
C. Dysplastic
D. Combined
E. Balloon cell
►D

This nevus is composed of two distinct populations of melanocytes; thus, it would be considered
a combined nevus.

548
302
The presence of which cell type confirms this diagnosis?
A. Lipidized histiocyte
B. Mast cell
C. Multinucleate keratinocyte
D. CD30+
E. Langhans giant cell
►C

Herpes infection is confirmed by the presence of multinucleate keratinocytes which often have
intranuclear inclusion bodies.

303
If a pregnant woman <15 weeks presents with pink scaly patches all over her body and has this
pathology, what is she at risk for?
A. LGA
B. death
C. SGA
D. Premature delivery
E. SIDS
►D

If pityriasis rosea occurs in women <15 weeks pregnant, they are at risk for premature delivery
and neonatal hypotonia.

304
Which of the following is commonly seen histopathologically in pityriasis rosea?
A. Absent spongiosis
B. Full thickness necrosis
C. Exravasated erythrocytes in the subcutaneous fat
D. Lichenoid lymphocytic infiltrate
E. Focal parakeratosis
►E

Histopathological features of Pityriasis Rosea: Mild subacute spongiotic dermatitis, focal


parakeratosis (corresponds to cigarette paper-like scale), perviascular lymphocytic infiltrate,
hemorrhage in papillary dermis, and sometimes a few necrotic keratinocytes.

305

549
Steatocystoma multiplex is associated with:
A. Jadassohn-Lewandowsky
B. Jackson-Lawler
C. Schaufer-Brunauer
D. Zinsser-Engman-Cole
E. Touraine-Solente-Gole
►B

Jackson-Lawler (Jackson-Sertoli) is known as pachyonychia congenital type 2. Multiple


steatocysts can be seen in this condition.

306
This presented as a papule with a tuft of hair on the face, what is this neoplasm?
A. Trichofolliculoma
B. Trichoepithelioma
C. Fibrous Papule
D. Acrochordon
E. Fibrofolliculoma
►A

Trichofolliculoma: Most often occurs on the face as a papule with a tuft of hair. Histologically
composed of a central dilated large follicle (Mama hair), from which many secondary smaller
hair follicles radiate (Baby hairs), with the entire unit often enveloped in a vascularized fibrotic
stroma. If sebaceous glands are associated with the hair follicles then the term used is a
sebaceous trichofolliculoma or a Folliculosebaceous cystic hamartoma. Within the mesenchymal
stroma there are increased CD34 and factor XIIIA fibroblasts and Merkel cells as is seen in the
ORS of the follicles.

307
Which of the following immunohistochemical staining profiles of a non-breast skin lesion best
characterizes extra-mammary Paget‘s disease secondary to an underlying visceral malignancy?
A. MART-1 positive/cytokeratin 20 negative/cytokeratin 7 negative/gross cystic
fluid disease protein-15 negative
B. MART-1 negative/pankeratin positive/cytokeratin 7 positive/cytokeratin 20
positive/gross cystic fluid disease protein-15 negative
C. MART-1 negative/pankerative positive/cytokeratin 7 negative
D. MART-1 negative/pankeratin positive/cytokeratin 7 positive/cytokeratin 20
negative/gross cystic fluid disease protein-15 positive
E. CD34 positive, Factor XIIIa negative
►B

550
The differential for Pagetoid cells in the epidermis includes melanoma in situ, Paget‘s disease of
the breast, extra-mammary Paget‘s disease, squamous cell carcinoma in situ, sebaceous
carcinoma, and others. MART-1 positivity in the setting of pagetoid spread suggests melanoma
in situ. The combination of MART-1 negativity, pankeratin positivity, and cytokeratin 7
negativity suggests squamous cell carcinoma in situ. Both Paget‘s disease of the breast and extra-
mammary Paget‘s disease are characterized by pankeratin positivity and cytokeratin 7 positivity.
Extra-mammary Paget‘s is defined clinically by its location outside of the breast. Extra-
mammary Paget‘s disease arising secondary to a visceral malignancy is further characterized by
cytokeratin 20 positivity and gross cystic fluid disease protein-15 negativity, whereas primary
extra-mammary Paget‘s disease is cytokeratin 20 negative and gross cystic fluid disease protein-
15 positive.

308
Which of the following is more commonly seen in lichen sclerosus et atrophicus compared to
morphea?
A. Inflammation and fibrosis in the subcutis
B. Marked edema in the papillary dermis
C. Minimal hydropic degeneration noted at the dermaldermal-epidermal junction
D. Epidermis relatively normal with no thinning of the rete ridges
E. No follicular plugging
►B

Morphea vs. Lichen schlerosus et atrophicus: Morphea has a relatively normal epidermis without
follicular plugging. Interface changes are rare. In the dermis, the collagen fibers appear
homogenized. Elastic fibers are preserved. The inflammation and sclerosis can extend into the
subcutaneous fat. LS&A has an atrophic epidermis with follicular plugging. There is often
prominent hydropic changes with clefting. There is often edema of the papillary dermis and
elastic fibers are often absent. The subcutis is uninvolved.

309
These skin changes are caused by which type of trauma?
A. Cryotherapy
B. Thermal burn
C. Electrocautery
D. Chemotherapy
E. Ultraviolet radiation
►B

551
The histology of thermal burns include epidermal and dermal necrosis, thrombosis of blood
vessels, destruction of pilosebaceous units, fusion of the collagen bundles and a sparse
inflammatory cell infiltrate. Electrocautery effect often shows vertical elongation of
keratinocytes and homogenization of dermal collagen. Topical chemotherapy will result in
bizarre keratinocyte mitoses. Ultraviolet radiation will result in necrotic keratinocytes.
Cryotherapy will cause the epidermis to become homogenous and a subepidermal bulla to form.

310
What is the diagnosis?
A. Sarcoid
B. Rheumatoid nodule
C. Granuloma annulare
D. Xanthoma
E. Gout
►E

Large pale pink acellular areas consist of dissolved urate crystals in lesions of gout.

311
A 74-year-old man presents with red-brown papules with horny scales involving the dorsal feet
and lower legs. Biopsy reveals a discrete region of hyperkeratosis overlying thinned granular and
spinous layers with irregular acanthosis and a bandlike lymphoplasmacytic infiltrate in the
papillary dermis, thus confirming the suspected diagnosis of Flegel Disease. What abnormality
of the epidermis is most likely causative?
A. Absent keratohyaline granules
B. Diminished loricrin
C. Diminished involucrin
D. Diminished lamellar bodies
E. Decreased Transglutaminase I activity
►D

Diminished lamellar, or Odland, bodies is characteristic of Flegel‘s disease, otherwise known as


hyperkeratosis lenticularis perstans. Complete absence of lamellar bodies is observed in
Harlequin Fetus. Consistent abnormalities of the other epidermal components listed are not seen
in Flegel‘s disease.

312
Clinically, this lesion often has a blue hue and usually occurs on the face:
A. Apocrine hidrocystoma
B. Epidermoid inclusion cyst

552
C. Bronchogenic cyst
D. Eccrine hidradenoma
E. Myxoid cyst
►A

apocrine hidrocystoma are usually solitary, tranlucent nodules which may have a bluish hue due
to the Tyndall effect. Histologically, they have one or several large cystic spaces with
decapitation secretion.

313
What is this lesion?
A. Spindle cell lipoma
B. Lipoma
C. Angiolipoma
D. Pleomorphic lipoma
E. Liposarcoma
►C

Angiolipomas are characterized by mature fat with an increase in blood vesses. Thrombi are
often found within the vessels.

314
What is the best diagnosis?
A. Cholesterol emboli
B. Masson's Tumor
C. Calciphylaxis
D. Leukocytoclastic vasculitis
E. Processing Artifact
►A

Cholesterol Emboli: Typically occurs after a vascular procedure but can occur spontaneously.
Usually presents as livedo reticularis of distal lower extremities, associated with eosinophilia and
acute renal failure. Often need multiple deeper levels on sectioning a punch biopsy to see the
characteristic cholesterol clefts and fibrin thrombi in the lower dermis or subcutis.

315
A patient presents with this path and erythema nodosum and iritis. What is their likely
diagnosis?
A. Lofgren Syndrome
B. Heerfordt Waldenstrom syndrome

553
C. darrier-roussy syndrome
D. blau syndrome
E. mikulicz syndrome
►A

Lofgren syndrome makes you SMILE: Sarcoid, migratory polyarthritis, iritis, LAD (hilar), EN,
because you laugh and grin.

316
The histologic finding of "shoulder parakaratosis", parakeratosis with prediliection for the
follicular ostia, is characteristic of pityriasis rubra pilaris as well as:
A. Stasis dermatitis
B. Atopic dermatitis
C. Seborrheic dermatitis
D. Nummular dermatitis
E. Allergic contact dermatitis
►C

Parakeratosis refers to pyknotic keratinocyte nuclei in the stratum corneum, where nuclei are not
usually present. It is common in diseases with changes in the epidermis. Histologically
seborrheic dermatitis can shows "shoulder parakeratosis" with epidermal spongiosis.
Histologically atopic, nummular and contact dermatitis present with spongiosis with or without
vesicles. Stasis dermatitis presents with more dilated papillary dermal small blood vessels and
hemosiderin.

317
What is the diagnosis?
A. Erythema multiforme
B. Fixed drug eruption
C. Graft-versus-host disease
D. Lichen planus
E. Rowell's syndrome
►C

On histology, graft-versus-host disease often demonstrates subtle patchy vacuolar degeneration


of the basal layer and necrotic keratinocytes often adjacent to lymphocytes.

318
This lesion was excised from the face, what is this neoplasm?
A. Clear cell syringoma

554
B. Sebaceous carcinoma
C. Microcystic adenocarcinoma
D. Renal cell carcinoma
E. Clear cell hidradenoma
►A

Syringoma: Multiple eyelids, cheeks, chests, can be eruptive and increased numbers in Down's
syndrome and a clear cell change has been associated with Diabetes. Histologically this is a
dermal tumor consisting of eccrine ducts, lined by two cell layers sometimes creating a tad pole
appearance, there can also be small basaloid islands or strands of cells. Within the lumens of the
ducts is eosinophilic material, the lumens are CEA+. The tumor is located superficially and fails
to display perineural extension as is seen in MAC. The glands are associated with a fibrous
sclerotic stroma, usually no (or minimal) keratin cysts or foreign body granulomas as seen in
desmoplastic trichoepithelioma. Enlargement of the glandular cells more of a clear or vacuolated
cytoplasm can be seen in patients with diabetes.

319
A biopsy was obtained from the nasal mucosa, what are the organisms in this biopsy?
A. Rhinosporidium
B. Coccidioidomycosis
C. Prototheca wickerhamii
D. Histoplasmosis capsulatum
E. Cryptococcus neoformans
►A

Rhinosporidiosis originally thought to be due to a fungus Rhinosporidium seeberi, now believed


to be caused by an aquatic protistan parasite or Cyanobacteria, Microcystics aeruginosa. Sri
Lanka & South America polypoid nasal/mucosal lesions in males and conjunctival lesions in
females. Obtained from water or soil. Histology: granulomatous dermatitis with mixed
inflammatory cell infiltrate with large thick walled birefringent sporangia which mature toward
the center of a cyst, the organism measures 10-200 microns contain 7-8 micron endospores
which contain eosinophilic globules. Watery environments causes cyst to rupture.

320
Pustulo-ovoid bodies of Milian can be found associated with which of the following conditions?
A. Granular cell tumor
B. Leishmaniasis
C. Schwannoma
D. Chromomycosis
E. Rhinoscleroma

555
►A

Pustulo-ovoid bodies of Milian can be seen in association with granular cell tumors. They appear
as relatively large and eosinophilic intracytoplasmic inclusions with a surrounding clear halo.
These inclusions characteristically stain positive with PAS and are diastase resistant.
Leishmaniasis is associated with Donovan bodies, which are intracytoplasmic basophilic
organismal (amastigotes) inclusions found in parasitized histiocytes. Verocay bodies may be
seen in schwannomas, which appear as pallisaded nuclei in rows. Medlar bodies (―copper
pennies‖) can be seen in cutaneous infectious of chromomycosis and appear as grouped ovoid
spores measuring 6 to 12 micrometers both within histiocytes as well as interstitially. Russell
bodies are associated with rhinoscleroma and are elliptical basophilic inclusions found in plasma
cells and are comprised of immunoglobulins.

321
What is the diagnosis?
A. Sarcoidosis
B. Leprosy
C. Rosacea
D. Crohn's disease
E. Foreign body reaction
►B

Tuberculoid leprosy is characterized by epithelioid granulomas that are arranged in and around
nerves and arrector pili muscles.

322
This keratin is associated with hair and nail:
A. Keratin 6
B. Keratin 8
C. Keratin 10
D. Keratin 16
E. Keratin 17
►E

Keratins 6 and 16 are found in the nail. Keratin 17 is seen in the nail as well as in the hair
follicle, and patients with pachyonychia congenital type 2, in which there is a mutation of
keratins 6b and 17, have nail findings as well as steatocysts.

323
Which of the following have been found within a nevus sebaceus?

556
A. Squamous cell carcinoma
B. Merkel cell carcinoma
C. Syringocystadenoma Papilliferum
D. Verruca vulgaris
E. Hidradenoma Papilliferum
►C

Cutaneous leiomyosarcoma, trichoblastoma, syringocystadenoma papilliferum and basal cell


carcinomas have all been reported to be found within a nevus sebaceous.

324
What is the diagnosis?
A. Calciphylaxis
B. Polyarteritis nodosa
C. Leukocytoclastic vasculitis
D. Erythema nodosum
E. Cryoglobulinemia
►A

Calciphylaxis: seen in patients with elevated Ca-PO4 product and secondary


hyperparathyroidism most commonly patients in renal failure. Calcification occurs within the
media of vessels in the lower dermis resulting in intimal hyperplasia and secondary intraluminal
thrombi. Vascular occlusion then results in epidermal/dermal infarction.

325
The patient just returned from a sunny vacation with this eruption. What is your diagnosis
A. rosacea
B. acute cutaneous lupus
C. phototoxic eruption
D. Porphyria cutanea tarda
E. seborrheic dermatitis
►C

The epidermis is still a basket weave so this process happened acutely. There is no lymphocytic
infiltrate that would be expected in lupus. We are not on sebaceous skin as would be expected for
seborrheic dermatitis and rosacea. Porphyria cutanea tarta results in a subepidermal split with
festooning of the dermal papillae.

326
Dorf balls are seen in which tumor?

557
A. Kaposi's sarcoma
B. Angiosarcoma
C. Tufted angioma
D. Kaposiform hemangioendothelioma
E. Dermatofibroma sarcoma protuberans
►A

Dorf balls are pink amorphous globules seen in vessels in Kaposi's sarcoma. Typical histologic
findings include proliferation of spindle cells, prominent slitlike vascular spaces, and
extravasated red blood cells.

327
The diagnosis is:
A. Dermatofibrosarcoma
B. Nodular fasciitis
C. Angiolipoma
D. Epithelioid sarcoma
E. Liposarcoma
►C

Angiolipomas are tumors of fat that are characteristically painful. Histopathologically, mature
adipocytes are seen with numerous vessels. Microthrombi are often present.

328
What lesion is shown here?
A. Chondroid syringoma
B. Hyaluronic acid filler nodule
C. Glomus tumor
D. Myofibroma
E. Scleromyxedema
►C

Glomus tumors show aggregates of glomus cells surrounding numerous blood vessels within a
fibrous and sometimes myxoid stroma.

329
Which fixative would best preserve the histologic features of a gout tophus?
A. Saline
B. Absolute ethanol
C. Formaldehyde

558
D. Michel's fixative
E. Tissue culture media
►B

Gout tophi are caused by deposition of monosodium urate monohydrate crystals. Using absolute
ethanol to fix the biopsy specimen will preserve the crystalline architechture. In formaldehyde
fixed tissure, the crystals are less obvious.

330
What is the diagnosis of this neoplasm?
A. Fibroma of tendon sheath
B. Dermatofibroma
C. Dermatofibrosarcoma protuberans
D. Spindle cell lipoma
E. Leiomyoma
►A

Fibroma of tendon sheath is a solitary, slow-growing tumor that is usually located on the hands,
wrists or fingers of middle-aged individuals. Spindle or stellate cells are embedded in a dense
fibrocollagenous stroma. This lesion demonstrates characteristic dilated or slit-like vascular
channels.

331
What is the staining profile of this neoplasm?
A. CD34+, FXIIa-
B. CD68+, CD34-, CD10+ procollagen 1+, SMA+
C. CD34-, FXIIIa+
D. CK5+, INI-1-. CD68+
E. S100+
►B

Atypical fibroxanthoma are CD68+, CD34-, CD10+ procollagen 1+, SMA+. DFSP are CD34+
and FXIIIa-. Dermatofibromas are CD34- and FXIIIa+. Epithelioid sarcomas are CK5+, INI-1-.
CD68+.

332
A biopsy was performed from the scalp of an elderly man what is the most likely diagnosis?
A. Metastatic renal cell carcinoma
B. Angiosarcoma
C. Carcinoid

559
D. Sebaceous Carcinoma
E. Clear cell hidradenoma
►A

Clear Cell Renal Carcinoma: Metastatic lesions are commonly located on the scalp. The tumor
itself is composed of cells with clear to slightly granular cytoplasm secondary to increased
glycogen and lipid. The tumors typically forms abortive tubes/ducts, cords or sheets of cells.
Immunohistochemical stains are + for EMA and CD10. Typically the tumor is very vascular with
scant stroma associated with extravasated RBC and hemosiderin. The differential diagnosis
includes clear cell hidroadenoma. This latter tumor is usually composed of a mixture of
components; solid areas composed of small poroid cells often with duct formation admixed with
clear cells and squamoid cells. The tumor can be solid or cystic or a combination of the two. The
large cystic spaces typically contain sialomucin. The stroma is delicate fibrovascular. The tumor
is + CAM 5.2, CEA, EMA, with glycogen and no lipid in the clear cells.

333
A 35-year-old man develops indurated, mildly erythematous plaques involving the face and
upper back. Biopsy reveals a perivascular and periadnexal lymphocytic infiltrate with no
overlying epidermal changes. Which histopathologic stain could be used to support a diagnosis
of tumid lupus?
A. Fontana-Masson
B. Giemsa
C. von Kossa
D. Colloidal iron
E. Osmium tetroxide
►D

Tumid lupus is characterized by increased mucin, and thus a mucin stain such as colloidal iron
may be used to accentuate these findings. Other disorders that may present with a superficial and
deep perivascular lymphocytic infiltrate without associated increased dermal mucin include
polymorphous light eruption, pernio, syphilis, and reactive lymphocytic infiltrate of Jessner,
among others.

334
A 54-year old woman who is healthy except for chronic osteoarthritis, for which she takes daily
naproxen, with bullae and milia on the dorsal hands. Biopsy is most likely to exhibit which of the
following histologic findings:
A. Asteroid Bodies
B. Caterpillar Bodies
C. Cowdry Type A Bodies

560
D. Donovan Bodies
E. Dutcher Bodies
►B

Caterpillar bodies represent degenerated type IV collagen and are classically associated with
porphyria cutanea tarda. In this case, it is most likely that the patient is experiencing
pseudoporphyria, which is often caused by NSAIDs such as naproxen. The other histologic
bodies listed are not associated with porphyria.

335
The PDGF gene has been shown to be translocated into the collagen I gene in:
A. Angiofibroma
B. Dermatofibroma
C. Dermatofibrosarcoma protuberans
D. Fibrous hamartoma of infancy
E. Solitary fibrous tumor
►C

A translocation involving the PDGF gene and collagen I gene is found in dermatofibrosarcoma
protuberans.

336
The mutated product in Darier's disease is:
A. SPINK5
B. SERCA2
C. ATP2A2
D. ATP2C1
E. SPINK5 and ATP2A2
►B

In Darier's disease, the ATP2A2 gene encoding the SERCA2 Ca(2+)-ATPase is mutated in some
patients. SPINK5 is the gene that is mutated in some patients with Netherton's syndrome, and
this gene the serine protease inhibitor LEKTI. ATP2C1 is the gene mutated in some patients with
Hailey-Hailey disease.

337
What clinical finding would be seen in this patient?
A. Folliculocentric morbilliform eruption
B. Gottron's papules
C. Target lesions

561
D. Malar erythema
E. Wickham striae
►A

Acute cutaneous graft-versus-host disease is characterized by a morbilliform eruption of abrupt


onset, often in a folliculocentric pattern representing follicular damage.

338
On histology there is fibrosis around nerves and blood vessels with nerve degenerative changes
diagnosed as Morton's neuroma. It is a benign neural degenerative reaction seen on the:
A. Sole
B. Trunk
C. Fingers
D. Head
E. Neck
►A

In a Morton's neuroma it is found on the sole of the foot and is considered to be a benign neural
degenerative reaction. It occurs in adults more commonly than females. Excision is curative.

339
Blue-gray pigmentation on the legs secondary to minocycline on biopsy stains with:
A. Fontana Masson
B. Perls
C. Sudan black
D. Fontana Masson and Perls
E. All of these answers are correct
►D

There are three types of pigmentary change that are caused by minocycline. The blue-gray
pigmentation on the legs and the blue pigment in scars is thought to be secondary to a drug-
protein complex deposited in the dermis. The blue-gray pigment on the legs stains with Perls and
Fontana-Masson. The blue in scars (often on the face) stains with Perls. The muddy-brown
discoloration on sun-exposed areas shows increased basilar pigment and melanin incontinence
on biopsy. It is likely secondary to phototoxicity.

340
This disorder is characterized by what process?
A. Vascular damage caused by infiltration of neutrophils
B. Vascular fragility

562
C. Large vessel involvement
D. Delayed-type hypersensitivity reaction
E. The presence of amyloid
►A

Leukocytoclastic vasculitis is characterized by vascular damage caused by the infiltration of


neutrophils.

341
Which disease process best describes Texier's disease?
A. Neutrophilic dermatosis
B. Deposition disorder
C. Infectious process
D. Panniculitis
E. Granulomatous disease
►D

Texier's disease is a panniculitis secondary to vitamin K injections causing sclerotic lesions with
lilac borders on the buttocks and thighs resembling a cowboy belt and holster.

342
What is the diagnosis?
A. Lichen planus-like keratosis
B. Malignant melanoma
C. Syphilis
D. Spitz nevus
E. Halo nevus
►E

A halo nevus is characterized by a symmetric proliferation of nevus cells in the dermis,


surrounded by an intense lymphocytic infiltrate.

343
What is this neoplasm?
A. Mastocytoma
B. Poroma
C. Glomus tumor
D. Langerhans cell histocytosis
E. Nevus
►A

563
Approximately 10% cases of mastocytosis present as a solitary lesion, otherwise known as a
mastocytoma. Typically they present in childhood on trunk or wrist and usually are self limited
and spontaneously resolve. Histologically the epidermis overlying the mast cell proliferation is
often hyperpigmented. Within the dermis there are increased numbers of ―fried egg‖• mast
cells. Often the mast cells are primarily located in the upper dermis where they fill and expand
the dermal papillae. Frequently admixed among the mast cells there are increased numbers of
eosinophils.

344
On histology, there is a pseudocyst within the dermis surrounded by fibrous pseudocapsule with
no epithelial lining. The center is composed of bluish myxoid material with overlying
hyperkeratosis. This describes a:
A. Mucous cyst
B. Focal mucinosis
C. Dermoid cyst
D. Myxoid cyst
E. Mucinous carcinoma
►A

This describes a digial myxoid cyst or mucous cyst. It is located overlying osteoarthritis. It is
usually benign with potential for recurrence. Mucin stains with colloidal iron or alcian blue.

345
From which tissue does this neoplasm develop?
A. Adnexal
B. Connective
C. Neural
D. Smooth muscle
E. Epithelial
►C

Granular cell tumors are derived from neural tissue. Granularity develops from the accumulation
of lysosomes in the cytoplasm.

346
What type of reaction is demonstrated here?
A. Connective tissue
B. Psoriasiform
C. Lichenoid

564
D. Foreign body
E. Neoplastic
►D

A foreign body reaction develops in response to the deposition of foreign material or altered
endogenous material in the dermis. Granulomatous inflammation with histiocytes and
multinucleate giant cells forms around the material. Tattoo pigment is present in this particular
case.

347
This patient has a black toenail. What is the diagnosis?
A. Medication deposition
B. Tattoo
C. Ungual melanoma
D. Tinea unguium
E. Intraungual hematoma
►E

Intraungual hematoma can develop as a result of trauma and can appear black on clinical exam.
A histologic examination will show blood within the nail plate.

348
Cicatricial pemphigoid antibodies directed against this are associated with high frequency of
malignancy:
A. Laminin 5
B. Laminin 6
C. Beta4-integrin
D. BPAg2
E. All of these answers are correct
►A

Anti-laminin 5 cicatricial pemphigoid (CP) is also known as anti-epiligrin CP. Anti-epiligrin CP


is associated with an increased frequency of internal adenocarcinomas. Laminin 5 is composed
of three chains (heterotrimer), alpha3, beta3, gamma2. Antibodies are frequently directed against
the alpha3 chain, and so cross-reactivity can be observed with laminin 6, as laminin 6
(alpha3beta1gamma1) has the alpha3 chain as well. Beta4-integrin antibodies have been
associated with ocular CP. BPAg2 antibodies are seen in CP patients that have mucosal as well
as skin disease.

349

565
An aquarium owner presents with an erythematous, indurated nodule on the right hand that is
tender and warm. The patient has a history of drug induced lupus when he used minocycline for
acne. What is the most serious adverse effect associated with the first line alternative treatment?
A. Sudden cardiac death
B. Idiopathic pulmonary fibrosis
C. Systemic lupus erythematosus
D. Acute renal failure
E. Pulmonary embolism
►A

In a patient with exposure to marine environments, M. marinum is a common cause of an


infection that presents with an erythematous, indurated nodule on the extremities. First list
treatment is minocycline. Second line treatment is clarithromycin. A serious adverse effect of
clarithromycin is sudden cardiac death, as shown in the CLARICOR trial.

350
Cellular neurothekeoma stains with:
A. Stromelysin-3
B. Desmin
C. S-100
D. PGP-9.5
E. Low molecular weight keratin
►D

PGP-9.5 and S100-a6 stains cellular neurothekeoma. Stromelysin-3 is positive in


dermatofibromas and negative in dermatofibrosarcoma protuberans. Desmin stains
rhabdomyosarcoma. S-100 stains neural tumors and melanocytic tumors among other things, but
cellular neurothekeomas are generally S100-negative.

351
A question for a nerve sheath myxoma is what stain is positive in this tumor?
A. EMA
B. S100
C. PAS
D. CD68
E. SMA
►B

Nerve sheath myxoma or myxoid neurothekeoma is S100 positive and EMA negative. It is a
collection of stellate and pale spindle cells in a myxoid matrix.

566
352
Which syndrome is associated with this lesion?
A. Proteus
B. Sturge- Weber
C. Parkes Weber
D. Klippel-Trenaunay
E. Cutis marmorata telangiectatica congenita
►C

Parkes Weber syndrome is associated with arteriovenous malformations. All other listed
syndromes are associated with capillary malformations.

353
Multiple trichoepitheliomas are seen in all except:
A. Bazex
B. Brooke-Fordyce syndrome
C. Brooke-Spiegler syndrome
D. Gorlin's syndrome
E. Rombo syndrome
►D

Gorlin's syndrome is nevoid basal cell carcinoma syndrome; multiple trichoepitheliomas are not
seen. Several syndromes have been associated with multiple trichoepitheliomas: Bazex, Brooke-
Fordyce, Brooke-Spiegler, Rombo, and possibly Rasmussen. (Rasmussen described one family
in 1975 with autosomal dominant inheritance of multiple trichoepitheliomas, milia, and
cylindromas.) Bazex (follicular atrophoderma, hypotrichosis, occasional trichoepitheliomas,
basal cell carcinomas, and localized or generalized hypohidrosis) is inherited in an X-linked
dominant manner. Dont confuse with Bazex syndrome or acrokeratosis neoplastica. Brooke and
Fordyce both described multiple trichoepitheliomas concurrently in 1892, and therefore multiple
familial trichoepitheliomas are sometimes called ―Brooke-Fordyce‖ sydrome. Spiegler described
patients with multiple cylindromas in 1899 and also noted that many of these patients had
mutiple trichoepitheliomas; more recently it has been noted that multiple spiradenomas may be
seen in patients with multiple trichoepitheliomas and cylinidromas; this co-occurrence of tumors
has been referred to as ―Brooke-Spiegler‖ syndrome. (Brooke-Fordyce and Brooke-Spiegler are
likely the same syndrome.) Rombo syndrome is characterized by vermiculate atrophoderma,
multiple BCCs, multiple trichoepitheliomas, cyanosis and peripheral vasodilation.

567
Chapter -6-
Benign and Malignant Neoplasms
1
The keratoacanthoma variant characterized by the sudden appearance during childhood or
adolescence of multiple KAs is called:
A. Gorlin syndrome
B. Xeroderma pigmentosa
C. Ferguson-Smith
D. Grzybowski
E. None of these answers are correct
►C

The Ferguson-Smith is a keratoacanthoma variant characterized by the sudden appearance during


childhood or adolescence of multiple KAs that may resolve and later-on reappear. This condition
is inherited in an autosomal dominant pattern.

2
The treatment of choice for this lesion shown is:
A. Radiation therapy
B. Imiquimod
C. 5 Flourouracil
D. Wide excision with 2cm margins
E. Mohs surgery
►E

The treatment of choice for Dermaotfibrosarcoma protuberans is Mohs surgery. Radiation


therapy has been used, however has limited value as solitary therapy for thsi tumor. Radiation
therapy can be used as an adjunct to wide surgical excision. Classically, these tumors should be
excised with 3cm margins. The recurrence rate associated with these tumors can be 10-20
percent with wide excision with 3 cm margins. With Mohs surgery, the recurrence rate ranges
from 0% to 6%. 5FU and Imiquimod are not effective modalitites in treating DFSPs, as it
infiltrates deep into the subcutaneous tissue.

3
Mutations in which gene would likely be found in the neoplastic cells of this lesion?
A. PATCH
B. p53
C. Fumarate hydratase
D. CREBBP

568
E. p63
►B

Squamous cell carcinoma is the second most common cancer of the skin. Mutations in the tumor
suppressor p16 and p53 are commonly found in SCC's. Normally, UV damage upregulates p53
thereby delaying cell cycle progression. DNA damage can then be repaired or the cell could
undergo apoptosis. In squamous cell carcinoma, p53 exhibits loss of heterozygosity due to C to T
or CC to TT mutations.

4
A patient with a innumerable disseminated keratoacanthomas, including lesions on the larynx
and oral mucosa:
A. Is unlikely to have palmoplantar involvement
B. Likely has an underlying immune deficiency
C. Is at high risk for myelodysplasia
D. Likely inherited their condition in an autosomal dominant manner
E. Likely developed them during adulthood
►E

This patient has the Grzybowski type of keratoacanthomas. Typically diagnosed in adulthood,
these patients have the sudden appearance of hundreds of small lesions in a disseminated
fashion. The lesions can be found anywhere on the body including palms, soles, larynx, and oral
mucosa.

5
The patient is a 45 year old male complaining of red, chapped lower lip. Which of the following
lasers is the most appropriate to treat this condition?
A. Pulsed Dye Laser
B. Nd:YAG laser
C. CO2 laser
D. Diode laser
E. Laser treatment is not an option
►C

The patient has actinic cheilitis. Notice the red, scaly lower lip, with erosions and fissures. The
CO2 laser is currently a common treatment alternative for this condition.

6
Which of the following ethnic groups are commonly diagnosed with dermatosis papulosa nigra:
A. Asians

569
B. Hispanics
C. African-Americans & Hispanic patients
D. Caucasians
E. No difference between ethnic groups
►C

Characterized by the presence of multiple, small, hyperpigmented, sessile SKs on the face, DPN
is typically diagnosed on the African-American and Hispanic population.

7
Intermittent sun exposure with painful sunburns is a predisposing factor for the development of:
A. Atypical nevi
B. Seborrheic keratosis
C. Malignant melanoma
D. Atypical nevi and Malignant melanoma
E. All of these answers are correct
►D

It has been reported that the risk for the development of atypical nevi and melanoma is higher
than twofold with a history of five or more episodes of painful sunburn during adolescence.

8
The following lesion is the classic presentation of:
A. BCC
B. Merkel cell carcinoma
C. Melanoma
D. CTCL
E. Keratoacanthoma
►E

Keratoacanthomas present as a solitary, firm, dome-shaped papule with a cratiform center.

9
A patient having Mohs surgery for a squamous cell carcinoma on the ear has tumor invading the
cartilage and perineural invasion. What stage disease does the patient have?
A. T0
B. T1
C. T2
D. T3
E. T4

570
►C

TX Primary tumor cannot be assessed T0 No evidence of primary tumor Tis Carcinoma in situ
T1 Tumor ≤2 cm in greatest dimension with <2 high-risk features∗ T2 Tumor >2 cm in greatest
dimension with or without one additional high-risk feature, or tumor any size with ≥2 high-risk
features∗ T3 Tumor with invasion of maxilla, mandible, orbit, or temporal bone T4 Tumor with
invasion of skeleton (axial or appendicular) or perineural invasion of skull base. *High risk
features: Depth/invasion >2 mm thickness or Clark level ≥IV,Perineural invasion, Primary site
ear, Primary site hair-bearing lip, Poorly differentiated or undifferentiated.

10
What would you expect to see under dermoscopy of this vascular neoplasm?
A. Red sacculae
B. Arborizing blood vessels
C. Hair pin telangectasia
D. Milky red globules
E. Blue-grey ovoid nests
►A

Hemangioma have a characteristic appearance under dermoscopy. Typically, they have a maroon
lagoon or red sacculae appearance.

11
The most important mutated gene associated with a predisposition to develop malignant
melanoma is:
A. PTCH
B. CDKN2A
C. PTCH2
D. None of these answers are correct
E. All of these answers are correct
►B

The CDKN2A gene located on chromosome 9p21 is the most important mutated gene associated
with MM of the listed choices. BRAF is another important mutation.

12
A patient is diagnosed with squamous cell carcinoma. As a doctor you explain to the patient that
the metastatic rate is:
A. 0.3-16%
B. 15%-20%

571
C. 18%-25%
D. 25%-30%
E. 30%-45%
►A

Patients that have SCC have a 0.3-16% of chance of metastasis. The location, size and type of
SCC can determine the possibility of metastasis.

13
Which of the following is the most common initial site of metastasis from a primary BCC?
A. Lungs
B. Regional lymph nodes
C. Bone
D. Liver
E. Pleura
►B

The metastatic potential of BCC is very low with rates ranging from 0.0028 to 0.1%. The head
and neck region is the most frequent location of the primary tumor with regional lymph nodes
being the most common site of metastasis. The lungs, bone, liver, and pleural are also potential
sites of metastasis.

14
Chloroma is a characteristic cutaneous manifestation of:
A. Tuberous sclerosis
B. Sweet‖s syndrome
C. Neurofibromatosis
D. Leukemia
E. Pseudomonas sepsis
►D

Chloromas, also termed granulocytic sarcomas, are a localized tumor composed of immature
granulocytic cells. They frequently have a greenish coloration due to the presence of
myeloperoxidase and most commonly affect the bone. The condition most often occurs in
patients with acute leukemia of the myeloid type.

15
Which phase of the cell cycle does p53 regulate?
A. G1
B. G2

572
C. S phase
D. Mitosis
E. Meiosis
►A

p53 is a tumor suppressor gene which arrests cell cycle in G1 as it controls the transition from
G1 to S. It also downregulates BCL-2. Mutations in p53 are associated with Li-Fraumeni
syndrome as well as the development of squamous cell carcinomas.

16
A 65 year-old female with multiple actinic keratosis on the face under treatment with 5-FU.
According to the image and aforementioned information, you may conclude that:
A. The patient must immediately stop treatment since unexpected side effects have
developed
B. The patient has been compliant with 5-FU treatment and the appearance of
inflammation, erythema and erosions are expected
C. The image is not relevant to 5-FU treatment
D. None of these answers are correct
E. All of these answers are correct
►B

Compliance is a key feature in treatment with 5-FU. Erythema, inflammation and erosion must
develop and is considered a sign of successful treatment.

17
Which of the following would you not expect to see under dermoscopy?
A. Maple leaf pattern
B. Arborizing blood vessels
C. Blue-grey ovoid nests
D. Orange crust
E. Milky red globules
►E

Dermoscopy is a useful tool in differentiating a pigmented basal cell carcinoma from melanoma.
Basal cell carcinomas may have arborizing blood vessels, maple leaf pattern, blue-grey ovoid
nests, and orange crust or ulcer. Milky red globules are sometimes seen in melanoma.

18
Which of the following melanoma subtypes is more consistently seen in dark-skinned
individuals?

573
A. Acral lentiginous melanoma
B. Superficial spreading melanoma
C. Lentigo maligna melanoma
D. Nodular melanoma
E. Superficial spreading melanoma and Lentigo maligna melanoma
►A

Acral lentiginous melanoma is the predominant type of melanoma in dark-skinned individuals. It


is usually located on the soles, palms, and subungeal region of patients in their fifth to sixth
decade of life.

19
Which of the following is an immunhistochemical marker for Merkel Cell Carcinoma?:
A. S-100
B. Vimentin
C. HMB-45
D. Neuron specific enolase
E. All of these answers are correct
►D

Neuron specific enolase stains merkel cells. Vimentin stains melanocytic lesions, sarcomas and
lymphomas. S-100 and HMB-45 stains melanocytic lesions, such as melanoma.

20
Which one of the following malignancies is associated with HPV infection?
A. Verrucous carcinoma
B. Metastatic melanoma
C. Basal cell carcinoma
D. Sebaceous carcinoma
E. Atypical fibroxanthoma
►A

Verrucous carcinomas are low-grade carcinomas which are slow-growing and metastasize very
late in the course. The presence of HPV has been demonstrated in cases both by electron
microscopy and DNA hybridization.

21
Dermatofibrosarcoma protuberans is:
A. Cytogenetically characterized by reciprocal translocation t(17;22)(q22;q13)
B. Factor XIIIa positive

574
C. CEA positive
D. CD 34 positive
E. Cytogenetically characterized by reciprocal translocation t(17;22)(q22;q13) and
CD 34 positive
►E

A chromosomal reciprocal translocation t(17;22)(q22;q13), and supernumerary ring chromosome


have been reported as cytogenetic characteristics of DFSP. Typically DFSP is CD34 positive and
factor XIIIa negative, allowing its differentiation from dermatofibroma.

22
Amplification of which of the following genes is associated with Merkel cell carcinoma?
A. L-Myc
B. C-Myc
C. GLI1
D. CDKN2A
E. PTEN
►A

The L-Myc gene has been found to be amplified in Merkel cell carcinoma but not in normal skin.
C-Myc has been found to be amplified in neuroblastoma. GLI1 is a transcription factor involved
in hedgehog signaling and a potential target in basal cell carcinomas. CDKN2A is implicated in
familial forms of melanoma that are associated with pancreatic cancer. PTEN is a tumor
suppressor gene that is mutated in Cowden Disease, Bannayan-Riley-Ruvalcaba Syndrome, and
Proteus Syndrome.

23
The most common location for a basal cell carcinoma is:
A. Lower eyelid
B. Forehead
C. Ears
D. Back
E. Shoulders
►A

The most common location for a BCC is the lower eyelid. It is the most common epithelial tumor
of the eyelid.

24
If left untreated, which of the following is not at risk for malignant transformation?

575
A. Bowenoid papulosis
B. Cutaneous horn
C. Actinic cheilitis
D. Leukoplakia
E. Stucco keratosis
►E

Cutaneous horn can overlie an AK or SCC or a benign lesion and it presents as a conical
protuberance arising from an erythematous base. Actinic cheilitis results from the confluence of
multiple AKs on the lips. Leukoplakia is a clinical diagnosis and is defined as a white patch in
the oral cavity. It is the most common premalignant condition of the oral cavity. Bowenoid
papulosis manifests clinially as multiple red-brown warty papules that histologically represent
high grade squamous intraepithelial lesions.

25
Currently, the surgical margin for melanomas that measure less than 2 mm in thickness is:
A. 1 mm
B. 0.5 cm
C. 1 cm
D. 2 cms
E. 3 cms
►C

The current surgical margins are 0.5 cms for melanoma in situ, 1 cm for melanomas that measure
less than 2mm in thickness and 2cm for melanomas >2mm.

26
Denileukin diftitox (ONTAK®):
A. is a systemic treatment option for CTCL
B. is a diphtheria fusion toxin
C. targets the interleukin-2 receptor
D. None of these answers are correct
E. All of these answers are correct
►E

Denileukin diftitox is as diphtheria fusion toxin that targets the IL-2 receptor. It is a systemic
treatment alternative for recalcitrant or advance CTCL.

27
Maple leaf-like structures seen on dermoscopy are characteristic of which lesion?

576
A. Seborrheic keratoses
B. Pigmented basal cell carcinoma
C. Dermal nevi
D. Melanoma
E. Hemangioma
►B

Maple leaf-like structures seen on dermoscopy are an important diagnostic criterion for
pigmented basal cell carcinomas. They correspond to the heavily pigmented cells in the basaloid
cells.

28
What is the diagnosis?
A. Condyloma acuminata
B. Molloscum contagiosum
C. Pearly penile papules
D. Lichen planus
E. Psoriasis
►C

Pearly penile papules are benign, filiform papules that form along the corona of the penis. THey
are often mistaken for a form of veneral disease. Histologically, they are similar to
angiofibromas and show ectatic vessels surrounded by dense connective tissue.

29
Which one of the following agents has demonstrated potential benefit as a chemopreventive to
UV-induced skin cancer?
A. Prostaglandin E2
B. Vitamin D
C. Arachidonic acid
D. Celecoxib
E. Vitamin E
►D

Cyclooxygenase-1 and -2 and enzymes that catalyze the conversion of arachidonic acid to
prostaglandins. It is believed the prostaglandin E2 (PGE2), whose levels are increased by
ultraviolet irradiation, is pro-inflammatory and may contribute to skin carcinogenesis. In a study
by Orengo et.al., hairless mice who were given celecoxib were found to have a significantly
longer latency period between exposure to ultraviolet light and the development of skin
carcinomas.

577
30
UVB induced mutations on the PTCH gene is associated with the development of:
A. BCC
B. Merkel cell carcinoma
C. Angiosarcoma
D. BCC and Merkel cell carcinoma
E. Merkel cell carcinoma and Angiosarcoma
►A

The p53 and PTCH genes are the major targets of UVB for the development of BCC. Other
genes involved include the Smoothened-activating mutations, and PTCH2 mutations.

31
Periungual Squamous cell carcinoma has been linked to which HPV type(s)?
A. 6, 11
B. 2, 4
C. 16
D. 13
E. 8
►C

Infections associated with Human Papilloma Virus can produce growths on the epithelial or
mucosal surfaces. There are over 100 strains of these viruses, and some of these strains can
predispose to intraepithelial carcinomas, particularly when involving the anal or genital mucosa.
In general, HPV strains 16 and 18 are classified as his risk virus types and can be associated with
cervical cancer , oral cancer, anal cancer and periungual cancers. HPV 6 and 11 are associated
with condyloma acuminata; HPV types 2, 4 are associated with common warts; HPV type 13 has
been associated with Heck's disease and HPV type 8 has been associated with epidermal
dysplasia verruciformis.

32
Which of the following features of thin melanomas (<1 mm thick) has not been associated with
an increased risk of metastasis?
A. Regression
B. Location
C. Age
D. Gender
E. P53 expression
►E

578
Regression in malignant melanoma appears histologically as a focal area of fibrosis with
lymphocytes and melanophages in the papillary dermis. The presence of regression precludes
accurate measurement of true thickness of the melanoma. Extensive regression in thin
melanomas has a significant association for the risk of metastases.

33
What is the most common site of metastasis for this dermal tumor?
A. Lung
B. Brain
C. Kidney
D. Liver
E. Bone
►A

Dermatofibrosarcoma protuberans is a rare, low-grade dermal sarcoma. Typically, the lesion


occurs as a painless subcutaneous mass that grows slowly. This malignancy typically has lateral
spread but invade deep. Metastasis is rare but has been reported to the lung.

34
Which substance does p53 normally activate to promote apoptosis via inhibition of bcl-2?
A. p21
B. p16
C. Puma
D. Mdm2
E. Akt
►C

p53 is the most commonly mutated tumor suppressor gene involved in human cancer and is often
mutated in SCC and BCC. p53 acts via two main pathways, 1) activation of p21 (Cdk inhibitor)
which leads to cell cycle arrest, and 2) activation of Puma which inhibits Bcl-2 (apoptosis
inhibitor) thereby leading to cell death. CKDN2A, a gene that when mutated leads to a risk of
melanoma, acts via 1) activation of p16 (another Cdk inhibitor) and 2) activation of p14ARF
which inhibits Mdm2 (which normally degrades p53). Akt is involved in the PI3K-Akt signaling
pathway and inhibits cell cycle arrest and apoptosis.

35
The most common location for primary mucinous carcinoma is:
A. neck
B. eyelid

579
C. areola
D. scrotum
E. nose
►B

Mucinous carcinoma presents as a slowly growing, asymptomatic, round, erythematous nodule


on the head and neck. Forty percent of cases occur on the eyelid. Histologically it is
characterized by the presence of large areas of mucin (―sea of mucous‖) with small islands of
basophilic epithelial cells. Primary mucinous carcinoma of the skin has an indolent course. Local
recurrence occurs in 1/3 of patients following excision. The rate of metastasis is low (9.6%).

36
All of the following disorders may manifest as exfoliative dermatitis except
A. Behcet's disease
B. Psoriasis
C. Pemphigus foliaceus
D. Drug reaction
E. Sezary syndrome
►A

The image shows erythrodermic patient with generalized desquamation of skin. Skin biopsy and
blood tests the diagnosis of Sezary syndrome. Sezary syndrome is a leukemic variant of mycosis
fungoides, is characterized by the triad of pruritic erythroderma, generalized lymphadenopathy,
and the presence of Sezary cells more than 1,000 cell/mm3 (abnormal, large hyperconvoluted
lymphocytes) in peripheral blood . Other manifestations include scaling and fissuring of palms
and soles, alopecia, pruritus, peripheral edema, and nail dystrophy. Exfoliative dermatits can be
manifested in many conditions other than Sezary syndrome inculude pemphigus foliaceus,
psoriasis and drug reaction.

37
The human papilloma virus type associated with red brown smooth and warty papules is
A. HPV 1
B. HPV 5
C. HPV 7
D. HPV 13
E. HPV 16
►E

Bowenoid papulosis manifests clinically as multiple red-brown warty papules or confluent


planques on the external genitalia. These lesions may resemble genital warts but histologically

580
represent high-grade squamous intraepithelial lesions. Bowenoid papulosis is caused by infection
by HPV and linked to HPV 16, 18, 31, 35, and 39.

38
A patient has a malignant melanoma 1.6mm thick with ulceration and a micrometastasis in 1
node. The patient's staging according to the American Joint Committee on Cancer Staging
System is:
A. IIC
B. IIIA
C. IIIB
D. IIIC
E. IV
►C

An ulcerated tumor of any size with micrometastasis in 1 node is T1-4b N1a M0. This
corresponds to stage IIIB.

39
The type #1 site of metastasis for dermatofibrosarcoma protuberans is:
A. Lungs
B. Kidney
C. Stomach
D. Bladder
E. Colon
►A

The #1 site of metastasis for DFSP are lungs. It has a low grade of metastasis <5% and treatment
is wide local excision with 3 cm margins.

40
Immunohistochemistry of dermatofibrosarcoma protuberans typically reveals:
A. CD34 negative and factor XIIIa positive
B. CD34 negative and factor XIIIa negative
C. CD34 positive and factor XIIIa positive
D. CD34 positive and factor XIIIa negative
E. None of these answers are correct
►D

Typically DFSP is CD34 positive and factor XIIIa negative, allowing its differentiation from
dermatofibroma.

581
41
All of the following statements are true regarding angiosarcomas EXCEPT:
A. They occur more commonly in Caucasians than in non-Caucasians
B. Men are more often affected than women
C. They are rarely symptomatic
D. Radiation is usually employed after surgical excision
E. Cervical lymph nodes are a common site of metastases
►C

Angiosarcomas are very rare, aggressive vascular tumors. They occur most commonly in the
head and neck region of white, elderly individuals. Men are more commonly diagnosed with this
neoplasm. The lesion initially arises as a painless, purple macule or plaque on the scalp or face.
Later on it becomes an elevated bluish or purple nodule that may ulcerate. Common symptoms
include bleeding, edema, and ultimately pain. Cervical lymph node and hematogenous
metastases commonly occur. Wide surgical excision is the treatment of choice, with radiation
therapy usually employed after surgical excision.

42
Which of the following are true for the sign of Leser-Trelat?
A. Lesions are commonly located on the chest and back
B. Classically described to appear in a ―Christmas tree‖ pattern
C. Usually associated with internal malignancies
D. Eruptive nature
E. All of these answers are correct
►E

Characterized by the sudden appearance of multiple SKs, the sign of Leser-Trelat may have
lesions located anywhere in the body. However, the ―eruption‖ is classically described to appear
in a ―Christmas tree‖ pattern commonly located on the chest and back. It may be associated with
internal malignancies such as adenocarcinomas.

43
The risk of metastasis from SCC increases with:
A. Tumor size
B. Depth of invasion
C. Degree of differentiation
D. Immunosupression
E. All of these answers are correct
►E

582
Tumor size (greater than 2 cms), location (lips, ears, eyelids), depth of invasion, degree of
differentiation, perineural invasion, immunosupression, as well as recurrent tumors and those
that arise in areas of chronic inflammation are all factors that increase the risk of metastasis.

44
All of the following statements regarding Bowen's disease are true EXCEPT:
A. Lesions arising on the lower limbs are more common in men than in women
B. The basement membrane remains intact on histopathology
C. 5% of patients with Bowen's disease develop invasive squamous cell carcinoma
D. Chronic sun exposure is a risk factor for Bowen's disease
E. Mucosal Bowen's disease may appear as a verrucous plaque
►A

The most common locations for BD include the head and neck regions and the extremities. BD
arising on the lower limbs is frequently found in women, whereas lesions located on the ears and
scalp are more common in men. The basement membrane remains intact on histopathology. 5%
of patients develop invasive SCC. Risk factors include chronic sun exposure,
immunosuppression, HPV, arsenic exposure, and ionizing radiation. Mucosal BD may appear as
a verrucous or polypoid plaque, as an erythroplakic patch, or as a velvety red plaque.

45
The incidence of cutaneous squamous cell carcinoma in organ transplant recipients is increased
by how much compared with the general population?
A. 2 fold
B. 5 fold
C. 10 fold
D. 20 fold
E. 65 fold
►E

The risk of skin cancer in organ transplant patients is dramatically increase and may be more
aggressive. In one study, SCC‖s had a 65x increased incidence, BCC 10x, and melanoma 3.4x.

46
What is the most common location for pagetoid reticulosis?
A. Trunk
B. Head and neck
C. Hands and feet
D. Flexural sites of upper and lower extremities

583
E. Genitals
►C

Pagetoid reticulosis is an indolent cutaneous T-cell lymphoma. Pagetoid reticulosis favors an


acral distribution and typically presents as scaly oval plaques. Epidermotropism is present, with
tumor cells being CD4 positive or CD8 positive. Another interesting immunohistochemical
finding is the absence of CD45 expression.

47
A patient presents with tender papules with a pseudo-Darier's sign. She has other family
members with the same condition. Screening should be performed to rule out which
malignancy?
A. Gastric carcinoma
B. Ovarian carcinoma
C. Renal cancer
D. Testicular cancer
E. Lung cancer
►C

Reed's syndrome is an autosomal dominant disease with incomplete penetrance characterized by


uterine and cutaneous leiomyomas. Clinically, leiomyomas appear as flesh colored or pinkish-
brown dermal papules or nodules that range from 0.2 to 2.0 cm in diameter. Their presentation
may otherwise be quite variable. They may be isolated or many in number, may be variably
distributed or dermatomal, and may be asymptomatic or painful in response to pressure or cold.
The predisposition gene for Reed‖s syndrome has been localized to chromosome 1q42.3-43 and
the gene encoding fumarate hydratase. Currently, the United States National cancer Institute is
recommending screening for all patients with leiomyomatosis to evaluate for an occult renal
malignancy.

48
All of the following cytokines have demonstrated therapeutic benefit in the treatment of
melanoma EXCEPT:
A. IFN-alpha
B. IL-2
C. TNF-alpha
D. IL-10
E. GM-CSF
►D

584
IFN-alpha, IL-2, TNF-alpha, have all been demonstrated to have some therapeutic benefit in the
treatment of melanoma. GM-CSF is the immune stimulator in the vaccine Talimogene
laherparepvec. IL-10 has been used to treat inflammatory disorders such as atopic dermatitis and
psoriasis.

49
A 60 year-old female presents with a well-demarcated, scaly, erythematous plaque on her right
shin. The biopsy shows full thickness epidermal atypia with scattered mitotic figures and
overlying parakeratosis. Howerver, the basement membrane remains intact. According to the
aforementioned information, the diagnosis is:
A. BCC
B. SCC
C. Bowen's disease
D. Angiosarcoma
E. CTCL
►C

Bowen‖s disease arising on the lower limbs is frequently found in women; whereas lesions
located on the ears and scalp are more common in men. The epidermal dysplasia does not
interrupt the basement membrane confirming the diagnosis of SCC in situ.

50
What would be the characteristic histopathologic findings of this lesion?
A. Full thickness keratinocytic atypia
B. Cornoid lamella
C. Pale staining cells
D. Horn pseudocysts
E. Wedge shaped granular layer with lichenoid infiltrate
►B

Five clinical variants of porokeratosis are recognized. They are the classic porokeratosis of
Mibelli, disseminated superficial actinic porokeratosis, prokeratosis palmaris et plantaris
disseminata, linear porokeratosis, and punctate porokeratosis. On histopathology they have
varying degrees of a cornoid lamella. This appears as a column of parakeratosis.

51
Topical 5-Fluorouracil:
A. Interferes with the synthesis of DNA and RNA
B. Is an alternative for the treatment of actinic keratosis
C. May cause pruritus and burning at the site of application

585
D. All of these answers are correct
E. None of these answers are correct
►D

Topical 5-FU blocks the methylation reaction of deoxyuridylic acid to thymidylic acid, thus
interfering with the synthesis of DNA and RNA. Normal side effects during treatment include
pruritus and burning at the site of application. It is a treatment option for patients with multiple
AKs within an area.

52
Knowing that the patient in this picture has a biopsy confirmed BCC, which of the following
would be the treatment of choice:
A. Conventional excision
B. Imiquimod
C. Radiation
D. Mohs micrographic surgery
E. Cryosurgery
►D

Providing the highest cure rates, Mohs micrographic surgery is the treatment of choice for large
BCCs (1cm on the face or 2 cms on the trunk), located on high-risk anatomic areas (ear, eyelids,
lips, genitals, nose, temples). , Morpheaform BCCs, tumors with positive margins after
conventional excision, and recurrent BCCs, are also an indication for Mohs surgery.

53
Mutations of the p53 gene has been associated with the development of:
A. Mmelanoma
B. Kaposi's sarcoma
C. Actinic keratosis
D. Molluscum contagiosum
E. All of these answers are correct
►C

UVB radiation triggers the formation of thymidine dimers both in DNA and RNA, resulting in
mutated keratinocytes. The mutations occur on the tumor suppressor gene p53 within the
keratinocytes resulting in impairment of the mechanism of apoptosis. Therefore, clonal
expansion of mutated keratinocytes may occur leading to the formation of AKs.

54

586
Spitz nevi can appear during childhood. The risk factors for metastatic Spitz include all of the
following except:
A. Ulceration
B. Increased Breslow thickness
C. Increased mitoses
D. More H-RAS mutations
E. Location on the back
►E

Features of atypical spitz include location in the back over one cm and can have irregular color
and border. The risk factors for metastatic Spitz is ulceration, increased Breslow thickness,
atypical mitotic features and more H-RAS mutations as opposed to BRAF and NRAS.

55
Histologically, this lesion is shows plump, polygonal cells arranged in nests and fascicles with
granular cytoplasm. Which immunohistochemical stain would be positive?
A. Colloidal iron
B. Von kossa
C. Warthin-starry
D. S-100
E. CD68
►D

Granular cell tumors are benign growths which typically occur on the tongue. They are typically
well-circumscribed, raised, firm nodules. Histologically, the cells have uniform nuclear
characteristics and granular cytoplasm due to presence of lysozyme. The lesions are PAS
positive and S-100 positive.

56
The most common site for intra oral melanoma is?
A. buccal
B. soft palate
C. hard palate
D. gingiva
E. uvula
►C

Multiple case series have shown that the hard palate (and specifically the anterior hard
palate/alveolar arch) is the highest risk location for intra-oral melanoma. Prognosis is generally

587
worse for oral melanoma, which is usually due to delay in diagnosis and presentation with more
invasive disease.

57
Which of the following variants of mycosis fungoides is best diagnosed using a punch biopsy
instead of a broad superficial shave biopsy?
A. Woringer-Kolopp pagetoid reticulosis
B. Syringotropic mycosis fungoides
C. Ketron-Goodman pagetoid reticulosis
D. Poikilodermatous mycosis fungoides
E. Sezary syndrome
►B

Syringotropic and folliculotropic mycosis fungoides (MF) are the variants of MF that should be
diagnosed by punch biopsy. Both variants of pagetoid reticulosis, Woringer-Kolopp disease and
the disseminated Ketron-Goodman disease, are best diagnosed with a broad shave biopsy.
Classic and poikilodermatous MF should also be diagnosed with a shave biopsy. To rule out
Sezary syndrome, flow cytometry should be performed.

58
Sentinel lymph node biopsy in malignant melanoma:
A. Has gained acceptance for the treatment of MM of intermediate thickness (1-
4mm)
B. Is mainly being used for a more accurate staging
C. Is not recommended
D. None of these answers are correct
E. Has gained acceptance for the work up of MM of intermediate thickness (1-
4mm) and is mainly being used for a more accurate staging
►E

Sentinel lymph node biopsy has gained acceptance as part of the work up for melanomas 1-4 mm
in thickness. Even though it has been widely accepted due to its low morbidity and high
feasibility, the role of SLN biopsy in survival has not been established. It is mainly being used
for a more accurate staging, prognosis and to determine if further adjuvant therapy is necessary.

59
Which of the following immunohistochemistry marker is negative in angiosarcomas?
A. CD31
B. CEA
C. Cytokeratin

588
D. CD34
E. Factor VIII
►B

Angiosarcomas are CD31, CD34, factor VIII, and cytokeratin positive. The carcinoembryonic
antigen (CEA) stains positive in Paget‘s disease, metastatic adenocarcinoma, and tumors with
eccrine differentiation.

60
Which of the following melanoma scenarios have the best prognosis?
A. Twenty-one year old female with primary lesion located on the right lower leg
B. Twenty-one year old male with primary lesion located on the chest
C. Thirty-six year old male with primary lesion located on the back
D. Twenty-one year old female with primary, ulcerated lesion located on the right
lower leg
E. Thirty-six year old male with primary lesion located on his left lower leg with
palpable inguinal lymph nodes
►A

Increasing age and male gender have a negative effect on survival. Primary lesions located on the
extremities have a better prognosis than those located on the trunk, head or neck region.
Ulceration is also considered a poor prognostic factor. Furthermore, there is a significantly lower
survival for those patients with palpable metastatic nodes (macrometastasis) when compared to
those with micrometastatic nodes (nonpalpable).

61
What is the most common location of metastasis of basal carcinoma?
A. liver
B. lung
C. bone
D. lymph node
E. skin
►D

Size of a BCC is related to its risk for metastasis. 3cm BCC has a 1.9% risk of metastasis. If a
BCC is >10cm it has approximately a 50% risk of metastasis. If metastasis does occur it most
commonly metastasizes to lymph nodes>lung>bone>skin>liver.

62

589
An elderly white man presents with a slowly enlarging, well-demarcated pink, scaly plaque on
the neck. Biopsy of the lesion reveals epidermal dysplasia and keratinocytic disorganization with
preservation of the basement membrane. Hyperkeratosis and parakeratosis are also present, and
numerous atypical keratinocytes are seen throughout the epidermis, with loss of polarity, atypia,
and mitoses. A chronic inflammatory infiltrate is present in the upper dermis. Which of the
following treatment(s) should be considered for this patient?
A. Conventional excision
B. Mohs micrographic surgery
C. Imiquimod
D. Conventional excision and Mohs micrographic surgery
E. All of these answers are correct
►D

The patient has Bowen's Disease, or squamous cell carcinoma in situ. Multiple treatment options
are available, including conventional excision, Mohs micrographic surgery, imiquimod,
cryosurgery, and 5-FU, among others. Conventional excision is the most commonly used
method. With this method, not only is the lesion removed, but it also provides the specimen for
histologic verification to rule out invasive SCC. Recurrence rates of 5% have been reported with
conventional excision. Mohs micrographic surgery is an option when the lesion is located in
areas where there is an increased risk of sub-clinical spreading, or when tissue sparing is a
priority. Imiquimod is not approved to treat SCCIS.

63
Compared with the general population, what is the overall risk of developing cutaneous and
systemic malignancies in organ transplant recipient?
A. 4 fold
B. 10 fold
C. 25 fold
D. 50 fold
E. 100 fold
►A

Organ transplant recipients are at increased risk of having both systemic and cutaneous develop.
The estimated increased risk is 3-4 times that of the general population. However, the risk of skin
cancer alone is much higher (e.g. SCC ― 65x increase, BCC 10-fold increase, Melanoma 3.4x
increase). This increased risk is thought to be partially due to the immunosuppressant agents
used to prevent graft rejection.

64
When performing a biopsy of a suspected keratoacanthoma:

590
A. Fusiform incision through the entire KA may be performed
B. It is necessary to biopsy down to subcutaneous fat
C. A full-thickness shave biopsy is acceptable
D. A complete excisional biopsy may be performed
E. All of these answers are correct
►E

When considering a keratoacanthoma, it is important to obtain a biopsy of the specimen down to


the subcutaneous fat. This can be achieved either by complete excisional biopsy, full-thickness
shave biopsy, or fusiform incision through the entire KA including its center and sides.

65
Merkel cell carcinoma should be treated with what size surgical margins?
A. 2mm
B. 5mm
C. 1cm
D. 3cm
E. 5cm
►D

Merkel cell carcinoma is an aggressive rare tumor of the skin accounting for less than 1% of
cutaneous malignancies. Also known as neuroendocrine cancer of the skin, this tumor presents as
a painless red to violaceous, firm, solitary, nodule that usually presents on sun exposed areas
such as the head, neck and upper extremities. These tumors present usually during the 6th and
7th decades, and have a 2 year survival rate of 50-70%. Because of this tumor's high potential for
regional and distal metastasis, this tumor should be excised with wide local excision with 3cm
surgical margins.

66
Which of the following is a clinical stage of keratoacanthomas?
A. Plaque
B. Mature
C. Proliferative
D. Resolving
E. All of these answers are correct except plaque
►E

Keratoacanthomas have three, consecutive, clinical stages: proliferative, mature and resolving.
The proliferative stage is characterized by the appearance of a rapid growing papule. This phase
is followed by the mature stage when the lesion acquires its characterisitic dome-shaped

591
appearance with a central, keratinous core. Tumor resorption occurs during the involution stage
resulting in a slightly depressed, hypopigmented scar.

67
All of the following are risk factors for metastasis from a primary squamous cell carcinoma
EXCEPT:
A. Increasing tumor size
B. Location on the ear
C. Recurrent tumor
D. Tumor within ulcer
E. All of these answers are correct
►E

The risk of metastasis from a primary SCC increases with tumor size (>2cm), location (lips, ear,
and eyelid, among others), depth of invasion, immunosuppression, degree of differentiation, and
perineural invasion. Recurrent tumors and SCCs arising in areas of chronic inflammation (such
as osteomyelitis, burn scars, or ulcers) are also considered high risk for metastases.

68
Which of the following lesions are ‖better felt than seen‖:
A. Seborrheic keratosis
B. SCC in situ
C. Actinic keratosis
D. Sebaceous hyperplasia
E. All of these answers are correct
►C

Typical AKs appear as flat, erythematous, rough papules that are better felt than seen.

69
What is the eponym for this metastasis to the umbilicus?
A. Tripe palm
B. Sign of Leser-Trelat
C. Pityriasis rotunda
D. Trousseau syndrome
E. Sister Mary Joseph Nodule
►E

Sister Mary Joseph nodule is a cutaneous metastasis that is most commonly associated with
stomach, large bowel, ovary, and pancreatic cancer. Tripe palms are most commonly associated

592
with lung, Leser-Trelat with adenocarcinomas of the stomach, colon and breast, Trousseau Sign
with carcinoma of the pancreas and pityriasis rotunda with hepatocellular carcinoma and gastric
cancer.

70
On histologic examination of the adenocarcinoma of the perineal area, which of the following
stains would NOT be positive?
A. PAS
B. Mucicarmine
C. CEA
D. EMA
E. HMB45
►E

Extramammary Paget's cutaneous adenocarcinoma of glandular differentiation. Approximately


25% of these tumors are associated underlying neoplasms. Histologically, the Paget cells stain
positively with PAS, mucicarmine, CEA, EMA, LMW keratin. They are HMB45 negative which
is a melanocytic marker.

71
The cure rate of cryotherapy as a treatment for actinic keratoses is:
A. 80%
B. 85%
C. 90%
D. 95%
E. 99%
►E

Cryotherapy is the most common treatment for AKs, with a cure rate of 98.8%.

72
Which of the following is true regarding poikilodermatous mycosis fungoides?
A. Majority of cases are predominantly CD8(+)
B. Patients typically have a later age of onset compared to classic mycosis
fungiodes
C. Can be associated with LyP
D. More women than men affected
E. Patients typically do not respond well to phototherapy
►C

593
While there is an overrepresentation of CD8+ cases compared to classic mycosis fungoides,
more cases are still predominantly CD4+. Patients typically present at a younger age (median age
44 years), with a slight male predominance. There is an excellent response to phototherapy with
clinical improvement in ~90% of patients treated. There is an increased association with LyP
compared to other types of mycosis fungoides. (Abbott et al, JAAD 2011; in press)

73
The most common locations of microcystic adnexal carcinoma include all of the following,
except:
A. Perioral
B. Nasolabial
C. Trunk
D. Periorbital
E. Perioral,nasolabial, and periorbital
►C

Microcystic adnexal carcinomas are more commonly located in the perioral, nasolabial, or
periorbital areas.

74
Merkel cell carcinoma can be summarizes with certain clinical features. All of the following are
features except:
A. Asymptomatic
B. Expanding rapidly
C. Older than 50
D. Older than 60
E. Ultraviolet-exposed area in person with fair skin
►D

Merkel cell carcinoma can be summarized with "AEIOU" that stands for expanding rapidly,
immune suppression, older than 50 years, ultraviolet-exposed area in person with fair skin.

75
In which of the following ethnic groups is squamous cell carcinoma the most common type of
skin cancer?
A. Asian Indians
B. Caucasians
C. Hispanics
D. Japanese
E. Chinese

594
►A

SCC is the most common type of skin cancer in Asian Indians and Blacks. BCC is the most
common type of skin cancer in Caucasians, Japanese, Chinese, and Hispanic people.

76
All of the following are true of melanoma-associated leukoderma except:
A. Lesions resemble vitiligo
B. Portends a worse prognosis
C. Seen in patients with metastatic disease but no primary lesion
D. Histology resembles that of a halo nevus
E. Lesions develop distant to melanoma
►B

Hypomelanosis associated with melanoma most commonly may be seen in three ways. The first
type is analogous to a halo nevus. The second is a remote leukoderma distant from the primary
lesion. The third is a vitiligo like leukoderma. It may portend a better prognosis in comparison to
others with the same stage of disease.

77
Which of the following is commony seen in seborrheic keratoses when examined with
dermoscopy?
A. Maroon lagoons
B. Network
C. Pigment globules
D. Milia like cysts
E. Blue-gray veil
►D

The main dermoscopic features of seborrheic keratoses include comedolike openings, fissures,
milialike cysts, fingerprinting and lack of true pigment network. In fact, it is the lack of true
pigment network, branched streaks, and pigment globules that differentiate SKs from
melanocytic lesions.

78
A 60 y/o male smoker with multiple sclerosis presents for evaluation of a painful 5cm x 4cm
plaque on right posterior shoulder, which appeared over 3 months. What is the most likely
diagnosis?
A. Irritated Seborrheic Keratosis
B. Squamous Cell Carcinoma

595
C. Basal Cell Carcinoma
D. Metastatic Carcinoma
E. Malignant Melanoma
►C

Basal cell carcinoma. It has a male to female ratio 2:1. UVB damages DNA and produces a C-T
transition mutation (65% of BCC). P53 and PTCH genes are commonly affected. It has a slow,
indolent growth potential: metastatic potential 0.0028% to 0.1%.; known to metastasize to lymph
nodes and lungs.

79
An elderly man presents to your office with a telangiectatic, violaceous 1cm dome-shaped
nodule on the neck. Biopsy reveals large, solid nests of cells of intermediate size, with a
trabecular pattern at the periphery. These cells involve the dermis and spread into the
subcutaneous fat, but spare the overlying epidermis. The cells are round and uniform in size,
with a round to oval nucleus, small nucleoli, and evenly dispersed chromatin. Numerous mitotic
figures and necrotic areas are seen. Neuron specific enolase is positive. Which of the following is
true regarding this patient's diagnosis?
A. Mohs micrographic surgery is contraindicated in treatment of this lesion
B. Vimentin and desmin stains may be positive
C. S-100 stains should be positive
D. This lesion may contain ACTH
E. The neoplasm should not contain gastrin
►D

This patient has a Merkel cell carcinoma. Vimentin, desmin, and S-100 are consistently absent in
MCC. Mohs micrographic surgery has been used successfully for the treatment of MCC, with or
without adjuvant therapy. This neoplasm may sometimes contain several neuropeptides
including vasoactive intestinal peptide, calcitonin, ACTH, gastrin, and somatostatin.

80
The translocation for dermatofibrosarcoma protuberans is:
A. Chr 17 and 22
B. Chr 7 and 10
C. Chr 9 and 13
D. Chr 20 and 25
E. Chr 29 and 30
►A

596
The translocation for DFSP is Chr 17 and 22. These chromosomes involve the fusion of PDGF
beta chain and collagen type I alpha I genes. DFSP are generally CD34+ and factor XIIIa
negative.

81
What virus is most closely associated with these lesions in this HIV infected patient?
A. Human herpes virus 2
B. Human herpes virus 6
C. Human herpes virus 8
D. Cytomegalovirus
E. Ebstein Barr virus
►C

Kaposi's sarcoma is an AIDS defining illness. It if found at a much higher incidence in


homosexual men. Human herpes virus 8 is thought to be pathogenic in this tumor.

82
What is the most likely diagnosis?
A. Acquired digital fibrokeratoma
B. Supernumery digit
C. Verruca
D. Infantile digital fibroma
E. Acrochordon
►A

The condition shown is an acquired digital fibrokeratoma. It is a pedunculated, acral lesion with
collarette which is defined by the lack of nerve twigs and bone on pathologic exam.

83
Dermoscopy would be the least helpful in evaluating which lesion?
A. Pigmented basal cell carcinoma
B. Congenital melanocytic nevus
C. Seborrheic keratosis
D. Amelanotic melanoma
E. Hemangioma
►D

Amelanotic melanoma are difficult to diagnosis with the naked eye and even with the
dermoscope. Pigmented basal cell carcinomas may have a characteristic maple leaf pattern,

597
arborizing blood vessels, and /or blue-grey ovoid nests. Seborrheic keratoses often have comedo-
like openings and milia cysts. Hemangiomas typically have red lagoons.

84
The most common location of Merkel cell carcinoma is:
A. Digits
B. Trunk
C. Genitalia
D. Extremities
E. Head and neck
►E

Merkel cell carcinoma usually presents as a violaceous, solitary, dome-shaped nodule, most
commonly located on the head and neck region.

85
A 40 year-old female patient presents with the following lesion (see image). The biopsy report
shows duct like structures, tadpole structures within a fibrotic stroma. According to the
aforementioned information, the most likely diagnosis is:
A. Microcystic adnexal carcinoma
B. Seborrheic keratosis
C. Dermatofibrosarcoma protuberans
D. Merkel cell carcinoma
E. None of these answers are correct
►A

One of the most common locations for MAC includes the perioral area. Histologically it presents
with poorly demarcated tumor cells invading the dermal and subcutaneous tissue. Islands of
basaloid keratinocytes, horn cysts and duct structures are also seen within a desmoplastic stroma.

86
All of the following may increase the incidence of SCC, EXCEPT:
A. Sun exposure
B. Immunosuppression
C. Increasing age
D. Proximity to the equator
E. All of the following may increase the incidence of SCC
►E

598
The incidence of SCC increases with age, and it is 35 times higher in individuals older than 75
years of age when compared to ages 50-55. The incidence doubles for each 8 to 10 degree
decline in latitude, therefore, individuals living closer to the equator have a greater risk.

87
Which of the following statements regarding prognosis for malignant melanoma is true?
A. Increasing age has a positive effect on survival.
B. Patients with primary lesions located on the extremities (except acral lesions)
have a worse prognosis than those with tumors located on the trunk.
C. There is a higher survival rate for patients with palpable metastatic nodes
compared to those with micrometastatic nodal disease.
D. For stage IV disease, patients with non-visceral metastases (eg skin, subcutis,
distant lymph nodes) have a better prognosis compared with those with visceral
metastases.
E. Male gender has a positive effect on survival.
►D

Increasing age and male gender have a negative effect on survival in melanoma. Patients with
primary lesions located on the extremities (except acral lesions) have a better prognosis than
those with tumors located on the head, neck, or trunk. There is a significantly lower survival for
patients with palpable metastatic nodes (macrometastasis) when compared to those with
micrometastatic nodes. For stage IV disease, patients with non-visceral metastases (eg skin,
subcutis, distant lymph nodes) have a better prognosis compared with those with visceral
metastases.

88
A child presents with a giant congenital nevus overlying the back of the skull, extending onto the
shoulders. It is ~15% body surface area, sparing the face and anterior neck. Which test should be
ordered?
A. A head CT
B. A head MRI
C. A skull plain film X-ray
D. A skin biopsy
E. A PET scan
►B

Giant congenital nevi overlying the spinal columna nd skull can be associated with
neurocutaneous melanosis. Presenting symptoms include increased cranial pressure, spinal cord
compression or leptomeningeal melanoma. A MRI should be performed in these children to rule-
out CNS involvement. The other tests would not be as useful in this situation.

599
89
What is the most common location for this rapidly growing tumor?
A. Head and neck
B. Chest
C. Back
D. Arms
E. Legs
►A

Merkel cell carcinoma a cutaneous neuroendocrine carcinoma that usually occurs on the head
and neck. It is a high grade malignant tumor with a 5-year mortality rate of 30-64%. On
histopathology, there are trabecular strands of basophilic cells that stain with a characteristic
paranuclear dot pattern with cytokeratin 20.

90
This lesion is associated with which of the following:
A. HPV 5
B. HPV 11
C. HPV 4
D. HPV 7
E. HPV 1
►B

HPV 6 and 11 are most often associated with anogenital warts or condyloma acuminata. They
can present as sessile lesions on the skin or peducnulated cauliflower lesions. HPV type 5 is
associated with epidermodysplasia verruciformis, HPV 4 is assoicatd with common warts, HPV
type 7 is associated with Butcher warts, and HPV type 1 is associated with common warts.

91
A 50 year-old female patient presents with a reddish indurated plaque located on her right
clavicular area (see image). The biopsy shows the presence of monomorphic spindle cells in a
storiform pattern, and deep subcutaneous infiltration. According to the aforementioned
information, the most likely diagnosis is:
A. Microcystic adnexal carcinoma
B. Seborrheic keratosis
C. Dermatofibrosarcoma protuberans
D. Merkel cell carcinoma
E. None of these answers are correct
►C

600
DFSP intially arises as an asymptomatic, reddish or skin colored indurated plaque, which may
later enlarge, and become raised, firm and multinodular. DFSP is histologically characterized by
the presence of monomorphic spindle cells arranged in a "storiform" or "cartwheel" pattern.

92
Dermoscopic features suggestive of malignant melanoma include:
A. Presence of 2 or more colors within the lesion
B. Blue-whitish veil
C. Asymmetric radial streaming
D. Abrupt interruption of pigment network in the periphery
E. All of these answers are correct
►E

Asymmetry, multicomponent pattern, blue-whitish veil, parallel-ridge pattern, atypical pigment


network, uneven radial streaming, localized irregular and diffuse pigmentation, irregularly
distributed globules, and regression structures are all dermoscopic features suggestive of
malignant melanoma.

93
Which of the following is the most important prognostic indicator in a patient with cutaneous
lymphoma?
A. Age
B. Primary vs secondary cutaneous
C. Gender
D. Extent of cutaneous involvement
E. Subtype of lymphoma
►B

When staging lymphoma, it is critical to determine whether the lymphoma is primary cutaneous
arising in the skin or secondary cutaneous arising in association with nodal or extranodal tumor.
The prognosis is worse in secondary when compared to primary lymphomas, irrespective of
histologic diagnosis.

94
Which of the following immunologic drugs has been shown to increase survival of patients with
stage III and stage IV melanoma?
A. Etanercept
B. Adalimumab
C. Ustekinumab

601
D. Ipilimumab
E. Infliximab
►D

A recent study in the New England Journal of Medicine reported an increase in survival of
patients with stage III and stage IV melanoma with a new immunologic therapy agent named
ipilimumab. The survival was increased from 6 months to 10 months (P < 0.001). All the other
medicines are not used to treat melanoma; they are used to treat psoriasis, among other disease.

95
The risk of a melanoma developing in a giant congenital melanocytic nevus is approximately:
A. 3%
B. 6%
C. 12%
D. 50%
E. Virtually all of these patients will develop melanoma
►B

Large/giant congenital nevi are greater than 20 cm or greater than 10% of the body surface area.
The risk of melanoma in this type of lesion is approximately 6% by the age of 60 (Rhodes, 1981;
Bett, 2005). 50% of these melanomas occur by 5 years of age.

96
Which of the following is not considered a high-risk location of SCC:
A. Eyelids
B. Nose
C. Ear
D. Lips
E. Chest
►E

High risk locations for recurrence and metastases include the ―H-zone‖ of the face, skin
overlying cartilage and bony structures (e.g. preauricular area, retroauricular sulcus, nasolabial
fold, inner canthus, philtrum, temple, upper lip, columella, nose, lower eyelid). Lesions located
on the trunk and extremities are usually considered low risk unless aggressive histologic pattern
is present.

97
What type of skin cancer is associated with recessive dystrophic epidermolysis bullosa?
A. Squamous cell carcinoma

602
B. Basal cell carcinoma
C. Merkel cell carcinoma
D. Melanoma
E. Actinic keratosis
►A

Patients with recessive dystrophic epidermolysis bullosa havea high risk of aggressive SCC in
the areas of chronic erosion and must be screened thoroughly and frequently. Actinic keratosis is
not skin cancer.

98
The mucin found in this lesion is produced by:
A. Fibroblasts
B. Keratinocyte
C. Merkel cells
D. Nail matrix
E. Osteoblast
►A

Digital mucous cysts are pseudocysts found on the dorsal digits between the distal
interphalangeal joint and the nail fold. The fibroblasts in digital mucous cysts produce large
quantities of hyaluronic acid.

99
Which of the following are features of dermatofibromas, which help differentiate it from
dermatofibrosarcoma protuberans?
A. Commonly located on the extremities
B. Factor XIIIa negative
C. Factor XIIIa positive
D. Commonly located on the trunk
E. Commonly located on the extremities and factor XIIIa positive
►E

Typically DFSP is CD34 positive and factor XIIIa negative, allowing its differentiation from
dermatofibroma.

100
What is the most common location for this lesion which lacks phosphorylase in the epidermal
cells?
A. Back

603
B. Buttock
C. Leg
D. Neck
E. Sacrum
►C

Clear cell acanthomas are slow growing, benign growths which typically occur on the leg. They
have a striking histologic appearance in the epidermis with sharp demaracation and enlarged,
pale cells. Excess glycogen in the cells accounts for their clear appearance and is due to a defect
in phosphorylase.

101
The Gorlin syndrome is characterized by:
A. Multiple BCCs during childhood
B. Macrocephaly
C. Odontogenic keratocysts of the jaw
D. Autosomal-dominant inheritance pattern
E. All of these answers are correct
►E

Gorlins syndrome is characterized by the appearance of multiple BCCs during childhood,


odontogenic keratocysts of the jaw, and skeletal defects. It is inherited in an autosomal dominant
pattern.

102
Seventy-five year old, male patient, with a 3cms pink, pearly nodule on his chest. You notice that
the patient has a scar on the same area. The treatment of choice is:
A. Radiation
B. Conventional excision
C. Cryotherapy
D. Mohs surgery
E. Imiquimod
►D

The most likely diagnosis in this patient is recurrent BCC, therefore Mohs surgery is the
treatment of choice. Furthermore, any NMSC measuring 3 cms located on any anatomical site is
an indication for Mohs surgery.

103
This type of keratoacanthoma is eruptive and disseminated in adulthood:

604
A. Grzybowski
B. Ferguson-Smith
C. KA centrifugum
D. Giant KA
E. Subungual
►A

Patients with Grzybowski keratoacanthoma have eruptive and disseminated KA during


adulthood. KA centrifugum are large and solitary and Giant KA can invade underlying.

104
Which HPV serotype has been associated with periungual SCC?
A. 6
B. 16
C. 11
D. All of these answers are correct
E. HPV is not associated with SCC
►B

HPV infection has been associated with the development of cutaneous SCC. HPV 6 and 11 have
been linked to genital lesions, whereas HPV 16 has been found in periungeal lesions.

105
Keratoacanthomas have been linked etiologically to:
A. Ultraviolet exposure
B. Human papilloma virus
C. Chemical carcinogens such as tar and pitch
D. Smoking
E. All of these answers are correct
►E

The origin of KAs has not been established. Ultraviolet exposure, exposure to chemical
carcinogens such as tar and pitch, as well as smoking, and a viral etiology, specifically the
human papilloma virus, have all been proposed as etiologic factors.

106
The treatment for Merkel cell carcinoma is all of the following except:
A. Wide local excision 2-3 cm
B. Mohs
C. Radiation sensitive therapy

605
D. Wide local excision 5-10 cm
E. Local excision of 1-2 cm
►E

Merkel cell carcinoma is a type of aggressive cancer with a high rate of recurrence. It is a
neuroendocine carcinoma that is most commonly on sun exposed areas like the head and neck in
the elderly. Treatment includes wide local excision of 2-3 cm, Mohs, radiation, and
chemotherapy.

107
Sezary syndrome:
A. Has characteristic Sezary cells in peripheral blood
B. The five year survival is estimated to be between 10-20%.
C. Is characterized by the triad of pruritic erythroderma, generalized
lymphadenopathy, and presence of Sezary cells in peripheral blood
D. Is associated with a poor prognosis
E. All of these answers are correct
►E

Sezary syndrome is characterized by the triad of pruritic erythroderma, generalized


lymphadenopathy, and presence of Sezary cells in peripheral blood. Sezary cells are abnormal,
large hyperconvoluted lymphocytes. The five year survival is estimated to be between 10-20%.

108
Which of the following best describes the incidence of skin cancer in transplant recipients from
greatest to least?
A. BCC>melanoma>SCC>Merkel cell
B. SCC>BCC>melanoma>Merkel cell
C. BCC>SCC>melanoma>Merkel cell
D. SCC>BCC>Merkel cell>melanoma
E. Merkel cell>SCC>melanoma>BCC
►B

The incidence of skin cancer in transplant recipient (from greatest to least) is as follows:
SCC>BCC>melanoma>Merkel cell.

109
Histopathological evidence of epidermotropism may be seen in which of the following
conditions:
A. Merkel cell carcinoma

606
B. SCC
C. CTCL
D. None of these answers are correct
E. All of these answers are correct
►C

Epidermotropism, defined as the presence of lymphocytes in the epidermis is characteristic of


CTCL.

110
40 year-old male presented with a slow growing large tumor on the back. The patient undergoes
wide excision with adjunctive chemotherapy. Which of the chemotherapeutic agents was used?
A. Imatinib
B. Cytarabine
C. Doxorubicin
D. Vincristine
E. Cisplatin
►A

Imatinib or Gleevec is used in the treatment of primary or locally recurrent dermatofibrosarcoma


protuberans. Even with wide excision, the 5 year recurrence rate for this tumor can be 20-25%.
Activation of the Platelet derived growth factor receptor associated with overexpression of
Platelet Derived growth factor is central to the development of DFSP Imatinib is a protein
tyosine kinase inhibitor for Bcr-Abl kinase and is used primary in chronic myelogenous
leukemia with the Philadelphia chromosome defect. Imatinib also inhibits the typosine kinases
associated with platelet derived growth factor and stem cell factor. Therefore, it directly inhibits
the Platelet Derived Growth factor receptor signaling cascade,that plays a critical role in the
pathogenesis and growth of DFSPs. While Imatinib has been successful in clinical trials, it is not
yet FDA approved for the treatment of DFSPs. The other listed agents are not adjunctive or
primary therapies for DFSP.

111
Which site of squamous cell carcinoma has the greatest risk of metastasis?
A. lip
B. ear
C. eyelid
D. trunk
E. nose
►A

607
The lip has an approximate rate of metastasis of 13%, which is slightly higher than the metastasis
rate of 11% from the ear. Risks for metastasis include size greater than 2cm, perineural invasion,
immunosuppression, treatment history, degree of tumor differentiation, or location on the ear or
lip.

112
All of the following are true regarding curettage of giant congenital melanocytic nevi except:
A. Treatment remains a controversial topic
B. Best performed during the first 2 weeks of life
C. Decreases the risk of melanoma
D. Offers an adequate alternative to surgical excision
E. Mandates careful long-term follow-up
►C

Curettage of giant congenital nevis is a highly controversial treatment option. De Raeve and
Roseeuw reported on 16 neonates who underwent curettage for treatment of their giant
congenital nevi. They noted that it was best performed within the first two weeks of life. They
claim that it offers an adequate cosmetic alternative to surgical excision. The risk of melanoma
developing within the giant congenital nevus is not decreased and long-term follow-up is
essential.

113
What is the mean time-frame for development of Stewart-Treves Syndrome?
A. Less than 1 year
B. 1 year
C. 5 years
D. 10 years
E. >20 years
►E

Stewart-Treves syndrome refers to an angiosarcoma arising from chronic lymphedema. It


typically occurs as a complication of long-lasting lymphedema of the arm after mastectomy
and/or radiotherapy for breast cancer. It is a very late complication of disease, usually developing
several years later. The mean time-frame for development of Stewart-Treves Syndrome in the
literature is 23 years (4-40 years).

114
What kind of T-lymphocyte is the most common neoplastic cell in Cutaneous T-Cell
Lymphoma:
A. CD4

608
B. CD8
C. Natural killer
D. None of these answers are correct
E. All of these answers are correct
►A

CTCL is a neoplasm of helper T cells (CD4) that originates in the skin.

115
Commonly used immunohistochemical markers that are positive in Merkel cell carcinomas
include all of the following EXCEPT:
A. Chromogranin A/B
B. Synaptophysin
C. Glial fibrillary acidic protein
D. Cytokeratin 20
E. Cytokeratin 8
►C

The most commonly used markers for MCC are monoclonal antibodies to cytokeratins 8, 18, and
20; neuron-specific enolase (the most constant marker); chromogranin A/B, and synaptophysin.
Leukocyte common antigen, vimentin, desmin, glial fibrillary acidic protein, and S-100 are
consistently absent in MCC.

116
Which area of the face receives the most cumulative exposure to UV radiation?
A. Dorsum of nose
B. Orbital region
C. Base of the nose
D. Chin
E. Central cheek
►C

The most frequent sites of basal cell carcinomas in one study was the base of the nose (bordering
the nasolabial fold and extending midway up the side of the nose), orbital region, apex of the
nose, ear, forehead, temporal region, nasolabial area, and the buccal region (in order). The nose
was by far the most frequent site of BCC‖s.

117
Desmoplastic trichoepithelioma:
A. Is most common in middle-aged patients

609
B. More commonly appears in males than in females
C. Does not display foci of calcification or ossification
D. Presents as a well-circumscribed lesions located in the upper dermis
E. None of these answers are correct
►D

Desmoplastic trichoepithelioma is a variant of trichoepithelioma, an uncommon adnexal tumor


with differentiation toward hair structures. It is three times more common in females and usually
occurs in young adults. The tumor is a well-circumscribed nodule lesions located in the upper
dermis. Strands or columns of basaloid cells are seen surrounded by fibrotic or desmoplastic
stroma. Horn cysts may also be seen, as well as foci of sebaceous cells, calcification and
ossification.

118
Which of the following is true regarding digital HPV-associated squamous cell cancers?
A. The rate of metastasis approaches 15%.
B. HPV18 is the most common associated sybtype.
C. Mohs micrographic surgery yields a 20% recurrence rate.
D. Women outnumber men 2:1.
E. These lesions only occur in association with immunosuppression.
►C

According to Riddel et al (JAAD 2011;64(6):1147-1153), Mohs micrographic surgery, although


the treatment of choice, results in a 20% recurrence rate, which is significantly higher than
cutaneous SCC. HPV16 is most often implicated. Men outnumber women 2:1. The rate of
metastasis averages between 2-3%. Although common in transplant patients, HPV associated
digital SCCs can occur secondary to trauma and in immunocompetent patients.

119
All of the following are true of thick melanomas (>3 mm) except:
A. Predominantly nodular type
B. Women affected more than men
C. Predilection for the head and neck
D. Mainly in older patients (>50 years)
E. Associated with fewer nevi
►B

According to a study performed by Chamberlain, et.al., thick melanomas (> 3 mm) were
predominantly nodular in type. They occurred in older men, mostly on the head and neck and
were associated with fewer nevi.

610
120
Merkel cell carcinoma stains positively for:
A. Leukocyte common antigen
B. Neuron specific enolase
C. Vimentin
D. S-100
E. None of these answers are correct
►B

Merkel cell carcinoma is frequently diagnosed in individuals older than 50 years of age.
However, there have been reports of MCC in children and young adults.

121
Pseudorosettes in Merkel cell carcinoma:
A. Are seen in the trabecular variant
B. Are seen in the intermediate-cell type
C. Are seen in the small-cell type
D. All of these answers are correct
E. None of these answers are correct
►A

There are three histologic patterns of MCC: trabecular, intermediate-cell type, and small-cell
type. The trabecular variant consists of interconnecting trabeculae separated by strands of
connective tissue. Pseudorosettes may be seen in this type.

122
A patient was referred from the oncology service because of tumoral lesions. There is no bone
marrow involvement. What is the stage on this patient?
A. IIA
B. IIB
C. IIIA
D. IIIB
E. IV
►B

This patient has the tumor stage of mycosis fungoides(MF). Patients with tumoral lesions are
automatically classified as IIB. IA is the limited patch or plaque disease with less than 10% BSA
involved. IB is the generalized patch/plaque stage with more than 10%BSA but less than 80%
BSA involved. IIA implies lymph node involvement. IIIA is erythroderma without blood

611
involvement. IIIB is erythroderma with low blood tumor burden. The stage IV of MF is divided
in three substages: IVA1(high blood tumor burden-Sezary syndrome), IVA2(very abnormal
nodes), IVB(visceral involvement).

123
Which of the following is true regarding actinic keratoses?
A. Salicylic acid, tretinoin, and alpha-hydroxy acids are not useful in treatment
B. Low fat diets may decrease the incidence of AKs
C. Cure rates for treatment with topical 5-fluorouracil are higher than for treatment
with cryotherapy
D. UVA from sunlight is most responsible for AK development
E. Male gender is not a risk factor for AKs.
►B

Salicylic acid, tretinoin, and alpha-hydroxy acids have been reported to be useful in treatment of
AKs. Low fat diets have been reported to be useful in decreasing the incidence of AKs.
Cryotherapy has a reported cure rate of 98.8% for AKs, whereas topical 5-FU has a reported cure
rate of 93%. UVB, not UVA, is most responsible for AK development. Male gender is a risk
factor for the development of AKs.

124
Which neoplasm is associated with the Stewart-Treves syndrome?
A. Renal leiomyomas
B. Basal cell carcinoma
C. Angiosarcoma
D. Keratoacanthoma
E. T cell lymphoma
►C

Stewart-Treves syndrome is the development of angiosarcoma in the setting of chronic


lymphedema. Originally, named after radical mastectomy for the treatment of breast cancer. The
term applies to the development of angiosarcoma in any chronic lymphedematous condition.

125
The most common location for angiosarcoma is:
A. Legs
B. Arms
C. Head and neck
D. Trunk
E. Digits

612
►C

Angiosarcomas present as painless, purple nodules in the head and neck region of elderly
individuals. Men are more frequently diagnosed with this aggressive neoplasm.

126
A patient with multiple skin nodules has a biopsy suggestive of mycosis fungoides. The next
appropriate step in making the diagnosis is:
A. Polymerase chain reaction
B. Complete blood count
C. Flow cytometry
D. Immunohistochemical stains
E. Chest xray
►A

In a patient with skin lesions and pathology suggestive of cutaneous lymphoma, gene
rearrangement studies should be done to detect a clonal population of lymphocytes. Polymerase
chain reaction (PCR) is approximately 1000 times more sensitive than Southern blot in the
detection of a clonal population of cells. In addition, PCR may be used on formalin fixed tissue
as well as fresh tissue, whereas Southern blot testing must be done of fresh tissue only.

127
All of the following statements regarding Basal Cell Nevus Syndrome are true EXCEPT:
A. Associated tumors include medulloblastoma
B. It is inherited in an autosomal recessive manner
C. Hypertelorism is a feature
D. The affected gene is mutated in 30-40% of sporadic basal cell carcinomas
E. All of these answers are correct
►B

Basal cell nevus syndrome (Gorlin Syndrome) is caused by a mutation in the PTCH gene,
located on chromosome 9q22. This gene is found to be mutated in 30-40% of sporadic basal cell
carcinomas. Gorlin Syndrome is inherited in an autosomal dominant pattern. Tumors associated
with this sydrome include medulloblastoma and meningioma. It is characterized by the
appearance of multiple BCCs during childhood, odontogenic keratocysts of the jaw, and skeletal
defects (including frontoparietal bossing and hypertelorism, among others).

128
What temperature must be achieved for adequate treatment of a small superficial squamous cell
carcinoma with cryotherapy?

613
A. -10 degrees Celcius
B. -20 degrees Celcius
C. -30 degrees Celcius
D. -40 degrees Celcius
E. -50 degrees Celcius
►E

Cryosurgery destroys tumor if it is frozen to -40 to -70 degrees Celcius, for at least 2 cycles with
a 60 second thaw. Benign lesions can be destroyed at temperatures around -25 degrees Celcius.

129
This tumor, also called a Shagreen patch, is characteristic of which of the following
genodermatoses?
A. Neurofibromatosis-1
B. Neurofibromatosis-2
C. Tuberous sclerosis
D. NAME syndrome
E. Buschke-Ollendorf
►C

Tuberous sclerosis a rare genodermatosis characterized by adenoma sebaceum, seizures and


mental retardation. Other cutaneous features include hypopigmented macules, periungual
fibromas, fibrous plaque of the face, cafe-au-lait macules and connective tissue nevus.

130
A 30 year-old gentleman presents with multiple self healing lesions similar to the one pictured.
Other family members also have the same disease. What is the most likely diagnosis?
A. Epidermodysplasia verruciformis
B. Ferguson-Smith syndrome
C. Nevoid basal cell carcinoma syndrome
D. Dyskeratosis congenita
E. Basex syndrome
►B

Ferguson-Smith syndrome is a rare autosomal dominant condition that is characerized by


multiple keratoacanthomas found in sun-exposed areas. Lesions typically regress over weeks to
months.

131
At what location is this tumor LEAST likely to recur after surgical excision?

614
A. Nasolabial fold
B. Lateral canthus
C. Chin
D. Preauricular
E. Scalp
►E

Basal cell carcinomas are slow-growing, epithelial tumors. Highest area of recurrences after
surgical excision is in the "H-zone" of the face which include nose, nasolabial folds, periocular,
periauricular. The scalp is a M-zone.

132
Which test should be used to detect monoclonal gene rearrangements in cutaneous T-cell
lymphoma?
A. Northern blot
B. Southern blot
C. Western blot
D. ELISA
E. Electrophoresis
►B

Monoclonality and gene rearrangements can be detected with a Southern blot. Southern blots can
be used to detect specific DNA fragments by gel-transfer hybridization.

133
All of the following are true regarding lentigo maligna except:
A. High rates of recurrence
B. Occurs mostly on head and neck
C. Mostly in sun-exposed areas
D. Margins difficult to evaluate
E. Spares oral mucosa
►E

Lentigo maligna is a type of melanoma in situ that usually presents as a poorly circumscribed,
variably colored patch. It occurs on sun-exposed areas on the head and neck. Margins may be
difficult to evaluate and recurrences are relatively common. There are rare reports of spread onto
the oral mucosa as well as conjunctiva.

134
Characteristics indicative of a high risk of metastasis with SCC include:

615
A. Mitotic rate
B. Extremity location
C. Well differentiated
D. Deep invasion
E. Papillomavirus infection
►D

The rate of SCC metastasis from all skin sites ranges from 0.5% to 5.2%. Local recurrences and
metastasis are related to: 1.treatment modality, 2. prior histological, 3. location, 4. size, 5. depth,
6. histological differentiation, 7. histological evidence of perineural involvement, 8. histological
evidence of desmoplastic features, 9. precipitating factors other than UV light, and 10. host
immunosuppression. In reference to metastatic disease, the highest rates occur from scars, the
lip, and the external ear. Patients with perineural spread have a local recurrence rate of 47.2%
and a metastatic rate of 34.8%. Demosplastic SCC's are 6 times more likely to metastasize than
other histological patterns.

135
What cancer in women most commonly metastasizes to the skin?
A. Breast cancer
B. Medullary thyroid carcinoma
C. Glioblastoma multiforme
D. Colon adenocarcinoma
E. Cervical cancer
►A

Breast cancer is the most common cancer to metastasize to the skin in women.

136
A young woman presents with a single small, firm, umbilicated papule on the face. Biopsy
reveals a well-circumscribed lesion located in the upper dermis. Strands of basaloid cells are
seen surrounded by fibrotic or desmoplastic stroma. Horn cysts and foci of sebaceous cells and
calcification are also noted. How should this patient and her lesion be treated?
A. Reassurance and no further treatment
B. Close clinical follow-up
C. Topical 5-fluorouracil
D. Cryotherapy
E. Local surgical excision
►E

616
This patient has a desmoplastic trichoepithelioma. Desmoplastic trichoepithelioma is a variant of
trichoepithelioma, an uncommon adnexal tumor with differentiation toward hair structures. It
presents commonly in young women as a small, firm, umbilicated papule on the face. Local
surgical excision is the treatment of choice.

137
A 55 year-old female presents with an ulcerated malignant melanoma with Breslow dept of
1.5mm. Sentinel lymph node biopsy is negative. Which of the following is the correct staging
classification?
A. Stage IB
B. Stage IIA
C. Stage IIB
D. Stage IIIA
E. Stage IIIB
►B

The current melanoma staging is developed by the American Joint Comittee on Cancer. This
system is based on four characterisitics: 1. Tumor thickness 2.Presence of ulceration within the
primary tumor, 3. Involvement of lymph nodes and 4.presence of distal metastases. Stage 1A
characterizes a melanoma that has less than 1mm in tumor thickness and no ulceration, no nodes,
and no distant mets. Stage IB characterizes a melanoma with tumor thickness less than 1mm
however with ulceration and no nodes or distal metastases. Stage IB also can characterize
melanomas with tumor thickness between 1.01-2.00 mm without ulceration. Stage IIA classifies
tumor with thickness between 1.01-2.00mm, with ulceration, and no nodes and no distal
metastases. Stage IIA also characterizes those melanomas with thickness between 2.01-4.0mm
without ulceration, nodes, or distal mets. Stage III and IV melanomas have nodal and distal met
involvement respectively.

138
Which of the following is most likely to present with cutaneous metastases in men?
A. Lung cancer
B. Colon cancer
C. Prostate cancer
D. Melanoma
E. Esophageal Cancer
►D

The most common malignancy to present with cutaneous metastases in men is from melanoma,
followed by squamous cell cancer of the head and neck. In women, breast cancer is most likely

617
to present with cutaneous metastases. Special presentations of cutaneous metastases include
alopecia neoplastica (scarring alopecia) and zosteriform metastases.

139
A 56-year old woman with a history significant for chronic lymphedema after radical
mastectomy twelve years ago presents with this growth on her arm. What is the diagnosis?
A. Angiosarcoma
B. Bacillary angiomatosis
C. Castleman's syndrome
D. Kaposi's sarcoma
E. Metastatic breast carcinoma
►A

Angiosarcoma may occur in association with chronic lymphedematous states. Stuart-Treves


syndrome applies to angiosarcoma arising in an area of chronic lymphedema, like upper arm
lymphedema after mastectomy.

140
Imiquimod is an immune response modifier that stimulates innate and cell mediated immune
pathways. It induces all of the following cytokines EXCEPT:
A. IL-1
B. IL-4
C. IL-5
D. IL-6
E. IL-8
►B

Imiquimod induces the synthesis and release of cytokines such as interleukins 1, 5, 6, 8, 10, and
12, among others.

141
Which syndrome is characterized by multiple keratoacanthomas beginning in childhood?
A. Muir-Torre
B. Ferguson Smith
C. Rombo
D. Grybowski
E. Keratoacanthoma centrifugum marginatum
►B

618
Ferguson Smith is familial (AD), and is characterized by regressing keratoacanthomas beginning
in childhood. Grybowski is a non-familial disorder in which patients develop generalized
eruptive keratoacanthomas beginning between age 40 to 60. Keratoacanthoma centrifugum
marginatum is most commonly a solitary lesion occurring later in life. Muir Torre is a syndrome
characterized by gastrointestinal or genitourinary neoplasms which precede development of
sebaceous neoplasms and keratoacanthomas by one to two decades. Rombo is not associated
with keratoacanthomas, but is associated with multiple basal cell carcinomas, atrophoderma
vermiculatum, hypotrichosis, and milia.

142
A 3-year-old girl presents with multiple small, angulated, firm nodules. There is a positive "tent
sign" and biopsy reveals ghost cells and germinative cells. Of the following conditions
associated with multiple pilomatricomas, which is thought to be most closely linked?
A. Turner's
B. Myotonic dystrophy
C. Rubenstein-Taybi
D. Sarcoidosis
E. Gardner's syndrome
►B

Pilomatricomas are the most common superficial pediatric tumor. The occurrence of multiple
pilomatricomas has been most closely associated with the development of myotonic
dystrophy(Steinert Disease). The onset of myotonic dystrophy, an autosomal dominant disorder
with variable penetrance, can occur before or after the onset of lesions. Cigliano et al. reported
myotonic dystrophy in 1 out of 2 patients with multiple pilomatricomas; Julian et al. reported
myotonic dystophy in 1 out of 4 patients. The other syndromes associated with multiple
pilomatricomas include Rubenstein-Taybi, Turner syndrome, Gardner syndrome, and
sarcoidosis.

143
What is the most common location for an epitheloid sarcoma?
A. head and neck
B. proximal extremities
C. hands and forearms
D. lower legs
E. groin and buttocks
►C

Epithelioid sarcoma is a rare soft tissue sarcoma that most commonly develops in young adults,
males greater than females, with a predilection for the distal upper extremities, namely hands and

619
forearms. Most tumors present as firm-to-hard palpable masses, either in the deep soft tissue or
in the dermis. The superficial lesions can present with ulceration. Five year survival and ten year
survival rate for patients with epithelioid sarcoma are approximately 50-70% and 42-55%
respectively (Journal of bone and Joint Surgery (Am), 70-A: 862-870, 1988). Female patients
have a more favorable outcome. Proximal lesions have been shown to have worse outcomes
compared to distal lesions.

144
What is the most common location of oral SCC?
A. Soft palate
B. Buccal mucosa
C. Gingiva
D. Dorsal tongue
E. Lateral tongue
►E

Squamous cell carcinomas are the most common carcinoma of the oral cavity. The most
common locations for this tumor are the lateral and ventral surfaces of the tongue and the floor
of the mouth.

145
Desmoplastic trichoepitheliomas are commonly located on:
A. Legs
B. Chest
C. Back
D. Face
E. Scalp
►D

Desmoplastic trichoepithelioma presents as small, firm, umbilicated papule on the face of young
adults.

146
The type of keratoacanthoma that is seen in children with a autosomal dominant pattern and
multiple in numbers is called:
A. Ferguson-Smith
B. Grzybowski
C. Verrucous carcioma
D. KA centrifugum
E. Buschke-Lowenstein

620
►A

Patients that have multiple keratoacanthoma in childhood have Ferguson-Smith type of KA. This
is autosomal dominant and are self-healing.

147
All of the followings can be used for treatemt of this condition except
A. Cryotherapy
B. Topical Imiquimod 5% cream
C. Topical 5-flurouracil
D. Topical retinoids
E. Surgical excision
►E

Attached picture is disseminated superficial actinic porokeratosis (DSAP) which is the most
common type of all porokeratosis, with multiple thin papules appearing most commonly on the
legs of adult women. Many treatments have been used for this condition which include:
cryotherapy, topical 5-fluorouracil (5-FU), topical retinoids, CO2 laser, and dermabrasion.
Although surgical excision might be used for treatment of other forms of porokeratosis, it is not
advised in this case because of number of lesions and increase risk of scarring. Other forms of
porokeratosis are: porokeratosis of Mibelli, punctate porokeratosis, linear, and Porokeratosis
palmaris et plantaris disseminata.

148
What is the most common location for a fibroepithelioma of pinkus variant of BCC is
A. head and neck
B. extremities
C. hands and feet
D. upper trunk
E. lumbosacral region
►E

Fibroepithelioma of pinkus variant of BCC is often a fleshy or pink colored nodule. The most
common location of a fibroepithelioma of pinkus is the lumbosacral region. Superficial variants
are more common on the trunk and extremities. BCC makes up the most common non melanotic
skin cancer at 75%.

149
Merkel cell carcinoma has been found to be associated with which of the following viruses?
A. Herpes virus

621
B. Polyomavirus
C. Paramyxovirus
D. Flavivirus
E. Enterovirus
►B

Merkel cell carcinoma has been found to be associated with the merkel cell polyomavirus
(MCPyV). Herpes virus causes diseases such as HSV, VZV, EBV, CMV, roseola, and Kaposi's
sarcoma. Paramyxovirus causes measles. Flavivirus causes diseases such as west nile virus,
dengue, and yellow fever. Enteroviruses are not associated with merkel cell carcinoma.

150
The Grzybowski type of keratoacanthoma:
A. Is characterized by rapid growth of a single lesion reaching a diameter of 9 cm
or more
B. Typically invades underlying cartilage
C. Demonstrates simultaneous central healing
D. Presents in childhood on sun-exposed surfaces
E. Presents with hundreds of disseminated lesions
►E

The Grzybowski type of keratoacanthoma is typically diagnosed in adulthood, with the sudden
appearance of hundreds of lesions in a disseminated fashion. The lesions are generally 2-3mm in
diameter and can be found anywhere on the body including the palms, soles, larynx, and oral
mucosa.

151
Which of the following are the most reliable prognostic factors in malignant melanoma?
A. Breslow‘s depth and ulceration
B. Breslow‘s depth and Clark level
C. Clark level and ulceration
D. All of these answers are correct
E. None of these answers are correct
►A

The most reliable prognostic factors in MM are Breslow‘s depth and ulceration of the primary
tumor. Breslow depth is the thickness of the melanoma measured from the granular layer to the
deepest point of tumor invasion.

152

622
Patients that are organ transplant patients have a higher risk of developing skin cancer such as
squamous cell carcinoma. The risk can be as much as:
A. 65x
B. 50x
C. 40x
D. 25x
E. 10x
►A

The risk of SCC in an organ transplant population is elevated by 65x. These patients should have
a year skin examination.

153
The Pinkus BCC is frequently located on the:
A. Digits
B. Eyelid
C. Lips
D. Ear
E. Lumbosacral area
►E

Pinkus BCC (also known as fibroepithelioma) appears as a pedunculated, dome-shaped papule


frequently located on the back (lumbosacral area).

154
What is the average time between a BCC primary tumor and its metastasis?
A. 6 months
B. 1 year
C. 2-4 years
D. 9 years
E. greater than 15 years
►D

The size of a BCC is related to its metastasis risk. Invasion of bone, cartilage, and muscle is not
common, however, spread does occur along perichondrium, periosteum, fascia, or tarsal plate.

155
All of the following statements regarding the patient pictured are true EXCEPT:
A. Spina bifida may be an associated finding
B. The patient likely has a mutation in the PTCH gene

623
C. Pheochromocytoma is a tumor associated with this disease
D. This patient likely had a similarly affected parent
E. These lesions appeared in childhood
►C

The patient has basal cell nevus syndrome, or Gorlin syndrome, which is characterized by the
appearance of multiple basal cell carcinomas in childhood, odontogenic keratocysts of the jaw,
and skeletal defects (i.e macrocephaly, hypertelorism, frontoparietal bossing, spina bifida, or rib
abnormalities). Tumors associated with this disease include medulloblastoma and meningioma
(not pheochromocytoma). It is caused by a mutation in the PTCH gene located on 9q22 and is
inherited in an autosomal dominant manner.

156
Which of the following neoplasms has demonstrated an association with HTLV-1 infection?
A. Mycosis fungoides
B. Adult T cell lymphoma
C. Follicular lymphoma
D. Multiple myeloma
E. Hodgkin‘s disease
►B

Mycosis fungoides is a rare form of cutaneous T-cell lymphoma, the etiology of which is not
completely known. Adult T Cell Lymphoma is also a type of T cell neoplasm that has been
linked to HTLV-1. Adult T-cell lymphoma may have an acute and chronic, smoldering form.
The chronic, smoldering form can be difficult to distinguish from mycosis fungoides.

157
Mohs micrographic surgery the treatment of choice for all of the following, except:
A. 1 cm SCC located on the chest
B. 2 cms BCC on lower extremities
C. 1 cm BCC on the eyelid
D. Morpheaform BCC on the cheek
E. Recurrent BCC on the chest
►A

SCCs that measure 1cm on the chest are not an indication for Mohs micrographic surgery.
Lesions must measure 2cms on the chest and extremities, or 1cm for those located on the face to
be considered for Mohs surgery. Due to the aggressive growth patter and subclinical spread of
morpheaform and recurrent BCCs are always and indication for Mohs surgery.

624
158
This type of verrucous carcinoma is locally aggressive but rarely metastasizes and is on the
plantar area is:
A. Epithelioma cuniculatum
B. Buschke-Lowenstein
C. Oral florid papillomatosis
D. Subungual
E. Giant SCC
►A

Epithelioma cuniculatum is a plantar SCC that is locally aggressive but rarely metastasizes, Oral
florid papillomatosis is the a type that is found in the mouth. Buschke-Lowenstein is the type that
you find in the genital area.

159
Which is the most common neoplasm in patients who have had long-term PUVA therapy?
A. Basal cell carcinoma
B. Atypical fibroxanthoma
C. Squamous cell carcinoma
D. Cutaneous T cell lymphoma
E. Melanoma
►C

The most common neoplasm to arise in patients who have been treated with long-term PUVA
therapy is squamous cell carcinomas. In a study by Stern, et.al. a 12.8 fold risk was found for the
development of squamous cell cancers in patients who received high doses than those that
received low doses.

160
Which of the following markers do not stain melanocytic lesions:
A. CK7
B. Vimentin
C. S-100
D. HMB-45
E. All of these answers are correct
►A

CK7 will stain keratinocytes. Vimentin, S-100 and HMB-45 stains melanocytic lesions.

161

625
Which of the following are features of microcystic adnexal carcinoma that help distinguish it
from desmoplastic trichoepitheliomas?
A. Deep subcutaneous infiltration
B. Perineural invasion
C. CEA positive staining
D. Commonly located on the face
E. Deep subcutaneous infiltration,perineural invasion, and CEA positive staining
►E

Desmoplastic trichoepitheliomas is one of the differential diagnosis for MAC. MAC show deep
subcutaneous and perineural invasion, as well as CEA positive staining, all features that may
help differentiate it from desmoplastic trichoepitheliomas. Both, MAC and desmoplastic
trichoepitheliomas are commonly located on the face.

162
Which of the following is a common location of melanoma in women?
A. Chest
B. Lower legs
C. Genitals
D. Digits
E. Scalp
►B

The most common locations of MM in women are the back, lower legs and upper extremities.

163
The most common location of superficial spreading melanoma in men is:
A. Lower legs
B. Back
C. Upper extremities
D. Head and neck
E. Digits
►B

Superficial spreading melanoma may be located anywhere, but the back is the most common site
in male patients.

164
Bazex syndrome can be differentiated clinically from Rombo syndrome by presence of:
A. Multiple basal cell carcinomas

626
B. Trichiepitheliomas
C. Milia
D. Follicular atrophoderma
E. Hypohidrosis
►E

Localized hypohidrosis is a feature found in Bazex syndrome but not Rombo syndrome. Another
differentiating feature is that Rombo syndrome classically has vermiculate atrophoderma, while
Bazex has follicular atrophoderma.

627
Chapter -7-
Medical Mycology
1
Which two characteristics combined form a significant risk for acquiring sporotrichosis? 1.
Animal handler 2. Male gender 3. Alcoholism 4. Filipino or African decent 5. Genetic
predisposition
A. 1, 2
B. 1, 3
C. 2, 5
D. 4, 5
E. 3, 4
►B

A variety of domestic animals can carry Sporotrichosis. Alcoholism increases the risk for this
cutaneous infection.

2
What is the most frequently reported cause of primary cutaneous aspergillosis?
A. Aspergillus flavus
B. Aspergillus fumigatus
C. Aspergillus niger
D. Aspergillus solani
E. Aspergillus marneffei
►A

Primary cutaneous aspergillosis is a rare disease reported mostly frequently in children with
hematologic malignancies who developed skin lesions at the site of IV canulas.

3
Which of the following DOES NOT typically cause white superficial onychomycosis?
A. Aspergillus species
B. Fusarium species
C. Trichophyton mentagrophytes
D. Scopulariopsis species
E. Trichophyton rubrum
►E

628
In white superficial onychomycosis, the organism only invades the superficial nail plate, and it
clinically appears as chalky white patches on the nails. The most common organism is T.
mentagrophytes, but Aspergillus, Cephalosporium, Fusarium, Acreconium and Scopulariopsis
are implicated as well.

4
A blood culture from a neutropenic patient with onychomycosis grew which of the following
organism:
A. Scopulariopsis sp
B. Aspergillus sp
C. Fusarium sp
D. Acremonium sp
E. T. rubrum
►C

Fusarium is one of the few moulds, which yield positive blood cultures, neutropenia is one of the
risk factors for Fusariosis.

5
Although this organism is not a fungus, it stains with PAS and GMS and produces 8-20 micron
spherules in tissue. This organism can be identified as:
A. Rhinosporidium seeberi
B. Coccidioides immitis
C. Penicillium marneffei
D. Prototheca wickerhami
E. Leishmania mexicana
►D

This achloic algae produces spherules or sporangia 8-20um in tissue. The mature form is called a
morula. Rhinosporidium is also not a fungus however, it produces sporangium 250-350 microns.

6
Which of the following is a fluorescent ectothrix dermatophyte?
A. Trichophyton rubrum
B. Trichophyton mentagrophytes
C. Microsporum ferrugineum
D. Trichophyton violaceum
E. Microsporum nanum
►C

629
Fluorescent ectothrix dermatophytes include M. canis, M. audouinii, M. distortum, M.
ferrugineum, and sometimes M. gypseum and T. schoenleinii ("Cats And Dogs Fight and Growl
Sometimes.") Nonfluorescent ectothrix dermatophytes include T. mentagrophytes, T. rubrum, T.
verrucosum, T. megninii, and M. nanum. Endothrix dermatophytes include T. rubrum, T.
gourvilli, T. yaounde, T. tonsurans, T. schoenleinii, T. soudanense, and T. violaceum.

7
Which of the following statements regarding candidal infection is FALSE?
A. Candida species displays true hyphae on potassium hydroxide examination
B. Predisposing factors for candidal infection include diabetes mellitus,
hyperhidrosis and broad spectrum antibiotics
C. Candida albicans is the number one cause of mucocutaneous infections
D. Candidal infections typically do not spare the scrotum
E. Candida species may be associated with granuloma gluteale infantum
►A

Candida species are part of the resident flora but they are also the most common cause of
opportunistic mycotic infection. Cutaneous clinical manifestations include thrush, perleche,
paronychia, onychomycosis, intertrigo, and folliculitis. Candida albicans is the most common
organism; however, C. dubliniensis is often implicated in mucosal disease in AIDS patients. C.
parapsilosis causes chronic paronychia and C. glabrata is fluconazole resistant. Infection is most
common in the extreme of age, and the mucosal disease is prevalent in HIV patients. Factors
predisposing patients to infection include impaired mucocutaneous barrier function,
immunodeficiencies, broad spectrum antibiotic use, malignancies, heat, humidity, friction,
diabetes, and indwelling catheters. Groin infections often involve the scrotum unlike tinea cruris
caused by dermatophytes. On KOH examination, budding yeast and pseudohyphae (not true
hyphae) are diagnostic. Predisposing factors for granuloma gluteale infantum include occlusion,
topical corticosteroids, and possibly Candida diaper dermatitis.

8
Erythematous to violaceous papules that may progress to nodular and necrotizing skin lesions in
neutropenic patients may be caused by which of the following organisms?
A. Trichosporon asahii
B. Trichosporon beigelii
C. Trichosporon ovoides
D. Trichosporon inkin
E. Trichophyton rubrum
►A

630
Trichosporon asahii can cause trichosporanosis, which presents with disseminated disease in
neutropenic patients. Patients may present with erythematous to violaceous papules that may
progress to nodular and necrotizing skin lesions. Trichosporon beigelii (new nomenclature:
Trichosporon ovoides and Trichosporon inkin) causes white piedra and other superficial
infections.

9
A biopsy shows broad-based budding thick walled yeast cells, 10-15 um with a double contoured
appearance.
A. This yeast has a yeast phase at room temperature
B. Does not grow at 37º C
C. Usually produces a severe characteristic pulmonary disease
D. May be found in dogs
E. Is transmitted by mosquitoes
►D

This biopsy describes Blastomyces dermatitidis which generally can be found in decaying
vegetation but can be carried by dogs.

10
Which of the following is true regarding Dermatophyte Test Media (DTM)?
A. DTM contains chlortetracycline and minocycline
B. Alizarin red is the indicator present in DTM
C. Non-dermatophytes cause the media to turn yellow due to acid byproducts
D. DTM is useful for culturing dermatophytes from skin and nails, but not hair.
E. Dermatophytes utilize glucose as a carbon source, producing alkaline by-
products.
►C

Dermatophyte Test Media (DTM) contains peptones, dextrose, gentamicin, chlortetracycline,


cycloheximide, and phenol red. Dermatophytes utilize protein as a carbon source producing
alkaline byproducts causing the media to turn from amber to red. Nondermatophytes cause the
media to turn yellow due to acid byproducts.

11
The most common eumycotic organism in the US produces which color grains in its
microcolony?
A. White
B. Black
C. Red

631
D. Yellow
E. Green
►A

The most common cause of fungal (eumycotic) eumycetoma in the US is Pseudallescheria


boydii. This and Acremonium produce white colonies. Black colonies are caused by Exophilia,
Madurella, and Curvalaria. Red colonies are created by Actinomadura pelletieri. Yellow colonies
are made by Streptomyces and Nocardia, both causes of bacterial mycetoma.

12
All of the following statements about paracoccidioidomycosis are true EXCEPT?
A. It is most common in male agricultural workers
B. It has a characteristic ―mariner‘s wheel‖ appearance on histopathology
C. Disease is almost always confined to the skin
D. It is endemic to Brazil
E. It may cause mucocutaneous lesions
►C

Paracoccidioidomycosis is caused by Paracoccidioidomycosis brasiliensis, and is endemic to


Central and South America, especially Brazil, Argentina, Venezuela, Ecuador and Colombia.
Male agricultural workers are at greatest risk. Infection is most commonly caused by direct
inoculation which leads to pulmonary disease. Lung disease may be followed by dissemination
to mucocutaneous surfaces, gastrointestinal tract, spleen, adrenal glands, and lymph nodes.
Cutaneous lesions are verrucous and/or ulcerative, and usually found on the face and in the nasal
and oral mucosa. Primary mucocutaneous disease exists as well and is caused by direct
inoculation. Biopsy specimens display multiple narrow-based budding yeast cells, described as a
―mariner‘s wheel.‖ The treatment of choice is itraconazole.

13
An elderly lady with moccasin-type tinea pedis has a fungal culture which demonstrates smooth,
teardrop-shaped microconidia which produce a ―port-wine‖ pigment. The organism is:
A. Trichophyton verrucosum
B. Microsporum canus
C. Trichophyton rubrum
D. Microsporum gypseum
E. Trichophyton mentagrophytes
►C

Trichophyton rubrum is an anthropophilic dermatophyte that is a cause of T. pedis, T. manum, T.


corporis, T. cruris, onychomycosis, Majocchi‖s granuloma, and rarely T. capitis. Colonies appear

632
as fluffy to granular white to cream colored with reverse non-diffusible port-wine or red
pigment. Macroconidia are rare, thin-walled pencil shaped. Microconidia are delicate teardrop
shaped.

14
A pet store owner comes into your office with pruritic scaly plaques on his arms. A fungal
culture demonstrates a yellow colony with spindle-shaped macroconidia which grows on
polished rice grains. The organism is:
A. Microsporum canis
B. Epidermophyton floccosum
C. Trichophyton rubrum
D. Microsporum gypseum
E. Trichophyton tonsurans
►A

Microsporum canis os a zoophilic dermatophyte, which causes ectothrix invasion when infecting
the hair. Colonies of M. canis are flat, spreading, white to cream-colored with a dense cottony
surface, The reverse of the plate is a canary yellow. Wet mount of the fungus shows spindle-
shaped macroconidia with 5-15 cells and often have a terminal knob.

15
The most reliable method for distinguishing between Trichophyton rubrum and T.
mentagrophytes is:
A. Morphology of microconidia
B. Morphology of macroconidia
C. Pigmentation studies
D. Hair perforation test
E. Colony morphology
►D

The diagnostic morphology of the Trichophytons overlap and may be difficult to differentiate. T.
mentagrophytes produces a positive hair perforation test (wedges in the test hair).

16
If the diagnosis is phaeohyphomycosis, what is the organism 1. Alternaria sp. 2. Bipolaris sp. 3.
Fusarium sp. 4. Exophiala sp.
A. 1,2,3
B. 2,3,4
C. 1,2,4
D. 1,3,4

633
E. All of these answers are correct
►C

All these organisms are dematiaceous, Fusarium is a hyalohyphomycete.

17
Which of the following is TRUE regarding coccidiomycosis?
A. First line treatment in pregnancy is itraconazole
B. Droplet transmission is the most common method of acquisition of disease
C. Southeast Asians are at a higher risk of disseminated disease
D. Erythema nodosum is associated with a poor prognosis
E. Approximately 75% of those contracting the disease will be symptomatic
►C

Southeast Asians and African Americans are at a higher risk for disseminated cocci, as are
pregnant women and the immunocompromised. Approximately 60% of those contracting
coccidiomycosis are asymptomatic, and the disease is infectious not contagious. The most
common method of disease acquisition is inhalation from the soil during natural events (dust
storms etc). Erythema nodosum is associated with a good prognosis and the first-line treatment
during pregnancy is amphotericin B. (JAAD 2006 CME)

18
A fungal culture demonstrates a suedelike cream-colored colony of teardrop and balloon-shaped
microconidia which produce a red-brown pigment. Which of the following is true of this
organism?
A. Causes ectothrix infection
B. Requires partial thiamine for growth
C. Does not have arthroconidia
D. Causes fluorescent hair infection
E. Is not a cause of tinea unguim
►B

Trichophyton tonsurans is an anthropophilic dermatophyte that causes non-fluorescent endothrix


hair invasion. It can cause black dot tinea capitis, tinea corporis, tinea pedis, and tinea unguium.
An important identifying feature of T. tonsurans is its red-brown pigment that diffuses into the
medium. Also, abundant tear-drop or club shaped microconidia can be found. It grows best in the
presence of thiamine.

19
This organism is often considered a contaminant but has been reported to cause onychomycosis:

634
A. Sepedonium
B. Curvularia
C. Scopulariopsis
D. Penicillium
E. Phialophora
►C

Causes white superficial onychomycosis.

20
Which of the following statements regarding histoplasmosis is TRUE?
A. Terbinafine is the treatment of choice
B. Histoplasma capsulatum is a yeast at 25°C
C. The organisms are intracellular on histological examination
D. The disease is most common in the San Joaquin Valley
E. The disease is primarily a mucocutaneous infection
►C

Histoplasmosis is primarily a pulmonary infection that can disseminate to other organs. It is


endemic to the Ohio, Missouri and Mississippi River Valleys where the bird and bat droppings in
the soil contain the fungus. Primary cutaneous disease is extremely rare. At 25°C it displays
septate hyphae and at 37°C H. capsulatum is a yeast. Itraconazole is the treatment of choice. On
histopathological examination, intracellular organisms that display a halo are seen. The halo is
not the result of a capsule but rather shrinkage artifact.

21
All of the following are common causes of chromoblastomycosis EXCEPT:
A. Cladosporium carrionii
B. Rhinocladiella aquaspera
C. Phialophora verrucosa
D. Pseudallescheria boydii
E. Fonsecaea pedrosi
►D

Chromoblastomycosis is a chronic fungal infection of the skin and the subcutaneous tissue
caused by traumatic inoculation of a specific group of dematiaceous (pigmented) fungi.
Fonsecaea pedrosoi is the most common causative organism, but Fonecaea compacta,
Rhinocladiella aquaspersa, Phialophora verrucosa, Exophilia jeanselmei and Cladosporium
carrionii are pathogenic as well. It is found most commonly in agricultural workers in the tropics
and subtropics, and it is notoriously resistant to therapy. It presents as verrucous papules and

635
plaques that may coalesce. Histopathological findings include brown, thick-walled cells known
described as ―copper pennies.‖ Early in its course, limited disease may respond to surgical
excision, electrodessication or cryosurgery. More extensive lesions may require systemic
antifungal agents including itraconazole or terbinafine, which are the treatments of choice.

22
A 37 year old woman from New Mexico, now 30 weeks pregnant, presents with flu-like
symptoms. Chest x-ray revealed diffuse miliary infiltrates, with blood cultures growing
Coccidioidis immitis. What is the treatment of choice?
A. Amphotericin B
B. Terbinafine
C. Itraconazole
D. Griseofulvin
E. No therapy
►A

There is an increased risk of dissemination of Coccidiomycosis in pregnant women, especially


during the third trimester and the post-partum period. In a review by Crum et al, maternal demise
correlated with disease diagnosed later in pregnancy, with only 45% of patients diagnosed in the
third trimester surviving (Am J Medicine 2006;119(11):Pages 993.e11-993.e17). Given that
azoles have been shown to cause teratogenicity, amphotericin B is recommended as the therapy
in pregnant women.

23
Which of the following organisms causes favus?
A. Trichophyton schoenleinii
B. Trichophyton mentagrophytes
C. Microsporum canis
D. Trichophyton rubrum
E. Microsporum distortum
►A

Favus is a chronic dermatophyte infection defined by the presence of yellowish crusts in the hair
follicles called scutula. Prolonged infections lead to cicatricial alopecias of the scalp and
glabrous skin. The infection is most commonly caused by Trichophyton schoenleinii.

24
Which of the following causes "black dot ringworm"?
A. M. canis
B. T. violaceum

636
C. T. verrucosum
D. M. gypseum
E. M. auddouinii
►B

"Black dot ringworm" is endothrix tinea capitis. Black dots are remnants of brittle hair broken at
the surface of the scalp (cuticle intact). On KOH prep, spores are seen within the hair shaft.
Causes are T. rubrum, T. gourvilli, T. yaounde, T. tonsurans, T. schoeleinii, T. soudanense, and
T. violaceum ("Ringo Gave Yoko Two Squeaky Violins."). T. verrucosum causes nonfluorescent
ectothrix tinea capitis. M canis, gypseum, and auddouinii also cause ectothrix tinea capitis.

25
Which one of the following is not caused by a Candida infection?
A. Angular cheilitis
B. Balanitis
C. Median rhomboid glossitis
D. Leukoplakia
E. Mycetoma
►E

Candidal infections may clinically present as thrush/leukoplakia, perleche/angular cheilitis,


vulvovaginitis, balanitis, paronychia, onychomcosis, intertrigo, and folliculititis. Median
rhomboid glossitis is now also associated with candidal infections. Mycetomas are caused by
true fungi (eumycetoma) or filamentous bacteria (actinmycetoma).

26
A patient with a pulmonary infection and cervical adenopathy has vesicles, papules and
ulcerations on the oral and nasal mucosa. Tissue culture reveals yeast with multiple buds giving a
―mariner‖s wheel‖ appearance. The organism is:
A. Penicillium marneffei
B. Coccidioides immitis
C. Paracoccidioides brasiliensis
D. Blastomyces dermatitidis
E. Leishmania mexicana
►C

Paracoccidioides brasiliensis causes a chronic progressive infection that is most commonly seen
in rural areas of South America. Patients may have constitutional symptoms, lung involvement,
ulcers of the upper respiratory and digestive tract, and verrucous/ulcerated cutaneous lesions.
Classically, Paracoccidioides brasiliensis has a ―mariner‖s wheel‖ appearance.

637
27
This 16 year-old patient was recently diagnosed with HIV, the diagnosis is:
A. Distal Onychomycosis
B. Proximal Subungual Onychomycosis
C. Proximal White Subungual Onychomycosis
D. White Superficial Onychomycosis
E. Paronychia with Candida Onychomycosis
►C

Proximal White Subungual onychomycosis is an AIDS marker, nails have a characteristic white
opaque appearance beginning in the region of the lunula and extending distally under the nail
plate.

28
The etiologic agent responsible for White Piedra is:
A. Candida albicans
B. Pityrosporum obiculare
C. Corynebacterium tenuis
D. Piedra hortai
E. Trichosporon ovoides
►E

Formerly known as T. beigelii.

29
Which of the following statments regarding pseudohyphae is incorrect?
A. Pseudohyphae are seen in yeasts
B. Are constricted at septations
C. Branching occurs at septations
D. The terminal cell is smaller than the others
E. Are not septated
►E

Pseudohyphae are seen in yeast and resemble true hyphae except that they are constricted at
septations, branching occurs at septations, and the terminal cell is smaller than the others. They
are septated.

30

638
Culturing T. rubrum from a white opacity on the fingernail plate should prompt testing for
what?
A. Diabetes mellitus
B. Hypothyroidism
C. Hyperthyroidism
D. HIV
E. Cirrhosis
►D

White superficial onychomycosis of the fingernails is a marker for immunosuppresion and


should prompt testing for HIV. Generally, T. mentag is the most common cause of white
superfical onychomycosis. However, in immunosuppressed patients, T. rubrum is the more
common dermatophyte.

31
A creamy white colony might be any of these organisms except:
A. Candida albicans
B. Prototheca wickerhamii
C. Curvularia
D. Sporothrix schenckii at 37º C
E. Cryptococcus neoformans
►C

Is a dematiaceous organism and is the only organism listed that does not produce creamy white
colonies.

32
White piedra is caused by:
A. Trichophyton cutaneum
B. Trichophyton inkin
C. Trichophyton asahii
D. Trichophyton ovoides
E. Trichophyton mucoides
►A

Patients with white piedra are infected with Trichophyton cutaneum also known as Trichophyton
beigelii. The treatment should be oral azole antifungals and shampoos without shaving the scalp.

33

639
This organism does not produce microconidia. The macroconidia are club shaped, and smooth
walled, they grow singly or in clusters.
A. Trichophyton rubrum
B. Epidermophyton floccosum
C. Microsporum gypseum
D. Microsporum canis
E. Trichophyton mentagrophytes
►B

This is the only organism in this list that fits this description.

34
Septate true hyphae are characterized by:
A. Transverse cross walls forming within the hyphae
B. Discrete unicellular bodies
C. Constrictions at septations
D. Branching occuring at septations
E. A terminal cell that is smaller than the others
►A

Hyphae are vegetative tube-like structures. In septate hyphae, transverse cross walls form within
the hyphae. Pseudohyphae, seen in yeast, resemble true hyphae except that they are constricted at
septations, branching occurs at septations, and the terminal cell is smaller than the others.

35
Which of the following is most likely to be a primary cutaneous infection?
A. North American Blastomycosis
B. Histoplasmosis
C. Paracoccidioidomycosis
D. Cryptococcosis
E. None
►A

All other mycoses are primary pulmonary infections, which may disseminate to the skin.

36
A 45 year-old agricultural worker from Brazil presented with ulcers of the buccal mucosa and
tongue. Cervical lymph nodes were tender and enlarged. The biopsy would most likely reveal:
A. Yeast cells in chains and a large thick walled round central yeast cell surrounded
by several thinly attached budding smaller yeast cells

640
B. Yeast cells with large capsules
C. Small budding yeast cells
D. Yeast cells with pseudohyphae
E. Copper pennies
►A

Yeast cells in chains and a large thick walled round central yeast cell surrounded by several
thinly attached budding smaller yeast cells p.18. This biopsy is describing Paracoccidioides
brasiliensis, an organism endemic in Brazil. Its yeast form seen in tissue produces a Mariners
wheel-like configuration. Generally this organism is inhaled and disseminates causing
mucocutaneous lesions with lymphadenopathy.

37
A patient with white nodules on the hair shaft has a KOH which shows hyphae and
arthroconidia. The etiology is:
A. Corynebacterium minutissimum
B. Exophiala wernickii
C. Piedraia hortae
D. Trichosporon beigelii
E. Corynebacterium tenuis
►D

White piedra is an infection of the hair shaft caused by Trichosporon beigelii. Unlike black
piedra which is firmly adherent to the shaft, white piedra presents as light brown nodules
composed of hyphae and arthroconidia, which move easily along the hair shaft. The most
common sites of infection occur on the mustache, beard and pubic area.

38
The most sensitive microscopic test for fungal infection is:
A. Potassium Hydroxide
B. Potassium Hydroxide with DMSO
C. Chlorazol Black E
D. Calcofluor white
E. Swartz Lamkins stain
►D

Calcofluor white is the most sensitive microscopic test for fungal infection. It is a glucan specific
immunofluorescent stain. The remaining options are useful in direct microscopic examination,
but not the most sensitive.

641
39
A patient with scaly feet has a positive KOH. Fungal culture reveals smooth, club-shaped
macroconidia attached to hyphae in groups. No microconidida are seen. The organism is:
A. Microsporum canis
B. Epidermophyton floccosum
C. Trichophyton rubrum
D. Microsporum gypseum
E. Trichophyton tonsurans
►B

The wet mount Epidermophyton floccosum shows smooth, thin-walled macroconidia without
microconidia. The appearance is occasionally referred to as ―snow shoes‖. Epidermophyton
floccosum is incapable of hair invasion, and therefore, does not cause tinea capitis.

40
A whitish, heaped and convoluted colony with growth submerged into the agar and a colorless
reverse was isolated from the scalp of a 35-year-old male. The organism did not produce any
conidia. The diagnosis most likely is:
A. Microsporum canis
B. Microsporum ferrugineum
C. Trichophyton rubrum
D. Trichophyton tonsurans
E. Trichophyton schoenleinii
►E

Produces a white cerebriform colony lacking any conidia. M. ferrugineum produces a rust
colored colony with bamboo-like hyphae without conidia. The other organisms produce micro
and macroconidia.

41
A horticulturist of sphagnum moss topiaries comes in with a nodular eruption with lymphangitic
spread and treatment with oral potassium iodide is initiated. What is the most well recognized
side effect of this treatment.
A. Gastrointestinal distress
B. Shortness of breath
C. Flushing
D. Angioedema
E. Pruritus
►A

642
This patient has sporotrichosis. Sporotrichosis is mainly an occupational disease of farmers,
gardeners, and horticulturists. Persons who handle thorny plants, sphagnum moss, or baled hay
are at increased risk. Outbreaks have occurred in nursery workers who handled sphagnum moss,
rose gardeners, children playing on baled hay, and greenhouse workers who handled bayberry
thorns contaminated by the fungus. Classic treatment is with oral potassium iodide for 3-4
weeks. The most recognised side effect of treatment is gastrointestinal distress. Thyroid function
tests should be performed during treatment as suppression can occur.

42
A slimy, mucoid colony growing on Sabouraud's agar at 37 degrees Celsius is observed to
produce urease. The organism is:
A. Candida albicans
B. Cryptococcus neoformans
C. Candida glabrata
D. Candida tropicalis
E. Aspergilus flavus
►B

Cryptococcus neoformans is an encapsulated yeast with a worldwide distribution.


Cryptococcosis is usually acquired by inhalation and subsequent dissemination to various organs
including the meninges and the skin can occur. Immunosuppressed patients are particularly
susceptible to infection. In approximately ten percent of disseminated cases, cutaneous lesions
may develop. These present as acneiform papules or pustules and may progress to infiltrated
plaques, nodules, or ulcers. Less commonly, cutaneous lesions can represent primary cutaneous
cryptococcosis via direct inoculation of organisms into the skin. Diagnosis is made by direct
microscopy and India ink stains can aid visualization. The organisms are large, encapsulated
budding yeasts. In histopathologic sections, capsules stain with alcian blue and mucicarmine
stains. In culture, C. neoformans is distinguished by production of urease as well as the ability to
pigment on Guizotia seed medium. Colonies are described as slimy and mucoid. Serologic
antigen-detection assays are also available. Treatment of disseminated cryptococcosis includes
amphotericin combined with flucytosine.

43
Which of the following statements regarding lobomycosis is FALSE?
A. Itraconazole is the treatment of choice
B. It resembles a ―chain of coins‖ on histopathology
C. It is also known as ―keloidal blastomycosis‖
D. Lacazia (formerly Loboa) loboi is the number one cause
E. The infection also occurs in dolphins
►A

643
Lobomycosis, also know as keloidal blastomycosis is caused by Lacazia loboi (formerly Loboa
loboi). It is endemic to Brazil and the Caribbean and is associated with dolphins. Clinically, the
disease presents with painless keloidal papules and plaques, ulcerative lesions and/or verrucous
lesions. Biopsy specimens display multiple budding thick-walled cells attached with a bridge,
often referred to as a ―chain of coins.‖ Surgical treatment is necessary as antifungal medications
are ineffective.

44
Medlar bodies are diagnostic of infection with which organism?
A. Blastomycosis
B. Chromomycosis
C. Coccidiomycosis
D. Histoplasmosis
E. Sporotrichosis
►B

Chromoblastomycosis, or chromomycosis, is a cutaneous mycosis caused by dematiaceous, or


pigmented, fungi. Several fungal species are associated with this infection including Phialophora
verrucosa, Fonsecaea pedrosoi, F compactum, Exophiala (Fonsecaea, Wangiella) dermatitidis
and Cladosporium carrionii. Infection is typically trauma-induced and involves the lower
extremities. Lesions appear as verrucous papules, nodules and plaques, with occasional
elephantiasis resulting from lymphatic blockage. Histopathologic findings include
pseudoepitheliomatous epidermal hyperplasia and a dermal infiltrate composed of epithelioid
histiocytes, multinucleated giant cells, and small clusters of inflammatory cells including plasma
cells, neutrophils, eosinophils and lymphocytes. The characteristic histopathologic feature is the
presence of dark brown, thick-walled, ovoid spheres in clusters or chains referred to as "copper
pennies" or Medlar bodies. They are visible without the use of special stains.

45
Which of the following is FALSE with regards to asexual reproduction of fungi?
A. Arthroconidia are formed by the fragmentation of hyphae
B. Sporangia are spores that are produced in a sac
C. Chlamydoconidia are thin-walled and are susceptible to environmental injury
D. Conidia are cells produced on the sides or ends of hyphae
E. Dematophytes produce conidia
►C

The structures formed during asexual propagation of fungi are termed either spores or conidia
depending on their mode of production and they arise following mitosis of a parent nucleus.

644
Conidia arise either by budding off conida-producing hyphae (on the sides or ends) or by
differentiation of preformed hyphae. The size and shape are generally characteristic of the
organism. Dermatophytes produce conidia. Arthroconidia are produced when hyphae fragment
or lyse. They may appear as thick- or thin-walled cells and are typically larger than the hyphae
from which they came. The separation occurs at the septae. The mold form of Coccidioides
immitis reproduce in such a fashion. Sporangia are sacs or cases that are multicellular structures
in which spores are produced. Zygomycetes reproduce in this way. Chlamydoconidia are thick-
walled, round conidia that are formed during unpleasant environmental conditions. Trichophyton
tonsurans produces these structures.

46
This dermatophyte has a growth requirement for inositol and thiamine
A. Trichophyton equinum
B. Trichophyton violaceum
C. Trichophyton concentricum
D. Trichophyton tonsurans
E. Trichophyton verrucosum
►E

Trichophyton verrucosum is a zoophilic fungus that requires thiamine and sometimes inositol for
growth. T. violaceum and tonsurans only have a partial requirement for thiamine. Trichophyton
equinum requires niacin. (Horses are nice).

47
Which of the following is not true regarding fungal culture media containing cycloheximide?
A. Cycloheximide is not found in Sabouraud Dextrose Agar (SDA) Emmons
Modification
B. Cycloheximide is found in Mycosel
C. Cycloheximide is found in Dermatophyte Test Media
D. Cycloheximide is found in Mycobiotic
E. Cycloheximide inhibits bacterial flora
►E

Sabouraud Dextrose Agar (SDA) Emmons Modification does not contain cycloheximide.
Mycosel/Mycobiotic (SDA with cycloheximide and chloramphenicol) and Dermatophyte Test
Media contain cycloheximide. Cycloheximide inhibits rapidly growing nonpathogenic molds and
some pathogens (Cryptococcus neoformans, some Candida species, Prototheca, Scytalidium
species, yeast forms of Histoplasma and Blastomyces). Chloramphenicol inhibits bacterial flora.

48

645
Which structure is found in a biopsy of Candida tropicalis, it is branching and pinching in at the
points of septations:
A. Septate hyphae
B. Arthroconidia
C. Mosaic fungus
D. Pseudohyphae
E. Pectinate bodies
►D

Pseudohyphae are products of yeast, which produce elongated yeast cells that do not pinch off,
branching and pinching in at the points of septations.

49
A 30 year-old male living in the Chicago suburbs complained of a slowly growing verrucous
plaque with sharp borders on his left wrist. A biopsy revealed yeast cells 10-14 um. Mucicarmine
was negative. A fluffy white colony grew at room temperature having small round conidia on
thin conidiophores. The diagnosis is:
A. Blastomycosis
B. Cryptococcosis
C. Candidiasis
D. South American Blastomycosis
E. Histoplasmosis
►A

The biopsy describes Blastomyces dermatitidis, the negative mucicarmine rules out
Cryptococcus. Chicago is an endemic area for Blastomyces. The colony morphology also
describes Blastomyces.

50
All of the following statements regarding superficial mycotic infections are true EXCEPT:
A. Trichosporon species can cause systemic disease in immunocompromised
patients
B. Malassezia furfur is the most common cause of tinea versicolor
C. Trichosporon beigelii causes white piedra
D. Malassezia species may be associated with neonatal cephalic pustulosis
E. Piedraia hortae causes black piedra
►B

Trichosporon asahii can cause disseminated disease in immunocompromised patients, especially


those with neutropenia. Patients develop nodular and then necrotic lesions and require systemic

646
antifungal agents. Malassezia globosa is the most common cause of tinea versicolor, not M.
furfur. M. pachydermatis, M. dermatis, M. obtuse, M. restrica, M. sympodialis and M. slooffiae
are all thought to be medically important as well. M.sympodialis is part of the skin‘s normal
flora. Trichosporon beigelii and Piedraia hortae are the etiological agents of white and black
piedra respectively. Several Malassezia species have been implicated in neonatal cephalic
pustulosis (‗neonatal acne‘).

51
A 6 year-old Hispanic girl came into the clinic with a history of alopecia and scaly scalp for 3
weeks. The Wood's lamp examination was negative and her cervical lymph nodes were not
enlarged. It was noted that the hairs were broken off at the surface of the scalp. The most likely
organism to be isolated would be:
A. M. audouinii
B. M. canis
C. M. gypseum
D. T. tonsurans
E. T. verrucosum
►D

T. tonsurans and T. violaceum cause the endothrix infection "Black Dot Ringworm". All other
choices cause Grey Patch Ringworm.

52
A solitary chancre-like lesion appeared on the arm of a florist who has a pet dog. No organisms
were seen on biopsy, however Sporothrix schenckii was cultured from the tissue. This infection
is known as ―fixed cutaneous sporotrichosis‖. It remains fixed because:
A. There is a resistance due to a prior exposure
B. There is a decreased resistance due to AIDS
C. The infection was acquired from the dog rather than from a plant
D. The infection occurred in the absence of tissue injury
E. This species of Sporothrix has limited infectivity
►A

There is a resistance due to a prior exposure.

53
Which of the following is the most useful morphologic feature in identifying the mycelial phase
of Histoplasma capsulatum?
A. Arthroconidia in every other cell
B. Encapsulated spores 2-5 um

647
C. Tuberculate macroconidia 8-14 um
D. Small oval conidia on long thin conidiophores
E. Microconidia laterally along the hyphae strand
►C

On artificial media the mould form of Histoplasmosis produces hyphae with tear drop
microconidia and round thick wall tuberculate macroconidia.

54
Nondermatophytes growing on Dermatophyte Test Media cause the media to turn what color?
A. Amber
B. Red
C. Yellow
D. Black
E. Green
►C

Dermatophytes utilize protein as a carbon source producing alkaline by-products causing


Dermatophyte Test Media to turn from amber to red. Nondermatophytes cause the media to turn
yellow due to acid by-products.

55
A patient has a positive Wood's light exam that is caused by pteridine. What is the causative
condition?
A. Microsporum ferrugineum
B. Corynebacterium
C. Trichophyton violaceum
D. Pseudomonas
E. Trichophyton tonsurans
►A

Wood's light positive dermatophytes (M. canis, M. audouinii, M. distortum, M. ferrugineum, M


gypseum, and T. schoenleinii) fluoresce secondary to pteridine. Corynebacterium can fluoresce
as well due to coproporphyrin III. Pseudomonas fluoresces from production of pycyanin. The
other organisms do not react with a Wood's light.

56
A young girl presented with a scaly annular facial rash and alopecia of her lower eyelashes.
Which of the following statements is/are true? 1. A KOH prep and fungal culture might confirm
the diagnosis. 2. The etiologic agent might be Microsporum canis. 3. The family puppy might be

648
infected. 4. Griseofulvin would be the drug of choice. 5. A topical azole cream would be the drug
of choice.
A. 1,2,4
B. 1,2,3,4
C. 1,3,4
D. 1,2,5
E. 1,2,3,5
►B

KOH and culture are the best and most reliable lab tests to diagnose a fungal infection. M. canis
is frequently found in kittens and puppies and is a common cause of tinea faciei in children. An
oral antifungal should be prescribed when a fungal infection involves hair.

57
Growth of which of the following is not inhibited on Mycosel media?
A. Cryptococcus neoformans
B. Yeast forms of Histoplasma
C. Yeast forms of Blastomyces
D. Microsporum gypseum
E. Scytalidium species
►D

Cycloheximide in Mycosel or Mycobiotic media (SDA with cycloheximide and


chloramphenicol) inhibits rapidly growing nonpathogenic molds and some pathogens
(Cryptococcus neoformans, some Candida species, Prototheca, Scytalidium species, yeast forms
of Histoplasma and Blastomyces).

58
Which of the following most accurately describe conidia?
A. Cells produced on the end or sides of hypha or conidiophore
B. Spores that are produced in a sac
C. Thick-walled round cell
D. Formed by budding
E. Formed by fragmentation of hyphae
►A

Conidia are cells produced on the end or sides of hypha or conidiophore, the size and shape
arrangement are generally characteristic of the organism. The remaining descriptions are of
specific types of conidia. Spores produced in a sac are sporangia, thick-walled round cells that

649
are resistant to the environment are chlamydoconidia, blastoconidia are formed by budding and
arthroconidia are formed by fragmentation of hyphae.

59
An organism producing an apricot colored colony and reflexive branching with few conidia was
cultured from a patient from Africa. Which of the following is/are true? 1. This mostly likely is
T. verrucosum 2. This most likely is T. soudanense 3. This most likely is T. violaceum 4. This
organism causes endothrix tinea capitis 5. This organism is zoophilic
A. 1,5
B. 2,4
C. 2,4,5
D. 3, 4
E. 3, 4, 5
►B

T. soudanense produces an apricot colored colony with reflexive branching. It is an


anthropophilic organism causing an endothrix tinea capitis. T. verrucosum produces a white
colony and requires thiamine and inositol for sporulation, T. violaceum produces a purple colony
and has a partial requirement for thiamine

60
Yeast:
A. Are filamentous fungi
B. Are characterized by tubular branching cells
C. Form fuzzy colonies
D. Form smooth colonies
E. Are unicellular oval to round cells that reproduce by budding or fission
►E

All of the listed features are those of molds. Yeast are unicellular, oval to round cells that
reproduce by budding or fission. They form moist colonies, not smooth or fuzzy colonies.

61
Clinically, actinomycotic mycetoma and eumycotic mycetoma appear identical. The importance
of identifying the etiologic agent is
A. Academic
B. Selecting the appropriate therapy
C. Ordering appropriate stains
D. Determine if the infection is contagious
E. Determine if amputation is necessary

650
►B

The etiologic agents of actinomycotic mycetoma are filamentous bacteria that require antibiotics,
whereas fungal agents causing eumycotic mycetoma require antifungal therapy.

62
Which of the following organisms will cause infections of skin, nails and endothrix hair?
A. Epidermophyton floccosum
B. Microsporum audouinii
C. Trichophyton schoenlinii
D. Microsporum canis
E. Trichophyton mentagrophytes
►C

E. floccosum does not infect hair; M. audouinii and M. canis cause fluorescent ectothrix T.
capitis, T. mentagrophytes ectothrix T. capitis. T.Schoenlinii can cause endothrix or ectothrix,
fluoresces blue white. It causese favus with hyphae with air spaces in them.

63
Which of the following statements about arthroconidia is correct?
A. Arthroconidia are formed by budding
B. Arthroconidia are formed by fragmentation of hyphae
C. Arthroconidia are thick-walled round cells
D. Arthroconidia are spores that are produced in a sac
E. Arthroconidia are yeast forms of dimorphic fungi
►B

Arthroconidia are formed by fragmentation of hyphae and may appear as thick or thin walled
rectangular cells. An example would be the mould form of Coccidioides immitis. The remaining
options are incorrect.

64
Numerous umbilicated nodules resembling molluscum contagiosum developed in a patient with
recently diagnosed HIV infection. Which of the following fungi might be the causative agent(s)?
1. Candida tropicalis 2. Cryptococcus neoformans 3. Aspergillus fumigatus 4. Histoplasma
capsulatum 5. Penicillium marneffei
A. 2
B. 3
C. 2, 4
D. 4, 5

651
E. 2, 4, 5
►E

All three organisms are seen in AIDS patients and can produce molluscum-like lesions.

65
Which actinomycotic organism has red grains?
A. Streptomyces somaliensis
B. Nocardia asteroides
C. Actinomadura madurae
D. Actinomadura pelletieri
E. Nocardia brasiliensis
►D

Mycetoma is a granulomatous infection of dermal and subcutaneous tissues usually occurring on


the foot. Draining sinuses containing grains characterize such infections. Three types of
mycetoma exist: eumycotic (true fungal), actinomycotic (filamentous organisms) and
botryomycotic (bacterial infections). Madurella and Leptosphaeria species produce black grains.
Pseudallescheria and Acremonium species, along with dermatophytes, make white grains.
Nocardia brasiliensis and N. asteroides produce white grains, while Nocardia caviae and
Actinomyces israelii tend to have yellow-white grains. Actinomadura madurae produce pink or
white grains and Actinomadura pelleteri make red grains. Streptomyces somaliensis produce
brown or yellow grains.

66
Which of the following statements regarding dermatophyte infection FALSE?
A. Trichophyton schoenleinii is a common cause of favus
B. Microsporum canis is associated with ectothrix tinea capitis
C. Trichophyton violaceum is normally associated with endothrix tinea capitis
D. Microsporum audouinii displays yellow fluorescence with Wood‘s lamp
examination
E. Trichophyton rubrum is always an ectothrix infection
►E

Organisms that cause an ectothrix pattern of tinea capitis include M. canis, M. audouinii, M.
ferruginosum, M. distortium, M. gypseum and occasionally T. rubrum. T. tonsurans, T.
violaceum, T. soudanense, T. gourvilli, T. yaoundei, and occasionally T. rubrum cause an
endothrix pattern. T. schoenleinii causes favus in which hyphae and air spaces are seen in the
hair shaft. A bluish-white fluorescence pattern is seen with Wood‘s lamp. Clinically, patients
have thick, yellow, cup-shaped crusts (scutula); scarring and secondary infection may result. M.

652
canis, M. audouinii, M. ferruginosum, M. distortium display a yellow fluorescence on Wood‘s
lamp examination.

67
Penicillium marneffei is an infection endemic to which part of the world?
A. Mexico
B. South America
C. Africa
D. Southeast Asia
E. The Caribbean
►D

Penicillosis is a recently recognized disease caused by infection with Penicillium marneffei, a


dimorphous fungus that is contracted through inhalation. Cases have been described in Vietnam,
China, and Thailand as well as imported cases in the U.S. in veterans returning from Vietnam.
Bamboo rats in these areas have been shown to be infected with P. marneffei and may represent
a reservoir. Although penicillosis has a tendency to occur in immunocompromised hosts
opportunistically, it has also been described in normal hosts. Clinical presentation is typically
with pulmonary or disseminated disease which can affect the skin, gastrointestinal tract, spleen,
lymph nodes, skin, and bone marrow. Cutaneous features include multiple umbilicated papules
which can enlarge and ulcerate occurring commonly on the face and trunk. Diagnosis is by
culture or histopathology. Treatment is with itraconazole. Severe cases may require amphotericin
B.

68
Trichosporon ovoides is a cause of:
A. Black piedra
B. White piedra
C. Tinea nigra palmaris
D. Ectothrix tinea capitis
E. Endothrix tinea capitis
►B

White piedra is cause by Trichosporon beigelii, or Trichosporon ovoides and T. inkin (new
nomenclature). It presents with tan to white soft, nonadherent small concretions ~1mm, seen on
the scalp, beard, moustache, and pubic areas. The hairs may fluoresce.

69
Which one of the following agents accounts for the depigmentation seen in pityriasis versicolor?
A. Thymidine kinase

653
B. Ketoconazole
C. Acetone
D. Dicarboxylic acid
E. Postinflammatory effect
►D

Pityriasis versicolor is primarily caused by the yeast M. furfur. M. furfur is a dimorphic,


lipophilic organism that can produce metabolites such as azaleic acid (a dicarboxylic acid) that
can inhibit tyrosinase and injure melanocytes.

70
What is the most likely cause of this infection?
A. T. tonsurans
B. T. schoenleinii
C. M. canis
D. T. mentag
E. T. rubrum
►C

M. canis and T. tonsurans both cause tinea capitis. M. canis is more commonly associated with
inflammatory tinea capitis and kerion formation.

71
Tinea imbricata is most frequently caused by:
A. T. mentagrophytes
B. M. Audouinii
C. T. concentricum
D. E. floccosum
E. M. furfur
►C

Tinea imbricata or Tokelau, an unusual type of tinea corporis causing polycyclic scaly lesions, is
endemic in the South Pacific Islands, Far East and C. & S. America.

72
The major endemic area for Histoplasmosis
A. Africa
B. Central America
C. Southwest United States
D. South America

654
E. Eastern United States
►E

The endemic area for Histoplasmosis includes: the Ohio, Mississippi and Missouri River Valleys
Syracuse NY area and the Caribbean. This organism prefers soil with a high nitrogen content
such as that enriched with bird and bat guano.

73
Black granules are found in mycetoma caused all the following organisms except:
A. Madurella grisea
B. M. mycetomatis
C. Exophiala jeanselmei
D. Curvularia
E. Scedosporium apiospermum
►E

All organisms listed other than S. apiospermum cause black granules. S. Apiospermum which is
the filamentous mold form of pseudallescheria boydii has white granules.

74
Which of the following methods of direct microscopic examination is chitin specific?
A. Chlorazol black E
B. Calcofluor white
C. Gomori Methenamine Silver
D. Fontana-Masson
E. KOH
►A

Chlorazol black E is chitin specific. Calcofluor white is glucan specific. Gomori Methenamine
Silver (GMS) and Fontana-Masson are histology stains. KOH is a rapid, easy, reliable method
for diagnosing fungal infections, but is not chitin-specific.

75
Which of the following statements regarding mycelium is NOT true?
A. A mass of hyphae is mycelium
B. Mycelium has reproductive capability
C. Hyphae forming corkscrew-like turns are spiral hyphae
D. Pectinate bodies are hyphae resembling a comb
E. Racket forms are club-shaped cells
►B

655
Mycelium does not have reproductive capabilities. The remaining statements are correct.

76
All of the following are features of mycetoma except:
A. Swelling
B. Self-limited
C. Granules
D. Fascia and bone involvement
E. Draining sinuses
►B

Mycetomas may be subdivided into eumycetoma and actinomycetoma . Mycetomas generally


begin as subcutaneous swellings occurring on the feet. Later, sinuses, granules, and nodules may
be seen. The skin and subcutaneous tissue may also become involved. Treatment is difficult in
later stages.

77
A landscape worker complained of several tender nodules on the right dorsal hand and forearm.
The biopsy of the lesion showed asteroid bodies but no organisms. The likely diagnosis is:
A. Nocardiosis
B. Blastomycosis
C. Sporotrichosis
D. Candidiasis
E. Coccidioidomycosis
►C

Sporothrix schenckii produces tiny yeast cells 4-6 um difficult to demonstrate on biopsy unless
there is an overwhelming infection. Asteroid bodies are seen on H & E and PAS as radiating
fingers of eosinophilic material.

78
A 49-year-old man has painless subcutaneous nodules on his feet with sinus tracts and abscesses.
Pathology shows "grain" in sinus tract drainage. A diagnosis of eumycotic mycetoma is made.
What is the most common cause in the United States?
A. Acremonium
B. Curvalaria
C. Exophilia jeanselmei
D. Pseudallescheria boydii
E. Nocardia

656
►D

Eumycotic mycetoma is fungal and can be caused by Pseudallescheria boydii (most common in
US), Madurella, Acremonium, curvalaria, and Exophilia jeanselmei. Actinomycotic mycetoma is
bacterial and maybe caused by Nocardia asteroids or brasiliensis, Streptomyces, and
Actinomyces.

79
Which of the following organisms would you expect to recover from a fluorescent tinea capitis?
A. T. tonsurans
B. M. gypseum
C. M. canis and M. gypseum
D. T. violaceum
E. T. rubrum
►C

M. gypseum may occasionally produce a dull fluorescence. M. canis is fluorescent.

80
A child presents with several yellowish, cup-shaped crusts on the scalp, some with single hairs
piercing through the center. KOH prep reveals arthroconidia and airspaces within the hair shaft.
A likely cause is:
A. M. gypseum
B. M. canis
C. T. tonsurans
D. T. verrucosum
E. T. mentagrophytes
►A

This child is presenting with the characteristic scutula of favus. Favus is most often caused by T.
schoenleinii, T. violaceum, and M. gypseum.

81
Which of the following is true regarding culture of the organisms that cause Pityriasis
versicolor?
A. Difficult to grow, requires olive oil overlay
B. Grow easily on Mycosel media
C. Cannot be cultured
D. Grow easily on Dermatophyte Test Media
E. Grow easily on Sabouraud Dextrose Agar

657
►A

Pityriasis versicolor (Tinea versicolor) is caused by Malassezia furfur, M. globosa (most


common cause), M. pachydermatis, M. dermatis, M. obtuse, M. restrica, M. sympodialis, and M.
sloofiae. The organisms that cause Pityriasis versicolor are difficult to grow. The culture requires
olive oil overlay.

82
Choose the correct statement regarding Coccidioidomycosis:
A. The most common form of primary inoculation is cutaneous
B. Dissemination may involve the bones, joints, viscera, brain and skin
C. Causative organism, C. immitits, is a thick-walled spherule with a
polysaccharide capsule demonstrated with Alcain blue
D. Erythema nodosum is a poor prognotic sign
E. Outbreaks occur in the Mississippi and Ohio River Valley
►B

Coccidioidomycosis: Primary inoculation is pulmonary. Erythema nodosum is a favorable


prognostic sighn. Dissemination from the localized pulomary lesions may occur in less than 1%
of infections. Target ogans of dissemination include the bones, joints, viscera, brain, meinnges,
and skin. The causative organisms is Coccidioides immitis, is a nonbudding, thick-walled
spherule, with a polysaccharide capsule demonstrated with Gridley or Gomori methanamine
silver stain. It is endemic in northern Mexico, Venezuela, and in southwest US, especially
California

83
This organism produces an endothrix tinea capitis:
A. Trichophyton mentagrophytes
B. Microsporum gypseum
C. Microsporum nanum
D. Trichophyton verrucosum
E. Trichophyton soudanense
►E

Only T. soudanense in this group of choices produces an endothrix type of tinea capitis.
Endothrix do not fluoresce. Trichophyton mentagrophytes, Microsporum nanum and
Trichophyton verrucosum produce nonfluorescent ectothrix tinea capitis. Microsporum gypseum
may produce a fluorescent or nonfluorescent ectothrix tinea capitis.

84

658
Mycelia can form structures with a comb-lke appearance called:
A. Racket forms
B. Favic chandeliers
C. Pectinate bodies
D. Spiral hyphae
E. Nodular bodies
►C

Racket forms (club-shaped cells), favic chandeliers (terminal hyphal branches having an antler-
like appearance), pectinate bodies (hyphae resembling a comb), spiral hyphae (hyphae forming
corkscrew-like turns), and nodular bodies (knot-like structure of hyphae) are among the types of
mycelia.

85
Which of the following stains is specific for chitin?
A. KOH
B. KOH with DMSO
C. Swartz Lamkins
D. Chlorazol Black E
E. Calcofluor White
►D

Chlorazol Black E is a chitin specific stain. The remaining options are useful in direct
microscopic examination for fungal elements, but are not chitin specific.

86
Sabouraud Dextrose Agar (SDA) Emmons Modification contains which of the following?
A. Peptones
B. Phenol red
C. Chloramphenicol
D. Gentamicin
E. Cycloheximide
►A

SDA Emmons Modification is the gold standard medium. It is nutritionally poor, containing
dextrose, peptones, water, and agar, and encourages sporulation. Phenol red, gentamicin, and
cycloheximide are found in dermatophyte test media (DTM). Chloramphenicol and
cycloheximide are found in Mycosel/Mycobiotic.

87

659
A 12 year old boy has a pruritic bullous eruption on his feet. A KOH is positive and a fungal
culture shows microconidia in grape-like clusters. The etiology is:
A. Trichophyton verrucosum
B. Microsporum canus
C. Trichophyton mentagrophytes
D. Microsporum gypseum
E. Trichophyton tonsurans
►C

Trichophyton mentagrophytes is the dermatophyte primarily responsible for bullous tinea pedis.
It typically has septate, spiral hypahe with cigar or grape like thin-walled microconidia.

88
Cutaneous lesions of Cryptococcosis may be 1. Nodular 2. Papular 3. Granulomatous-ulcerative
4. Herpetiform 5. Cellulitis-like:
A. 1, 3, 4
B. 3, 4, 5
C. 2, 3, 4
D. 1, 3, 4, 5
E. All of these answers are correct
►E

Polymorphous lesions have been reported in Cryptococcosis

89
A neutropenic patient has erythematous nodules for several days. A biopsy shows branching
septate hyaline hyphae. Which of the following conditions should be included in the differential?
1. Coccidioidomycosis 2. Fusariosis 3. Aspergillosis 4. Zygomycosis 5. Phaeohyphomycosis
A. 1
B. 1, 2
C. 2, 3
D. 3, 4
E. 2, 3, 5
►C

Aspergillus and Fusarium produce similar appearing morphology in biopsy. Coccidioidomycosis


produces spherules, very few if any septations are seen in Zygomycosis, dark hyphae, yeast cells
are seen in Phaeohyphomycosis.

90

660
Trichophyton tonsurans sporulates via thick-walled round cells resistant to the environment
known as:
A. Arthroconidia
B. Blastoconidia
C. Chlamydoconidia
D. Sporangia
E. Mycelia
►C

Chlamydoconidia are thick-walled round cells, resistant to the environment. Arthroconidia are
formed by fragmentation of hyphae, and may appear as thick or thin-walled rectangular cells.
Blastoconidia are formed by budding. Sporangia are spores that are produced in a sac.

91
Which of the following methods of direct microscopic examination for fungi is glucan specific?
A. KOH
B. Swartz Lamkins
C. Chlorazol black E
D. Calcofluor white
E. Mayer's mucicarmine
►D

Calcofluor white is a glucan specific stain. Chlorazol black E is chitin specific. KOH is a rapid,
easy, reliable method of diagnosing fungal infections but is not glucan specific; Swartz Lamkins
contains a counterstain but is not glucan specific. Mayer's mucicarmine is a histology stain.

92
Which of the following causes tinea imbricata?
A. Epidermophyton floccosum
B. Trichophyton concentricum
C. Trichophyton rubrum
D. Trichophyton mentagrophytes
E. Trichophyton tonsurans
►B

Tinea imbricata is characterized by concentric, annular, scaly rings resembling erythema


annulare centrifugum. The causative agent is T. concentricum, which is endemic to the South
Pacific, South and Central America and Asia. The infection is usually chronic.

93

661
Which of the following statements regarding superficial mycotic infections is TRUE?
A. Malassezia species do not fluoresce under a Wood‘s lamp
B. Phaeoannellomyces wernecki causes tinea nigra
C. Trichophyton beigelii causes white piedra
D. Trichosporon hortae causes black piedra
E. Micrococcus sedentarius causes tinea nigra
►B

Malassezia species fluoresce pale yellow with Wood‘s lamp examination. Trichosporon beigelii
(not trichophyton) causes white piedra, an infection that causes tan to white, soft non-adherent
concretions on head and pubic hairs. Piedraia hortae causes black piedra, which forms dark
concretions that are adherent to scalp, beard, and pubic hairs. Micrococcus sedentarius is a
bacterium that is responsible for pitted keratolysis, which is characterized by crateriform pitting
that primarily affects the pressure-bearing aspects of the plantar surface of the feet and,
occasionally, the palms of the hands. Tinea nigra (superficial phaeohyphomycosis) presents with
asymptomatic brown to black, macular, non-scaly macules and patches on the palms or soles.
Phaeoannellomyces werneckii is causative.

94
The outstanding characteristics of Aspergillus species on biopsy include:
A. Blastoconidia
B. Hyaline, septate dichotomously branching hyphae
C. Copper pennies
D. Ribbon-like filaments that may be twisted and distorted branching at right angles
E. Stain with Fontana-Mason stain
►B

Aspergillus is seen as wide (3 um) septate dichotomously branching (45‖ angles) hyphae.

95
Of the following Candida species, which is likely to cause disseminated Candidiasis with
cutaneous lesions?
A. Candida albicans
B. C. parapsilosis
C. C. tropicalis
D. C. krusei
E. C. neoformans
►C

662
C. albicans is responsible for approximately 50% of all Candidemia, but C. tropicalis is seen with
increasing frequency among patients with dissemination and cutaneous manifestations.

96
This dermatophyte is not an anthropophilic organism:
A. Trichophyton rubrum
B. Epidermophyton floccosum
C. Microsporum gypseum
D. Microsporum audouinii
E. Trichophyton soudanense
►C

M. gypseum is a geophilic organism, all others listed are anthropophilic.

97
Two weeks after a bicycle accident a 25-year-old female diabetic patient complained of a
swollen tender right cheek The PAS biopsy demonstrated ring forms and distorted wide hyphae
with few septations and right angle branching. The diagnosis is:
A. Aspergillosis
B. Actinomycosis
C. Mucormycosis
D. Dermatophytosis
E. Fusariosis
►C

Having diabetes is a risk factor for Mucormycosis, ring forms in the biopsy are the x-section of
the wide sparsely septate hyphae.

98
Which of the following usually does not fluoresce bright green upon Woods lamp examination?
A. M. audouinii
B. M. canis
C. T. violaceum
D. T. schoenleinii
E. M. distortum
►C

Fluorescent-positive infections are caused by:"Dogs And Cats Fight Some Gypsies" M.
Distortum M. audouinii M canis M. ferrugineum T. schoenleinii M. gypsium (occasionally)

663
99
Which of the following is NOT used for histologic examination for fungal infection?
A. Chlorazol Black-E
B. Gormori Methanamine Silver
C. Periodic Acid Schiff
D. Fontana-Masson
E. Mayer's mucicarmine
►A

Chlorazol Black-E is a stain specific for chitin and is used in direct examination, not histologic
preparations. The remaining options are histology stains useful in fungal identification in tissue.

100
This organism causes a resistant tinea pedis indistinguishable from dermatophytosis:
A. Aspergillus flavus
B. Scytalidium dimidiatum
C. Curvularia sp.
D. Scopulariopsis
E. Fonsecaea sp
►B

This fungus is very difficult to treat due to its resistance to most antifungals. It is sensitive to
cylcoheximide so it should be cultured on media free of this antifungal.

101
Which of the following organisms is most likely to cause a sporotrichoid nodule on the arm:
A. Nocardia brasiliensis
B. Phialophora verrucosa
C. Rhinospiridium seeberii
D. Fusarium
E. Fonsecaea pedrosoi
►A

Nocardia brasiliensis is a common cause of actinomycotic mycetoma and the ulcerative, draining
lesions can be misdiagnosed as sporotrichosis.

102
The organism that causes white piedra:
A. Does not cause onychomycosis
B. Is known as Piedraia hortae

664
C. May also cause post-operative wound infections
D. Grows as small, compact, black-greenish velvety colonies
E. Can only be treated by cutting off affected hairs
►C

Trichosporon beigelii (new nomenclature: Trichosporon ovoides and Trichosporon inkin) causes
white piedra as well as other superficial infections, including post-operative wound infections,
paronychia, and onychomycosis. It grows as cream to yellow-colored pasty colonies when
cultured. Treatment is with topical amphotericin B lotion, benzoic acid, salicylic acid, or cutting
off affected hair. Black piedra is caused by Piedraia hortae, which grows as small, compact,
black-greenish velvety colonies when cultured, and can be treated only by cutting off affected
hairs.

103
A 56-year-old male with HIV presents with multiple umbilicated brown papules on the face.
Which of the following is the least likely cause of his lesions?
A. Cryptococcus
B. Histoplasmosis
C. Coccidiomycosis
D. Penicilliosis
E. Blastomycosis
►E

This patient has molluscum-like lesions on the face in the setting of immunosuppression. The
differential for such papules includes cryptococcus, histoplasmosis, coccidiomycosis, and
penicilliosis. Blastomycosis can cause sporotrichoid lesions, warty vegetations, papillomatous
proliferations, and osteomyelitis; it usually does not produce molluscum-type papules.

104
Which of the following statements about dimorphic fungi is INCORRECT?
A. Are in the mold form in the environment
B. Are in yeast/spherule form in tissue
C. Are in mold form at 25 degrees C
D. Are in yeast/spherule form at 37 degrees C
E. Are in mold form in tissue
►E

Yeasts and molds are not mutually exclusive. Dimorphism can occur - a dimorphic fungus will
be in a mold form in the environment at 25 degrees and in a yeast/spherule form at 37 degrees C.

665
105
A patient returns from a vacation in Brazil with keloidal-like nodules on the face and arms. The
patient denies any sick contacts or exposures and reports only lying on the beach and swimming
with dolphins. She likely has:
A. Actinomycosis
B. Lobomycosis
C. Chromoblastomycosis
D. Mucormycosis
E. Sporotrichosis
►B

Lobomycosis, also called Keloidal blastomycosis, is caused by Loboa loboi. Lesions


characteristically appear keloidal with or without fistulas. A common location for the infection is
on the ears and has been associated with dolphins. Histologically, double contoured refractile
spherules with budding may be seen.

106
What organism produces round thick walled spiny macroconidia and pear shaped microconidia?
A. Sporothrix schenckii
B. Blastomyces dermatitidis
C. Paracoccidioides brasiliensis
D. Cryptococcus neoformans
E. Histoplasma capsulatum
►E

At room temperature Histoplasma capsulatum produces round thick walled spiny macroconidia
and pear shaped microconidia.

107
Which of the following stains is NOT used to stain fungal elements?
A. Mucicarmine
B. Periodic Acid Schiff
C. Fontana-Masson
D. Von Kossa
E. Gormori methenamine silver
►D

Mucicarmine stain is used to stain adenocarcinoma in addition to infectious organisms including


Cryptococcus and Rhinosporidium. The Cryptococcus capsule stains red. Periodic Acid Schiff
(PAS) is useful in identifying fungi, parasites, glycogen and the basement membrane. The wall

666
of the organism stains red. Fontana-Masson stains melanin black and therefore helps identify
dematiaceous (melanin-producing) fungi. Gormori methenamine silver stains fungi, parasites,
Donovan bodies and Rhinoscleroma black. Von Kossa stain is used to identify calcium by
staining it black.

108
Cryptococcus neoformans resides in:
A. The Mississippi Valley region
B. Chicken roosts
C. Pigeon droppings
D. Sandy soil
E. None of these answers are correct
►C

This organism is ubiquitous and is abundant in soil enriched with pigeon droppings.

109
All of the following parameters are used to distinguish dermatophytes except:
A. Nutritional requirements
B. Colony morphology
C. Sucrose hydrolysis
D. Hair fluorescence
E. Growth temperature
►C

Sucrose hydrolysis is not a useful parameter to distinguish dermatophytes from one another.
Growth temperature, colony morphology, hair fluorescence and nutritional requirements can be
used to identify dermatophytes.

110
Septate hyphae with 90‖ branching and thick walled barrel shaped arthroconidia alternating with
empty cells best describes the microscopic morphology of:
A. Histoplasma capsulatum
B. Sporothrix schenckii
C. Microsporum gypseum
D. Trichophyton tonsurans
E. Coccidioides immitis
►E

This is the only organism in this list that fits this description.

667
111
A mass of hyphae is known as a:
A. Sporangia
B. Conidia
C. Mycelium
D. Chlamydoconidia
E. Pseudohyphae
►C

A mass of hyphae (vegetative tube-like structures) is known as a mycelium. Sporangia, Conidia,


and Chlamydoconidia are structures formed through different types of asexual reproduction.
Pseudohyphae are seen in yeast.

112
Which pathogen is not inhibited by cycloheximide?
A. Cryptococcus
B. Coccidioides immitis
C. Prototheca
D. Scopuloniopsos brevicaulis
E. Scytalidium species
►B

Cycloheximide is used in fungal culture media to reduce contaminants.

113
A 34-year-old male patient presents with a verrucous lesion of the nasal mucosa; the biopsy
showed spherules ranging in size from 200 to 325 um. The mature endospores have a rough
appearance. The diagnosis is:
A. Coccidioidomycosis
B. Protothecosis
C. Rhinosporidiosis
D. Pseudallescheriosis
E. Cryptococcosis
►C

The large size of the spherule and the rough endospores describe this organism along with the
clinical symptoms. The other organisms that produce spherules are Prototheca (8-20 um) and
Coccidioides immitis (10-80 um)

668
114
The most common cause of tinea capitis in the United States today is:
A. Trichophyton schoenleinii
B. T. tonsurans
C. T. mentagrophytes
D. Microsporum audouinii
E. M. canis
►B

M. audouinii was previously the primary cause of T. capitis in the US.

115
Which of the following dermatophytes causes endothrix tinea capitis?
A. Microsporum gypseum
B. Microsporum audouinii
C. Trichophyton tonsurans
D. Microsporum canis
E. Microsporum ferrugineum
►C

Organisms that cause an ectothrix pattern of tinea capitis include M. canis, M. audouinii, M.
ferruginosum, M. distortium, M. gypseum and occasionally T. rubrum. T. tonsurans, T.
violaceum, T. soudanense, T. gourvilli, T. yaoundei, and occasionally T. rubrum cause an
endothrix pattern. T. schoenleinii causes favus in which hyphae and air spaces are seen in the
hair shaft. A bluish-white fluorescence pattern is seen with Wood‘s lamp. Clinically, patients
have thick, yellow, cup-shaped crusts (scutula); scarring and secondary infection may result. M.
canis, M. audouinii, M. ferruginosum, M. distortium display a yellow fluorescence on Wood‘s
lamp examination.

116
This agent causes a superficial, asymptomatic infection usually on the palms of the hands. The
lesions are flat, nonscaly and appear as irregularly shaped brown macules:
A. Piedra hortae
B. Cladosporium carrionii
C. Aureobasidium pullulans
D. Phaeoannellomyces werneckii
E. Exophiala jeanselmei
►D

669
These lesions often have a deeper pigment at the advancing border. The other organisms are
dematiaceous but are not the etiologic agent of tinea nigra.

117
This organism is the most common cause of eumycotic mycetoma in the US:
A. Madurella mycetomatis
B. Nocardia asteriodes
C. Scedosporium apiospermum
D. Phialophora verrucosa
E. Wangiella dermatitis
►C

Although fairly uncommon, S. apiospermum is the most frequently recovered organism causing
eumycotic mycetoma in the US. This name represents the asexual stage of the organism which
grows as a filamentous mould, the sexual state, Pseudallescheria boydii, grows as a cleistothecia
(round sac containing ascospores).

118
Which dermatophyte is most commonly responsible for tinea corporis gladiotorum?
A. T. mentag
B. T. tonsurans
C. T. schonlenleinii
D. T. rubrum
E. E. floccosum
►B

Tinea corporis gladiotorum is caused by Trychophyton tonsurans. Clinically, it appears as well-


defined, scaly plaques usually located on the arms, neck, and head.

670
671
672

You might also like